量子力学讲义
张永德
一九九七.九
第一章 量子力学的物理基础
§1.1 实验基础
1,第一组实验 —— 光的粒子性实验
2,第二组实验 —— 粒子的波动性实验
§1.2 基本观念
1,de Broglie关系式与波粒二象性
2,波粒二象性的初步分析
3,de Broglie波的初步分析
4,波粒二象性的再分析 —— 某些基本推论
§1.3 测不准关系
1,Heisenberg测不准关系
2,进一步解释及某些应用
§1.4 量子力学的基本假设
1,第一公设 —— 波函数公设
2,第二公设 —— 算符公设
3,第三公设 —— 测量公设
4,第四公设 —— 微观体系动力学演化公设
5,第五公设 —— 全同性原理公设
6,公设应用举例 —— 广义测不准关系推导
第二章 方程的一般讨论 dingeroSchr &&
§2.1 方程 dingeroSchr &&
§2.2 方程基本性质讨论 dingeroSchr &&
1,线性性质与态叠加原理
2,几率流与几率的定域守恒
3,稳定势场下方程的一般解 dingeroSchr &&
4,势场奇点和界面处波函数的一般行为
5,能量平均值下限问题
6,能谱分界点问题
7,本征函数族完备性与能量可观测性问题
§2.2 方程向经典力学的过渡 dingeroSchr &&
1,h过渡方式 0→
2,取平均值过渡方式
§2.2 力学量期望值的运动方程和对易子计算
1,力学量期望值的运动方程
2,对易子运算
第三章 一维问题
§3.1 一维定态的一些特例
1,一维量子谐振子问题
2,一维势垒透射问题
3,一维周期势(Kronig-Penney势)问题
4,均匀势场中的运动
§3.2 一维定态的一般讨论
1,本征函数族完备性定理
2,束缚态存在定理
3,无简并定理
4,零点定理
§3.3 一维高斯波包的自由演化
第四章 中心场定态问题
§4.1 前言
§4.2 轨道角动量及其本征函数
§4.3 几个一般分析
1,m量子数简并的离心势
2,径向波函数的边界条件 0→r
3,径向解的完备性问题
4,粒子回转的角动量和波尔磁子
§4.4 球方势阱问题
1,束缚态问题
2,无限深球方势阱问题
3,自由粒子球面波解
4,非束缚态问题
§4.5 库仑场 —— 氢原子问题
1,Schr方程及其求解 dingero&&
2,讨论
§4.6 三维各向同性谐振子问题
1,方程及其求解 dingeroSchr &&
2,讨论
第五章 表象与表示
§5.1 幺正变换和反幺正变换
1,幺正算符定义
2,幺正算符的性质
3,幺正变换
4,反幺正变换
§5.2 表象概念
1,波函数的标记和分类
2,量子力学的表象概念
3,几种常用的表象
§5.3 量子力学的Dirac符号表示
1,Dirac符号
2,Dirac符号的一些应用
3,关于Dirac符号的一个注记
§5.4 Wigner定理
1,Wigner定理
2,讨论
§5.5 量子力学的路径积分表示
1,传播子与Feynmann公设
2,和方程的等价性 dingeroSchr &&
3,传播子( )
00
,;,trtr
r
U
r
的再研究
4,路径积分计算之一 —— 自由粒子情况
5,路径积分计算之二 —— 谐振子情况
§5.6 非惯性系中的量子力学
1,等效原理
2,Schr方程的广义Galilean变换 dingero&&
3,COW实验
4,引力红移
第六章 对称性及其应用
§6.1 一般叙述
1,对称性的含义
2,量子力学中的对称性
3,对称性与守恒律及守恒量
§6.2 时空对称性及其应用
1,时间均匀性和能量守恒定律
2,空间均匀性和动量守恒定律
3,空间各向同性和角动量守恒定律
4,空间反射对称性和宇称守恒
5,时间反演对称性
§6.3 内禀对称性
1,同位旋空间旋转对称性和同位旋守恒
2,微观粒子全同性原理
第七章 电子自旋角动量
§7.1 电子自旋角动量
1,电子自旋的实验基础和其特点
2,电子自旋态的表示法
3,自旋算符与Pauli矩阵
4,例算
5,
2
1
自旋态的极化矢量与投影算符
§7.2 自旋角动量与轨道角动量的耦合
1,与的合成 S
r
L
r
2,角动量的升降算符
3,自旋 — 轨道耦合作用与碱金属原子光谱的双线结构
4,耦合表象与无耦合表象基矢的相互展开
§7.3 两个
2
1
自旋角动量的耦合
1,自旋单态和自旋三重态
2,两套基矢 —— 耦合表象基和无耦合表象基
3,例算
4,自旋交换算符
第八章 电磁作用问题
§8.1 电磁场下的方程 dingeroSchr &&
1,最小电磁耦合原理及电磁场下的方程 dingeroSchr &&
2,方程的某些考察
§8.2 均匀磁场下粒子的运动 —— 无自旋情况
1,均匀磁场下不考虑自旋的中心场方程 dingeroSchr &&
2,正常Zeeman效应
3,均匀磁场下自由带电粒子运动
§8.3 均匀磁场下自由中性带磁矩粒子的运动 ——
2
1
自旋情况之一
1,中子极化矢量在磁场中的进动
2,中子的旋量迭加与干涉 —— 中子干涉量度学
§8.4 均匀磁场下束缚粒子的运动 ——
2
1
自旋情况之二
1,均匀磁场下
2
1
自旋粒子中心场方程求解 dingeroSchr &&
2,讨论
§8.5 Aharonov-Bohm效应
1,磁AB效应
2,向电磁AB效应的推广
3,几点讨论
§8.6 超导量子理论基础
1,超导体中的流密度与London方程
2,Meissner效应
3,磁通量子化及磁荷
4,超导Josephson结的AB效应
第九章 定态微扰论
§9.1 非简并态微扰论
1,基本方程组
2,一阶微扰论
3,二阶微扰论
4,例算
§9.2 简并态微扰论
1,简并态微扰论要旨
2,简并态微扰论
3,例算
第十章 势散射理论
§10.1 一般描述
1,散射(碰撞)实验的意义及分类
2,基本描述方法 —— 微分散射截面
3,入射波、散射波和散射振幅
§10.2 分波法 —— 分波与相移
1,分波法的基本公式
2,分波法的一些讨论
3,光学定理
4,低能散射的一般特征
§10.3 格林函数法与波恩近似
1,格林函数法与势散射基本积分方程
2,一阶波恩近似
3,波恩近似适用条件分析
4,例算
§10.4 全同粒子散射
1,全同性原理在散射问题上的应用
2,例算
§10.5 考虑自旋的散射
1,散射分道的概念
2,分道散射振幅计算 —— 带自旋的波恩近似
3,自旋权重平均
4,例算
第十一章 含时问题与量子跃迁
§11.1 含时问题求解的一般讨论
1,量子力学中时间相关问题的一般分析
2,含时系统初始衰变率的一个普遍结论
3,初态衰变系统长期衰变规律的一个分析
4,量子Zeno效应
5,含时问题在相互作用图象中的处理
§11.2 时间相关微扰论与量子跃迁
1,含时扰动及状态之间量子跃迁的概念
2,量子跃迁系数基本方程组及其一阶近似
§11.3 几种常见含时微扰的一阶近似计算
1,常微扰情况
2,周期微扰情况
§11.4 Sudden及不撤除微扰的情况
§11.5 光辐射与物质的相互作用
1,概论
2,受激原子的量子跃迁
3,电偶极辐射
4,受激氢原子的光电效应
第十二章 WKB近似方法
§12.1 WKB近似方法的原理
1,经典近似讨论
2,WKB近似方法原理
3,成立的条件
4,一般讨论
§12.2 WKB近似方法的应用
1,一维WKB解
2,中心场WKB解
3,势垒贯穿
4,矩阵元计算
§12.3 WKB近似的进一步讨论
1,一般讨论
2,几个数学细节
前言
到19世纪末,经典物理学的两个主要支柱——牛顿力学和光学(包括电磁波理论)取得了辉煌的成就。在巨大的成就灿烂光辉的眩惑下,原本对立的粒子概念和波概念,更被物理学家们将其普适化和绝对化了。与此同时,牛顿力学和波动力学的描述方法也被普适化和绝对化了。仿佛物理学所研究的全部对象,它们的性质真的非此即彼了。与此相应,拉格朗日决定论这种其实是因果论的最简单形式也被普适化和绝对化起来,看成是因果论的唯一正确的形式。用爱因斯坦的话说就是:“上帝是不玩掷骰子的”。
实际上,物理世界并非如此。爱因斯坦这句话虽然并非论据,仅仅表达了他的一种信念,
却显然是一种混入了主观主义的信念。要知道,我们能有什么“根据”去肯定“上帝”一定是“不玩掷骰子”的呢?就根据经典物理学的巨大成就和伴随着的拉格朗日决定论吗?这显然是一种不可靠的根据,一种主观的推测。因为经典物理学(以及与之相关的拉格朗日决定论)所取得的辉煌成就有明显的极大的局限性。牛顿力学(包括后来建立的相对论力学)并未涉及物质的内部构造、物体的内禀性质,而只局限于研究物体在其外在时空中的低速(高速)机械运动;光学(包括后来的电磁波理论)也并没有涉及光的产生和吸收、光和物质的相互作用,而只局限于研究光的传播。经典物理学,一旦超出原来的范畴,进入这些新领域,
便呈现出了捉襟见肘和漏洞百出的状态。就在经典物理学处于颠峰状态的19世纪末,也已经发现了许多无法用经典物理学去理解的现象。比如,1896年贝克勒耳发现放射性、黑体辐射问题中出现的紫外灾难、光电效应等等。这些都是经典物理学晴朗天空边际的朵朵乌云,
预示着暴风雨的即将来临。
话得说回来,人们总是习惯于根据已有的知识和经验去思考新问题、理解新现象。尤其当已有的理论已经取得了辉煌成就的情况下,这种已有的理论就常常会转化为精致的鸟笼,
束缚或局限住人们的创造性思维。不幸的是,这种“鸟笼”作用常常是习惯性的、不自觉的、
不易觉察的,从而也就是不易挣脱的。因此,在从经典物理学过渡到量子理论的时候,在学习量子力学的初始阶段,必须善于剖析自己的积习观念,必须善于从经典物理学光芒所编织的鸟笼中挣脱出来,按照新的实验事实所指引的方向,利用缜密的逻辑思维前进。应当说,
新的实验事实、新的实验现象永远是医治人们物理思想僵化的特效药方;严密的逻辑思维是我们最可靠的工具。两者的恰当结合,才是正确指引物理学家前进的唯一灯塔,是肯定或扬弃一切物理理论的唯一裁判。其中,实验检验又是肯定或扬弃一切理论的最高和最后裁判。
当今的量子理论已经发展成为庞大的理论群体。可以毫不夸张地说,量子理论是物理学家迄今建立的最宏伟的物理理论,它博大精深、包罗万象,小至夸克和胶子的量子色动力学,
大至宇宙的量子创生理论,量子理论无所不在。量子理论已经取得了前所未见的巨大的辉煌成就。
正如在经典物理学辉煌成就面前不应当目眩神夺一样,在量子理论辉煌成就面前,也应保持清醒的头脑。量子理论仍然不是人类不懈追求的最终真理。从量子理论诞生的时刻起,
成功和困难就想躯体和影子那样,一直相伴相随着:成功的躯体越长越高大,困难的阴影也愈来愈浓重、愈清晰。这一切都说明,量子逻辑中正在积聚着一场火山爆发所需的能量,预示着比本世纪初的那场暴风雨更猛烈更深刻的革命。
编写这本教材所遵循的主导思想有三点,一是偏重物理,也就是偏重物理思想的发掘、
物理思想的阐述、物理内涵的剖析,对数学论证则力求简洁有力,以求得对量子力学物理原理的深刻领悟。二是尽力向前看,尽可能包括量子力学的一些新进展,以期求得稍稍变更基础教材难脱的老面貌。当然,由于作者水平和知识所限,也由于教学大纲和学时的约束,做好这一点是困难的,只能说是尽量而已。三是论述中尽量做到封闭与开放相结合,也就是阐述量子逻辑的自洽性和力量美的同时,尽量可能随时随地指出其开放的一面,指出目前认识的边界所在,以便或是明了该问题描述的局限性,或是增加对量子力学固有困难的了解,这既有助于加深对现有内容的理解,又能活跃思想,尽量不使量子力学僵化成为新的教条,成为束缚人们思维的新“鸟笼”。
一九九六年二月二十九日 作者谨识
第一章 量子力学的物理基础
§1.1 实验基础
这里简述一些上个世纪末直到这个世纪30年代所进行的著名实验,这些实验奠定了量子力学 的基本观念,触发了从经典物理学向量子理论的跃变,并为这种跃变提供了最初一批实验事实。
1,第一组实验 —— 黑体辐射、光电效应、康普顿散射。它们给出了能量分立、光场量子化概念,从实验上揭示了光的粒子性质。
上个世纪末,黑体辐射谱已被实验物理学家很好地测定了,但从经典物理学观念出发却无论如何难以通盘地理解。1896年,Wien从经典统计理论和黑体辐射经验规律出发,给出了黑体辐射谱的公式。若记黑体辐射空腔单位体积中辐射场的频率在ν ν ν→+d间的能量密度为dE d
ν
ε ν ν= (),则该公式可以明确地写为
dE d c e d
c
ν
νβ
εν ν ν ν==
()
1
3
2
(1.)
这里、是两个常系数,c
1
c
2
β = 1/ kT。这一公式在短波长或高频率区间内与实验符合,
但在中、低频区,特别是低频区与实验差别很大。1900年Rayleigh、1905年Jeans将腔中黑体辐射场看成是大量电磁波驻波振子的集合,利用能量连续分布的经典观念和麦克斯韦-
波尔兹曼分布律,导出了黑体辐射谱的另一个表达式——Rayleigh-Jeans公式。若记
ε ν ε
νν
() N=,这里是腔中辐射场单位体积内频率N
ν
ν附近单位ν间隔内电磁驻波振子自由度数目,简单计算可得它为
8
2
3
πν
c;而ε
ν
则是对应于ν的驻波振子的平均能量,由麦-
波分布律可得
ε
εε
ε
ν
εβ
εβ
==




ed
ed
kT
0
0
于是得到Rayleigh-Jeans公式为
dE d
kT
c
d
ν
εν ν
πν
ν==()
8
2
3
(1.2)
与Wien公式的情况正相反,这个公式在低频部分与实验曲线符合得很好,但在高频波段不但不符合,而且出现黑体辐射能量密度趋于无穷大的荒谬结果。这就是著名的所谓“紫外灾难”,是经典物理学最早显露的困难之一。1900年Planck引入能量子的概念,假设黑体辐射空腔中振子的振动能量并不象经典理论所主张的那样和振幅平方成正比并呈连续变化,而是和振子的频率ν成正比并且只能取分立值,
ν:023,,,,......hhhν ν ν
与此相应,空腔场和腔壁物质之间交换能量也是这样一份份的。进一步,按经典统计理论的麦-波分布律,与上述能级对应的比例系数分别为
1
23
,,,,......ee e
hhhνβ νβ νβ
将这些系数归一化(即除以这些系数的总和)使它们变成为权重系数。从而,对应频率ν的驻波振子的平均能量成为
ε
ν
β
ν
νβ
νβ
νβ
==?
=

=

=




nh e
e
e
nh
n
nh
n
nh
n
0
0
0
ln( )
=?=
β
ν
νβ
νβ
ln( )1
1
e
h
e
h
h
将此ε
ν
乘以自由度数目,即得Planck公式
εν ν
πν ν
νβ
()d
h
c
d
e
h
=
8
1
3
3
(1.3)
显然,这一公式在高频和低频波段分别概括了Wien公式和Rayleigh-Jeans公式,体现了关于辐射谱峰值位置的Wien位移定律。总之,此公式在全波段范围内与实验曲线十分符合。
这说明了,在解释黑体辐射这一辐射场与腔壁物质相互作用的实验规律中,必须假定腔内电磁场和腔壁物质之间所交换的能量是断续的、一份一份的,即,对所有频率,相应的能量都是量子化的。
光电效应的实验规律,自1887年赫兹起,直到1916年密里根止,逐步被揭示出来。其中,不能为经典物理学所理解的实验事实有:反向遏止电压(和逸出电子的最大动能成正比)
和入射光强无关;反向遏止电压和入射光的频率成线性关系;电子逸出相对于光的照射而言几乎无时延。它们难于理解是因为按经典观念,入射光引起金属表面电子作强迫振动,入射光强越大,强迫振动的振幅也越大,逸出的电子的动能也应当越大,从而反向遏止电压和入射光强应成正比关系,而且也应当和入射光的频率无关。此外,自光照射时起,电子从受迫振动中积聚能量直至逸出金属表面,这需要一段时间,因为电子运动区域的横断面积很小,
所能接受的光能很小,电子积聚到能逸出金属表面那样的动能需要一定的时间。然而,实验却表明,这个弛豫时间很短,它不大于10秒。为了解决这些矛盾,1905年,爱因斯坦在普朗克的能量子概念基础上,再大胆地前进一步,提出了光量子概念,并指出光量子和电子碰撞并被电子吸收从而导致电子的逸出。他的光电效应方程是
9?
hν =+Φ
0
2
1
2
max
mv (1.4)
这里是实验中所用金属的脱出功,比如,对Cs为1.9eV,对Pt为6.3eV。等式右边用了逸出电子的最大速度,是因为有些电子在从金属表面逸出的过程以及在空气传播的过程中,
可能因遭受碰撞而损失了部分动能。这样,不仅光场的能量是断续的、量子化的,而且光场本身也是量子化的,显示出微粒的集合的形象。简单地说,爱因斯坦认为光这种波场是一团
“光子气”。沿着这一思路前进,我们甚至可以引入光子的“有效”质量,即
Φ
0
m
m
c
h
c
==
ε ν
22
(1.5)
于是,若在重力场中,一个光子垂直向上飞行了H距离,其频率要由原来的ν
0
减小为ν,
hh
h
c
gHνν
ν
0
2
=+,从而ν ν<
0
这说明垂直向上飞行的光子频率会产生红移
a
。这一现象在1960年由R.V.Pound和
a
这里,等式右边第二项在地球条件下比第一项小很多,所以作了一级近似计算。
G.A.Rebka Jr.在哈佛大学校园的水塔上实验观测到了。爱因斯坦的光电方程被密里根用10
年时间的实验所证实。
在此稍后一点的时间(1923年),发现了康普顿效应,更进一步证实了光量子的存在。
在这个效应里,散射光的能量角分布完全遵从通常微粒碰撞所遵从的能量动量守恒定律。即当初始电子为静止时
mc h mc h
h
c
h
c
p
0
22
+= +′
=

+
νν
νν
,
.
v
将第二个方程的
h
c
′ν
项移到等号左边,再平方,利用第一个方程和,
即得
pc mc mc
22 24
0
24
=?
h
h
h
mc
′ =
+?
ν
ν
ν
θ11
0
2
(cos)
记λ
c
h
mc
=
0
为电子的康普顿波长
a
,上式转换为
′? =?λ λ λ θ
c
(cos1 ) (1.6)
这为实验所证实。这里推导所使用的光的粒子性以及散射光的频率改变(减小)是经典概念所无法理解的,因为经典观念认为电子在受迫振动下所发射的次波其频率应和入射光的相同。
总之,这一组实验揭示了作为波场的光其实也具有粒子的性质。
2,第二组实验——电子杨氏双缝实验、电子在晶体表面的衍射实验以及中子在晶体上的衍射实验。它们表明,原先我们认为是粒子的这些微观客体,其实也具有波动的性质,
呈现出只有波才具有的干涉、衍射现象,从实验上揭示了微粒的波动性质。
1961年J?nsson用电子束做出了单缝、双缝衍射实验。由于电子的波长短,在这种实验中缝宽和缝距要十分狭小,加之低能电子又容易被缝屏物质散射衰减,这种实验是很难做的。J?nsson在铜膜上刻了五条缝宽为03,μm、缝长50μm、缝距1μm的狭缝,并分别用单、
双、三、四、五条缝做了衍射实验。实验中电子的加速电压为50keV,接受屏距离缝屏35cm。
下面我们对双缝实验作些初步的概念性分析
b

a
电子的康普顿波长。 λ
c
A= 0 0242.
o
b
电子杨氏双缝实验是最富于量子力学味道也是最奇特的实验之一。关于这个实验的各种翻版,直到现在仍不断在文献中出现;关于它的严格计算可见费曼,量子力学与路径积分;张永德,Young双缝实验的唯象量子理论,大学物理,第11卷,第9期,1992。
这时在接受屏上x处探测到电子到达的几率P x()并不简单地等于两个缝单独开启时的几率、之和,而存在两缝相互影响的干涉项; Px
1
() P
2
(x)
Px P x P x() () ()= + +
12
干涉项
此干涉项可正可负。它存在并随x迅速变化,从而使P x()呈现明暗相间的干涉条纹。如果通过缝屏的是光波、声波,出现这干涉项是很自然的,因为在x处的总波幅ψ()x是由孔1、
孔2分别传播过来的波幅ψ
1
(x)、ψ
2
()x之和
ψ ψ ψ() () ()xxx= +
12
而Px x
11
2
() ()= ψ、Px x
22
2
() ()= ψ,并且
Px x x x() () () ()==+ψψψ
2
12
2
= + +Px Px x x
12 12
2() () Re( () ())ψ ψ (1.7)
但现在是电子,这个干涉项的存在,从经典粒子观念来说,是很别扭费解的。每当我们在实验中探测到电子的时候,它总是有一定的能量、有一个静止质量,特别是,有一个相当局域的位置!正是这些给我们以“粒子”的感觉,并且,我们从未探测到半个电子。拿这个概念去理解上面这个电子杨氏双缝实验总觉得怎么都协调不起来。因为,如果电子是以粒子的“身份”穿过狭缝的话,那它不论穿过的是哪个缝,总是只能穿过其中的一个。这时,另一个缝的存在与否对这个电子的这次穿过的行为并不产生影响。就是说,只要入射的是粒子,两个缝的作用就应是独立的、互不干扰的。换句话说,如果把电子理解为经典概念中的“粒子”,
干涉项的存在是无法理解的。那有没有可能这个“经典粒子”的电子是以很复杂的方式同时通过这两个缝?比如,穿过缝1之后又绕回来倒穿过缝2,接着再次穿过缝1出射到接受屏去,甚或多次绕双缝穿行后再出射到接受屏去?但是,这样一来,引起的疑问就更多了。比如,电子为什么会返回来再倒穿过另一个缝呢?到底电子要绕几圈呢?如何计算?这将必然陷入混乱和不可知论。并且,更不可以说电子是以“经典粒子”的身份同时从两个缝穿过去的(比如,一半是从缝1,一半是从缝2穿过去的)。这种图象显然和我们从未测到过一个电子的一部分这一事实相违背。因为,既然一个电子能分开并部分地穿过一个缝,总应该能部分地测到它(比如,将探测器装在一个缝上)。
那么,电子到底是怎样穿过缝屏上的这两条缝的呢?
正确答案已经包含在上面的分析中了。总结上面的分析,若认为电子是经典的“粒子”,
就不能同时穿过两条缝,便不会产生干涉项;若认为电子是经典中的某种“波”,就必定同时穿过两条缝,从而产生干涉项。由此,我们若用简单(但不严格)的语言来说,电子是以经典“波”的行为同时穿过这两条缝的。若严格表述,即是:由于电子同时有些象经典波又有些象经典粒子这样的双重特异性质,它是以“自己独特”的方式“同时”穿过两个缝的。这里说“自己独特”的方式,是因为这种方式既完全不同于经典粒子的通过方式,也“不完全相同”于经典波的通过方式。“不完全相同”是由于,电子可以在其传播途径上的任一点(包括在缝前、缝中、缝后、接受屏上)以一定的几率被探测到,而且一旦被探测到,它总是以一个完整的粒子的形态(有一定质量、一定电荷、一个相当局域的空间位置)出现,即表现出
“波行为到粒子形象”的突变,这是与经典波本质不同之处。其实这正说明:以波的行为穿过双缝的电子同时也具有粒子性的一面。这里要强调指出,事情之所以如此奇异,是由于测量严重地干扰了电子电子原来的状态,使它发生了不可逆转的状态突变。更明确地说,正是对电子位置的测量,导致电子显现出经典粒子的面貌,并不一定是对电子位置测量之前电子就客观地以“粒子”形象存在着
a
!为进一步说明这一点,设想如下实验
b
:在缝屏后放置一个照明光源,若光源足够强,则每个穿过缝屏出来的电子都会和光源的光子发生散射,由散射光可以判断该电子是通过哪个狭缝的。结果发现,每个电子都是只穿过一个缝过来的,从未观察到从两个缝同时穿过来的情况,正如同从未观测到半个电子那样。如前面所分析的,
这并不意味着电子(在作这种观察之前)是以经典粒子的方式穿过这两个缝的。这只是表明,
我们的这种测量造成了穿过狭缝的电子状态突变,从而呈现出了粒子的面貌而已。
总之,在这个电子杨氏双缝实验中,电子穿过双缝时表现出它具有波的性质,而在位置测量中被抓住时,又表现出粒子的特征。这一切只能说明,作为微观客体的电子,它既
具有经典粒子的性质,又具有经典波的性质,但它既不是经典粒子,又不是经典波。如果一
定要使用经典的语言,作经典的类比,则可以简单地说,电子具有波粒二象性(英文为duality
或dualism,关于这个问题后面还将作进一步阐述)。这正是这个实验传出的最重要的信息。
或者,更确切地说,这个实验表明,最重要的量是几率幅ψ()x:到达x点有两条可能的路径,相应两个几率幅ψ
1
()x、ψ
2
()x,在x点找到电子的几率正是这两个几率幅等权叠加的模平方,干涉就正来自这种叠加。
20年代做成了几个出色的电子衍射实验。其中,1925年Davisson-Germer无意中采用镍单晶做了电子的衍射实验,显示了电子的波动性
c
。这个实验可以简明地示意如下。截取单晶的一个面作为表面。该表面形成2维平面点阵,画出其中一维图象如图。对垂直入射的一束电子,在和垂直方向夹角θ的方向能测到的电子,按点阵衍射理论,必为多光束干涉极大的方向。比如,对第一级极大,应有
d sinθ λ=
于是对一定能量的入射电子,具有一定
的波长λ,从而在按上式决定的θ方向,
可以探测到反射电子的峰值。
后来,又用NaCl晶体做了中子衍射实验。到1969年,曾用中性钾原子束做了单缝衍射实验
d
,证明了量子理论的正确性。实验中所用的缝宽为23 10
6
×
米。
a
从后面测量理论叙述知道,对状态ψ()x进行某个力学量的测量,实质是将ψ()x按该力学量的本征态进行展开,测得的力学量是本征值中的一个,出现的几率是该展式相应项的系数的模方。而该次测量完毕时,ψ()x即突变为该本征态。
b
R.P.Feynman,A.R.Hibbs,Quantum Mechanics and Path Integrals,McGraw-Hill Book Company,1965。
c
Anton Z.Capri,Nonrelativistic Quantum Mechanics,P.18。
d
Am.J.Phys.,37,905(1969)。
除上面两组关于波粒二象性的基础实验之外,1911年Rutherford根据α粒子被金属箔散射的实验提出了原子的有核模型,特别是1913年Bohr建立了原子的初等量子理论,它们对量子力学的诞生起了直接的推动作用。Bohr理论要点有三,一是定态概念,二是定态间的跃迁概念,三是角动量量子化概念,后者导致量子化条件。这里,定态概念主张原子的有核模型只对某些分立的能量Em
m
(,,.)= 12才是稳定的,这是为了解决电子绕原子核转动时不辐射而稳定存在的问题。因为经典的电动力学主张带电粒子只要有加速度就会产生辐射而损失自己的能量,从而这种(负电荷电子绕原子核的)稳定转动是不可能的。定态之间跃迁概念是为了解决原子光谱及其里兹组合原则
ω
mn m n
EE=?()h/
就是说,原子发光是由于原子从一个能量较高的定态向一个能量较低的定态的跃迁。原子核外电子所具有的角动量量子化条件
Jm= h
进一步地揭示了核外电子呈分立的定态存在的原因,丰富了量子化的内容。至于Bohr利用经典的轨道概念进一步将这个量子化条件表达为动量对坐标的回路积分形式则是一种不成功的尝试,已被后来诞生的量子力学所否定。除这部分内容外,Bohr理论的定态及定态间跃迁概念均被后来的量子力学汲纳和发展。
§1.2 基本观念
1,de Broglie关系式与波粒二象性
1905年爱因斯坦通过提出下列关系
Eh= =ν ωh,
vv
h
v
p
h
e==
λ
k (1.8)
引入了光子的概念,在原先认为纯粹是电磁波的图象上添加了粒子的图象,这由上节中第一
组实验所证实。于是,已知等式右边的波动参数ω、k,便可由这组关系求得它所具有的微粒子的特性。经过18年之久,de Broglie克服积习的约束,反过来读这组关系,将上面这组关系从针对的情况推广到m = 0 m ≠ 0的情况,提出原先是微粒的微观粒子也具有波动性,
ω =
E
h

v
v
h
k
p
=,(1.9)
就是说,若已知等式右边的粒子参数E、p,便可由这组关系式求得它所具有的波动特性。
这中间的桥梁便是普朗克常数,形象地写出便是 h
(,) (,)Ep k
v
v
h
←→? ω
这组公式中,关于波长的第二个公式已为上节中第二组实验所证实,而关于频率的第一个公式则被原子光谱实验所证实。注意,这组de Broglie关系是物质世界的普遍规律,只是由于普朗克常数很小,h的数值便成为这种联系能够明显表现出来的尺度,也就是波粒二象性能够明显显示出来的尺度。假如所研究的问题中,相对而言,可以认为,波和粒子便截然分开,波粒二象性的现象便可以忽略。这时,由原先粒子
h
h → 0
)(,E p
v
,利用这组关系便得到λ →
h
0

,从而与此粒子相联系的波性便可忽略。就这个意义上可以说,经典力学是量子力学当是的极限情况。当然,这里的h是相对而言的,并非真要(本就是常数0 → 0
的)变小,而是要求所研究的对象h (,)E p够大(从而波长λ够短),运动所在的空间尺度够大,以致于
l
h
h
λ
e
λ
n
λ
γ
=
λ << l
即可。简言之,可以按普朗克常数在所研究的问题里是否能忽略,来决定波粒二象性是否存在,从而决定经典与量子的界线。于是,经典力学只不过是其研究对象的能量、动量以及运动的区间尺度如此之大,以致于能使的作用可忽略情况下的力学。
综上所述,不论静止质量为零和不为零的微观物质,都普遍地有波和粒子两重属性,这两种截然不同的属性通过普朗克常数连结成为de Broglie关系,统一在所有微观物质上。对初学者而言,波粒二象性是理解微观物质普遍属性的基本图象,也是初学者理解量子力学的基本图象。
2,波粒二象性的初步分析
然而,这种波粒二象性的基本图象,使初学者常常感到迷惑和不习惯。产生迷惑和不习惯的原因是他们所学到的、所经历到的物理现象中均可忽略,不存在这种二象性:波就是纯粹的波,粒子就是道地的粒子。于是,常常会问:电子一会儿象波,一会儿又象粒子,那它到底是什么?为了回答这种问题,可以打个比方:某个人,在今天早晨遇到某事时笑了,
表现出他的一副笑面孔。但在今天中午碰到某事时却哭了,表现出了哭面孔。由于某人表现出了这么两付截然不同的面孔,那我们能不能据此发问:他到底是怎样的面孔?!显然,不应当这样发问,因为它们都是这个人的面孔,而只应当问:他在什么情况下表现为笑面孔,
在什么情况下表现为哭面孔?将这种论述平移到电子波粒二象性问题上来。我们可以回答说:波性和粒子性都是电子的固有属性,当它表现出两种属性的时候,人们不应当问它“到底”是什么属性,只应当问它在什么条件下表现出类似于经典波的性质,在什么条件下却又表现出类似经典粒子的性质。电子既不能用纯经典波(波包)来理解,也不能用纯经典粒子来理解。只能说它有时象经典波,有时又象经典粒子。“象什么”本身的前提就是“不等同”。
归根到底,电子就是电子!这个意义上说,电子的波粒二象性图象,只是我们使用经典的语言,用经典类比的方法去描述电子这个微观客体时,所必然得到的一种并非十分贴切的图象。
鉴于人们总是习惯地以已有知识和经验去理解和描述新的东西,因此保留波粒二象性的图象还是有助于初学者的理解和形象思维,只是不要过分地执着和拘泥而已。这里,正如杨氏双缝实验所启示的,根本性的东西是几率幅,是有关几率幅的计算的理论,而不是借助经典语言所得出的波粒二象性图象。但为了初学者的方便,本书仍沿用波粒二象性的说法。
h
3,de Broglie波初步分析
对常用的非相对论电子、非相对论中子、光子,它们的de Broglie波长和它们的能量的关系式为
E
=
12 26.
E
=
0 286.
×1241 10
4
.
E
这里E (对的粒子,m ≠ 0 E为其动能)的单位为eV,λ的单位为。 A
o
对宏观物体,如上所述,其波动性可以忽略。例如,1克小球,速度v =1米/秒,de Broglie
波长为
λ ==×
h
mv
66 10
31
.米
显然,这个尺度和小球本身尺度以及和小球作宏观机械运动的空间尺度相比,完全可以忽略。
从而,在研究小球作任何宏观机械运动时,与这个波长相联系的波动性质(也就是与小球运动相关的量子效应)完全可以忽略。
这里再说一下de Broglie波的群速度和相速度的问题。对m = 0和两种情况,虽然de Broglie关系相同,但它们的相速度还是有差别的,
m ≠ 0
m ≠ 0:相速度V
k
E
p
c
V

====νλ
ω
2
m = 0:相速度==
ω
k
c
就是说,对光子来说,相速度也是c;但对m ≠ 0的粒子来说,相速度不但不等于粒子的运动速度V,而且大于光速。但可证明,粒子的de Broglie波波包的群速度等于粒子的运动速度
c
V
d
dk
d
dk
dE
dp
d
dp
pc mc V

== == + =
ω ω()
()
h
h
22
0
24
据此,以前曾有人主张微观粒子实际上是de Broglie波的某种波包。但进一步考察表明,这会导致不能接受的结果(比如,电子的de Broglie波包会扩散而电子却是稳定的粒子)而被否定,
4,波粒二象性的再分析——某些基本推论
由微观粒子具有波粒二象性这一基本图象,派生出三个重要观念:描述的几率观念、能量取值分立化的量子化现象、测不准关系式。
首先,由微观粒子的波粒二象性,可以导致量子力学的一个重要观念:在描述粒子运动中的几率幅和几率观念。
按de Broglie关系,和一束匀速直线运动的粒子流相联系的应是一个平面波。它们的经典形式是(为书写简明,这里只写一维情况)
e
ikx t()?ω
代入de Broglie关系,便得到和这束粒子流相联系的de Broglie平面波
ψ(,)
()
xt e
i
px Et
=
h
(1.0)
这时,如果定义ψ(,)xt
2
为在x处单位体积内找到这束匀速直线运动粒子的数目,则这种数目分布是空间均匀的。更一般地,我们来研究如下的de Broglie波波包
ψψ(,) ()
()
xt pe dp
i
px Et
=

h
这里p和E满足如下关系
Epcmc
222
0
24
=+
取,于是de Broglie波包成为 t = 0
ψψ() ()xpe
i
px
=

h
dp
这里ψ()x是粒子在x处的de Broglie波波幅,即几率幅。我们将ψ()x
2
理解为在x处附近单位体积内找到粒子的几率,或说成,粒子取坐标x的几率。而ψ()p
2
则理解成是粒子取动量p的几率。
显然,用这样的方式去理解所引入的de Broglie波,是能够统一描述微观粒子波粒二象性的唯一方法:ψ()x本身是波幅,可以叠加并产生干涉,体现微观粒子的波动性;一旦(以
ψ()x
2
几率)在x处被观察到,却又是个完整的粒子形象。但是,我们把这两种(从经典物理学看来)完全不同的禀性用如此方式统一起来描述的时候,已经付出了沉重的代价:放弃了经典物理学中惯用的拉普拉斯决定论,描述时引入了不确定性,引入了几率观念。显然,为了做到统一的、兼顾两种属性的描述,这种代价是必须付出的。
描述方式上的这种不确定性和微观实验中表现出的不确定性是相对应、相配合的。再拿电子的杨氏双缝实验为例。假定电子源的强度十分弱,以致可以认为这个实验是用单个电子一次一次地串接着进行的。如果认定某个电子,当它穿过缝屏之后,它到底在接受屏上哪一位置被观测到,无法用实验事先测定,也无法(至少在目前)在理论上以拉普拉斯决定论方式去实现准确预计。当这个电子穿过缝屏时,它的动量和穿缝之前相比究竟有多少改变,实验上也无法事先确定,并且理论上也无法以拉普拉斯决定论的方式事先计算。单个电子在穿过狭缝时的状态突变、电子在接受屏上被测到时的状态突变都是一种深邃的、事先无法预计的、
不可逆转的变化。只有大量的同种类型的突变所表现出的统计规律才是可以事先了解和准确预计的。实验中表现出来的这种不定性,也迫使我们采用几率幅、几率的观念。或者说,实验中表现出来的这种不定性,正是电子既是波又是粒子、既不是波又不是粒子的结果。于是,
以电子杨氏双缝实验为例,在接受屏上x处观测到电子(表现出粒子的面貌)的几率P x()是该处de Broglie波波场振幅的模方,而该处的振幅又是由(作为波源的)两条缝传播过来的波幅的叠加,所以
Px x x P x P x x x() () () () () Re[ () ()]=+ =++ψψ ψψ
12
2
12 12
2
可以看到,上面这种带有“不定性”的描述方法,不仅能以统一的方式描述电子的波粒两种属性,而且和带有“不定性”的双缝衍射实验紧密配合。
对于这些不定性,即,实验中突变的不定性和波函数几率描述中的不定性,存在两种观点。其一,实验中的突变的不定性,并非是我们实验方法、实验仪器的不完善造成的,而是微观客体固有的,它不能靠改进实验方法提高实验精度来消除。正由于存在这种客观的、固有的不定性,我们现有的包含与之相匹配的不定性的描述是完备的,并非理论描述方式上的先天不足。就是说,量子力学描述中的几率观念并不说明描述方式的不完备,而是客观现象本就如此。所谓的未知的“隐变数”是不存在的。量子力学现有的描述方式是完备的。其二,
这些不定性的存在说明我们对微观世界事物了解上的不完备。实验测量中的不定性固然说明了实验方法上的局限性,描述方法中的不定性更说明描述方法的不完备、说明隐变数的存在尚未被量子力学包括入描述方法中去。长期以来这两种观念争论不休。值得指出的是,到目前为止,实验事实仍然不支持隐变数的存在,而支持量子力学的描述是完备的。但有鉴于目前量子理论存在巨大的困难,因此狄拉克说:“它是到现在为止人们能够给出的最好的理论,
然而不应当认为它能永远地存在下去。我认为很可能在将来的某个时间,我们会得到一个改进了的量子力学,使其回到决定论,从而证明爱因斯坦的观点是正确的。但是这种重新返回到决定论,只有以放弃某些基本思想为代价才能办到,而这些基本思想我们现在认为是没有问题的。如果我们要重新引入决定论的观点,我们就应当以某种方式付出代价,这种方式是什么,现在还无法推测。”
a
其次,看看和微观粒子相联系的波动性质是怎样导致该粒子能量和状态的间断分立或量子化现象的。
这里,即使在经典物理的领域里,也存在一个重要的、普遍的、众所周知的事实。那就是,对于任意波场,当它们展布或传播在无限空间中时,波参数可以取连续变化的数值;但是,一旦用任何方式将这些波局限在有限空间的时候,波场所取的波参数必将分立化,它们的频率和波长均要断续化、分立化。从傅立叶频谱分析的观点来说,任意局域的波均是一个傅立叶级数,而不是一个傅立叶积分。或者说,任何波动方程其局域解的问题总都是一个本征值和本征函数的问题
b

转到微观粒子的情况。局域de Broglie波的波动性同样将造成频率和波长的断续性。而且进一步,频率和波长的这种断续性又通过de Broglie关系转化为该粒子的能量和动量的断续性。因此,可以说,任何局域化的de Broglie波必将伴随其能量的量子化。这正是粒子的
de Broglie波波动性的结果,是局域de Broglie波自相干涉(由边条件反射)的结果。这正是和经典物理中,从一维琴弦振动、二维鼓膜振动到三维微波腔中电磁波驻波等现象相对应。
最后,考察和微观粒子相联系的波动性质是怎样导致所谓测不准原理的。
按照前面所说ψ()x和ψ()p的物理解释,可以定义一个微观粒子坐标x和动量p (相对任一选定值、)的测量均方根偏差 x
0
p
0
()
()()
()
x
xx xdx
xdx
2
0
2
2
2
=

+∞

+∞


ψ
ψ
()
()()
()
p
pp pdp
pdp
2
0
2
2
2
=

+∞

+∞


ψ
ψ
这里,由于ψ()x和ψ()p是傅立叶变换对,因此根据傅立叶变换的带宽定理
c
,立即可得
a
P.A.M,Dirac:物理学的方向,科学出版社,1981年。
b
对物理中常见的一些波动方程来说,本征值是分立的或是部分分立的。
c
傅立叶带宽定理,
若fx Fye dy
ixy
() ()=

+∞

1

,,Fy f xe dx
ixy
() ()=

+∞

并定义 ()
()()
()
x
xx fxdx
fx dx
2
0
2
2
2
=

+∞

+∞


,()
()()
()
y
yy Fydy
Fy dy
2
0
2
2
2
=

+∞

+∞


,
这里,为任意固定值,则有 x
0
y
0
xp?≥
h
2
(1.1)
这说明,不论de Broglie波波包形状如何,与之相应粒子的动量均方根偏差与坐标均方根偏差的乘积不小于
h
2
。或者简单地说,微观粒子的坐标和动量是不能同时测准的。这里要强调指出,这种不能同时测准是原则性的,并非由于实验方案欠周到、实验技术欠精密所带来的实验误差。这个测不准关系的存在正是根源于微观粒子的波动性,或者准确地说,正是由于微观粒子的波粒二象性。显然,随着研究的问题向宏观领域趋近,h的作用逐渐减小,就从x、p不能同时测准约略成为能同时测“准”了。其实,由上面所用的带宽定理可知,任何波(弹性波、光波、……)均存在类似的关系式。这是对波动过程进行傅立叶分析的基本结论之一。关于测不准关系问题后面还将专门讨论。
§1.3 测不准关系的讨论
1,应当强调指出,由于测不准关系的物理根源是微观粒子的波动性,因此它是个普遍成立的关系式。这就是说,在任何普朗克常数h的作用不能忽略的现象里,在任何明显显示波粒二象性的事例中,总之一句话,在任何量子物理实验中,都能分析出这一测不准关系。
前面的关系式还可以改变一下形式。设电子沿x方向运动,由于粒子在x方向的位置有一个不确定,从而用照明光与其散射以确定其位置时,发生散射的时间也就有一个不确定
t
x
v
x
=
这里是散射前粒子的速度。显然,这也是用显微镜观察粒子的观测时间的不确定量。另一方面,粒子的能量
v
x
E
m
p
x
=
1
2
2
,所以和?p
x
相应的粒子能量的不确定量为
Evp
xx
=
两者相乘,可得
E th? ≈ (1.2)
这里可以将观测时间的不确定量看成是观测的持续时间。于是,测量粒子能量E的不确定量和对它进行观测的持续时间之间,也存在类似的测不准关系。换句话说,测量过程的分析表明,为了精确地测量能量(精确度达到?E ),要求测量所花费的时间至少为
t
h
E

另外,根据一定时间间隔波包的傅立叶频谱分析,启发我们可以对上面这一关系作另一种解释:对持续在短时间间隔内的任何不稳定现象,其能量必有一不确定性(或其所含频率必有一宽度),使两者之间满足上面的关系
t
a

2,测不准关系的进一步解释及某些应用
对上面这两个测不准关系作一些进一步的解释,并讨论它们的某些应用。
xy?≥
1
2

a
这两种解释参见:费米著《量子力学》,西安交通大学出版社,1984年。
首先,应当强调指出,上面这两个关系不仅对大量相同微观粒子相同实验的所谓量子系综是统计正确的,而且也有不少主张认为它对单个微观粒子的单次实验也是正确的。
比如拿E th? ≈来说。设想一个用光辐照原子使原子激发的实验。这里光的照射只持续一个短时间τ,尽管在这个短时间τ内它的振荡频率是ω。于是这个光束可记为
θθτ
ω
() ( )tt
it
e
这里。无论从经典还是量子的观点,若作频谱分析,这束光都不是频率为θ()
,
,
t
t
t
=
<
>
00
10
ω
的单色光:经典观点,将它作傅立叶频谱展开,由于τ不是无穷大,频谱将有一个宽度,是个非单色光;量子观点看,这是一束非单色光子的集合,它们的能量以hω为中心有一个展宽。这样一束非单色光子集合入射到大量原子上,并不如通常那样将原子一个个激发到高于基态hω (假定这个激发态以及其附近的激发态均存在)的激发态上,从而退激时发出锐细的频率ν
ω
π
=
2
的光谱,而是被激发到以上面这个激发态为中心的各种激发态,退激时发出一定宽度的谱线。实验结果正是这样。并且实验还指出,只当上面这个照射时间持续很长情况下,退激发时原子所发出的谱线才是hω。这里,对此公式作单个解释和统计解释都不困难。
然而下面的例子就只能用单个解释,并且意味着:在时间量级?
t
E
~
h
上,经典能量守恒定律的?E量级的破坏是可能的。例子是,近代关于核力的概念可以形象表述如下。
n和p仿佛两个相向滑冰的运动员。当划到处时,向n x
1
p抛去一个球(虚介子),同时在离开p的方向上受一反冲。p在处接到抛来的球,也产生了另一个方向的要离开的反冲。假定我们只能看见这两个运动员而看不见球,那我们一定觉得在与
x
2
n
A B之间存在着某种斥力。n和p之间抛接小球的最大距离便是这种斥力的力程。n和p之间的距离若大于这个力程,n和p之间的这种斥力便可以忽略。如果之间是吸引力,可以设想在它们之间抛接飞去来器(虚介子)。就是说,n在处向背着(不再是朝向)x
1
p的方向抛出飞去来器,它飞向p并绕圈后被p接受。于是,当抛接的是光子,两粒子间的作用力便是电磁力。抛出和接受光子的能力便是电荷。质子和中子之间抛接的是π介子,表现出的便是介子场论中的核力的图象。现在的问题是,在一个核内,某对核子之间所抛接的π介子,原先并不存在,是从“无”中生出来的。这就以为着在数量上破坏了经典意义下的能量守恒定律。
但是,按测不准关系,只要这个
Em
μ
c=
2
π介子存在的时间(从抛出到接受,即从产生到吸收)?
t
E mc
≈=
hh
μ
2
就是可以的
a
。可以如下估计核力的力程(或由核力的力程估计π子的质量)。设π介子以近于光速从一个核子向另一个核子飞行,则 c
≈≈
10
o
~
22
D
~
o
V原子尺度
22
23
h
(.
eV
.~
原子核尺度
Ep
10
15
cm
rct
mc
核力
h
μ

实际上,这个力程就是π介子的康普顿波长。由此可知,π介子质量越大,由之产生的力程就越短。代入,可得。当然,这种图象是简化了的,实际上,在核子之间抛接的不仅是
rc
核力
≈×
13
mmc V
μ
2
132≈15,Me
π介子,而且也不止就抛接一个π介子。毫无疑问的是,π介子是其中最轻的,因而承当了核力中的较长程的部分。
另一个需要讨论的问题是,由于粒子的位置和动量不能同时测准,因此在量子理论中不存在经典物理中常用的粒子轨道的概念。这是因为,经典的轨道概念是以粒子的位置和动量能同时定准为前提的。
最后,讨论一下这两个测不准关系的某些应用。第一,能量尺度与空间尺度的关联。原子尺度为Ac10
8?
,m
p
m
c
mc
MeV s cm s
MeV cm
e e
222 22 2 10 2
82
2 2
658 10 3 10
2 0511 10
≈=
×?××
××
h (,) ( / )
.()
= 38,eV A
∴ Ae
o
~10
原子核尺度为10,
13?
cm
p
m
c
mc Fermi
MeV cm
MeV cm
n n
2
22
11 2
13 2
2 3
1973 10
2 940 33 10
≈=
×?
×?×
)
(,)
(,)
= 20 33,~,M Fermi
33 20.Fermi MeV
弱电统一时的高能物理尺度10:这时粒子已很接近光速,所以
16
cm?t
cm
c

10
16
,有
c
c
x
MeV cm
cm
≈?= =
×?
×
h
2
1973 10
210
11
16
.

2
GeV
高能物理的尺度弱电统一的尺度
10 10
10 10
16 2
1
GeV
cm GeV
~
(~)
第二,前面我们已经严格证明了对任一de Broglie波包,有
xp
x

h
2
早在1926年,薛定格就已构造了所谓“最小不准确度波包”,现在通常称之为波色子的相干态。这个波包其实是一个高斯波包(由于在数学上,高斯型波包的傅立叶变换仍为高斯型,
所以它的动量展开也是一个高斯波包),随时间的演化也仍保持为是一个高斯波包。这个“最
a
R.Shankar,Principles of Quantum Mechanics,1980,P.253。
小不准确度波包”不仅可以用于自由运动粒子,也可运用于束缚粒子,得到一些新的相干态
a
。关于相干态的问题将在第三章中详细讨论。
§1.4 量子力学的基本假设
上面讲述了导致量子力学诞生并构成它的实验基础的一些实验事实,以及由这些实验事实所抽引出的一些基本观念。这些基本观念构成了量子力学的基础,体现了量子力学最本质的特征。遵循这些基本观念,利用公设加逻辑的公认科学体系,便能构筑起整个量子力学框架。全部量子力学的理论基础可以归纳为5个公设,下面简要阐述一下量子力学的这
些基本假设。
若进行逻辑的归纳,非相对论量子力学是建立在五条基本假设或称为公设之上。当然,
如同任何科学理论那样,作为公设和整个理论出发点的这些基本假设分别都是许多实验经验
(以及这些实验经验所揭示的基本观念)的概括。这些假设是在量子力学建立的过程中和建立之后才归纳抽象出来的。
1,[第一公设称为波函数公设]
,一个微观粒子体系的状态,用一个波函数ψ(,)
v
x t来完全描述,它是粒子的坐标和时间的函数,而且在ψ(,)
v
x t的分布区域中找到粒子的几率由 dV
dP dV=?
ψψ
v
表示,这里为ψ
ψ的复数共轭。从而,ψ(,)x t在其分布区域中必须处处单值、连续、可微(除个别点、线、面之外),对此区域的任意部分都是平方可积的。”
这个波函数公设可细分为三点内容:状态由波函数表示、波函数的几率解释以及随
之而来的对ψ()x函数的数学要求。ψ
2
称为几率密度。
比如,经典物理中匀速直线运动的自由粒子,对应有量子理论中的动量为确定值的微观粒子,完全描述其状态的波函数是平面de Broglie波
ψ (,)
()
v
h
vv
rt Ae
i
pr Et
=

这里为某个常数。确切些说,这个波函数描述了动量值为A
v
p的无尽的粒子流,在这个束流中每单位体积内平均有ψψ
= A
2
个粒子存在。因此,最好不用它去代表一个(动量为定值的)粒子的波函数。因为如果这样,那在全空间肯定总能找到这个粒子,也就是存在如下归一化条件
ψ()
v
rt dV
2
1
全空间

=
由于积分发散,指数前面的归一化系数趋于零。物理上这当然是合理的,因为这时在任意地方的单位体积里找到这一个粒子的几率几乎是零。这导致无法写出波函数的后果。所以,代
表一个粒子,最好不用平面波
b
,而用某种波包,一个展布在有限区域,从而模平方积分收
a
J.R.Klauder,Bo-Sture Skagerstam,“A Coherent State Primer”,P.58。
b
当然,如果引入δ -函数,将平面波归一化成为δ函数,在数学运算上也是通的。并且实际大多数情况也是这样做的。所以,这里的说法并不意味着放弃使用平面波。事实上,由于它的理想化和简单,将给数学描述(例如在散射理论中)带来简化。而它所带来的问题可以用一些办法予以补救。
敛,可归一化的波函数。这里强调指出,从实验测量的观点,只要求ψ()
v
rt dV
2
处处单值、
连续、有限,或写为
ψ()
[]
v
rt dV
M
2

=单值、有限
这里[]M表示被测点M附近任意小但有限的小体积。这是因为,任何测量粒子位置的实验,
无论其精确度多高,也不能精确到一个几何点,所以测量精度使得测定的区域虽然很小但总为有限。这样,实验测量几率值必须单值有限这一要求就体现为上述积分等式。这里并没有要求ψ()
v
rt
2
本身处处有限。“ψ(,)
v
r t处处有限”的要求其实是人为主观的要求。比如,
就球坐标原点附近这一情况而言,实验测量只要求到
ψ
2
2
0
rdr
r=

=
附近任意有限区间单值、有限
v
就是说,按实验测量的观点,波函数ψ(,)r t在r = 0点可以发散,只要它的发散满足下面边界条件
ψ()
v
rt
r→
→? ∞
0
应慢于r
32/
这就是从测量观点出发所得的物理要求。这个问题在第四章中还将谈到。
2,[第二公设称为算符公设]
,所有力学量(可观察的物理量)均分别以线性厄米算符表示。这些算符作用于态的波函数。在这种由力学量到算符的众多对应规则中,基本的规则是坐标A
$
A x和动量p向它们算符$x、的对应。这个对应要求 $p
$$ $$xp px i? = h。” (1.3)
这里需要解释几点。第一,线性算符是指
$
()
$$
Ac c cA c A
11 2 2 1 1 2 2
ψψ ψ+= +ψ
A
$
A
(1.4)
这里、一般为复常数。第二,一个算符的厄米共轭算符由下面内积定义 c
1
c
2
$
A
$
A
+
(
$
,) (,
$
)A
+
=?ψ? ψ (1.5)
当时称为自共轭算符,或称为厄米算符。上式又可写为
$
A
+
=
$
A
(,
$
)(,
$
)ψψAA
+?
=
因为内积(,)? ψ用波函数积分表示即为(,) () ()?ψ? ψ=

v v v
rrdr,于是上式又可写为波函数积分形式
[ ]
ψψ?ψ
+?
∫∫∫
==()
$
() ()
$
() ()(
$
())
vvv vvv vv
rA rdr rA rdr r A r dr
v
这里表示取复数共轭。如算符是厄米的,则应有?
$
A
[ ]
ψψ

∫∫
=()
$
() ()
$
()
vvv vvv
rA rdr rA rdr
当然,上面表述中的?和ψ均属于某一类函数(参见后面叙述)。第三,可以证明,厄米算符的本征值均为实数,且对应于不同本征值的本征函数是正交的。因为,设ψ
1
()
v
r和ψ
2
()
v
r分别是厄米算符的对应本征值为和的本征函数,即
$
A a
1
a
2
$
() ()Ara rψψ
111
v v
=,
$
() ()Ara rψψ
222
v v
=
v v
对这两个方程分别左乘以和ψ
2
()r ψ
1
()r并积分,得
ψψ ψψ
21 121

∫∫
=()
$
() () ()
v v v v v v
rA rdr a r rdr
ψψ ψψ
12 212

∫∫
=()
$
() () ()
v v v v v v
rA rdr a r rdr
另一方面,由的厄米性可得
$
A
[ ]
ψψ ψψ
21 12

∫∫
=()
$
() ()
$
()
vvv vv
rA rdr rA rdr
v
将上面两个等式代入此式,得
( ) () ()aa r rdr
12 2 1
0?=


ψψ
v v v
如果取ψ
2
()
v
r为ψ
1
()
v
r,则由于ψ
1
2
0()
v v
rdr

≠,可得,即是实数;如果
,这导致
aa
1
=
1
a
1
a
1
≠ a
2
ψψ
21
0

=() ()
v v v
rrdr
v v
就是说,分属于不同本征值的ψ
1
()r和ψ
2
()r是互为正交的。第四,一般而言,一个厄米算子的本征函数族并不一定是完备的
a
。我们把那些本征函数族是完备的(这里,完备性定义是可以对任一波函数——指定类型中的任意函数作展开)厄米算符所对应的力学量称为可观测的力学量
b
。这样说的详细解释参见公设三。第五,本公设中从力学量到厄米算符的对应,
以及$x和的对易子可如下理解。按$p ψ()
v
r的几率解释,坐标算符的平均值应为
v
v v v
vv
v v v
vv
r
rrdr
rdr
rrdr
rdr
≡=




$
()
()
()
()
ψ
ψ
ψ
ψ
2
2
2
2
v v
可以看到,由于状态波函数ψ用坐标r来描述,坐标算符
$
r可直接就取为
v
r
c
。这时,的表示式又如何呢?以对应确定动量值
v
$
p
p的一维de Broglie平面波波函数为例,它是动量算子的本征函数,其本征值为p。对之写出本征方程,为
$
() ()
pe pe
i
px Et
i
px Et
hh

=
可以看出,
v
p算子的表达式可取成
v h
$
p
i
d
dx
= (1.6)
=
a
可以证明,如果一个线性算符能满足某一多项式为零的方程
$
,
$$
......
nn
n
+++
αα
1
1
0
这里α
1
,……,α
n
为常系数,则的本征函数族是完备的。参见狄拉克,量子力学原理,第二章。
$
b
这里预先指出,定态的一维薛定格方程(包括它在任意中心场下的径向分离方程)可归入Sturm-Liouville型方程。它的本征函数的完备性能严格证明。见钱敏、郭敦仁译,柯朗、希伯特著“数学物理方法(I)”,科学出版社,1958年,第328、262、225页。或见吴大猷著“量子力学(甲部)”,科学出版社,1984年,第
86页。从而可以说,就一维这一特殊情况来说,可以严格证明,系统的能量是可观测量。但是,对一般系统,能量是否为可观测量,也即其本征态是否为完备集,这种证明已超出现有的数学能力,只能是个假设。
这参见陈咸亨译,狄拉克著“量子力学原理”,科学出版社,1965年,第36页。也参见§2.2相应注记。
c
由于ψ()
v
r并非是
v
$
r的本征函数,从而无法对之写出本征方程。但若把δ()
v v
rr?
0
作为某种实际的、十分局域于点的粒子波函数的理想化表示,则可对它写出坐标算符
v
r
0
v
$
r的本征方程如下,
v v vvvv
$
()()rrr r rrδδ?=?
00 0

在$x、如此取法下,它们之间的对易子为 $p
[ ]
$,$ $ $$$xp x p px=
=x
i
d
dx i
d
dx
x
hh
= ih
当然,对三维空间的另两维、自由度也有类似计算。从而,就三维空间来说,y z
{ }
v
$
$,$,$rxyz=,
v
h
$
pi=。
由此,非相对论的动能算符、势能算符等均可以用这两个算符的函数去构造。列表如下
非相对论动能算符
$
$
T
p
mm
==?
v
h
22
22
v
势能算符 VV
$$
(
$
)r=
v
角动量算符
v v
h
v
$
$$
Lrp ir=×=? ×?
v v
粒子密度算符
$
(
$$
)ρδ=?′rr
流密度算符
v
vv
v v
vv
$
[(
$
)
$$
(
$
)]jrr
p
m
p
m
rr=?′ +?′
1
2
δδ
而系统的总能量算符(即系统的哈密顿量)
$$$
H T V= +。在球坐标中,角动量算符的各个分量分别为
$
$
$
$
$
()(sincotcosLyPzP iy
z
z
y
i
xzy
=?= =? +hh )
θ
θ?

$
(cos cot sin )Li
y
=h?
θ
θ?

$
Li
z
=?h

动能算符为
$
$
T
mr r
r
L
mr
=? +
h
22
2
2
2
2
1
2
这里的进一步注意事项参见第二章第三节。第六,以为例阐述一下动量算符的厄米性问题。由上面第二点叙述知,此时
$
P
x
$
Pi
d
dx
x
=?h的厄米条件相应为以下等式成立
ψ
ψ

=?
∫∫
() ( )i
d
dx
dx i
d
dx
dxhh
这里?()x、ψ()x是两个任意的波函数。由于上式右边为
i
d
dx
dx i i
d
dx
dxhh h
ψ
ψ? ψ

∫∫
=+?
上限下限
()
这就是说,仅当分部积分项为零时,P的厄米性才能被保证。就束缚态而言,当
$
x
x →±∞时,
ψ()x和?()x均趋于零,的厄米性不会出现问题。至于对有限区间
$
P
x
[ ]
ab,情况,的厄米性只在满足周期(或反周期)边界条件的函数族中才被保证,这时分部积分项仍为零。
$
P
x
3,[第三公设称为测量公设或平均值公设]
,一个微观粒子体系处于波函数为ψ()x的状态,若对它测量可观测力学量的数值,
$
A
所测得的的平均值(期望值)为
$
A
r()
v
α
n
A
ψ
A
rA rdr
rrdr
ψ
ψψ
ψψ
=


()
$
()
() ()
v v v
vvv
(1.7)
若ψ()
v
r是归一的,则
ArAr
ψ
ψψ=

()
$
()
v
d
v v
。”
除波函数几率解释之外,这又是一个将量子力学的理论与实验联系起来的公设,它将力学量的量子力学理论计算与实验观测联系起来,和波函数几率解释一道构成量子力学中实验观测的理论基础。这里指出几点。第一,这里的平均值是指对大量相同的态ψ()
v
r作多次观测的平均结果。这里有所谓多次平均测量结果和单次测量结果。第二,如果ψ()
v
r不是算符的本征函数,只要是可观察力学量,也即的本征函数构成完备集,则
$
A
$
A
$
A ψ(
v
)r一定可用的本征函数族
$
A
{ }
r()
v
ψ
n
展开,
ψαψ() ()
v v
rr
nn
n
=

这里ψ
n
是的本征值为的本征函数,
$
A a
n
$
() ()Ara r
nnn
ψψ
v v
=
展开系数一般为复数。将展式代入A
ψ
的表达式,得
A
rA rdr
rrdr
a
nn
n
nn
n
nn
n
nn
n
nn
n
n
n
ψ
αψ αψ
αψ αψ
α
α
==


∑∑

∑∑



(()
$
(()
(()(()
v v v
vvv
2
2
(1.18)
即平均值为实数本征值的加权平均,加权系数等于a
n
ψ()
v
r用
{ }
ψ
n
r()
v
展开时的系数的模方。注意,在单次测量中,测得的数值必定总是的本征值之一,不可能是本征值以外的数值,这是和经典力学测量截然不同之处;得到该力学量某个本征值的几率是被测态波函数对该力学量本征态展式的相应系数的模方。注意,作为决定几率权重的这些系数随被测态的演化可能会随时间变化。第三,即使在量子力学实验中,测量的数值总应当是实数(力学量的取值总应当是实数),所以要求对任一波函数
A
$
A
ψ()
v
r,A
ψ
均为实数。事实上这是被保证了的。因为是厄米算符,于是有
$
A
{ }
ψψ ψψ

∫∫
=()
$
() ()
$
()
vvv vvv
rA rdr rA rdr
由于单次测量结果总是的本征值之一,显然也应当总是实数。第四,每次测量之后,态
$
A
ψ()
v
r即受严重干扰,并总是向该次测量中所得本征值的本征态突变过去。就某一单次测量而言(除非ψ()
v
r已是该被测力学量的某一本征态),究竟向哪个本征态突变,就象测得的本征值一样,是完全不能预先预言的。就是说,由测量引起的突变总是向被测力学量的本征态之一突变,而且这种突变是随机的、无法预计的、不可逆的、超出量子力学描述范围的。总括说来,对被测态ψ(
v
)r进行力学量的每一次测量可分为三个阶段:A ψ()
v
r按的本征态分解,称为谱分解;
$
A
ψ(
v
)r (以谱分解展开式系数模平方为几率)随机不可逆地向的本征态之一突变,称为波函数坍缩;坍缩后的态作为初态重新开始测量后的新一轮演化。这就是量子测量的全过程。现在可以解释公设二中所提的可观察力学量的说法了。如果力学量是
$
A
$
A
个可由实验观测的量,那么对任给的一个状态ψ()
v
r,都应当可以对其实行的数值的测量。
由于测量就意味着本征函数展开和向其中之一的坍缩,这就意味着
$
A
ψ()
v
r必能用的本征函数族展开。由于
$
A
ψ()
v
r是(满足一定条件下的)任意态,这就要求,就分解这一类态而言,A的本征函数族必须是完备的。这就是作为算符它的本征函数族的完备性和作为力学量它的可观测性之间的联系。如果一个厄米算符,它的本征函数族不是完备的,原则上它就不是一个可以观测的量。就一般系统而言,它的任一力学量的本征函数是否完备很难证明。因此,
一般说来,力学量是否为一个可观测量是个凭物理直觉作出的假设和信念。第五,可以证明如下重要结论:两个力学量、
$
$
A
$
A
A B可以同时观测的充要条件是它们的对应算符彼此对易,

$
,A
A
$
AB BA=?
$
ψ
ab
ab
[]
$
B,ab ba=?
$
A
$
A
$
B
ab
}
ψ
ab
[ ][
$
,AB
ab
ab
ψ==
ab
ψ

0
]
$
= 0
]
$
= 0
$
B
$
A ψ
a
$$
AB
aa
ψψ
)(
$
aBψψ=
0
$
,A
$
(
a
a
Bb
aa
ψψ=
B B
[]
$$ $
B =0
这里要指出,零算符的含义是它作用到任何物理波函数上结果均为零。条件的必要性论证如下:若对任一状态都能够同时测定、B,根据上面所说,必存在、
$
A B的共同本征函数。
因为,这时在测量之后,原先的波函数必以某一几率向、
$
A
$
B的某个共同本征函数ψ坍缩。于是就有
$
()A
ab ab
ψψ=0
这里ψ
ab
分别是、
$
B的某一本征态。由于、是可观测物理量,所以
{
序列是完备的,任一波函数总可按
{
展开,于是对任给的波函数
}
ψ
ψ,有
] [ ]
,
$$$$
,
$
AB AB
ab
ab
ab
ψαα

=
从而得到
[
$
,AB
条件的充分性论证如下:如果
[
$
,AB
并且假定、
$
A中有一个是非简并的,就是说对应每一个本征值只存在一个本征态。不失一般性,假定如此,于是取的一个本征态
$
A,有
[]
==?
$$
B BA ψ
也即
$
)AB
aa
这就是说,也是的本征值为a的本征态。根据假定,的这个本征态不简并,因此和
$

$
A
$
A
$
B
a
ψ ψ
a
必定只差一常系数,即
$
说明此ψ
a
也是的(本征值为b的)本征态。也就是说,力学量、A可以同时观测。对于有简并的情况,结论依然如此,这在以后论述。
以上五点,构成了量子力学的全部测量理论。
4,[第四公设——微观体系动力学演化公设或Schr?dinger方程公设]
,一个微观粒子体系的状态波函数满足如下薛定格方程
i
rt
t
Hr p rt Hr i rth
v
vv v v
h
v?ψ
ψ
()
$
(,
$
)()
$
(,) ( )==ψ (1.9)
这里
$
H为体系的哈密顿算符,又称为体系的哈密顿量,
$$$
$
()HTV(r)
p
m
V(r)
m
Vr=+ = + =? +
v
v
v h v
22
。”
这里强调指出,如果说在“测量公设”中所涉及的状态坍缩是随机的、不可预测的,不符合经典观念的因果律的话,那么在本公设中完全规定了状态波函数随空间和时间的变化规则。这里不存在任何随机的、不可预测的成分。就是说,描述状态的波函数是完全遵循经典观念下的因果律的。这两方面——态演化的决定论形式和态测量的随机坍缩形式的有机结合就是微观世界的新的因果律,是de Broglie波达到因果律。
5,量子力学的第五个公设——全同性原理公设将在后面第六章中叙述。
6,公设应用举例——广义测不准关系推导
假定任意两个非对易的算符和,可以证明
1
2
[ ]
ψ
ψψ 2121
,
2
1
))((≥ (1.20)
这里记

=?
rd
v
ψψ
ψ
11

为在态
1
ψ中的平均值,而
ψ
ψ
ψ
2
111
)

()(=
是力学量对应的算符在态
1
1
ψ中的方均根偏差。
证:令
IA

11
ψ
=,IB

22
ψ
=
I
为单位算符,于是

=== rdAA
v
ψψ
ψ
ψ
ψ
ψ
222
11
2
1

)

()(
2

)
( ψψψ ArdAA ≡?=

v
这里ψA
是态的模长。同样有 ψA
ψ
ψ
B
)(
2
=
注意到[ ] [ ]BA
,

,
21
=,所以只需证明
[ ]
ψ
ψψ BAAB
,
2
1

≥?
即可。根据Schwarz不等式,
ψ?ψ?ψ?=?≤
∫∫∫
rdrrdrrdrr
vvvvvvv 22
)()()()(
我们有
∫∫

=≥? rdBArdBABA
vv
ψψψψψψ

)
()
(

利用恒等式
[ ] [ ]BABABA
,
2
1
,
2
1

+=
+
这里[ ] ABBABA

,
+=
+
是两个算符的反对易子。于是
[ ] [ ]

+≥
+
rdBABA
v
ψψ
ψψ
)
,
2
1
,
2
1
())((
21
等式右边包含反对易子的第一项为实的(取厄米不变),第二项为纯虚的(取厄米将反号)。但对任何实数值和b,总应有a
22
baiba +=+,
于是得
[ ] [ ]
2/1
22
21
,

,
2
1
))((
+≥
∫∫
+
rdBArdBA
vv
ψψψψ
ψψ
[ ]

≥ rdBA
v
ψψ
,
2
1
[ ]

=
rd
v
ψψ
21
,
2
1
这个不等式说明,如果两个算符彼此不对易,在任何态中都不可能同时测得它们的准确值,
它们没有共同的本征态。并且,它们的量子涨落(测得值的均方根偏差)应当满足这一不等式。
作为特例,可得
2
h

x
px。同时,值得注意的是,在这个广义测不准关系(包括海森堡测不准关系)的推导中,只用到了前三个公设,并未用到Schr?dinger方程公设,说明此关系和状态进行怎样的动力学演化无关,而是植根于微观粒子基本秉性 —— 波粒二象性和量子测量理论之上的。
第二章 方程 dingeroSchr &&
§2.1 方程 dingeroSchr &&
第一章中说到,这个非相对论量子力学的基本方程是个公设,它既不能由其余公设导出,
更不能由经典观念导出,它的正确性只能由它导出的结论和实验是否符合来检验。下面我们来试着理解它。
现在我们已经知道,无外场的自由粒子波函数为
()
()Etrp
i
Cetr

=
rr
h
r

显然,这个(tr,
r
)ψ表达式所满足的波方程为
( )
(tr
m
p
t
tr
i,
2
,
2
r
)
rr
h ψ
ψ
=
(2.1)
这就是自由的微观粒子的方程。我们也可以用一种简明的公设性程式,即“一次量子化”的方法直接得到这个方程:将经典物理学关于自由粒子能量的等式
dingeroSchr &&
m
p
E
2
2
r
=,按以下对应分别替换为算符
pp
t
iE
,
rr
h →

并将所得的算符方程作用到系统的状态波函数( )tr,
r
ψ上即可。
若存在外场V()r
r
,这时按经典物理学,这一经典系统的总能量为()rV
m
p
E
r
r
+=
2
2
,若转到对应的量子系统,仍可沿用上述“一次量子化”的程式,做经典物理量→算符的替换,
即在进行以上替换之外,再加上() ( )rVr

V
rr
→,并将所得到的算符方程作用到波函数()tr,
r
ψ
上,就得到与此经典系统对应的量子系统的方程,dingeroSchr &&
( )
() ( )trrV
m
p
t
tri
,
2
,
2
rr
rr
h
ψ
ψ
+=
(2.2)
这里,我们已用了方程( ) () ( ) ( )trrVtrr,,

V
rrrr
ψψ =。通常记
() () HrV
m
rV
m
p
22
22
=+=+
rhr
r

这里H
o&&
称为此量子系统的哈密顿量算符,简称为系统的哈密顿量。于是,一般量子系统的方程即为 dingerSchr
( )
(trH
t
tri
,
,r
)
r
h
ψ
ψ
=
(2.3)
这就是非相对论量子力学的基本方程。这里应当补充指出三点:第一,如同已经说过的,这里的一次量子化程式充其量只是一种理解而已,不是严肃的论证。,一次量子化”程式代表着由经典力学向量子力学的飞跃,实质上它已概括了前面许多
a
量子力学公设。第二,对于复杂些的经典系统,比如势V中也含有动量p
r
时,在一次量子化过程中,由于算符r
r
和p
r
不对易而出现一个经典势的表达式可能对应几个量子势的表达式(它们之间的差别仅在于其中的r
r
和的排列顺序不同)的情况,这对其它的经典力学量也可能会出现,比如 p
r
2222222
,, xpxpxpxpx
xxxx
量子算符经典力学量一次量子化
→?
这在前面关于流密度算符中已经领略到了。这里存在一些一般的对应规则,详细可参见第五章叙述
b
。但归根结底,对应是否正确仍要由实践来检验。第三,若V (trV,
r
)=,这是含时的经典系统,对应于经典粒子在交变势场中运动,粒子系统与外界系统之间存在能量交换,
机械能一般不守恒。在量子系统转变时,仍然如此对应。这是由于V中含有时间参数,量子系统的哈密顿量,成为含时的量子系统。相应的问题为非定态问题,详见第十一章。
()tHH

=
若势场V不显含t,在经典对应情况下(经典粒子在势场()r
r
( )rV
r
中运动)机械能守恒;
在对应的量子情况下,后面将会阐明,依然有总能量守恒。这时,方程通过变换(其中
dingeroSchr &&
E为某一常数参量)
() ()
Et
i
E
ertr
h
rr
=ψψ,
转变为关于()r
E
r
ψ的不含时方程
( ) ( )rErH
EE
rr
ψψ =
(2.4)
此方程称为对应能量E的定态方程。一般而言,求解定态方程问题是一个求本征值和本征函数的问题。就是说,对一个给定的哈密顿量
dingeroSchr && dingeroSchr &&
H
,一般说,不是对任意给定的E值都有对应解( )r
E
r
ψ存在。有对应解( )r
E
r
ψ存在的E值集合称为该定态方程的能谱。一般地说,一个系统的能谱既有分立部分也有连续部分,但有些形式的势只存在分立谱或只存在连续谱。全部对应解的集合
dingeroSchr &&
( ){ }r
E
r
ψ称为这一问题的(能量)本征函数族。
a
除测量公设和全同性原理公设,后一条公设在所谓“二次量子化”方法中才被概括进来。这一方法是当量子力学向量子场论发展时所采用的。
b
亦可参见C.J,Isham著“Lectures on Quantum Theory —— Mathematical and Structural Foundations”。
从初条件的观点来看,上面的( ) ( )0,rr
E
rr
ψψ =为初态波函数,并且它还是H
的某个本征函数。
如果初始波函数(0,r
r
)ψ并不是H
的本征函数,后来任意时刻的( )tr,
r
ψ如何计算呢?这就是一般情况下出事波函数随时间演化的问题,即初值问题。解决的一般办法是,首先将
(0,r
r
)ψ按此问题的本征函数族展开
( ) ( )
() ()?
=
=

rErH
rcr
nnn
n
nn
rr
rr
ψψ
ψψ 0,
一般而言,这个展式既包含求和(对分立谱部分)也包含积分(对连续谱部分)。于是,以后时刻的波函数即为 t
() ()

=
n
tiE
nn
n
erctr
h
rr
ψψ,(2.5)
这可以用代入办法直接检验。对于能谱为连续的本征函数族( ){ }r
n
r
ψ的情况,如自由的正能量的de Broglie平面波族情况,可类比这里的程式进行计算,具体可见第三章第三节高斯波包时间演化计算。
§2.2 方程基本性质讨论 dingeroSchr &&
这里分几点讨论一下方程的一般性质。 dingeroSchr &&
1,线性性质与态叠加原理
注意方程对dingeroSchr && ψ而言是线性的,从而保证了“态叠加原理”成立。就是说,

1
ψ、
2
ψ是系统的两个状态,则对任意复常数
1
α和
2
α,
2211
ψαψαψ +=也必是系统的状态。同时,由于从物理上坚信一个量子系统的能量总应是可观测的,于是此系统的哈密顿量H算符本征函数族应是完备的。就是说,任一状态总可以用这组完备集态{}
n
ψ将其展开。
这些解释已在前面说过了。但是,这里要强调的是量子力学的态叠加原理和经典波中的拨叠加概念有重大的不同。这表现在测量的突变、单次测量的不确定性以及每次测得的力学量均是本征值等等。这是德布罗意波的波叠加原理。
2,几率流密度与几率的定域守恒
对方程取复数共轭,得 dingeroSchr &&
()
() (trVtr
mt
tr
i,,
2
,
2
rrh
)
r
h

+=
ψψ
ψ
将方程左乘以,将此方程左乘以dingeroSchr &&
ψ ψ,前者减去后者,即得所谓粒子几率流的连续性方程
j
t
r
=
ρ
(2.6)
这里
()() ()
() () () () ()[]几率流密度几率密度
trtrtrtr
mi
trj
trtrtr
,,,,
2
,;,,,
rrrrhr
r
rrr

=
=
ψψψψ
ρψψ
显然,它们分别为前面的算符( )rr
rr
′?=δρ?和() ()[ ]rrpprr
m
j
rrrrrr
r
′?+′?= δδ

2
1
在态
(tr,
r
)ψ中的平均值
a
。这在后面还将说及。
上述方程表明,r
r
处体积内几率密度的变化(比如增加)是由此处体积内外之间粒子的流动(流入)造成的。这正是非相对论量子力学的粒子数定域守恒的数学表示。如果粒子被局域在有限空间范围内,则对上面连续性方程进行全空间积分,利用高斯公式将第二项的体积分化为在无穷远表面的通量积分。接着利用局域条件,即假定此量子系统为局域的,
在无穷远处不存在粒子的注入或流出(或不存在净粒子流),于是这项为零,即得
dv dv
() 0,=

全空间
rdtr
t
rr
ρ
就是说,Schr方程规定,粒子在全空间的总几率不变。这表明整个非相对论量子力学都不涉及(质量为m的)粒子的怎样产生和如何湮灭,也就是不考虑粒子间的转化问题。这是非相对论量子力学的基本范畴。这一点和能量范围是非相对论正相匹配,说明非相对论量子力学在概念上是逻辑自洽的(不像单粒子相对论量子力学)。
dingero&&
3,稳定势场方程的一般解 dingeroSchr &&
上面已叙述过,当V中不显含t时,一般初条件下方程的普遍解可写为 dingeroSchr &&
() ()

=
n
tE
i
En
n
n
erctr
h
rr
ψψ,
于是根据量子力学的基本公设,在这个叠加态中,测量时出现态
n
E
ψ的几率为
2
n
c。从而在(tr,
r
)ψ态中所测得的能量平均值为


=
n
n
n
nn
c
cE
E
2
2
(2.7)
4,势场界面和奇点处波函数的性质
显然,由(tr,
r
)ψ的几率解释,一般地说,要求( )tr,
r
ψ在其分布区域内应当处处连续、
模为单值而且在任一有限区域内平方可积。此外,还要求除在势场的奇点、突变的界面之外,
处处可微。在势场发生突变的界面或界点上,如突变是有限的,则由边界两边几率流应当相等可得,(不仅要波函数连续)微商也须连续。只在势场的无限大跃变的地方波函数微商才会不连续。在势为无穷大的区域,根据方程中ψV项必须有限以保持方程在此区域成立这一考虑得知,在此区域ψ必须为零。在势场的奇点处,也许ψ会有相应的奇点,但仍必须保持在奇点附近区域内平方可积;而当势场处处有限时,ψ也必处处有限。
= 0≥
r
rdV
=


r
r
ψ
ψ
min
n
)n
VVT +
,
( →r

0<
∞→r

V 0<E
5,能量平均值下限问题
由于动能算子
m
p
2
2
T
r
的全部本征值
2
2
m
p
r
,从而不论对任何态,0≥T。设V是的最小值,则有
min
()rV
r
min
min
V
drd
rdV
V ≥=



r
r
ψ
ψ
ψψ
ψψ
于是得到
min
VTE ≥≥+=
这一不等式对任何态均成立,包括对任意本征态,比如第个能量本征态亦成立,从而得到
(对任何VE
n min
≥ (2.8)
6,能谱分界点问题
设外场在无穷远处消失,即假定)∞→,0V。这时,对应于能量的所有定态都是波函数分布在有限区域(局域)的束缚态。这是因为,粒子的动能是非负的,若粒子在无穷远处有存在的几率,则它在那部分空间里的方程将为
0<E
∞→
=
r
m
p
r
ψ
2
2
然而这个等式是不能成立的,因为已设定E,除非0=。这里强调指出,可以用简单的讨论说明
b
,在()∞→→ r,0的情况下,所有的态不仅都是束缚态,而且它们的负能量本征值均呈分立谱;而所有的态不仅都包含无限运动,而且它们的正能量本征值均呈连续谱。
0>E
7,本征函数族完备性与能量可观测性问题
对任意V形式,()r
r
H
的本征态族{ }
n
ψ是否为完备系(即,可以用它们展开任意给定的初始波函数)是一个数学上尚未解决的问题。这等价于说,对任意量子体系在数学上尚不能证明系统的能量是个可观测量。当然,在物理上,我们有理由相信,任何量子体系其能量都是可观测的。事实上,一类相当普遍的体系(含常见情况),已证明其哈密顿量的本征函数族是完备的
c

a
亦可参见:戴显熹,贺黎明,徐炳若,黄静宜,大学物理,1984年第1期。
b
Л.Д,朗道,E.M,栗弗席茨著,量子力学(非相对论理论) 上册,第36页,1980年。
c
李政道著“场论与粒子物理学”(科学出版社,1980年)上册第12页,有一个有关完备性的定理(它也参见李著“物理学中的数学方法”)。该定理断言,如果一个厄米算符H
有下限而无上限(含义见该书第11
§2.3 方程向经典力学的过渡 dingeroSchr &&
量子力学向经典力学的过渡有多种途径。这里只给出两种从方程出发的过渡。
dingeroSchr &&
1,h过渡方式 0→
首先经典的波动光学在数学形式上是怎样过渡到几何光学的。众所周知,几何光学是麦克思韦方程当波长0→λ时的极限情况,就是说,令
( ) ( )
( )[ ]
() ()
()[trLki
trLki
erhtrH
eretrE
ω
ω
=
=
r
]
r
r
r
r
r
rrr
r
0
0
,
,
(2.9)
这里,e、h是
r
r
r
r
的复矢量函数,是为了考虑光场可能存在的各种偏振状态。e、h、均与
r
r
L
0
0
2
λ
πω
==
c
k无关(这里,
v
c
nnkk ===,
2
0
λ
π
)。将这两个表达式代入空间非均匀但各向同性的非导体介质(0=σ)的麦克斯韦方程组,令∞→
0
k,即取极短波长近似,很容易得到对于光程函数的程函方程——几何光学的基本方程,()r
r
L
( )
22
nL =? (2.10)
与此同时,平均波印亭矢量的空间轨迹即为几何光学中的光线。这就从波动光学过渡到了几何光学。
与此类似,我们可以指望牛顿力学是量子力学当时的极限情况。就是说,对接近经典的量子系统,如令
0→h
() ()
( )
h
r
rr
trS
i
etratr
,
,,=ψ
这里和均为实函数,可以预计,当时S将服从牛顿力学规律而成为经典粒子的作用量。为说明这点,将此表达式代入方程,可得
a S 0→h
oSchr &&dinger
a
() 0
222
2
2
=++
Vaa
m
aS
m
i
Sa
m
i
S
m
a
t
a
i
t
S
a
hhh
h
分开实项和虚项,可得两个方程
()
=+?+
=+?+
0
1
2
0
22
1
2
2
aS
m
S
m
a
t
a
a
ma
VS
mt
S h
}
n
页),则它的本征函数族{ψ是完备的。但是,除了少数特殊情况外,在一般情况下断定一个厄米算符是否“有下限无上限”却是困难的。
a
Л.Д,朗道,E.M,栗弗席茨,量子力学(非相对论) 上册,高等教育出版社,1980年。也参见K,Gottfried,
Quantum Mechanics,vol.1,p.70 (1965)。
从第一式中略去项,并将第二式乘以,得
2
h a2
()
()
=

+
=+?+
0
0
2
1
2
2
2
m
S
adiv
t
a
VS
mt
S
(2.1)
第一式就是关于单粒子作用量的经典哈密顿——雅可比方程S
a
。第二个方程可看成连续性方程。如果不把看成是粒子的几率密度,而看成是粒子密度,则该方程完全是个经典力学方程。
2
a
综上所述,当方程中(实质上是说,所研究体系的动量或能量足够大,以至相应的德布罗意波长
dingeroSchr && 0→h
0≈λ)时,若略去h项,则得到经典力学的规律。说得仔细些,当时,若在波函数的振幅和位相中,只计及到h的一次方项,则它们均服从经典力学规律。
2
0→h
2,取平均值过渡方式
也可以换一种方法来研究这种过渡的问题。这就是研究势场中波函数的平均动量变化的规律。以
dingeroSchr &&
x方向为例,

=
dv
x
ip
x
ψψ h? (2.12)

()
()
∫∫
∫∫
∫∫
+

=
+
+=
=


dv
x
V
x
Vdv
xxm
dvV
mx
dv
x
V
m
dv
tx
idv
xt
ip
dt
d
x
ψ
ψ
ψ
ψ
ψ
ψ
ψ
ψ
ψψψ
ψ
ψψ
ψ
ψ
ψψ
2
22
2
22
h
hh
hh
这里,第一个积分中的两项均包含?算子,对之进行分部积分并利用无穷远处的边条件(在那里ψ及其一阶偏导数均为零),可得第一项积分为零;第二项积分在消去两项将之化简之后,即得

=
dv
x
V
p
dt
d
x
ψψ?
a
参见例如,吴大猷著,古典动力学,第251页,科学出版社,1983年。
Hamilton-Jacobi方程为
0,=
+
t
S
x
S
xH
i
i
这里
i
i
p
x
S
=


() FVdvVp
dt
d
r
r
===

ψψ
(2.13)
这表明,微观粒子运动在期望值(平均值)的意义上是遵从牛顿第二定律的,量子效应只是围绕经典平均值的一种量子涨落。
§2.4 力学量期望值的运动方程和对易子计算
1,力学量期望值的运动方程
力学量在态?
(tr,
r
)ψ中的期望值为
() ( ) ( )

=?≡?
dvtrtr,
,
rr
ψψ (2.14)
对它求时间导数,得
()
()




+

=


+?=


+?=
+

+?
=?=?



dvHH
i
t
dvH
i
t
H
i
dvH
i
t
H
i
dv
tttdt
d
dt
d
ψψ
ψψψψψψ
ψψψψψψ
ψ
ψψψψ
ψ






h
hh
hh
(2.15)
这里,
t?

是当算符中显含有时间参数时的偏导数。接着,引入两个定义:其一是所谓
“量子泊松括号”,
[ ] [ ] ( )ABBA
i
BA
i
BA
QP

1
,
1
,
=≡
hh
(2.16)
其二是算子的时间微商算子A
dt
Ad
,这个新算符用其平均值来定义,即
dt
Ad
的平均值是平均值的时间微商,
A
=≡
dt
Ad
A
dt
d
dt
Ad
代入上面计算结果,得
[]

+

=
=
dvH
tdt
d
dt
d
QP
ψψ
,

由于ψ态是任意的,于是我们得到如下算符的时间导数算符公式,
[ ]
QP
H
tdt
d
,

+

=
(2.17)
这个表达式可看作是与力学量
dt
d?
对应的算符?
&
的定义式。比如,若,则此式即相应于速度算符的定义式。
x?
=?
如果算符?不显含,则时间导数的平均值(也即平均值的时间导数)成为
t
[ ]

=
dvH
dt
d
QP
ψψ
,
(2.18)
举例,对位置算符x,它显然不显含时间,于是它的导数算符为
[ ] ( )
()
m
p
xim
xx
mi
xTTx
i
Hx
i
v
x
x
2
22
1

1
,?
1
2
=
=
=
==
hh
h
hh
(2.19)
又比如,
[ ] [ ] VVp
i
Hp
idt
pd
F===≡,
1
,
1
r
h
r
h
r
r
(2.0)
于是,一个力学量的厄米算符,如果它既不显含t,又与系统的哈密顿量对易,那么这个力学量便是这个系统的一个守恒的物理量,简称为守恒量,或称为运动常数。就给定的系统而言,说某个力学量守恒,具体的含义是:第一,该力学量在此系统任意(束缚)态内的平均值不随时间变化;第二,如果在所给的态中该力学量具有确定值(即此波函数是该力学量算符的本征函数),则在所有以后时刻该态仍具有该确定值,或者说,这个定值是守恒量子数,又常称为好量子数,
在分立谱的定态上,任何不显含时间的力学量的平均值不随时间变化。证明如下,


=?
=
dv
EH
nn
nnn
ψψ
ψψ
*
于是
()
0
}{
1
}

{
1
}

{
1
)

(
1
*
**
*
=
=
=
=
=

∫∫

nn
nnn
nnnnn
nn
EE
i
dvHE
i
dvHdvE
i
dvHH
idt
d
h
h
h
h
ψψ
ψψψψ
ψψ
证毕。
2,对易子运算
量子力学的对易子运算全部建立在下面这个基本对易子的基础上,
[ ] hipx
x
=?,? (2.1)
这个对易子是量子力学基本假设内容之一。由这个对易子,便可决定当时,xx =?
x
ip
x
= h?。将此结论自然地推广到3维( )3,2,1=α
α
x,得
[ ]
αββα
δhipx =?,(2.2)
此外,对易子运算中还常用到一些恒等式,如
[ ] [ ] [ ]
[][]
[][][]
[][][][][][]等等,0
,
,

,
,

,
,

,

,

,
,

,
,

,

,
=++
+=
=
+=+
BACACBCBA
CBACABCBA
ABBA
CABACBA βαβα
作为运算的一个例子,我们证明如下结论,
设,能表达成,的幂级数,且对易子()pxAA?,?
= (pxBB?,?
= ) x? p? [ ] hipx =?,?,求证
[ ] { }
CPQP
BABA,
,
lim
0
=
→h
(2.3)
这里,右边是当,A
B
的算符变数换为经典变量x,p时的泊松括号。
证:设(这是可以做到的,因为如有在右边的项,均通过基本对易子将之调换,直到每项的均在的左方)。


=
=
0,

ml
ml
lm
pxaA
x?
x? p?
p?
首先,注意有以下对易关系,
()[]
()[]
()[]
()[]
++
+
=
+
+
=
+
=
=
121
223
2
,?,?
,?,?

,?,?
,?,?
nnnn
x
p
f
x
p
f
x
p
f
xipxfx
x
p
f
x
p
f
x
p
f
xipxfx
x
p
f
p
f
xipxfx
p
f
ipxfx
Lh
h
h
h
类似有
()[]
++
+
=
121
,?,?
mmmm
p
x
f
p
x
f
p
x
f
pipxfp Lh
∴ ()()[] ()[ ]

→→
=
ml
ml
lm
pxBpxa
i
pxBpxA
i
,
00
,?
,
1
lim?,?
,?,?
1
lim
hh
hh
[ ] [ ]{ }
++
+
+
++
+
=
+=






mlll
ml
mmml
lm
ml
mlml
lm
px
p
B
x
p
B
x
p
B
xi
p
x
B
p
x
B
p
x
B
pxia
i
pBxBpxa
i


1
lim
,?
,
1
lim
121
,
121
0
,
0
Lh
Lh
h
h
h
h
()
() ()
{}
CP
ml
ml
lm
ml
ml
lm
ml
mlml
lm
ml
mlml
lm
A
BA
p
A
x
B
p
B
x
A
pxa
px
B
p
B
pxa
x
p
px
x
B
p
B
x
px
a
p
B
plxpx
x
B
ma
,
,,
,
,
11

=
=
=
+

∑∑



(2.4)
这里步等号是由于当时,与对易,从而()A 0→h x? p?
x
B
与可交换,成为对应的经典力学量;其余类似论证。
k
p?
对易子计算的注意事项详见附录4。
再举个例子。如前所说,粒子密度算符可取为( ) ( )rrrr ′?=′?
rrrr
δδ
。这个算符也不显含时间,但同样也不能说它的时间导数(算符)为零。事实上,按上面关于一个算符的时间导数(算符)的公式,可得它得时间导数(算符)为
( )
()[]()
()[]()[]{}?′+′=
′?=′?=
′?
,
,
2
2
,
1
,
1
2
rrrr
mi
m
p
rr
i
Hrr
idt
rrd
rrrrh
r
rr
h
rr
h
rr
δδ
δδ
δ
由于()[ ] ()( )rrrr ′=?′?
rrrr
δδ
,
,这里的小括号表示算符?只对( )rr ′?
rr
δ
(中的r
r
)作用,于是得
()
()() (({}rrrr
midt
rrd
′+′=
′? rrrrh
)
rr
δδ
δ

2
(2.25)
方程右边就是粒子密度算符的时间导数(算符)表达式。将它对态( )tr,
r
ψ求平均,利用时间导数算符的平均值等于该算符平均值的时间导数这一定义,
()( )()
( )( )
()
dt
trd
dt
trtrd
rdtrrrtr
dt
d
,
,,
,,

=
′′
=′?=

r
rr
rrrrr
ρ
ψψ
ψδψ上式左边
() ( )( ) ()( )[ ] ()
() ( )() ()()[]()
()( ) ( )()
()( ) ()()
() ()() (trtrtrtr
mi
j
rdtrrr
m
p
m
p
rrtr
rdtrrrrrtr
mi
rdtrrrrrtr
mi
rdtrrrrrtr
mi
,,,,
2
,
2
2
,
,

,
2
,

,
2
,

,
2
′?′′?′?′′′?=′?=
′?+′′?=
′+?′′
=
′′′′?=
′+′=





rrrrh
r
rrrr
rr
rrr
rrrrrrrh
rrrrrrrh
rrrrrrrh
ψψψψ
ψδδψ
ψδδψ
ψδδψ
ψδδψ上式右边
)
这正是前面的连续性方程。同时也看到,流密度矢量
() () ()() ()( )trtrtrtr
mi
trj,,,,
2
,′?′′?′?′′=′

rrrrhr
r
ψψψψ (2.26)
正是流密度矢量算符() ()rr
m
p
m
p
rrj ′?+′?=
rr
rr
rr
r
δδ
2
2
在态( )tr,
r
ψ中的平均值。
第三章 一维问题
§3.1 一维定态的一些特例
求解方程是量子力学的中心任务。我们从最简单的情况——一维定态的一些特例开始讨论,下节中再给出一维定态问题的一般性讨论。
dingeroSchr &&
1,一维量子谐振子问题
在经典力学中,一维经典谐振子问题是个基本的问题,它是物体在势(或势场)的稳定平衡位置附近作小振动这类常见问题的普遍概括。在量子力学中,情况很类似。一维量子谐振子问题也是个基本的问题,甚至更为基本。因为它不仅是微观粒子在势场稳定平衡位置附近作小振动一类常见问题的普遍概括,而且更是将来场量子化的基础。
众所周知,当粒子在势场的平衡位置附近作小振动时,势场V(x) 总可作泰勒展开并只取到最低阶不为零的项。设平衡位置0
0
=x,并选取能量尺度的原点使V(0)=0,则
2
2
1
)0()( xVxV ′′?
这里,含V的一次项由于平衡位置)0(′ 0)0( =′V而消失,也由于是稳定振动而有V>
。除非振动的幅度较大,否则不必考虑展开式中非简谐的高阶项。这类问题的物理例子比如,原子核内核子(质子或中子)的简谐振动、原子和分子的简谐振动、固体晶格上原子的简谐振动、甚至一个多自由度系统在其平衡态附近的小涨落小振动,在通过引入简正坐标后也可以化为一系列退耦的一维振子之和。
)0(′′
0
一维量子简谐振子的位势为
22
2
1
)( xmxV ω= (3.1)
dingeroSchr &&方程为
)()(
2
1)(
2
22
2
22
xExxm
dx
xd
m
ψψω
ψ
=+?
h
(3.2)
为方便,引入自变数变换
x
m
h
ω
ξ =
并令
ω
λ
h
E2
=,可得
0)()(
)(
2
2
2
=?+ ξψξλ
ξ
ξψ
d
d
这里,)()( xψξψ =。为消除)(ξψ方程中含的项,引入函数变换
2
ξ
a
a
关于求解方程中的函数变换和自变数变换问题可参见附录二。 dingeroSchr &&
1
)()(
2
2
xe?ξψ
ξ
=
转化为关于?的方程
0)1(2
2
2
=?+λ
ξ
ξ
ξ
d
d
d
d
为求解此方程,设解为广义冥级数形式代入并逐项决定此级数的系数,


=
=
0
)(
n
n
n
l
a ξξξ?
其中和均为待定常数。代入l
n
a?方程后得
∑∑
∑∑∑
==
+
=
=

=
=+?
++?
01
11
2
2
1
11
0
2
0)21(2
)1(2)1(
n
n
n
l
n
n
n
l
n
n
n
l
n
n
n
l
n
n
n
l
alna
annnalall
ξξλξξ
ξξξξξξ
这要求相同冥次的系数之和均为零。于是得到以下关于系数的代数联立方程组,
n
a
0)21(2)1)(2()2(2)1(:
0)21(222)1(:
02)1(:
0)1(:
222
0222
11
1
0
2
=+?++++?+?
=++?+?
=+?
=?
+++
+
nnnnn
nl
l
l
l
alnaannanlall
alaalall
laall
all
λξ
λξ
ξ
ξ
LLL
由微分方程广义冥级数求解理论知,指数方程(第一个方程)
0)1( =?ll
的两个解l、0= 1=l
0=
相差是整数,对应的两个广义冥级数解不相互独立,因此可只取l值中的一个,比如l。由此得到系数之间的递推关系
nn
anann )21()1)(2(
2
λ?+=++
+
如果参数λ不是正整数,这个递推公式将一直工作下去直到无穷(只是在奇次幂项之间和在偶次幂项之间的递推各自独立进行),于是将得到一个无穷幂级数。验算可知,这时解)(ξ?
将有如下渐进形式,
2
)(
)(
ξ
ξ
ξ? e→?
∞→
于是可得)(ξψ的渐进行为
2
2
1
)(
)(
ξ
ξ
ξψ e→?
∞→
这在物理上是不合理的,因为这样的)(ξψ不是平方可积的。再说,当∞→ξ时,∞→)(ξV,
那里的波函数)(ξψ应当趋于零才是。由这些分析可知此冥级数应当截断,即应当存在如下
2
关系,
12 += nλ (3.)
由E与λ的关系可知,这相当于要求E不能任意取值。
这时比如,当E的取值相应于时,
0
nn = 0
20
=
+n
a,由此以后的系数均为零
a
,)(ξ?变成阶的多项式,从而
0
n )(ξψ便满足上面关于平方可积和局域性这些物理要求了。接着,由上述本征值定出多项系数,便可得到与此本征值相对应的本征函数。这时)(ξ?的方程
022 =+′?′′ξ? n
是厄米特方程,其解为厄米多项式)()( ξξ?
n
H=。前几个)(ξ
n
H是,
53
5
42
4
3
3
2
2
10
32160120)(164812)(
812)(42)(
2)(1)(
ξξξξξξξ
ξξξξξ
ξξξ
+?=+?=
+?==
==
HH
HH
HH
它们具有以下的正交归一性

∞+
∞?
=

=
nmn
nm
deHH
nnm
,!2
,0
)()(
2
π
ξ
ξ
ξξ
于是,量子谐振子的波函数和能级为
=+=
=

),2,1,0()(
)()!2()()(
2
1
22
1
4
1 2
Lh
hh
h
nnE
x
m
H
x
en
m
x
n
n
m
n
n
ω
ω
π
ω
ψ
ω
这里的)(x
n
ψ是正交归一的。
见图
a
下面简要地讨论一下这些结果。
首先,研究一下谐振子处于本征态上时的平均动能和平均势能。
由于前面叙述过,不显含时间算符在任一定态中的平均值将不随时间改变,现将此结论用到算符上,有(略去算符记号“xp
=? ∧”)
a
注意偶次幂项系数由初条件a(奇次幂项系数由初条件a)递推而得。
0 1
3
[]
[][]
[]
[] []
VT
x
x
V
i
m
p
i
i
x
x
V
ipxp
m
p
ppx
m
p
i
xVp
m
p
xp
i
xHpHxp
i
Hpx
i
px
dt
d
22
1
,
2
,
2
1
,
2
,
1
,,
1
,
1
0
2
2
=
=
+=
+
=
+=
==
hh
h
h
h
h
h
h
(3.4)
由此式,再考虑到VTE
n
+=,于是可得,谐振子任意本征态的动能平均值和势能平均值相等,各为该本征态总能量的一半。这又一次说明,平均值的情况是符合经典图象的。
其次,注意到不论谐振子处在量子数n为多少的本征态)(x
n
ψ上,随x增大)(x
n
ψ均迅速减小。说明在谐振子势中运动的微观粒子,较多机会集中在平衡位置附近,离平衡位置较远处的势能变大,粒子的徳布罗意波“渗透”进这个区域去的振幅变小。应当强调指出,
在能量本征值问题上有两点与经典振子显著不同的特点。第一点,能量本征值随量子数n
的变化(如前所述,它称为系统的“能谱”)不但是断续的,而且是等间距的,间距为ωh,
只和振子的固有频率有关。这正好说明了普朗克能量子ωh假设的根源。因为,任何量子系统,如果可以认为它在做简谐振动,那么它的能谱特征都是如此(黑体腔内电磁场也不例外)。
能谱的这种均匀间距的特征是和势场为型密切相关的。第二点,这里最低能态(通常称为“基态”)的总能量并不为零,而是大于零,
2
x
x
e
m
xE
m 2
2
4
1
00
)(,
2
1
h
h
h
ω
π
ω
ψω
==
这个称为“零点能”。这就是说,按量子力学的观点,即使在温度为绝对零度,无论是电磁场的简正振动模还是晶体点阵上的原子,它们作为谐振子的振动能量并不为零,依然在振动着。这时的平均动能和均方位移分别为
0
E
()
==?
=
ω
ω
m
xx
T
h
h
2
1
4
1
22
(3.5)
由于这两个量不为零,说明这时的确存在振动。事实上,低温下?x射线布拉格弹性散射强度依然和刚性点阵结果不符,说明这时点阵的零点振动依然存在,造成了漫射背景甚至非弹性散射(加真空涨落造成自发跃迁)。
“能量量子化”和“零点能存在”是量子振子的能谱不同于经典振子能谱的两大特点。
而且,“零点能存在”问题即使在普朗克假设中也是没有表现出来的。这两点都是粒子波动
a
L,泡令,E.B,威尔迪著“量子力学导论”,第69-70页,科学出版社,1964年。
4
性的体现,前者是由于粒子德布罗意波的自身干涉;后者是由于动能为零值的德布罗意波是没有意义的
a
,更准确地说,是由于粒子德布罗意波所固有的测不准关系。关于后一论断可以简述如下。由于
()
()
2222
22
2
1
2
1
4
1
2
1
2
1
4
1
xmxmV
p
m
p
m
T
===
===
ωωω
ω
h
h
从而若令
() ()
22
,xxpp?==?
则有
2
h
= px (3.6)
所以,量子谐振子的基态是具有“最小测不准度”的状态(这称之为相干态,这在以后还有叙述)。
再三,当谐振子处于量子数n为某一确定值的本征态时,哈密顿量的平均为
n
n
n
xmTnE
22
2
1
)
2
1
( ωω +=+= h (3.7)
这里用
n
T代表动能算符在
n
ψ态中的平均值。这就是说,在平均值的意义上,经典的能量守恒定律依然成立,这是从爱伦弗斯特定理可以预料的。但是,按经典力学,振子的振幅有一最大值,它满足
max
x
2
max
2
2
1
2
1
xmn ωω =
+ h
于是
() (12,12
max
+=+= n
m
nx ξ
ω

h
) (3.8)
然而,按量子力学,粒子在此之外仍有一定的几率被找到
max
x
ξ
ξ
ξ
π
deH
n
P
n
n
n
n
=


+
2
)(
!2
2
2
12
(3.9)
对于基态,随增大,减小。这当然是和德布罗意波的“渗透性”相16.0,0
0
≈= Pn n
n
P
a
一个态不可能平均动能为零。证明:(用坐标表象表示也可,因为可转到动量表象)
0)(
)(
2
)(
2
)(
2
0
2
2
2
*
2
=?
=
===


∞+
∞?
∞+
∞?
p
dpp
m
p
dpp
m
p
p
m
p
T
ψ
ψ
ψψ
5
关连的量子效应。
再四,研究一下在谐振子问题上,量子力学向经典力学过渡的问题。图一是当n较小时的情况,图二是n较大时的情况。两个图
a
中的虚线代表按经典观点,在谐振子势阱中找到质点的几率密度(单位长度内发现粒子的几率)的空间分布。由两个图可以看到,就平均而言,当量子数n越大,量子结果和经典结果越接近。
其中,经典
n
ρ是按下面近似计算出的,
当质点能量为E时,它被“绝对”地限制在由下式决定的区间[ ]XX,?之内,
22
2
1
XmE ω= (3.10)
在区间[内,质点在]XX,? x处间隔内的几率正比于它在该处dx间隔内停留的时间,即
dx )(xdP
dt
22
1122
)(
xX
dx
dx
vTT
dt
xdP
===
π
(3.1)
这就是上面两个图内虚线的来由。
第五,注意对第个能级,共有个n n ωh,也就是有个量子。当为偶数时,波函数n n
()x
n
ψ为偶函数;当n为奇数时,( )x
n
ψ为奇函数。本书稍后将说明,这表明量子谐振子的单个量子 —— 光子的内禀宇称为负的。
2,一维势垒透射问题
这个问题也称为入射粒子在一维势垒上的散射问题。它是如下一类(体现隧道贯穿量
子效应)问题的概括:核中α粒子衰变、外电场下金属电子的冷发射等。
为简化计算,先假定如下矩型势垒,粒子自左向右朝向势垒运动并经受势垒的散射。
6
这时能量为E的定态方程可分为三个区域写出 dingeroSchr &&
=?
≤≤?=+?
=?
)()()(
2
)()()()()(
2
)()()(
2
2
22
0
2
22
2
22
axxEx
dx
d
m
axaxExxVx
dx
d
m
axxEx
dx
d
m
IIIIII
IIIIII
II
ψψ
ψψψ
ψψ
h
h
h
(3.12)
分两种情况来求解这个方程组。第一种情况,粒子的总能量,这时,在自左向右入射到势垒的初条件下,上述方程的解为
0
VE?
=
≤≤?
+=

+=
)()(
)()(
)()(
1
22
11
xa
ik
Fex
axa
xik
De
xik
Cex
ax
xik
Be
xik
Aex
III
II
I
ψ
ψ
ψ
(3.13)
这里mEk 2
1
=h,)(2
02
VEmk?=h 。考虑到时间因子e
ti
e
iEt ω?
=
h/
,因此代表向右运动的波数为的平面波,
ikx
e k
ikx
e
则是向左运动的平面波。在I、II两个区域内存在向左运动的反射波。而在III区中则只存在向右运动的透射波,不存在向左运动的反射波。
利用在ax ±=边界上波函数及其导数连续的边界条件,得
aik
Fek
aik
De
aik
Cek
aik
Fe
aik
De
aik
Ce
aik
De
aik
Cek
aik
Be
aik
Aek
aik
De
aik
Ce
aik
Be
aik
Ae
122
122
2211
2211
12
21
)(
)()(
=
=
+
=?
+
=+
为便于计算,将上面这些方程写成矩阵形式
=
D
C
aik
e
k
k
aik
e
k
k
aik
e
aik
e
B
A
aik
e
aik
e
aik
e
aik
e
22
22
11
11
1
2
1
2
F
aik
e
k
k
D
C
aik
e
aik
e
aik
e
aik
e
1
22
22
2
1
1
=
记起任意满秩的二阶矩阵和其逆矩阵的关系为
a
取自A.A,索科洛夫等著:“量子力学原理及其应用”,第197页,上海科技出版社,1983年。
7
=
δγ
βα
M
=
αγ
βδ
βγαδ
1
1
M
于是可得
+
+
+
+
=
D
C
a
k
ki
e
k
ka
k
ki
e
k
k
a
k
ki
e
k
ka
k
ki
e
k
k
B
A
)(
1
)(
1
)(
1
)(
1
2
1
2
1
2
1
2
1
2
1
1
2
1
2
1
2
1
2
F
aik
e
aik
e
k
k
aik
e
k
k
D
C
1
2
2
2
1
2
1
1
1
2
1
+
=
由这两组方程,可得势垒两边波函数振幅间的关系
F
aik
e
k
k
k
k
aik
e
k
k
k
k
a
k
ki
e
k
k
k
ka
k
ki
e
k
k
k
k
B
A
++
+
+
+
+
+
=
2
11
2
11
)(2
11
)(2
11
4
1
22
2
1
2
1
2
1
1
2
2
1
1
2
2
1
1
2
2
1
1
2
() ()
()
F
ak
k
k
k
ki
aik
eak
k
k
k
ki
ak
+?
=
2
2
1
1
2
2
2
1
1
2
2
2sin
2
2
2sin
2
2cos
1
由此很易求得透射系数T和反射系数R,
() ()ak
k
k
k
k
ak
A
F
T
2
2
2
2
1
1
2
2
2
2
2sin
4
1
2cos
1
++
==
+
=
h
)(22
sin
)(4
1
1
0
2
0
2
0
VEma
VEE
V
(3.14)
()
() ()ak
k
k
k
k
ak
ak
k
k
k
k
A
B
R
2
2
2
2
1
1
2
2
2
2
2
2
2
1
1
2
2
2sin
4
1
2cos
2sin
4
1
++
==
8
+
=
h
h
)(22
sin
)(4
1
)(22
sin
)(4
0
2
0
2
0
0
2
0
2
0
VEma
VEE
V
VEma
VEE
V
(3.15)
注意,由于在边界上使用了波函数及其导数连续的条件,从而保证了边界两侧的几率流密度相等,这导致上面T、R的表达式满足几率守恒条件
1=+ RT (3.16)
另外,应当指出的是,当满足下面条件
πna
VEm
=
2
0
)(2
2
h
时,透射系数为1,即入射波没全部透过势垒,不存在反射。这种现象出现的原因是,这时从势垒的第一个界面上产生的反射波和势垒的第二个界面上反射并经多次反射向第一区的透射波相消干涉,从而使第一区中的反射波消失。光学中已知,在同一介质层两侧的两个反射波是反号的,这就是说,它俩之间有附加π位相存在。目前在势垒的a±两个界面的反射情况就如同一个介质层两侧的反射情况,从而知道两个反射波之间已有π相差存在。于是,
只要在势垒中往返的程差为πnak 24
2
=,或πnak =
2
2,即导致两束反射波的相消干涉。
这种共振透射现象在许多波动现象里均存在着,比如,光学中无反射薄膜的透射、波导中的阻抗匹配短线以及低能电子从惰性气体上散射的Ramsauer-Townsend效应等。
第三种情况,E
0
V?,这时在同样的由左入射的初条件下,在I、III区中的解形式不变。
第II区的解改变为
)()( axa
x
De
x
Cex
II
+
=
λλ
ψ (3.17)
这里指数λ为
h
)(2
0
EVm?

这是因为,在II区中,这时的方程为 dingeroSchr &&
)(
)(2
)(
2
0
x
EVm
x
IIII
ψψ
h
=

右边
II
ψ前的系数是正值,其解应为指数函数。这个问题的求解可利用前面的结果。这只要在前面公式中,将λik →
2
,并利用
chzizishziz == )cos(,)sin(
即得
9
() ()ash
k
k
ach
A
F
T
λ
λ
λ
λ 2
4
1
2
1
2
2
1
1
2
2
+
== (3.18)
()
() ()ash
k
k
ach
ash
k
k
A
B
R
λ
λ
λ
λ
λ
λ
λ
2
4
1
2
2
4
1
2
2
1
1
2
2
2
1
1
2
+
+
== (3.19)
同样,依然有
1=+ RT
这是因为和前种情况一样,方程遵从连续性条件并且还使用了几率流密度守恒的边界条件的缘故。当
dingeroSchr &&
1aλ,即势垒比较高、透射不容易的情况下,
2
22
1
1
2
1
1
4
16
4
4
1
4
1
4
4
1
1
+
=
+

λ
λ
λ
λ
λ
λ
λ
k
k
a
e
a
e
k
k
a
e
T (3.20)
这时T和的关系呈负指数衰减的形式。 a
3,一维周期势(Kronig-Penney势)问题
首先从两个相邻的方形势阱讲起。假定有两个完全相同的底宽为b 的方阱,a很大彼此分开很远并各有一个粒子在基态上。
则此系统的总能量是两个底宽为b的方阱基态能量之和。或者说,图二的a、b两种情况,
当中间的势垒相距很宽时,能量是一样的。现在设想它们中间的势垒十分缓慢地逐渐变薄(绝热变化),这时两个阱彼此相互影响。对a情况来说,将逐渐过渡到阱宽为2b 的基态,总能量是此基态能量的确一倍(两个粒子均在这个加宽了的阱的基态上)。由于阱加宽了,故随着两个阱的合并,系统的总能量下降。对情况b来说,将逐渐过渡到阱宽为2b的第一激发态上,总能量是此第一激发态的一倍(两个粒子均在这个加宽了的阱的第一激发态上)。
阱加宽虽使得能级下移,但毕竟过渡到第一激发态,系统的总能量仍稍有上升,如下图。
实际上,在靠近时,波函数究竟
是对称还是反对称“连接”,这
要由这两个全同的粒子是玻色子
还是费米子、它们的自旋状态
10
等待因素来决定。
上面的分析定性地说明了,对一个相距很远时两阱能量相同的双阱系统,当两个阱相互靠近时,两个阱的相互影响会使原先的每一个能级按空间对称或反对称的组合而劈裂成两个能级。
其次,由此推广,对于从晶体周期势抽象出的Kronig-penney势(图三),预计在此势中运动的电子,其能谱的每个能级都将劈裂、拓宽成为一个能带。实际情况正是如此,这是晶体中电子运动的最重要特征。由此出发可以定量或半定量地解释固体中的许多现象。现在来处理这一重要问题。
设电子总能量
0
VE?,一般地我们考虑第n谷和第n+1谷。假定第n谷中的波函数为
a
() (anlxaln
nlx
ik
eB
nlx
ik
eAx
nnn
+? )
+
= 1,
)()(
)(
11
ψ
(3.21)
为书写简便,先解第0、第1谷情况,这时
()

+
+
+?
+
=
alxa
xik
e
lik
eB
xik
e
lik
eA
axa
x
De
x
Ce
axal
xik
eB
xik
eA
x
在第一谷里垒里在第谷里在第
,
0,
0,
1111
11
11
00
λλ
ψ (3.22)
这里)(2,2
01
EVmmEk?== λh
l1
1
A
1
B
h。注意,这里系数、分别乘有相因子
,就是说,、采用了和前面例子不同的新定义。参照上面例子,把
1
A
1
B
ik
e
±
ax ±=处的四个边界条件写成矩阵形式,
=
=
=
D
C
ax
x
e
x
e
x
e
x
e
B
A
ax
xik
eik
xik
eik
xik
e
xik
e
λ
λ
λ
λ
λλ
0
0
11
11
11
=
=
=
1
1
11
)()(
)()(
11
11
B
A
ax
lx
ik
eik
lx
ik
eik
lx
ik
e
lx
ik
e
D
C
ax
x
e
x
e
x
e
x
e
λ
λ
λ
λ
λλ
于是
a
参见E,Merzbacher,Quantum Mechanics,P.100,John Wiley & Sons,Inc.,1961。
11
=
0
0
11
1
1
11
1
1
11
11
11
11
)()(
)()(
B
A
aik
eik
aik
eik
aik
e
aik
e
a
e
a
e
a
e
a
e
a
e
a
e
a
e
a
e
la
ik
eik
la
ik
eik
la
ik
e
la
ik
e
B
A
λ
λ
λ
λ
λλ
λ
λ
λ
λ
λλ
按上例中逆矩阵的一般公式求出两个逆矩阵,即得
() () ()
() () ()()
+
=
0
0
1
1
111
111
2
222
2
2
22
B
A
lik
e
aik
eashiach
lik
eashi
lik
eashi
lik
e
aik
eashiach
B
A
λελληε
ληελελ
这里
+=
=
λ
λ
η
λ
λ
ε
1
1
1
1
2
1
,
2
1 k
k
k
k
。令( )ash ληβ 2
2
=和,
于是
() ()
11
2
22
1
βαλελ i
aik
eashiach?=
( ) ( ) ( ) ( )
()( ) ()(
+=
=
akashakach
akashakach
111
111
2cos22sin2
2sin22cos2
λελβ
λελα
)
)
(3.23)
最后即得系数递推公式
=
=
0
0
1
1
B
A
B
A
B
A
n
n
n
n
n
()
()
+

=?
lik
ei
lik
ei
lik
ei
lik
ei
11
11
112
211
βαβ
ββα
这里,(+∞∞?=,,2,1,0,1,2,,LL
±
n。现在来研究这个关于系数的递推矩阵。设的两个本征值为

ω,即
() ( )0det0det
2
=?+=
±±±
trωωω即
由于,可得det,于是 1
22
=?ηε 1=?
() ()
=?
+=?=+
±?=
+
+
±
1
sincos2
1
2
1
2
1
1111
2
ωω
βαωω
ω
lklktr
trtr
(3.24)
根据在此周期阱中波函数必须有限的物理要求,由下面分析可得限制条件
1
2
1
≤?tr (3.25)
也就是
12
( ) ( ) 1sincos
1111
≤+ lklk βα (3.26)
这是因为,如果1
2
1
tr,由
±
ω表达式可知在两个
±
ω中必有一个的模值大于1,比如
1?
+
ω,于是+∞=
+
+∞→
n
n
ω
lim
,而同时
+
=
ω
ω
1
,又有+∞=
∞→
n
n
ω
lim
。这将导致
±∞→x处阱中波函数发散。因为,这时两个根
±
ω不相等,对任何给定的初条件,
0
0
B
A
总可以将它用
±
ω对应的本征态
( )
()?
±
±
0
0
B
A
展开,这两个本征态满足
( )
()
( )
()?
=
±
±
±
±
±
0
0
0
0
B
A
B
A
ω
于是可得
( )
()
( )
()?
+
=
+
+
0
0
2
0
0
1
0
0
B
A
B
A
B
A
δδ
这里
1
δ、
2
δ是两个依赖、的系数。于是根据态叠加原理,在第个阱中,对应这个初条件
的解可表示为
0
A
0
B n
0
0
B
A
()
()
()
()
()
()
()
()?
+
=
+
=
+
+
+
+
+
0
0
2
0
0
1
0
0
2
0
0
1
B
A
B
A
B
A
B
A
B
A
nn
n
n
n
ωδωδ
δδ
当,即±∞→n ±∞→x时,
。由此即得上面的限制条件。 ∞→
n
n
B
A
由于这时
±
ω均为复数,且,但又由于
*
+?
=ωω 1=?
+
ωω,因此
±
ω的模均为1,从而可将它们记为
ikl
e
ikl
e
==
+
ωω
这里实参数k和能量E有关并由下面条件决定
()?=+
+
tr
2
1
2
1
ωω (3.27)

1sincoscos
1111
≤+= lklkkl βα (3.28)
13
对于现在的方形周期势阱,将前面的
1
α、
1
β表达式代入(注意l ba 22 +=),
()(
()() ()()bkashbkach
bkakakbkakak
bkakbkak
bkakbkakkl
11
1111111111
111111
111111
2sin22cos2
2sin2cos2sin2cos2sin2cos
2sin2cos2cos2sin
2sin2sin2cos2coscos
+?=
+?++=
+?+
)
+=
λελ
βαβα
ββ
αα
(3.29)
这是在情况下,决定电子能谱的公式。由下面讨论知它具有带状结构。这里补充指出,
如果令每个势垒区域在保持面积的前提下无限减薄,此问题即转为一维Dirac梳,其相应解已在许多参考书中叙述过。
0
VE?
若,只需用替换
0
VE >
2
ik?=λ,并注意双曲函数与三角函数的关系:
() ( ) zizchizsh cos,=zi sin=,于是得另一公式
()( ) ( ) ( )bkakbkakkl
1212
2sin2sin2cos2coscos= (3.30)
这里,
()
+=?=
2
1
1
2
02
2
1
,2
1
k
k
k
k
VEmk?
h
现在来讨论上面两个能谱公式。它们决定电子在这种周期势中运动时,所容许具有的能量E,第一个公式,即情况下,由于
0
VE? 12?ach λ(当0≠λ),当E取值满足
整数== mmbk π
1
2
时,等式右边第一项为ach λ2,第二项为零,从而使等式不成立。显然这些E(及其附近值!)是被禁止的。于是电子能谱便被分割成一系列的能量带,中间所夹的被禁止的能量间隙包含着上面方程所决定的点。对第二个公式,即情况下,由下式决定的能量附近是禁止的
0
VE?
整数==+ mmbkak,22
12
π
从而构成了在这些点附近的一个个能量间隙。这个条件可如下得到,将之代入第二个公式得
() ( )
() ()
() ( ){}bk
bkbk
bkbkmbkbkmkl
m
mm
1
2
1
21
1
2
1111
2sin11
2sin2cos
2sin2sin2cos2coscos
+?=
=
=
+
π?π
(3.31)
由于1,大括号中的量的模大于1,无k解,即由该式所决定的能量(及其邻值)是被禁止的。从而,当时,电子能谱也具有带状结构。
0
VE?
这里电子能谱呈带状结构的结论虽是在方阱周期势这一特殊情况下得出的,实际上,对任何形状的周期势,电子能谱均呈带状结构,只是间隙的位置和宽度等具体细节不同而已。
14
这一来源于电子的徳布罗意波波动性质的结论对了解固体物质许多基本性质是十分重要的,
并且是金属中电子传导量子理论所不可少的内容。
以上是就两个方程成立与否谈起的,说明当E取某些区间内的值时,等式右边不成立,
即无k值对应。就每一单个的能隙(禁带)来说,能隙的上、下限能量值必定总是使得
1cos ±=kl
也就是说,能隙的上下限必满足
整数== mmkl π
于是若作k
mE
k—
2
2
1
2
h
=的图,则为
下面讨论本征函数问题。
当取定的对应本征值?
+
ω、
ω的两组本征矢量
( ) ( )
( )
( )()
( )
++
0000
,,,BABA,于是
( )
()
( )
()?
=
+
+
+
+
0
0
B
A
inkl
e
B
A
n
n

( )
()
( )
()?
=
0
0
B
A
inkl
e
B
A
n
n
下面来求
() ()
( )
++
00
,BA。由本征方程
( )
()
( )
()?
=
+
+
+
+
+
0
0
0
0
B
A
B
A
ω
可得其中一个,比如第一个方程
()
() () ()+++
=
002011
11
A
ikl
eB
lik
eiA
lik
ei ββα
所以
()
()
()
kllklk
lik
e
ikl
e
lik
ei
lik
ei
B
A
sincossin
1111
2
11
2
0
0
1
1
1

=

=
+
+
βα
β
βα
β
(3.32)
这里第二步等号是用到了下述等式
15
lklkkl
1111
sincoscos βα +=
由此,可取
()?
=
=
+
+
lik
ekllklkB
A
1
sincossin
1111
)(
0
2
)(
0
βα
β
(3.33)
从而第n个谷中的电子波函数为(
()
)(x
+
ψ ( ) alnlxa)
[]
[]
)(
))1((
sincossin
)()(
)(
sincossin
)(
)()(
)()(
)(
1
2
1
1
11
11
1111
2
1111
2
)(
0
)(
0
)()()(
xu
ikx
e
lnx
ik
ekllklk
nlx
ik
e
nlxik
e
ikx
e
nlx
ik
ekllklk
lik
e
nlx
ik
e
inkl
e
nlx
ik
eB
nlx
ik
eA
inkl
e
nlx
ik
eB
nlx
ik
eAx
k
nnn
=


+

=


+
=
+
=
+
=
++
+++
βα
β
βα
β
ψ
( )[ ] ( ) ( ) ( ) anlxalanlxaln++?,1
这里是周期函数,其周期和势的周期相同,即 )(xu
k
)()( xulxu
kk
=+
由此,电子波函数具有 )(
)(
x
+
ψ
()
)()(
)(
x
ikl
elx
++
=+ ψψ (3.4)
这里k称为Bloch波矢,这种波函数(Bloch平面波乘以适当的和势同周期的周期函数)称为Bloch波函数。的情况类似。这一结论具有普遍性。这就是说,尽管周期势具体形式不同将导致
)(
)(
x
ψ
( )
)(x
±
ψ的具体形式不同,但这些)(xψ都满足上式。这就是说,电子在任一周期势中的波函数为Bloch波矢的平面波乘以适当的(与周期势的周期相同的)周期函数。
这称为Floquet定理。
最后还应当指出,这两个波是线性无关的,除非
()
)(x
±
ψ πmkl =。当πmkl =时,这两个解相同,都代表着驻波。这是因为由
16
=?+
+=
1
sincoscos
2
2
2
1
2
1
1111
ββα
βα lklkkl
代入πmkl =,将第一个方程平方,再将第二个方程代入后再开方,即得
21111
cossin ββα ±=? lklk
于是
()


=

±
=
+
])
2
1
([sin
])
2
1
([cos
1
2
12
))1(()(
)(
1
1
2
22
)( 11
lnxki
lnxk
lk
i
e
lnx
ik
e
nlx
ik
e
iknl
ex
nm
β
ββψ
这是驻波解。由于解对应于)(
)(
x
ψ ( )
*
+?
= ωω,故
[ ]
*
)()(
)()( xx
+?
= ψψ (3.5)
这两个驻波解相应于能隙上下限的带边情况。
4,均匀势场中的运动
这是如下一些问题的概括,
重力场中的粒子
a
和均匀电场中电荷q的粒子。当重力方向和电场强度E
v
的方向指向-x
时,势能均为
FxxV =)(
这里或mgF = Eq
v
。这两个势能里的零点(也即x坐标的零点选择)可根据题意选定。所选的V的零点不同只相当于系统总能E的零点定义不同,并不给问题带来实质性的变化。
考虑如图的问题。在处,有一刚性墙壁(在此处势突变升至)。
0
xx?= ∞+
dingeroSchr &&方程为
0)()(
2
)(
2
=?+′′ xFxE
m
x ψψ
h
(3.36)
引入无量纲参数

=
F
E
x
mF
3/1
2
2
h
ξ
当0=ξ时,
F
E
xx ≡=
1
,从经典观点来看,这是能量为E的粒子所能达到的最大高度。
a
关于量子力学和重力、非惯性参照系及等效原理的讨论可见 张永德、裴寿镛,量子力学与引力,大学物理,第11卷,第4期,第1页,1992年。
17
这就是说,以此点为分界线,0?ξ为经典不容许区域;此界线以下为经典容许区域。在这样的自变数替换下,方程变为Airy方程 dingeroSchr &&
)(ξAi (ξBi
=) α(ξψ
)
()
()
=?
=
ξα
π
α
ξα
Ai
Ai
0)()( =?′′ ξξψξψ (3.7)
它的两个线性无关解、)均称为Airy函数
a
,其中,由于,
不符合物理边条件。故舍去(这是因为,当
+∞→?
+∞→ξ
ξ)(Bi
+∞→x时,+∞→)(xV,必有0)( →xψ,所以在[区域内有限且平方可积的解为 ),
0
+∞?x


+=
0
3
3
cos
2
)( duu
u
Ai ξ
π
ξ
这里α为[区间上的归一化系数。为运算方便可区分,
0
+∞?x ξ大于小于0,分别将()ξψ写为
() (3.38)
+
=
0,
2/3
3
2
2/3
3
2
3
0,
2/3
3
2
3
3/13/1
3/1
ξξξ
ξα
ξξ
ξ
ξψ
JJ
K
应用()0
0
=? xψ的边界条件,可得能量本征值
0
3
2
3
2
2
3
0
3
1
2
3
0
3
1
=
+
ξξ JJ (3.9)
上式中的
0
ξ含有E,所以是个决定能量E本征值的方程式,其根的集合即为此问题的能谱。
注意,由于处的边界条件已将粒子局域化,所以能谱呈现出分立的情况。
0
x?
讨论,
i,如上所述,[]0,
0
ξ?区域是经典容许区域。现计算向经典趋近时,此区域中粒子在不同位置出现的几率分布P(x)。这时可假设,也即0→h +∞→ξ,利用贝塞尔函数的渐进表达式)(zJ
v
b
,
a
关于Airy函数可参见M.Abramowitz,I.A.Stegum,“Handbook of Mathematical Functions”,P.446,Dover
Publications,Inc.,New York; Л.Д.朗道等著“量子力学(非相对论理论)“,上册,附录b;
J.Phys.A.:Math.Gen.15(1982)L463-L465; J.Phys.A.:Math.Gen.16(1983)L451-L453等; 泡利物理学讲义,5。波动力学,第110页,人民教育出版社,1983。
b
M.Abramoutitz and I.A.Stegun,Handbook of Mathematical Functions,P.364。
18
→?
+∞→
42
cos
2
)(
ππ
π
v
z
z
zJ
z
v
代入上面()ξψ在0?ξ区域中的表达式,得
()
+=
=
+
+
→?
+∞→
43
2
sin
43
2
cos
463
2
cos
463
2
cos
3
2
2
3
2/34/1
2/34/1
2/32/3
2/3
π
ξξ
π
α
π
ξξ
π
α
ππ
ξ
ππ
ξ
ξπ
ξα
ξψ
ξ
由于势场V(x)随x变化,这相当于折射率n为非均匀的介质中光波波色的运动
a
,这时等效波长)(xλλ =是位置x的函数,
)(
2
)(
3
2 2/3
x
x
xxk
λ
π
ξ =?=

()( )
(
10
2/3
1
2/1
2
3
)( xxx
xxmF
x
x
=

λ ) (3.40)
当时,0→h () 0→xλ,发生快速振荡。将此快速振荡抹平,也即在宏观尺度下讨论位置分布几率时,需对很多个λ的空间范围取平均。这样一来,
xx
xxP
∝∝
+==
1
2/3
2
2/1
2
2
11
43
2
sin)()(
ξ
π
ξξ
π
α
ψ
(3.41)
另一方面,按经典观点有
1
2
2
1
FxFxmv =+

()xx
m
F
v?=
1
2
并且在内找到此经典粒子的几率正比于它在此处的速率,dx
a
E,费米,量子力学,第1、2章,西安交通大学出版社,1984年。
19
xx
v
xP
∝∝
1
11
)( (3.42)
ii,对0>ξ的区域,按经典观点是禁止区域。但按量子力学,仍能有一定的几率在此区域内发现粒子。这显然仍是物质粒子徳布罗意波波动性的表现,是纯粹的量子效应。当
+∞→x即+∞→ξ时,由于
()
z
e
z
zK
z
v
→?
+∞→
2
π
由()ξψ在0?ξ区域的表达式可得
()
2/3
3
2
4/1
2
ξ
ξ
π
α
ξψ
ξ
→?
+∞→
e (3.43)
于是在此区域内的几率分布随x增加而迅速衰减。这显然是由于外场的势能呈线性增长并变化很大的缘故。注意此区域情况并无经典类比,不存在向经典过渡的问题,仅当

0→h
+∞→ξ)时,() 0→ξψ,成为经典运动的禁区。
iii,现在来研究取消处刚性墙的约束而出现的现象。这时可认为,利用的渐进表达式,可将前面的能量本征值方程简化为
0
x? +∞→
0
x
)(zJ
v
0
43
2
sin
2/3
0
=
+
π
ξ
也即
)1,1(
43
2
0
2/3
0
=+ nn或等效的当ξπ
π
ξ
于是
()1,
4
19
2
1
)(
00
3
2
3
1
222
0
∞→?
= nxFxn
m
F
xE
n
并且

这里n的选取要保证。此式表明,当0)(
0
xE
n
∞→?
0
x时,只要n也足够大(从而
0
ξ便足够大),E的这个近似表达式便成立。由于是待定的参数,x和都是独立变数
(只是n的变化只限于整数范围而已,但
)(
0
x
n n
E
0
n
0→?
∞→nn
dn
dE
),于是→
+∞

→+∞→ n
n
E
,
(?
x
0
x
0
)
20
连续变化,从而过渡到连续谱情况。详细可参见文献
a

§3.2 一维定态的一般讨论
有了上面一些具体认识,现在来研究一维定态的一些普遍特征。除了第二章中所讲的方程的普遍特征之外,一维方程还有如下几个一般性结论。 dingeroSchr && dingeroSchr &&
1,本征函数族完备性定理
[定理1] 一维哈密顿量)(
2
2
xV
m
p
H +=中,若V在任意态中的平均值有下界,即对任给的一个单值、连续、可微(除有限个孤立点外)、平方可积函数
)(x
)(xψ,若存在一个不依赖于ψ的常数 c,使得
cdxxxxVV ≥=

)()()(
*
ψψ (3.4)
则此哈密顿量的本征函数族是完备的。(这相当于证明了此系统的能量是可观察量。)
证:这只需证明此系统的哈密顿量H有下界、无上界,然后应用前面叙述的完备性定理即可。
事实上,H是有下界的。因为按定理条件,有
cVVTH ≥≥+=
此式对任意态均成立。同时,H又是无上界的。因为,如果取
2
2
)(
λ
ψ
x
ex
=,则
2
2
2
2
2
22
2
2
22
2
2
2
2
2
2
2
λ
λ
π
λ
π
λ
λλ
m
m
dxe
dxe
dx
d
m
e
T
x
xx
h
h
h
==
=


∞+
∞?
∞+
∞?

当0→λ时,+∞→T,于是不论V有无上界,H将随0→λ而无上界。
讨论:1,此定理有一个特例,即势V(x)作为x的函数有下限Vmin
b
。在这种情况下,
显然导致H有下界无上界的结论。这将包括简谐振子等重要情况,然而却未能概括库伦势场等重要情况。后者如采用绝热近似机制计算库伦势的积分,就可以概括在这里的定理1
中。这就是说,定理1表明,库伦场的波函数(包括正能量的连续态)族也是完备族。
注意,此处的定理比文献
c
中的斯特姆—利欧维方程本征函数完备性定理要宽松得多。
a
朗道,非相对论量子力学,上册,第92页,高等教育出版社,1983年。
b
这就是李政道著“场论与粒子物理学”,上册,第13页例1的论断。它现在是本定理1的一个特例。
c
柯朗。希伯尔特,数学物理方法(I),科学出版社,1958年。
21
那里要求:势函数( )xV在定义域内为连续函数。
2,束缚态存在性定理
[定理2] 在一维哈密顿量)(
2
2
xV
m
p
+
)(x
中,若非常数势V(x)满足,i,V,ii,
,iii,对任意波函数
0)( =±∞
0)( ≤xV ψ,有常数c存在,使得

≥ cdxV
*
ψψ,则此系统至少存在一个束缚定态。
证:设势阱V(x)如图。
在此势阱内,总可以选取一方势阱
()
<<?
=
其他,0
,
)(
0
1
bxaV
xV
使得V对所有x值均成立。 )()(
1
xVx ≥
众所周知,在此方势阱V中,至少有一个束缚定态)(
1
x )(x?存在,它对应的本征能量为负值,若令
0
E
(xV
1
+ )
m
p
H
2
1
2
=,有
() () ( ) ( ) 0
00
*
1
*
<==
∫∫
EdxxExdxxHx
于是在()x?态下,
() () () ()
() () 0)(
2
)(
2
01
2
*
2
**
<=
+≤
+=

∫∫
EdxxxV
m
p
x
dxxxV
m
p
xdxxHx


同时,按前面的定理1,H的本征函数族{ })(x
n
ψ是完备的(这里为方便,只写断续情况。若谱是连续或包含连续,则下面推导只需将相应的求和化为积分即可)。于是可写
( ) ( )

=
n
nn
xcx ψ?
所以
( )()




<==
n
nnn
mn
mmnmn
dxHcEccdxxHx 0
*
2
,
**
ψψδ
可见至少有一个态()x
i
ψ,满足
( ) ( )

<= 0
*
iii
EdxxHx ψψ
这个定态()x
i
ψ对应负能量,当±∞→x时,( ) 0→x
i
ψ,是个束缚态。证毕。
22
讨论:此定理意味着,在这种一维势场中运动的粒子,其能谱一定包含分立的负值部分
a

3,无简并定理
[定理3] 若一维势V(x)在有限x处无奇点,则对应的全部束缚定态都是不简并的。也就是说,此一维问题的分立能级均无简并存在。
证:假定有两个束缚态
1
ψ、
2
ψ,均对应同一分立能级E,则
()
()
=′′
=′′
2
2
2
1
2
1
2
2
ψψ
ψψ
VE
m
VE
m
h
h
由此得
0
2112
=′′?′′ ψψψψ
将此式作不定积分,得
的常数不依赖于x=′?′
2112
ψψψψ
由无穷远处0
21
==ψψ,定出此常数为零。于是
2112
ψψψψ ′=′
或再积分一次,得
( ) ( ) cxx ′+=
21
lnln ψψ
于是
( ) ( )xcx
21
ψψ =
这里c是常数。从而按波函数的含义,( )x
1
ψ、( )x
2
ψ代表同一个束缚定态。上面第二次积分中假定了()x
1
ψ和()x
2
ψ无节点存在。如果存在节点,可在两个最靠近的节点之间的区间内应用上面的结果,接着利用在节点处两侧
1
ψ、
1
ψ′、
2
ψ、
2
ψ′连续(这由V的正规条件所保证),可得在节点的两侧所得常数c相等。从而定理仍成立。证毕。
讨论:i,需要强调指出,这里并未涉及正能量的非束缚态。对非束缚态(比如周期势),
结论并不成立,详见
b

ii,这个一维束缚态无简并定理有一个简单的推论:一维束缚态波函数总可以取成实函数。这是因为,H是实的,若()xψ是解,( )x
*
ψ也必是同样解。又由于非简并,要求两态相同
a
可用微扰论方法定出一维浅势阱中的能级。参见朗道“量子力学(非相对论理论)”上册,第197页。高教出版社,1983年。
b
柯朗。希伯尔特,数学物理方法,第227页,科学出版社,1958年。
23
( ) ( )xcx ψψ =
*
考虑到ψ、均是归一的,c只能是个相因子
*
ψ
δi
e。于是可得
() ()xexe
ii
ψψ
δδ
2
*
2
=
可见,只要取新的波函数
() ()
+=Ψ
ψψ
δδ
2
*
2
2
1
ii
exex
即得实的归一化了的波函数。由此可知,以前的一维问题中,( )xψ上的复数共轭记号其实是多余的。
4,零点定理
[定理4] 如将一维问题的分立谱波函数( )x
n
ψ按其本征值递增顺序编号,则属于第个能级的本征函数
1+n
n
E ()x
n
ψ,在其定义域内有限x值处共有个零点。 n
证明参见文献,因为一维方程即是该处所研究的Sturm-Liouville型方程的特例。
dingeroSchr &&
讨论:i,应当指出,对二维、三维问题,分立谱(0<E)的基态也无零点。这对任意有限维问题均成立。详见上面文献
a
第346页。
ii,从分立谱基态无零点这一结论出发,可以证明二维、三维情况的分立谱基态也是不简并的。因为,如果是简并的,便有至少两个不相关的本征函数( )q
)1(
0
ψ、对应同一个能级,而它们的线性组合也属于同一能级。但调整组合系数、c,
总可以使(这N维空间中)任一给定点成为零点(确切些说,成为零值的超曲面)。这是和原来假设相矛盾的。 证毕。
()q
)2(
0
ψ
c
0
E
)2(
02
)1(
01
ψψ cc +
0
q
1 2
这一推论也可表述为:在方程情况下,不存在真空态自发破缺。这里的真空态就是基态,是相对于激发粒子(准粒子)来说的空态。
dingeroSchr &&
§3.3 一维高斯波包的自由演化
这里用高斯波包自由运动作为例子进行随时间演化的计算。
设初始时刻给定的波包为高斯型波包,即
()
()
2
2
0
2
0,
σ
ψ
xx
ex
= (3.45)
研究它在自由运动情况下随时间的演化。
a
柯朗。希伯尔特,数学物理方法(I),第348页,科学出版社,1958年。
24
按第二章所述,第一步是将此( )0,xψ展为平面波,即
() ()

=
p
dp
xip
epx
h
h
/
2
1
0,ψ
π
ψ
于是
() ()


=
x
dx
xip
exp
h
h
/
0,
2
1
ψ
π
ψ
第二步是将(0,x )ψ展开式中的每个平面波(它们均是自由粒子运动的定态方程的解)乘以时间因子
dingeroSchr &&
t
m
p
i
e
2
2
h
,于是即得
() ()
() xddp
t
m
p
i
xxp
i
ex
dp
t
m
p
i
xp
i
eptx
px
p


′=

=
∫∫


2
2
)(
0,
2
1
2
2
2
1
,
hh
h
hh
h
ψ
π
ψ
π
ψ
代入(0,x′ )ψ表达式并完成对p的积分,得
()
()
()
()
() ()
′+?′′

=


′=



22
0
2
2/1
2/1
22
1
exp
2
1
2
2
2
1
2
2
2
0
,
xx
t
im
xxxd
t
mi
xx
t
m
i
e
t
mi
xx
exdtx
x
hh
h
h
σπ
π
σ
ψ
计算中用到了富耐涅尔积分
(i
iu
edu ±=
±

+∞
∞?
1
2
2
π
) (3.46)
完成对积分,最后得到 x′
()
( )
()?
=
2
2
0
2
2
exp
2/1
22
1
2
1
,
σσ
ψ
imt
xxim
t
im
t
mi
tx
hhh
于是,演化到时,原先峰高为1、峰宽为t σ(在此宽度内峰的面积为整个峰面积的68.3%)
的几率分布
2
()
2
0,xψ
25
()
( )
=
2
2
0
2
exp0,
σ
ψ
xx
x
变成为峰高
2/1
1
42
22?
+
σm
th
,峰宽为
2/1
2
22
22
2
+
σ
σ
m
th
的分布
()
()
+
+=
22
22
2
2
0
42
22
2
exp
2/1
1,
σ
σ
σ
ψ
m
t
xx
m
t
tx
h
h
(3.47)
就是说,波包的高度逐渐变矮,宽度逐渐加宽。这就是所谓的“波包弥散”。这说明,粒子的德布罗意波即使在自由传播时仍存在色散(这和光波在真空中传播时无色散呈鲜明对照)。
这一结论决定性的否定了把粒子看成纯粹的波包的偏颇观念,说明了不能用波动学说片面代替波粒二象性,正如同不能按经典观念用粒子学说片面代替波粒二象性那样。
最后,计算此高斯波包的位置和动量的不准度。
设此高斯波包的平衡位置以匀速
m
p
0
向正x方向运动,于是(不计时间相关因子
t
m
p
i
e
2
2
0
h
)此波函数为
()
( )
h
xipxx
ex
0
2
2
2
0
+
=
σ
ψ
这里,t
m
p
x
0
0
=,于是
()
()( ) ()
()()
()
()
()
2
2
2
0
*
2
0
*
2
0
2
2
2
0
2
2
2
0
σ
ψψ
ψψ
σ
σ
=
=
=?




∞+
∞?
∞+
∞?
∞+
∞?
+∞
∞?
dxe
dxexx
dxxx
dxxxxx
xx
xx
xx
(3.48)
26
()
() ()
()()
() ()
()
()
2
2
2
0
4
2
2
2
2
00
2
2
2
*
2
0
*
2
0
2
1
2
2
2
0
0
2
2
2
00
2
2
2
0
σ
σσ
σπ
ψψ
ψψ
σ
σσ
h
hh
hh
h
hh
=
=
++?
=

=?




∞+
∞?
∞+
∞?
+
∞+
∞?
∞+
∞?

dxexx
dxep
dx
d
pi
dx
d
e
dxxx
dxxp
dx
d
ix
pp
xx
xxxx xipxip
(3.49)
最后得到
2
h
= px
这说明继谐振子基态之后,高斯波包(及其演化态)也是一个相干态。
27
第四章 中心场定态问题
§4.1 前言
自然界存在着各式各样的相互作用,有的只决定于两体的两两(两体)相互作用,也有的决定于三个物体的三体相互作用等等。但自然界中最常见的是两体相互作用。如多个电荷之间的相互作用总可以拆开成为两两电荷间的两体相互作用 —— 库仑相互作用,多个天体之间的引力作用也总可以分解为一系列两两之间的两体相互作用 —— 万有引力相互作用,如此等等。
两体之间的相互作用,最简单的情况是只决定于这两个物体之间连线的长度,即它们的相互作用势可写成VVrr Vr=?=(| |) ( )
vv
12
vv
rr
12
,如库仑作用、引力作用等均如此。一般些说,
两体相互作用V应当是的函数,即VVrr=?()
v v
12
。在量子力学中,这种两体问题由下面的哈密顿量决定
)(
22
)(
22
2
2
2
1
1
2
21
2
2
2
1
2
1
rV
mm
rrV
m
p
m
p
H
vhh
vv
+=
++=
(4.1)
这里
2
2
2
2
2
2
iii
i
zyx?
+
+
=?,i 2。由于两粒子间的相互作用,V中耦合了两个粒子的坐标,体现了它们运动之间的关联。和经典力学十分相似,量子力学中的两体问题也可以通过引入它们的质心坐标和相对坐标
= 1,
a
,把它们作为整个体系的质心运动和彼此相对运动这两部分运动分离开。也即令
v
v v
R
mr mr
mm
=
+
+
11 2 2
12

v v v
rrr=?
21

H
M
Vr
Rr
= +
hh v
22
22

μ
() (4.2)
这里
Mmm= +
12
,μ =
+
mm
mm
12
12
M是总质量,μ是折合质量。注意,经这样代换之后,哈密顿量H被分成相互不关联的两项之和。这里HH H=+
Rr
H
M
RR
=?
h
2
2
,H。按照分离变量的观点可以得出,当
Vr
r
=? +
h v
2

)
r
(
H可以分成互不关联的几部分之和时,相应的能量本征值便可以分成互不关联的几部分之和,而波函数便能分解成互不关联的几部分之积。因为,这时可令
ΨΨ(,) (,) () ()
vv
v
v
v
v
rr Rr R r
12
==ψ
于是这个两体系统的方程成为 dingeroSchr &&
a
这是Jacobi坐标在两粒子情况下的特例。一般多粒子系统的Jacobi坐标参见布洛欣采夫,量子力学基础”,俄文第581页。
Hrr ErrΨ Ψ(,) (,)
v v v v
12 12
=
v
HRrHRr rHR RH
Rr
ψ?ψ ψ( ) () ( ) () () ( ) ( )
v
r
v
v
v v
v
+=+=ER r?ψ()()
v
v
(4.3)
v
v
等式两边同除以?ψ()()Rr,得
11
ψ
ψ
()
()
()
()v
v
v
v
R
HR
r
Hr+
v
E= (4.)
v
左边两项分别属于独立坐标R和r,因此必定各自等于常数、,它们的和为E
R
E
r
E。即得
==+
=
)()()()()()(
2
)()(
2
2
2
rEErErrVr
RER
M
Rrr
RR
vvvvvh
vv
h
ψψψψ
μ

(4.5)
第一个方程表明,这两个相互作用着的微观粒子,作为一个整体(用它们的质心坐标代表)是自由运动,因为作为一个整体,并没有受到外界的作用。第二个方程表明,两体的相对运动,
当相互作用只和它们之间的连接矢量
v v v
rrr
21
=有关时,可以转化为单体运动,这时只要将质量替换成折合质量即可。通常把关于质心坐标
v
R的运动称为运动学问题,因为它不涉及相互作用;而把关于相对坐标
v
r的动力学问题,因为它包含了相互作用。通常对不含相互作用的运动学问题不感兴趣,只对包含相互作用的动力学问题感兴趣,后者这时将转化为中心场V r()
v
中的单体运动问题。由于采用这一坐标和折合质量概念,以下所研究的中心场问题,
既包容了两粒子质量相差很大,以致对轻粒子而言,重粒子构成了不动的力心这一情况,也包容了两粒子质量相差不很大这一情况。总之,在得出两粒子相对运动之后,再结合它们的质心运动就能构成这两粒子体系运动的完整描述。
§4.2 轨道角动量及其本征函数
在许多常见情况,如库仑势和各向同性谐振子下,V r()
v
简化为相对于坐标原点各向同性的中心势V r()。将前面关于ψ()
v
r的方程中的EE
R
记为E并略去?
r
的脚标,描述相对运动的Schr方程成为 dingero&&
H r E rψ ψ() ()
v v
=
H=? +
h
2

()Vr (4.6)
v v
这时,由于? =,正如r r r
2
=?一样,在绕原点转动下表现为一个标量,即不变化。势V r()显然也如此。可以证明,这时轨道角动量
v
L和是守恒量。比如对,将L
2
L
2
H
和均采用球坐标表述,即可清楚地看出这一结论,L
2
H
r r
r
L
r
Vr=? + +
h
22
2
2
2
2
11

μ
()
L
222
=h
(,)
,
θψ
= +
(,)
sin
sin
sin
θψ
θ
θ
θ
θ θ

2
2
2
2
11
(4.7)
显然,它们是对易的
[ ] 0,
2
=LH
由第二章中力学量守恒的叙述可知,在中心场V r()中运动的粒子的轨道角动量平方是个守恒量。同样论证可得,的三个分量也是守恒量,因为在球坐标中,,,同样
L
2
L
z
v
L L
x
L
y
均只涉及对θ、ψ的偏导数,并不涉及对径向r的偏导数。而且由下面可知
[

]
LL
i
2
0,=
zyxi,,=。
=
=
=
v
L
]xy
==
L
iL
xy
]
=
]
L
2
v
L
L
2

=
α
)
if
ml)(,θψ?
)
m
Y
这里,先来讨论一下的本征函数和本征值问题。 L
2
经过计算可得
[ ]
LL iL
xy
,h
z
[ ]
LL iL
yz
,h
x
[ ]
LL iL
zx
,h (4.8)
y
或简记为
v v
hLLi×= (4.9)
由此可得
[][ [ ]
LL LL LL
z
222
0,,,(4.10) =
有时为了方便,也引入如下复合算符来代替和,L
± x
L
y
LL
+
= +,LLiL
x? y
=? (4.1)
这时可得
[
LL L
z+?
,2h (4.12)
[
LL L
z
,
±
=±h
±
L
z
(4.13)
LLLL
z
2 2
=++
+?
h (4.14)
由上面对易关系看出,的任何两个分量彼此均是不对易的,按第一章中的测量公设的叙述知道,决不能同时测准轨道角动量
v
L
v
L
L
的三个分量中的任何两个。或者说,不存在这种状态波函数,它既是的本征态,又是的本征态,等等。但是,L
x y
v
L
2
和三个分量均对易,所以,和中的任一分量可以同时测量。于是,可以寻找这样的状态波函数,它既是的本征函数,又是的本征函数。就是说寻找函数L
z
f (,)θ ψ,使下面两个方程都成立
Lf f
2
Lf f
z
β (4.15)
这里、β是相应的本征值。用球坐标表示
=h
22
(,θψ
αff
=h

β (4.16) f
满足这两个方程的解为
fY
lm
lm
l
Pe
lm l
mim
) (,)
()!
()!
(cos ),(| |θψ ε
π
θ
==
+
+

21
4
(对,,对mm > 0 ε =?(1 < 0,ε = 1) (4.17)
相应的本征值,α =+ll()1
2
h β = mh。这里
lm
(,)θ?是归一化了的球谐函数,其中缔合勒让得多项式用Ferrer的定义
Px
l
x
d
dx
x
l
m
l
m lm
lm
l
()
!
() ()=
+
+
1
2
2
2
2
1ml,(||≤ )
a
(4.18)
此外还有
Px Px
l
mlm
l
m
()() (?=?
+
) (4.19)
Px
lm
lm
Px
l
mm
l
m=
=?
+
() ()
()!
()!
() (4.20)
注意球谐函数在球面上是正交归一的
YY d
l m lm ll mm′′

∫∫
=(,) (,)sinθ? θ? θθ? δ δ
ππ
00
2

2
(4.21)
并且
YY
lm
m
lm
=?(,) () (,)
,
θ? θ? (4.2)
综上所述可得
LY ll Y
lm lm
2
1(,) ( ) (,)θ? θ?=+h (4.23)
LY m Y
zlm lm
(,) (,)θ? θ?= h
l
mll ll
=
=+
012
1101 1
,,,.......
,,...,,,,...,,.
(4.24)
这里称为轨道角动量量子数,m称为磁量子数。对一个给定的,m可以取()个不同的值。对应个不同的态。球谐函数的有关计算参见附录7。
l l 21l +
(21l + )
§4.3 几个一般分析
上面论述了中心场V r()情况下,轨道角动量
v
L守恒,从而波函数的(,)θ?部分为
Y
lm
(,)θ?并且lm是守恒量子数,可用于态的分类。在求解一些具体的中心场问题之前,这里再进行一些不依赖于V r()具体形式的一般讨论。
1,量子数简并和离心势 m
球坐标下的方程为 dingeroSchr &&
111 12
0
2
22 2
2
22
r r
r
r
EVr
ψ
θ
θ
θ
ψ
θ θ
ψ

μ
ψ() [
sin
(sin )
sin
][ ()]+++?
h
=
(4.25)
将波函数作变数分离
ψ θ? θ?(,,) () (,)rRrY
lm
= (4.26)
代入上面的方程,得
LY ll Y
r
d
dr
rR
ll
r
R
EVr
R
lm lm
22
2
22 2
1
112
0
(,) ( ) (,)
()
() [ ()]
θ? θ?
μ
=+
+
+
=
h
h
(4.27)
波函数的径向部分也可记为R
r
r
=
χ()
,相应χ()r的方程为
a
见郭敦仁著“数学物理方法”,第286、287页,人民教育出版社,1979年。此处的也即Abramowitz
书P.332中的。P还有另一定义,称Hobson定义,与此处相差因子。另外,Y
Px
l
m
()
m
Px
lm
() x
l
m
() ()?
lm
(,)θ?
还有另一定义,与此处相差一个因子(),见朗道“量子力学”,第112页。
||
mm
l
i
2
χ
μ
χ() { [ ()]
()
}()rEVr
ll
r
r′′+
+
=
21
0
22
h
(4.28)
这里强调指出两点。第一,径向波函数方程中未含磁量子数m,于是,由此方程得出的E的允许值中将不包含m。这就是说,中心场V r()的能级是关于磁量子数简并的,简并度为重。这是因为现在的问题是绕O点转动对称的,并无特殊方向可言,现在的轴只是人为选取的,实际也不应特殊;从而轨道角动量对轴投影的大小不应影响系统的能量。这也就是说,若要解除这种简并,必须再加外场以破坏现在的绕点的各向同性性质。第二,正如从
m
(21l + )
z z
O
χ()r方程中所见到的,r方向的有效势为
VVr
ll
r
eff
=+
+
()
()1
2
2
2
h
μ
(4.29)
这里第二项
ll
r
()+1
2
2
2
h
μ
(只当轨道角动量不
为零时才存在)常称为离心势。原因是它在
r = 0附近构筑了很高的势垒,产生自中心
向外的斥力,使粒子在r = 0附近的
存在几率明显下降
a
,而且越大这种 l
现象越突出。这和经典图象是一致
的。经典力学中有心力场的有效势的表达式也是如此
b

2,径向波函数r → 0时的边界条件
c
总共存在三种。即
i。有限,或平方可积。 ||
[]
ψ
22
rdrd
O
=

|()|χ rdr
2
0


ii。,或。 r
r
ψ


0
0 χ()r
r→

0
0
iii。ψ()0或R()0有限,或不慢于χ()r
r→

0
0 r。
这里的三个条件是不同的,而且一个比一个苛刻。到底应当用哪一种? 物理的和数学的根据如何?
从方程在直角坐标和球坐标中解的集合的等价性出发加以探讨,就可以解决这个不确定性。
dingeroSchr &&
众所周知,球坐标中的拉普拉斯算符在r = 0点是不确定的。从直角坐标转入球坐标时,
拉普拉斯算符是经过了除r (它的定义域包含零,为[,)0 +∞ )这种带有奇性的运算的。
于是,可以疑问,球坐标下的方程的解是不是都是直角坐标下同一方程的解呢? 事实上并非如此。以自由粒子的定态方程为例,
下面
dingeroSchr &&
dingeroSchr && dingeroSchr &&
a
由后面知,当r → 0时,R r()将以r。
l
→ 0
b
例如参见V.巴杰,M.奥尔森著“经典力学新编”,第114页,科学出版社,1981年。
c
这一节的内容详细可参见,张永德,ψ =
1
r
e
ikr
是自由粒子状态吗?——兼谈运算中的奇异性,大学物理,1989年第9期第1页。
ψ
μ
α
()r
r
e
r
e
i
E
r
ir
==
±
±
11
2
2
h
(4.30)
满足球坐标下能量为E的自由粒子方程 dingeroSchr &&
=
h
22
2

ψ
d
dr
rEr() (ψ) (4.31)
但这个ψ()r就不是直角坐标下能量为E的自由粒子的方程的解。因为代入dingeroSchr &&
1
r
e
irα
后会得到
=+
h
22
2
2
μ
π
μ
δ
αα
() () (
e
r
E
e
r
r
ir ir
h
) (4.32)
在代入验算时,上面方程右边含δ函数的第二项常常被遗漏了。右边第二项是否应当存在可用下面简便办法予以检验,即将此方程两边对任一半径R的球体进行积分,这时
左边 = =
=
∫∫∫∫∫
hh
v
22
m
e
r
dV
m
e
r
dS
ir ir
rRR
() ()
αα

= =?
=
∫∫
2
2
2
2
2
1
2
1
m
e
r
ir rd
m
iRe
ir
iR
rR
α
α
α
π
α() ( )? (4.33)
右边
=+=
∫∫∫ ∫
E
e
r
rdrd
m
Eredr
m
ir
R
ir
R
α
α
π
π
π
2
2
0
2
2
4
2

hh
+
=?
2
1
2
π
α
α
h
m
iRe
iR
()
(4.34)
于是,形如
1
r
e
ir± α
(包括零能量解
1
r
)的解,虽然是球坐标下自由粒子方程的解,并且在
dingeroSchr &&
r ≠ 0的区域也满足直角坐标下自由粒子方程,但毕竟(至少在dingeroSchr &&
r = 0附近)不是直角坐标下自由粒子方程的解。显然,一个真正的物理解不应受坐标系选取的影响,而应当在无论什么坐标系中都满足同一方程。
dingero&&Schr
由此,需要拟定在r = 0处的自然边条件,以便将这一类由球坐标拉普拉斯算符的奇性引入的额外的解排除掉。这就是r → 0时的边界条件ii
r
r
ψ


0
0或 χ()r
r→

0
0
的由来。显然,按物理的要求,只需条件I.即可,再进一步的更严的要求其实已是非物理的了。现在,根据直角坐标下和球坐标下解的两个集合必须等价这一数学要求,拟定了比条件
i更严的条件ii。至于比条件i,ii都要严的条件iii,则已是既没有物理的根据也无数学的根据了。
顺便指出,
1
r
e
ir± α
虽然不是全空间内的自由粒子球面波解,但还是可以把它作为渐近解用于渐近区域( r值较大的区域)中。正如后面散射理论中所做的那样。
3,径向解的完备性问题
注意到,现在的径向解χ()r方程很像以前所研究的一维问题。根据第三章第二节的定理1,只要下面积分有下界
(()
()
)[ () () ]Vr
ll
r
RrRrrdr C+
+



1
2
2
0
(4.35)
则径向方程的解序列便构成关于径向波函数的完备函数族。这里限定R r()是任意单值、连续、可微(除个别点)和平方可积的任意函数。C为某一常数。就一般常见的各种中心场问题而言,这是能满足的。比如各向同性谐振子势
Vr r()=
1
2
22
μω
由于V r()≥ 0,下限C可取为零;再比如球方势阱
Vr
Vr
ar
()
,
,
=
<<
<
0
0
0
a
下限C可取为。所以这里集中讨论吸引库仑势?V
0
Vr
Ze
r
()=?
2
由于l ≥ 0,所以
ll
r
()+

1
0
2
,上面不等式(4.35)左边第二项积分,不论此积分发散(视在零点的行为而定)与否,终归有利于总积分下限的存在。于是作为充分条件,只需要求
≥ 0
fr()
V rRrRrrdr V r r rdr C() () () () () ()
2
00
∞∞
∫∫
=χχ ≥ (4.36)
存在即可。这是成立的,因为由中心场的自然边界条件知道,作为径向波函数( R
r
=
χ
)要求其满足
χ()r
r→

0
0
从而这个积分在r → 0时收敛,下限存在。而对于由χ()r线性叠加而成的(平方可积)的任意,下限依然存在。总之,这里根据上一章的定理1证明了常见的中心势的径向波函数族是完备的。这里,只要求V
fr()
r()在r → 0处的发散不超过?1次幂,其对应的径向本征函数族都将是完备的
a

4,粒子的回转与角动量及波尔磁子
现在利用第二章中的ψ态下的流密度表达式
v
h
j
i
=


ψψψψ[]
来计算中心场V r()里粒子的流密度,并进而讨论有关的问题。
将梯度算子写入球坐标中
= + +
vv v
e
r
e
r
e
r
θ θ

θ?
11
sin
(4.37)
由于这时波函数中与r及θ有关的部分均为实函数,从而jj
r
= =
θ
0。就是说,中心场
V r()里的粒子只绕(其实是任选的)轴回转。这时 z
a
当然,如前面所说的,这个完备函数族应包括整个能谱范围,就是说,应包括库仑场中正能量的非束缚态解,这部分解的集合对应能谱的连续部分。
j
i
RY
r
RY RY
r
RY
lm lm lm lm?
μθ
θ

=?

h
2
11
[
sin
()
sin
()]
=≡
m
r
RY
r
m
lm
h
h
μθ
ψ
μθsin
||
||
sin
22
2
也即
v
h
v
j
r
me=
||
sin
ψ
μθ
2
(4.38)
根据这个的表达式先来 j
计算角动量的分量。是 z j
几率流密度,乘以粒子的质量μ
即为粒子的质量流密度(或称动量
密度),再乘以体积元dτ,即
成为dτ内粒子的动量,故方向 z
的角动量密度为
dL dL e e r j d
zzz
=?=?×
v
v v v
v
()μτ
v v
v
= ×? =() siner jd jr d
z
μ τ μ θ τ
(4.39)
这里
vv v
err e
z
×=sinθ
。对全空间积分后考虑到ψ是归一的,从而得到
Lm
z
= h (4.0)
这里之所以能用经典观念得出正确的量子结果,是因为使用了量子力学的几率流表达式,如前面一章中所说,这个几率流密度表达式是几率流密度算子在ψ
lm
态中的平均值,而以前论述过,量子力学的平均量与在其基础上的运算带有经典的性质。
如将乘以负电荷,就描述了氢原子的核外电子“云”的回转。这时表示电子云的电流密度,再乘以和
j
>ee(0)?ej
v
e
相垂直的面积元dσ,就是环形流动的电流元dI
dI ej d=?
σ (4.1)
此电流元对指向轴的磁矩的贡献为 z
dM
S
c
dI
z
= (4.2)
这里Sr,是绕轴环流的圆面积。由此,沿轴的总磁矩为 =πθ(sin)
2
z z
M
c
SdI
c
re
r
md
z
==
∫∫
1
22
2
π
θ
ψ
μθ
σsin ( )
||
sin
h
=? =?

e
c
mrd
e
c
m
hh
2
2

ψπ θσ
μ
ψ|| sin ||

dτ (4.43)
这里2π θ σ τrddsin =是截面为dσ,半径为r sinθ的“轮胎形”细环的体积元。最后得
Mm
zB
=? μ (4.4)
其中μ
μ
B
e
c
== ×
h
2
9 273 10
21
.尔格/高斯,称为波尔磁子。和之比为 M
z
L
z
M
L
e
c
z
z
=?

(4.5)
是个常数,称为电子的轨道回磁比。于是,由于轨道角动量是量子化的,磁矩也是L
z
M
z
量子化的并决定于量子数。这是被称为磁量子数的由来。这里补充指出,(按相对论量子力学结果),电子的自旋回磁比(电子内禀磁矩和自旋角动量之比)是
m m
e

,于是这两个回磁比的比值等于2。关于这个比值的讨论参见后面第7章。
ra
a
+
+
r
r
kE2μ
)krA=
kr′
=
2
()
§4.4 球方势阱问题
考虑一有限球形势阱
Vr
Vr
()
,
,
=
<
>
0
0
分以下几个问题讨论。
1,束缚态( EV<
0
)问题
′′ ′+?
+
=<
′′ ′+ ′?
+
=>
R Rk
ll
r
Rra
R Rik
ll
r
Rr
21
0
0
2
2
2
2
[
()
].()
[( )
()
].(
(4.46)
a)
这里
=
1
h
,′ =?kV
1
2
0
h
μ()E
第一个方程是球贝塞尔方程,后一个方程是修正的球贝塞尔方程(Modified Spherical Bessel
Equation)或虚宗量贝塞尔方程,它们的解如下
)(),(
2
)(()(
2
1
arkrJ
kr
AkryAJrR
l
kllkllkl
<=′+
+
π
(4.47)
Rr B KkrB
kr
Ikr
kl
l
kl
l
() ( ) ( )′ + ′



+

+
ππ
22
1
2
1
2
,(r a> (4.8) =



+
B
kr
Kkr
kl
l
π
1
2
() )
这里,r a<的解中已略去第二类球贝塞尔函数
ykr
kr
Ykr
l
l
() ()=
+
π
2
1
2
(()
!
{()y
l
O
l
l
ρ
ρ
ρ
ρ→
+

+
0
1
21
1 ),r a>的解中已略去
π
2
1
2


+
kr
Ikr
l
()
,因为它们分别在r = 0和r = ∞处发散,不满足波函数的物理条件。
}
利用r a=处波函数及其微商连续,得到决定本征值E的超越方程
(4.49)
ππ
ππ
22
22
0
1
2
1
2
1
2
1
2
ka
Jka
ka
Kka
kr
Jkr
kr
Kkr
ll
l
ra
l
ra
++
+
=
+
=




′′
=
() ( )
(()( ()
例如,对l = 0这一特例,由于
a
ππ
22
1
2
1
2
z
Jz
z
zz
Kz
z
e
z
()
sin
,()==
π
2
` (4.50)
上面超越方程转化为
sin( )
() (
sin
)
ka
ka
k
Z
e
ka
ek
Z
Z
Z
Zka
ka
Zka
′′?

′ =
= ′

=
π π
22
0 (4.51)
整理即得
tan( )ka
k
k
=?

(4.52)
这一结果和一维半壁无限高方阱结果完全相同。这是必然的,因为这里的χ()r方程和一维半壁无限高方阱的方程完全相同(这时l = 0,在r = 0处χ() ()|0
0
= =
=
rR r
r
=
=
A
k
kr
r00
1
0sin( )|,与在r = 0有一无限高势垒效果相同)。于是那里关于能谱和波函数的结果可以全部移至此处的能谱和χ()r上。特别地,至少有一个束缚态存在的条件是
Va
0
2
22
8

π
μ
h
(4.53)
2,无限深球方势阱
这是上面有限深球方势阱的极端情况,即V
0
→∞,也即′→∞k。所以
Kkr
l+
′ →
1
2
0()。于是在r a=边界上,
jka
l
()= 0 (4.54)
假定的第n个根为,即,则能量的本征值为 j
l
α
n
l
n
()
,(,,,......)= 123 j
ln
l
()
()
α = 0
E
k
a
n
nl
n
l
n
l
== =
hh
222 2
2
2 2
12
() ()
,(,,......)
() ()
μ
α
μ
(4.55)
相应的径向波函数为
Rr Njkr
nl nl l n
l
() ( )
()
= (4.56)
归一化系数可如下求出
1
22
0
22 2
0
==

∫∫
Rrrdr N jkrrdr
nl nl l n
l
a
() ( )
()
=? =
+ +
N
a
jj N
a
j
nl l n
l
ln
l
nl l n
l2
3
11
2
3
1
2
[ ( )( )] [( )]
() () ()
αα α
于是
N
aj
nl
ln
l
=
+
2
3
1
2
()
()
α
(4.57)
这里利用了数学公式
b
a
分别参见M.Abramowitz,et.al.,Handbook of Mathematical Functions,第438页、第444页。
b
参见王竹溪、郭敦仁著“特殊函数概论”,第469、471页,科学出版社。
jx x dx xjx j xj x l
JJJ
l
x
lll
nnn
22
0
32
11
11
1
2
3
2
0
()() [ () () ()],( )
() (),( ())
() () ()
′′ ′=? >
=? =

+

ν
ν
ν
ν
ν
ααα当
(4.58)
3,自由粒子球面波解
这时a。由于不论为任何正值均成立,于是对的约束方程(即能量的本征值方程)消失,过渡到连续谱。这时波函数为
→∞ jk
l
()?∞ =0 k k
ψ θ? θ?(,,) ( ) (,)rAjkrY
kl l lm
= (4.59)
利用连续参量下的球Bessel函数归一化公式
a
jkrjkrrdr
k
kk
p
EE
ll
()( ) ( ) ( )′ =?′ =?′


2
0
2
3
2 2
π
δ
π
μ
δ
h
(4.60)
所以归一化波函数为(归一到δ()EE? ′ )
ψθ?
μ
π
θ?
klm
l
llm
ri
p
jkrY(,,) ( ) (,)=
2
3
h
(4.61)
这里添加的相因子i是为了以后考虑时间反演运算的方便。量子力学中有两组常用的自由粒子解,一组是平面波解,另一组便是这组波函数
l
b
。与平面波具有确定的三个动量分量不同,
这组定态具有确定的能量、角动量及其第三分量。由于
c
jkr
kr
l
Okr
l
r
l
()
()
()!
((


+
+
0
21
1 ),(4.62)
不论l是否为零,总满足r的边界条件。所以这一组解确实可称为自由粒子球面波解。显然,如同平面波集合一样,这组解的集合也是完备的。于是,两组解之间可互相展开,详见
r
ψ


0
0
d

4,非束缚态问题
这时,系统总能量处处超过势垒。方程的解为 EV>
0
Rr Aj kr r a
Rr Bj kr Cy kr r a
l
ll
() ( ),
() ( ) ( ),
=<
= ′ + ′ >
(4.63)
这里′ =
k
EV2
0
2
μ()
h
。r a=处的边条件为
Aj ka Bj k a Cy k a
Aj ka Bj ka Cy ka
ll l
ll l
() () ()
() () ()
= ′ + ′
′ = ′′+ ′′
(4.64)
这里第二个方程中微商对r进行。这里只有两个方程,但有三个待定系数。因此,若不添加
a
见J.R.Taylor,“Scattering Theory,The Quantum Theory on Non-relativistic Collisions”,P.183,John Wiley
& Sons,Inc,1972。或直接利用球Bessel函数方程,乘积后积分,采用球Bessel函数的边条件,即能算得。
b
原则上可以在无穷多种坐标系中写出自由粒子方程。只要满足相应的边条件便是真正的自由粒子解。所以应当有无穷多组自由粒子波函数族。
dingeroSchr &&
c
M.Abramowitz,et.al.,“Handbook of Mathematical Functions”。
d
J.R.Taylor,“Scattering Theory,The Quantum Theory on Non-relativistic Collisions”,P.183,或张永德,
大学物理,1989年第9期。
在无穷远处条件,将不存在决定E的允许值的本征值方程,所以是连续谱。和散射边界条件相应的散射问题的解参见第11章。
θ,
χρ
ρ
()+
u(
χρ()
χ
§4.5 库仑场——氢原子问题
1,方程及解 dingeroSchr &&
这时,Vr,
e
r
()=?
2
ψ? θ?(,) () (,)rNRrY
lm
=;径向方程为
d
dr
rR E
e
r
ll
r
rR
2
22
2
2
21
0()[ ( )
()
]( )++?
+
=
μ
h
(4.65)
并附带下面这两个边条件
Rr rRr
r
(),()
→∞ →
→0
0
(4.6)
为解这个方程,引入无量纲变量ρ和参量ε,
ρε==
r
a
E
I
,
2
(4.67)
这里a,称为波尔半径,
e
cm== ×
h
2
2
8
0 529 10
μ
,I
a
e
a
eV===
h
2
2
2
2 2
13605
μ
.,由下面知道,这是氢原子基态的电离能。于是χ ρ χ( ) () ()= =r rR r的方程为
d
d
ll
2
2
2
21

ρ ρ
χρ
()
[
()
]()++?
+
= (4.68)
为消去方括号内项系数,作函数变换(ρ
2
δ为待定系数)
χρ ρ ρ
δ
() ()= v
这时关于δ的指标方程
a
,δ δ δ? + =

10
12
pq变为
δ δ()() + =11ll 0 (4.69)
解为δ
1
1=+l,δ
2
=?l,取函数变换。接着,为消去χρ ρ ρ() ()=
+l
v
1
v()ρ前的2ε系数项,作函数变换。而指标方程变为 ve()= )ρ
βρ?
ρ
0
ββ
2
00
0++=pq
βε
2
2+= (4.70)
解为β=?2ε。总起来,作如下函数变换
ρ ρ β ε
βρ
(),( )==
+?l
eu
1
2? (4.71)
代入χ ρ()微分方程。这时
′ =
+
+

ρ
βχ χ()
lu
u
1
(4.72)
′′ =? + +
+
+
+

+
′′
χ
ρ
χ
ρ
βχ
ρ
βχ χ() ( ) ( )l
llu
u
u
u
1
11
2
1
2
2
(4.73)
于是得到
′′+
+
′?
+?
=u
l
u
l
u[
()
]
()21
2
212
0
ρ
β
β
ρ
(4.74)
a
这个方程是将广义幂级数形式的待定解代入微分方程后,令幂次最低的项系数之和为零 所得的方程。见附录2。
或除以,并令2()2
2
β βρ ξ=,uu() ()ρ ξ=,得
du
d
ldu
d
l
u
2
2
21
1
1
1
0
ξ ξξ
β
ξ
+
+

+?
=[
()
]
()
(4.75)
u()ξ方程的指标方程为δ δ δ()()?+ + =12 1l 0。所以,δ
1
0=,δ
2
21=? +(l )
a
。这正是下面合流超几何方程的特例( rl= +21(),α l
β
=+?1
1
)
′′ +? ′?Wz
r
z
Wz
z
Wz() ( ) () ()1=0
α
(4.76)
它的指标方程为δ δ δ()?+ =1r 0。所以,δ
1
0=,δ
2
1=? r。按合流超几何方程通解理论,当r ≠整数时,可得两个线性无关的独立解
Wz F rzWz z F r rz
r
12
1
12() (,,),() ( ),,)==?+
αα?
这里F r z(,,)α是合流超几何函数,定义为
Frz
kr
z
rk
krk
z
k
k
k
k
k
k
(,,)
()
!( )
()
()
()
!( )
α
α
α
α
==
+
+
=

=

∑∑
00
Γ
Γ
Γ
Γ
=+ +
+
+
+1
1
1
1
2
1
1
2
!!
()
()
......
α α α
r
z
rr
z (4.7)
但是,现在的情况是r l=+=22正整数,按合流超几何方程通解理论,在的邻域只得到一个独立的广义幂级数形式的解(当
z = 0
r = 1时,W与Wz相同;当z
2
()
1
() r ≥ 2时,Wz
无意义。现在是
2
()
r ≥ 2的情况)。这时,另一个线性无关解应为
Wz A zWz z Cz
l
m
m
m
21
21
0
() ln ()
()
=?+
+
=


(4.78)
这里第二项前z的幂次为δ
2
21=? +(l ),并且C
0
0≠
b
。但是,这个解应当放弃,它不满足r → 0时χ()r → 0的边条件。这是因为,按此解
χχρ
β
() () () ()r
r
a
eu
r
a
l
r
a
==
+
1
=
+
() ( )
r
a
eW
r
a
l
r
a
1
2
β
β
=+
+
+
=


() {ln( ) ( ) ( ) ( )}
()
r
a
eA
r
a
W
r
a
r
a
C
r
a
l
r
a
l
m
m
m1
1
21
0
222 2
β
βββ β
=+
+
+?+
=


() { ln( ) ( ) ( ) ( )}
()
1222
11
1
21
0
a
eAr
r
a
W
r
aa
rC
r
a
l
r
a
ll
m
m
m
β
βββ β2
(4.79)
于是,不论是否为零,(由于Cl
0
0≠ )大括号内的第二项均决定了。所以,只有一χ /→
→r 0
0
a
按用广义幂级数求解二阶线性微分方程的通解理论,如δ δ
12
≠整数,可得两个线性无关解;若
δ δ
12
=整数,只能得到一个独立的广义幂级数形式的解,另一个需另外求得。见下面叙述。
b
这可用朗斯基行列式配合幂级数展式积分予以证明。详见梁昆淼编“数学物理方法”,第259页,人民教育出版社,1960年。也可参见吴大猷著理论物理第六册量子力学(甲部),第103页,科学出版社,1984
年。但后者未阐述理由。
个解留下来
Rr
r
r
r
r
a
eF r
r
l
r
a
() () ( ) (,,)==
+
11
2
1
χα
α
β
β (4.80)
但是,这个解也存在问题,因为当足够大时,k Fr
r
a
(,,)α
β2
中相邻两项的比值为
11
1
21 2
k
k
k
rk
rk
r
ak
k
rk
r
a
++
+
+
++
=
+
+
Γ
Γ
Γ
Γ
()
()
()
()
() ()
α
α
β α β
k
k
r
a
足够大

12
()
β
(4.81)
这说明,当r →∞时,如果α ≠负整数,将有
Frz e
r
a
(,,)α
β

2
这使R r()变成不是平方可积的。由此应当令α =? =nn
rr
(,,,......)012,使无穷级数形式
F r z(,,)α截断为阶多项式。这里n称为径向量子数。这时(注意n
r r
α
β
=+?l 1
1
),
β ==++=
1
112
n
nn l n
r
,,(,,......)
n为主量子数。于是能量本征值和本征函数为
EI
e
a n
me
n
rNRrY NreFnll
n
r
a
Y
n
nlm nl nl lm nl
l
n
r
a
lm
=? =? =?
==?++
β
ψθ? θ? θ
2
2
2
4
22
1
2
1
2
1
12 2
2
h
(,,) () (,) (,,) (,)?
.(4.82)
这里,ψ θ?
nlm
r(,,)的径向部分(除负指数外)是个关于r的nln
r
+ =?(1)
)
阶多项式,即阶数只与主量子数有关。 n
2,讨论
i,求归一化系数。由N
nl
a
知,上面的合流超几何函数Fn l z
r
(,,? +22可化为广义拉盖尔多项式,
Lz
m
m
Fm z
m
ν
ν
ν
ν()
()
!( )
(,,= )
+ +
+
+
Γ
Γ
1
1
1 (4.83)
它有如下积分公式
zeL zdz
nn l
nl
lz
nl
l22
1
21 2
0
2
1
+?

+


=
+

[()]
()
()
!
!
(4.84)
于是,根据归一化要求
Nr e F n l l
r
na
dr
nl
l
r
na
222
2
2
0
12 2
2
1
+

++ + =

[(,,)] (4.85)

N
na na l
ln
nl
nl
l
=
+
+

()
()!
()!
()
/
22 1
21 1
232
!
(4.86)
ii,以前曾说过,方程实质上是粒子德布罗意波的波动方程。这里,用dingeroSchr &&
a
王竹溪、郭敦仁“特殊函数概论”,第362页,科学出版社。
德布罗意波干涉的观点和对平均值的经典处理再次求出氢原子能谱,以此来再次证实这种观点。根据圆形轨道上德布罗意波自相干涉的极大值条件(或称驻波条件),设第个电子轨道的平均
..
半径为,则应有
n
r
n
2
2
πλ
π
rn n
p
n
==
h
,或r
n
p
n
=
h
(4.87)
这里p是电子在此轨道上回旋的平均
..
动量。同时,由平均
..
的离心力和向心力相等给出
μ
μ
v
r
p
r
e
r
nnn
22
2
==
2
,或p
e
r
n
2
2
=
μ
(4.8)
和上面公式结合,得 r
n
r
n
e
na
n
=≡
22
2
2
h
μ
(4.89)
于是
E
pe
r
e
r
e
r
e
r
e
a n
n
nnn n
=? =? =? =?
22 2 2 2 2
2
22 22
1
μ
(4.90)
关于对平均值可以直接实施经典运算的问题,其理由前面已经讲过。
iii,关于简并度
注意,库仑场的能谱公式中只含主量子数( nnn l
r
= + +1),不含也未显含。于是对某个能级,总共的简并度为
m l
n f
n
fl
n
l
n
=+=
=

()21
0
1
2
n (4.91)
这里,关于m的简并其实对所有中心场均存在,但对的简并只对l
1
r
形式的这种中心场才存在。后面这种简并称为库仑简并。这一简并的原因在第六章宇称节中给以说明。
关于氢原子能级、谱线和波函数见下面图
iv,按经典电动力学观点,氢原子模型是不稳定的。因为圆周运动的电子将不断辐射能量,最后应当坠落到核上,发生原子的坍缩;但实际上氢原子很稳定。量子力学定态的观点解决了这一困难,说明了氢原子的稳定性。但由此却产生了一个新问题,也是按量子力学定态的观点,当电子处在激发态时,如无外界的扰动,应当继续保持下去,不应有自发的向低能级跃迁。这一新的困难在量子逻辑继续向前推进后,在场的量子化框架内得到了出色的解决。
v,关于径向波函数的零点 Rr
nl
()
当时,l ≠ 0 r = 0处是的零点。即当 Rr
nl
() r → 0时,。这说明离心势影响电子分布,使之偏离中心点。当
Rr
nl
l

l = 0时,r = 0不是波函数的零点。
一般地说,由于χ()r的方程形式上是个一维方程,所以关于一维问题零点的定理4可以应用到这里。于是,不计r
dingeroSchr &&
= +∞0、两处,χ
nl
r()(因而Rr
nl
())共有个零点 (这由波函数表达式中函数为阶拉盖尔多项式也可看出)。
n
r
F n
r
vi,一些重要的修正
应当强调指出,上述方程对氢原子问题的描述仍是近似的。虽然在氢原子光谱问题上,理论结果和实验基本相符,然而同时也表现出了理论与实验之间明显偏离。
这些偏离说明,这里的理论描述还需要作进一步的修正。修正后的理论将与光谱实验数据更好地符合。这些修正包括,用
dingeroSchr &&
p
m
2
2
代表电子的动能是最低阶的非相对论近似,进一步考虑遂需要高一级的修正;电子有自旋,从而有磁矩。这个内禀磁矩和轨道角动量所产生的磁矩之间有相互作用,称为旋—轨耦合;电子并非是一个位置用几何点表示的质点,而是德布罗意波,在其康普顿波长λ
e
范围内的位置是不定的,这导致库仑场对它作用时有弥散效应,就是说,加在电子上的库仑场并非V r()
v
(其中
v
r是电子作为几何点的矢径),而是
v
r附近λ
e
范围内的场,这项修正称Darwin项。以上称为精细结构修正。此外还有,核电荷分布有限体积的修正、核磁矩和电子磁矩相互作用的修正、多电子原子中电子之间电磁相互作用的修正等等。这些修正不仅会使能级发生移动,也会使能级产生劈裂。详见微扰论叙述。
vii,原子单位
进入库仑场方程的物理常数总共三个:,、。为简化表述,可采用原子单位制,在计算中,形式上略去这三个常数,即相当于令它们为1;在最后结果中,
再加上它们适当幂次的组合,凑得量纲正确即可。具体说,
dingeroSchr && h e m
e
如计算质量,在最后结果上乘以电子质量m。
e
如计算长度,在最后结果上乘以a。
me
e
=
h
2
2
如计算时间,在最后结果上乘以h。
34
/ me
e
如计算速度,在最后结果上乘以e。
2
/ h
如计算动量,在最后结果上乘以me。
e
2
/ h
如计算能量,在最后结果上乘以me。
e
a
e
42
2
/ h =
§4.6 三维各向同性谐振子问题
1,方程和解 dingeroSchr &&
这时势场为
Vr r()=
1
2
22
μω (4.92)
径向方程为 dingeroSchr &&
′′+ ′+
+
=R
r
RE r
ll
r
R
22 1
2
1
0
2
22
2
[( )
()
]
μ
μω
h
(4.93)
边条件和库仑场相同。令
k
E
==
2
2
μ
α
μω
h h
,
径向方程变为
′′+ ′+
+
=R
r
Rk r
ll
r
Rr
21
0
242
2
[
()
]()α (4.94)
为消去R r()前面系数中的r
2
项,作函数变换
Rr r vr() ()=
δ
选δ满足指标方程
a
δ δ δ() ()? +? + =12 1ll 0 (4.95)
即可达到目的。这时 δ
1
= l,δ
2
1=? +(l )。取第一个根,即决定函数变换为
Rr rvr
l
() ()= (4.96)
作第二步函数变换,为消去v r()前的r
2
项,作下述函数变换
vr e ur
br
() ()=
2
(4.97)
只要选择满足 b
+ =α
42
4b 0 (4.98)
得到b。于是,总计作变换 =
1
2
2
α
Rr re ur
l
r
() ()=
1
2
22
α
(4.9)
得到u r()的方程
′′++? ′+? + =u
r
lruk lu
2
12
3
2
0
22 2 2
()[()]αα (4.100)
引入无量纲参量并作自变数变换(为消去p中的正一次项)
S
kE
r== =
2
2
22
2α ω
ρα
h
,(4.101)
若仍记u r u() ( )= ρ,得u()ρ方程如下
du
d
l
du
d
S
l
u
2
2
3
2
1
2
3
2
2
1
0
ρ ρρ ρ
ρ+
+
+?
+
=[][ ]()
(4.102)
这是合流超几何方程,这时
α =+? =+
1
2
3
2
3
2
[],lSrl (4.103)
a
(若′′+ ′+ =zWQzW() () 0WP )δ的指标方程为δ δ δ()? + + =

10
12
pq,是p
1
P z()中项系数,是中项系数;满足的方程是。 z
1
q
2
Qz() z
2
b qbpb
21
2
240++=
由于r ≠整数,可得两个线性无关的广义幂级数解
Fr F l Sl(,,) ( ( ),,)αρ ρ=+?+
1
2
3
2
3
2
(4.104)
ρα ρρ
1
1
2
12
1
42
1
2
+
+? =?
+
r
l
Fr r F
ls
l(,,) (,
()
ρ,)
(4.105)
但第二个解不满足r → 0时的边条件rψ → 0,因为第二个解当r → 0时,有乘子
rr
l
l
l
ρ
+
+
=
()
()
1
2
1
即使l = 0,也不满足rψ → 0。所以此解应当略去。同时,保留下来的第一个解由于指数发散,必须予以截断(参见中心场相应叙述),为此令
1
2
3
2
012( ),(,,,......)lSnn
rr
+? =? = (4.106)
n
r
为径向量子数。这时合流超几何函数截断成为n阶广义拉盖尔多项式。于是能量本征值和本征函数分别为
r
Enl N N
Nr
=++ ≡+ =( ) ( ),(,,,......)2
3
2
3
2
012hhωω (4.107)
ψθ? α α θ
α
nlm nl
l
r
rlm
rr
rNreFnlrY(,,) () (,,)(,=?+
1
2
22
22
3
2
)
=
+
++
+
N
nl
nl
re L rY
nl
r
r
l
r
n
l
lm
rr
!( )
()
() ( ) (,
Γ
Γ
3
2
3
2
1
2
1
2
22
22
αα
α
)θ? (4.108)
2,讨论
i,往求归一化系数的表达式 N
nl
r
借助数学公式
zeL zdz
nl
n
l
z
n
l
r
r
r
+
+∞

=
++1
2
1
2
2
0
3
2
[()]
()
!
Γ
,
Γ()
()!
,(,,......)m
m
m
m
+=
=
1
2
21
2
12π,

N
l
nl
n
nl
ln
r
r
r
r
=
+
++
+?
α
π
3
2
2 1
2
21
2221
()!
[
()
!
]
!
(4.109)
ii,能级和简并度
能级:仅与N有关,而Nn
r
= l+2,所以与l N奇偶性相同。和一维谐振子一样,
三维各向同性谐振子仍表现了谐振子能级的特点——间距均匀,有能量子存在。
简并度:对同一个N,有不同的,而对每一个又有不同的。简并度计算如下 l l m
Nf l k
N
l
N
k
N
为偶数.()(
,,,......,,,......
/
=+=
==
∑∑
21 4 1
024 012
2
)+,( l k= 2 )
=? + + += + +4
1
2
1
22 2
1
1
2
1()()()(
NN N
NN2) (4.110)
Nf l k
N
l
N
k
N
为奇数.()(
,,,......,,,......
()/
=+=
==
∑∑
21 4 3
135 012
12
)+,( l k= +21)
=? +

+
+= + +4
1
2
1
1
2
1
2
3
1
2
1
1
2
1() ()()(
NN N
NN2) (4.111)
于是不论N为何正整数,简并度均为
1
2
1()(NN++2)。简并度与l无关。
iii,三维各向同性谐振子在直角坐标中的解法
哈密顿量为
HrH
xy
=? + = + +
h
2
22
2
1

μω? H
z
(4.112)
这里H
xxx
=? +
h
2
2
2
1

μωx
2
等。于是能量本征值和本征函数分别为
Ennn N Nnn
xyz x y z
nnn x y z x y z
nnn n n n
xyz
xyz x y z
=+++ =+ =++
=
()(),
(,,) () () ()
3
2
3
2
hhωω
ψψψψ
n
(4.113)
这里ψ
n
x
x()是一维谐振子的对应本征值为的波函数。 n
x
计算这时对某一固定N的简并度。先算两个,即设f
N
nn
yz
n+ =,则有重简并。
于是对给定的一个n,有
n +1
x
Nn n? n
xyz
= +,对、n有n
y z
(Nn
x
)? +1重简并。所以总简并度为
fNnNN
Nx
n
N
x
n
N
x x
=?+=++?
==
∑∑
()()()111
00
=+? + = + +()() ()(NNNNN1
1
2
1
1
2
12
2
)
和前面解法的简并度果然一致。于是,对同一个能级,即同一个N,也即在同一个
1
2
1()(NN++2)维子空间中,两套基矢彼此可以相互展开。
附录2,
设二阶齐次微分方程
′′ + ′ + =Wz PzWz QzWz() () () () () 0
在处(若不在z = 0 z = 0处可作平移变换)有正则奇点,即在z = 0处P z()有不超过一阶的极点,有不超过二阶的极点,Qz()
Pz
p
z
ppz Qz
q
z
q
z
qqz( ),..; ( ),..=+++ =++++
1
01
2
2
1
01
,
用广义幂级数待定解Wz代入,最低幂次为z Cz
n
n
n
()=
=


δ
0
()δ? 2,共有三项;令它们系数之和为零,即得指标方程
δ δ δ()? + + =

10
12
pq
第五章 表象理论
§5.1 幺正变换和反幺正变换
1,幺正算符定义
对任意两个波函数?()
v
r、ψ()
v
r,定义标积符号
(,) () ()?ψ? ψ=

vv
rrd
v
r (5.1)
则幺正算符的定义是,
“对任意两个波函数?、ψ,算符U均使下式成立
$
(
$
,
$
)(,UU?ψ?ψ= ) (5.2)
而且U算符有逆算符U存在,使得UU
$ $
1
$$ $$
UU I

= =
11a
,称这个U为幺正算符。”
$
由于任一算符的厄米算符是如下定义的:“对任意
$
A?、ψ,的厄米算符被定义为
$
A
$
A
+
(,
$
)(
$
,?ψ?ψAA=
+
)。” (5.3)
因此,上面的幺正算符U的另一个等价的定义是
$
“如果U是幺正的,其充要条件是
$
$$ $ $
UU U U I
++
= =
或者说
$$
UU
+?
=
1
。” (5.4)
证明:若(成立,则按U定义,有
$
,
$
)(,UU?ψ?ψ= )
$
+
(,) (
$
,
$
)(
$$
,)?ψ? ψ?ψ==
+
UU UU
由于?、ψ任意,所以
$$
UU I
+
=
又因为U有唯一的逆算符U存在,假定取,则有
$ $
1
′ = ′ =

ψ
$
,
$
UU
11
ψ
I
(,) (
$
,
$
)(,)(
$
)
$
,)?ψ? ψ? ψ?ψ= ′′= ′′=
+?
UU U U
11
所以
(
$
)
$
UU
+?
=
11
由于,上式即 (
$
)(
$
)UU
+ +?
=
11
$$
UU I
+
=
这就从第一种定义导出了第二种定义。类似,也能从第二种定义导出第一种定义。从而,幺正算符的这两种定义是等价的。
2,幺正算符的性质
幺正算符有如下几条性质,
i,幺正算符的逆算符是幺正算符
这是显见易证的。
ii,两个幺正算符的乘积算符仍是幺正算符,
证明:设U、V是两个幺正算符,则
$ $
(
$$
)(
$$
)
$$$ $ $$
UV UV U V V U UU I
+++
= = =
+
(
$$
)(
$$
)
$$$$$$
UV UV V U U V V V I
+++ +
== =
a
这里强调写出了U对U的右乘和左乘,是因为对无限维空间(这在量子力学中是常见的)情况下,一个算符的左逆(算符)常常不等于其右逆(算符)。或者说,一个算子并不一定同时具有左、右逆算符。这和有限维空间情况是不同的。
$
1
$
所以UV也是个幺正算符。
$$
iii,若一个幺正算符U和单位算符
$
I相差一无穷小,这个幺正算符被称为无穷小幺正算符。这时U可记为
$
$$
Ui=?1 εF
ε为一个无穷小参数。于是U的逆算符(准确到
$
ε的一阶,以下同)为
$$
Ui
+
=+
1
1 εF
1
利用U的幺正性,
$
$$
(
$
)(
$
)(
$$
)UU iF iF i F F
++ +
=+? =+? =11 1εεε
得到等式
$$
FF
+
=
这说明,如将一个无穷小幺正算符U表述为上述形式,则其中的为厄米算符。也常称为幺正算符U的生成元。由此用厄米算符
$ $
F
$
F
$ $
按以下方式可以构造出一个幺正算符U
$
$
$
Ue
i
=
α?
这里,α为任意实数。
3,幺正变换
幺正算符给量子系统带来的变换称为幺正变换。具体地讲,一个幺正算符对量子系统的幺正变换是
对波函数,;
$
()
U
U
ψψ≡
对力学量算符,
$$ $ $
()
UU
U


1
,
对态和算符这两种变换,配合起来使任意几率幅保持不变,即有
(,
$
)(,
() () ()
ψ? ψ=
UUU
)
这可以检验:右边=?。 = =
+
(
$
,
$$ $$
)(
$
,
$$
)(
$$
,
$
)(,
$
)UUU U UU UU? ψ? ψ?ψ?ψ?
1
v v
举例来说,对一个量子系统施以傅立叶积分变换,将波函数ψ ψ() ()r p→,将算符
,也就是通常所说(下面将要谈及)的由坐标表象向动量表象变换,便是一种幺正变换。这时
$
()
$
()
vv
r→ p
$
()
/
Udr
i
pr
=


1
2
32
πh
v
h
vv
e (5.)
$
()
/
U dpe
i
pr
=

1
32
1
2πh
v
h
vv
(5.6)
v
注意,这里的算符U是一种积分变换,其中,
$
r为积分变数,
v
p为参量(因此当U和后面的算符或坐标函数作相乘运算时,
$
v
r必须和后面的自变量取成一致并进行积分,而
v
p此时仍保持为参量);U中
$
1
v
p为积分变数,
v
r为参量。在几个算符连乘的运算中,为避免混淆,常把积分变数记为带撇的。于是有
$
()
()
() ()
/
Ur dre r p
i
pr
ψ
π
ψψ
v
h
vv
h
vv
=


1
2
32
v
= (5.7)
$
()
()
() ()
/
U p dpe p r
i
pr

1
32
1
2
ψ
π
ψψ
v
h
vv
h
vv
v
(5.8)
$$
() ()
//
UU dre dp e
i
pr
i
pr
′?
=?′
∫∫
1
32 32
1
2
1
2ππh
v
h
v
h
vv
h
vv
= ′?
∫∫
′?
dp e dr
i
pp r
v
h
v
h
vvv
1
2
3
()
()
π
= ′′ =

dp p p
v v v
δ()1 (5.9)
$
$
$
() ()
//
U
p
m
Udre
m
edp
i
pr
i
pr
v
h
v h
h
v
h
vv
h
vv2
1
32
2
2
32
2
1
2 2
1
2
′?
=?

∫∫
ππ
=


′?
∫∫
1
2
1
2 2
32 32
2
() ()
//
ππh
v
h
v
v
h
vv
h
vv
edr
p
m
edp
i
pr
i
pr
=

′?
∫∫
′?
v
v
h
v
h
vvv
p
m
dp e dr
i
pp r
2
3
2
1
2()
()
π
=

′′=

v
vvv
v
p
m
ppdp
p
m
22
δ() (5.10)
$
$
()
$$$
() ()U
p
m
rU
p
m
UUr
p
m
p?=?=
v
v
v
v
v
v
22
1
2
22 2
ψψψ,或
$
$
()
()
()
/
U
p
m
rdre
m
r
i
pr
=?


v
v
h
v h v
h
vv2
32
2
2
1
2 2
ψ
π
ψ?
=
+

∫∫ ∫
1
2 2
32
2
()
(){ () ()
/
π
ψσ ψ
h
h v v
h
vv
h
vv
h
vv
m
erd
i
pre
i
pr
i
pr
}
v
dr
=

=


1
2 22
32
2
2
2
()
()() () (
/
π
ψ
h
h
h
vvv
h
vv
m
i
pdrre
p
m
p
i
pr

v
(5.1)
$$ $
() ()
//
UrU dre r dp e
i
pr
i
pr
v
h
vv
h
v
h
vv
h
vv
′?
=?′
∫∫
1
32 32
1
2
1
2ππ
= ′


′?
∫∫
1
2
3
()
()
πh
vv
h
h
vv
h
vv
dre dp
i
e
i
pr
p
i
pr
= ′?
′?
∫∫
1
2
3
()
()
πh
vv
h
h
vv
h
vv
dp dr
i
ee
p
i
pr
i
pr
=?′
′?
∫∫
1
2
3
()
()
πh h
vv
h
vvv
i
dp dre
p
i
pp r
=? ′? ′ =?

i
dp p p
i
h
vvv
h
δ()
p
(5.12)
于是在这个U变换下,坐标表象中的哈密顿量
$ $
$
()H
p
m
Vr=+
v
v
2
2
改变成为
$
H
p
m
V
p
=+?
v
2
2
(
i
h
)
1
h
。这里利用了无穷远处粒子的密度和流均为零的边条件。
应当强调指出,量子体系在任一幺正变换下不改变它的全部物理内容。这个“全部物理内容”包括:基本对易规则、运动方程、全部的力学量算符方程、全部的测量几率。比如,
基本对易规则在U变换下保持不变
$
$
( $$ $$)
$$
$
$$
$
$$
$
$$
$
$
U xp px U UxU UpU UpU UxU?=
1111
== = =

$$ $$ ()
() () () ()
xp px UiUiU i
UU UU
hh
11
即有
$$ $$
() () () ()
xp px i
UU UU
=h
而且,全部测量几率不变,因为,已如上所说,全部的几率幅不变,比如,在?()
v
r态中找到ψ()
v
r态的几率幅为
fr
r
ψ
ψ?
()
()()
v
rd
v v v
=

(5.13)
这里的上标(f )
v
r表示是在坐标表象中算出的。当系统经幺正变换转到动量表象之后,
() (
vv
)r p→,ψ ψ() ()
vv
r p→,这个几率幅应表示为
fppd dprdrre
r
i
pr
i
pr
ψ
ψ?
π
ψ?
()
/
()()
[( ) ]
()()
v
h
vv
h
vv
vvv
h
vvv v v
== ′′


∫∫
1
2
32 2
= ′′? ′

drdr r r r r
v v v v v v
ψ?δ()()( )
v
==

dr r r f
r
v v v
ψ?
ψ
()()
()
,
的确不变。反过来也可以说,两个量子体系,如能用幺正变换联系起来,则它们在物理上是等价的。
4,反幺正变换
反幺正变换全名是反线性的幺正变换。为阐述其内容,我们先定义反线性算符。
一个反线性算符满足
$
A
$
()
$
AAα? βψ α? β ψ+= +

$
A (5.14)
这里α、β为任一复常数,?、ψ为任意波函数。这就是说,在此算符作用下的常数若抽出算符作用之外时,需要取复数共轭。这是与线性算符唯一的然而是极重要的差别。
反线性算符的厄米共轭算符的定义是
$
A
$
A
+
(,
$
)(
$
,) (,
$
)?ψ?ψ ψ?AA A==
+? +
(5.15)
这里第一步等号的等式右边的复数共轭是为了使定义在逻辑上自洽所必要的。这只需从标积内?或ψ中抽出一个常系数来检查便可知道。
反线性的幺正算符(简称反幺正算符)定义为
$
A
(,
$
)(
$
,) (,
$
)?ψ?ψ ψ?AA A==
11
(5.16)
根据这个定义,立即知道,对反幺正算符也有
$$
AA
+?
=
1
这导致
$$ $ $
AA A A I
++
==。这和幺正算符相同。
反线性算符的详细叙述参见附录7。
§5.2 表象概念
1,波函数的标记和分类
三维空间德布罗意平面波(或三维自由运动方程解的集合)需用三个本征值(、p、p )来标记分类,缺少一个则波函数的标记就不完全,出现对该量子数的简并。
但可以用另外三个本征值(或量子数)来标记、分类这个解的集合,比如自由粒子球面波,这时需用(
dingeroSchr &&
p
x y z
E、、)(详见中心场球方势阱那一节)来标记分类。又如三维各向同性谐振子,
既可用(、n、)来分类它的全部状态,又可以用(、、)来分类全部状态。显然,
用于分类的这些态必定都是这组(与量子数所相应的)力学量算符的共同本征态。因此,这组力学量算符必须互相间都能对易,可以同时测量的。
l
y
m
nn
x z
n l m
由上面分析可以得出结论:任一量子体系的波函数集合能用一组可对易力学量算符的本征值来区分和标记。如果这组算符数目选少了就出现(态对未被选入的力学量本征值的)分类不彻底,从而波函数标记不明确的现象。能够对一个量子体系全部状态进行彻底地分类并标记(即不出现简并)的最少数目的力学量算符,成为这个量子体系的完备力学量组。为了方便,
完备力学量组中各力学量总是尽可能地选用该体系的守恒力学量,也就是说,尽可能用好量子数对态进行分类。
应当指出,由于力学量的本征值有的呈现连续变化,有的呈现分立的可数的变化,因而不同的量子体系,其状态的分类和标记有的是连续的,有的则是分立的,有的还是两者兼有。
甚至同一量子体系,若从不同的观点对其状态进行分类,也可以有时是分立的有时则是连续的。这要看分类时所选用的算符完备组性质而定。比如氢原子问题,在束缚态问题的范围内,
完备组可以选能量、轨道角动量和它的第3分量做完备力学量组,对应的好量子数完备集为
( ),本征函数族为nlm {()}ψ
nlm
r
v
。但如果考虑与自旋有关的效应,则还应计入自旋角动量及其第3分量,否则就会无法彻底区分各种状态。而如果还包括连续谱,即还考虑电离和散射等非束缚态,除上述这些之外,还应引入别的(比如动量)量子数或本征值来作区分。此外,
如果此问题采用
v
r力学量的本征值来分类(不是好量子数),则这些状态便被表示为关于
v
r的一系列平方可积的连续函数及其线性叠加。这等于用波函数来标记状态。
2,量子力学的表象概念
上面叙述了如何对一个量子体系的各种不同的状态进行分类和标记。这里讨论这些状态
(包括完备组的共同本征态)如何以某种统一的观点进行描述,也就是量子力学的表象问题。
先从三维空间的任一矢量的表示方法说起。众所周知,三维矢量的线性组合构成了三维空间。对这三维空间中的任一矢量,既可以抽象地记为
v
A、
v
B、......,并用标积、矢积和各种矢量表示方法来构成它们之间的代数或微积分运算,而不涉及任何坐标的选取,这显然是一种抽象的、因而是普适的、不依赖于笛卡儿坐标选取的表示方式,也可以在三维空间中事先选定一个坐标系(比如某个笛卡儿坐标),于是任一矢量
v
A在这个坐标系中便由与之相应的三个数(是与坐标轴的单位矢量
v
A
v
e
i
的标积,也称其为这个矢量在这个坐标系中的分量
)来表示。但坐标系有无穷多种选取办法,这种表示也就有无穷多种。
它们之间只相差坐标轴的平移和旋转,就是说,各种表示之间可用平移和正交变换相联系。
v
v
Ae?= =,,123Ai
ii
,
在量子力学中,按照态叠加原理(或Schr方程关于波函数的线性性质),可以知道,一个量子体系的所有可能状态将构成一个线性空间,这个关于状态集合的线性空间通常又称为Hilbert空间。于是,体系的每一个状态将对应于体系的这个状态Hilbert空间中的一个矢量,称为状态矢量,简称态矢,所以状态Hilbert空间又常称为态矢空间。这个Hilbert
空间的范数便是状态之间的标积
dingero&&
N = (,)? ψ
a

和三维空间类似,可以不必事先选取任何坐标,而采用抽象的态矢符号,以普适的方式描述态矢空间中态矢的变化和态矢间的关联;也可以事先选定一组作为基底的态矢(又称为基矢,一般为无穷多个,有时是可数无穷多,有时是不可数连续变化的无穷多,视基矢所属力学量算符性质而定,其作用类似于三维空间中三个坐标轴上的三个单位矢量)。于是,
态矢空间中的全部态矢均按这组基矢进行展开,并用它们与这组基矢的内积这组数(可数无穷多或不可数的连续变化的无穷多)(也称为态矢向这组基矢的投影,相当于三维空间中某个矢量的坐标)来表示。由于Hilbert空间的基矢通常为无穷多个,于是用于描述某个态矢的投影(相当于三维空间一个矢量的坐标)的个数也是无穷多。而作用在态矢上并造成态矢变化的
v
e
i
各种力学量厄米算符,便成了无穷维(可数或不可数)的厄米矩阵,这些无穷维的矩阵关联着态矢空间中各个态矢并决定它们的变化。这些厄米算符表示为可数抑或不可数的厄米矩阵,
要看所取的基矢是可数或不可数来决定。
每选择一族基矢,与之相应的,每一状态以至整个态矢空间便有一种描述方式,便相当于选取了一种表象。表象的改变便意味着态空间中基矢的改变,这是一种幺正变换;反之亦然。从物理上说,选用不同基矢来描述应是等价的,不会改变任何物理结论;从数学来说,
体系经受一个幺正变换并不改变任何物理结论,两种说法是一致的(这种等价性在前面坐标表象到动量表象的幺正变换中已经看到)。
举例来说,迄今所讲的波函数(及相应的量子力学描述 —— 方程,常称为波动力学)便是选取了坐标表象,只是以前未便明确指出而已。可以在所考虑量子体系的
dingeroSchr &&
a
注意,量子力学中的状态空间 —— Hilbert空间不完全等同于数学中的Hilbert空间。因为前者还包括了归一化到δ -函数的矢量,而后者无此类矢量。
态矢空间中,选取坐标算符本征态矢集合{( ),}δ
v v v
rr r ′作为基矢。由于作为本征值的坐标
v
r在数值上是连续变化的,也即本征值的编号是连续的,从而这组基矢的编号也是连续的。
于是,一个态矢在这组基矢上的投影的全体便构成了
v
r的一个连续函数——波函数。就是说,
ψ(
v
)r是系统处在这样一个态矢上,当坐标取
v
′r时的几率幅为ψ()
v
′r,等等。这个ψ()
v
′r正是这个态矢和本征值取
v
′r时坐标本征矢的内积,或者说,是这个态矢向本征值为
v
′r的坐标本征态矢的投影(坐标)。
v
ψ(
ψ()
v
p
H
}
C
nlm
ψ()
v
rC r
nlm nlm
nlm

C ()
v
r
)
C
nlm
==

ψ? ψ,r)()
v v
}
Φ
$
() ()Ψ
v v
rt rt=
ψ
ψ
1
2
1
2
()
()
=
t
t
M
M
M
M
()
nlm j
ψ
j
i
E
rrdr=?
$
()
vvv
() ()
E
t
j
()
v
t r
ii
i

ψ
() ()
v
t r
ii
i

ψψ
1
2
M
$
3,几种常用的表象
几种常用的表象是坐标表象、动量表象、能量表象等。如上所说,这些表象均对应于态矢空间中基矢的不同选取。坐标表象中选择了坐标算符的本征态作为态矢空间的基矢,而任一态矢可用它在这组基矢上投影的坐标 —— r的函数)
v
r来表述。这就是通常所称的波函数;动量表象选择了动量算符的本征态作为基矢,这时任一态矢被它的投影(坐标)的全部集合所表示,这个集合即为ψ()
v
p (有时称为动量表象波函数);能量表象通常取哈密顿量、和的共同本征态矢作为基矢。这时一个态矢在此表象中的“波函数”(投影坐标的集合)是如下展式中的一组系数集合
L
2
L
z
{
,
()
v
= (5.17)
这里便是这个态矢(它的波函数为
nlm
)向基矢(它们的波函数为ψ
nlm
r()
v
)的投影坐标或内积,用波函数表述出来便是
(
nlm nlm
(
v
(5.18) dr
注意,由于这时基矢的编号( )是断续的,所以能量表象中的“波函数”
{
也是断续的,这和坐标表象、动量表象不同。与此相应,造成态矢变换的各种力学量的算符便具有了可数的无穷维厄米矩阵的形式。比如,在坐标表象中的一个波函数
nlm C
nlm
()
v
rt在算符作用下的变换方程
$
Φ
在能量表象中写出来便成为
ωω
ωω
11 12
21 22
,,......
,,......
....................
....................
()
t
t
,
为记述简明,这里脚标i、j表示量子数组( )分别取第i、第组数值。而
ωψ
ij i
e
i

()
hh
v
Φ()rt =
v
Ψ()rt =
这里矩阵
(
、矢量和分别是能量表象中的算符
)
ω
ij
M
ψ
ψ
1
2
M
M
、态Φ(它在坐标表象中的表示是波函数)和态(它的波函数是Φ(
v
rt) Ψ Ψ()
v
rt )。注意这里ω
ij
中时间因子里的频率
EE
ij
h
i
ω
j
=?ω体现了光谱学中的里兹组合规则。实际上,这正是矩阵力学(见下面叙述)创始人海森堡思考中的出发点之一。
从上面讨论我们知道,一个量子体系,如取x表象来描述它,它的状态便用波函数表示,
它的力学量算子便一般地表现为微分算子(除了纯由坐标组成的力学量,由于在自身表象中,
所以表现为普通的函数)。量子力学的这种表述方式最先由提出,称为波动力学,所以坐标表象也常称为表象。与此同时,如果取能量表象来描述这个量子体系,它的状态便用由一组可数的常数所组成的列矢量来描述,而力学量算符便相应地变成矩阵形式(一般说,这些矩阵是无限维的。但若只研究属于某个能量的状态子空间,则状态列矢量的分量个数便是有限的,算符矩阵也是有限维厄米方阵)。量子力学的这种表述方式最先由海森堡提出,称为矩阵力学,因此能量表象也常称为海森堡表象。
dingeroSchr &&
dingeroSchr &&
§5.3 量子力学的狄拉克符号表示
1,狄拉克符号
上面的全部叙述,如用狄拉克符号表示出来就十分简洁明了。
如前所述,由于态叠加原理(或Schr方程的关于波函数的线性性质),可以指望一个量子体系全部状态将组成一个线性空间。在这个空间中每一个状态用一个矢量来表示,
简称态矢或右矢,并记为
dingero&&
A。这里右矢符号里的是对此态矢的某种标识,办法可以视方便来选择,比如用态矢的好量子数组来标记态矢,例如
A
nlm,也可以用态矢的波函数(它和态矢的关系下面即将谈及)来标记,例如这里的nlm态矢也可记为
nlm
ψ。还比如,r,′
v
p′
v
等等,前者是坐标算符的本征态,对应本征值为
v
′r,即rrrr ′′=′
vvvv
,等等。
与此同时,对每一右矢A,也可对应地给出一个左矢A
a
,它是右矢的厄米共轭,即
+
= )( AA (5.19)
当然,右矢也是左矢的厄米共轭。于是,和基右矢相应,也有一组基左矢。
有了右矢和左矢的概念,便可以引入内积(标积)定义。对任意右矢A和左矢B,定义它们的标积
的几率幅中发现在某个复数BAAB ==
这个内积应满足对A为线性,对B为反线性。于是
= BAAB (5.20)
这可以从它们之中各自抽出一个常系数予以检验。于是AA将是非负的实数。如果标识(编号)为分立的左、右态矢,它们之间的内积为零,自己和自己的内积为1,即称它们为正交归一的;对标识(编号)为连续的左、右态矢,若它们之间的内积是δ -函数,即称它们为正交归一的。
为了运算方便,常常选定一组正交归一的态矢作为基矢。就是说,规定基矢组{和}ξ
{ξ }有如下正交归一性质
对分立编号
ijji
δξξ =
对连续编号 )( ξξδξξ ′′?′=′′′
前者例如
mmllnn
mlnnlm
′′′
=′′′ δδδ,后者例如)( rrrr ′′?′=′′′
vvvv
δ,)( pppp ′′?′=′′′
vvvv
δ

b
。如此选定基矢之后,上面的态矢A、A以及和另一态矢B、B之间的内积便可
a
左矢常称bra,右矢常称为ket,这是bracket一字的左三个字母和右三个字母。
b
后两者为连续表象。在这类表象中正交归一化为δ -函数。这使量子力学的Hilbert空间大于数学中由平方可积函数组成的传统的Hilbert空间。详细还可参见下面叙述。
以具体地写出。一般情况下,一个完备的右基矢组常常既包含分立的基矢集合{
i
ξ },又包含着参数为连续变化的基矢集合{。相应地,左基矢组情况也如此。于是一般而言,}ξ A、
B可展开为


+= ξξξξ daaA
i
ii
)( (5.21)


′′′+=

ξξξξ dbbB
i
ii
)( (5.22)
于是


′?′′+=

ξξξξδξξδ ddababAB
ij
ijji
)()()(
=+



ba b a d
ii
i
()()ξξξ (5.23)
按量子力学的基本假设,这些内积应被解释为在A中发现B的几率幅。若取坐标表象的连续基矢{r
v
},按此观点就应当有
)(rAr
A
vv
ψ=
这是因为,等式左边的含义是在态中找到粒子位于A
v
r处的几率幅,这正是ψ
A
r()
v
的含义。
同时,就分立编号基矢{情况而言,有 }
i
ξ
j
i
iji
i
ijij
aaaA ===
∑∑
δξξξ (5.24)
于是有
∑∑
==
i
ii
i
ii
AAA ξξξξ (5.25)
假如这套基矢是完备的,那对于任一右矢都可以作这种展开。于是由于A的任意性,即得
I
i
ii
=

ξξ (5.26)
这是已经假定{能够展开任意右矢}
i
ξ A的结果。从而,这个条件便是表明这组基矢完备的完备性条件。对于连续编号基矢,类似有
)()()()(})({ ξξξξδξξξξξξξξξξ adadadaA =′′?′=′′′=′′′=
∫∫∫
(5.27)
于是
∫∫
== AddAA ξξξξξξ (5.28)
也是由于A的任意性,得到这组连续编号基矢的完备性条件
Id =

ξξξ (5.29)
比如对能量表象和坐标表象,分别有
Inlmnlm
nlm
=

Ippdprrdr ==
∫∫
vvvvvv
当然,一般而言,基矢的完备性条件应当写为
Id
i
ii
=′′′+


ξξξξξ
可以将这些完备性条件作为单位算符,插入运算式中适当地方,有利于运算的进行。这从后面的例算中可以看到。
此外,从内积定义的物理解释可以得到
h
vv
h
vv
/
2/3
)2(
1
rpi
epr
=
π
h
vv
h
vvvv
/
2/3
)2(
1
rpi
eprrp

==
π
这里前面的系数是为了保证连续态归一到δ -函数,比如
)(
)2(
1
/)(
3
rrpderppdprrr
rprpi
′?==′=′
∫∫

vvv
h
vvvvvvv
h
vvvv
δ
π
量子力学的第三公设(测量公设)所说的对状态A进行力学量?的多次测量后所得的平均值,现在用狄拉克符号表示即为
AA
AA?
=?
(5.30)
通常所研究的态总是归一的,于是
AA?=?
(5.31)
2,狄拉克符号的一些应用
在应用狄拉克符号作一些具体演算之前,先证明如下两个态矢等式


=′


=′
r
r
irp
r
r
ipr
v
v
h
vv
v
v
h
vv
(5.32)
证:第一式的左边右乘以任一右矢A,并插入两个完备性条件,得
∫∫
′′′′′′′′′′=′ pdpdAppppprApr
vvvvvvvvvv

= ′′ ′? ′′ ′′ ′ ′′
′? ′
∫∫
1
2
32
()
()()
/
/
π
δψ
h
vvv vvv
vv
h
eppp pdpdp
ip r
A
= ′′
′? ′

1
2
32
()
()
/
/
π
ψ
h
vv
vv
h
pe p dp
ip r
A

v
=?

′′
′? ′

i
r
ep
ip r
A
h
v
h
vv
vv
h
dp
π
ψ
1
2
32
()
()
/
/
Ar
r
ir
r
i
A


=′

=
v
v
h
v
v
h
ψ
)(,
由于A是任意的并且不依赖于参量
v
′r,这个结果便意味着如下的关于左矢的一个等式
r
r
ipr ′

=′
v
v
h
vv
第二个等式其实是第一个等式的厄米共轭,但为了熟悉运算,这里再予以证明。为此用任意左矢A作用,则其右边为
∫∫
′′′′′′′′′′=′ pdpdrpppppArpA
rrrrrrrrrr

()
∫∫
′′′′′?′′′′=
′?′′
pdpdepppp
rpi
A
rvrvrv
h
h
vr
/
2/3
)(
)2(
1
δψ
π

′′′=
′?′
pdepp
rpi
A
vvv
h
h
vv
/
2/3
)(
)2(
1
ψ
π

′′

=
′?′
pdep
r
i
rpi
A
vv
h
v
h
h
rv
/
2/3
)(
)2(
1
ψ
π?
rA
r
ir
r
i
A


=′

=
v
v
h
v
v
h
ψ
)(
同样由于左矢A中并不含变数
v
′r并且是任意的,可将等式右边的A向左抽出并从等式两边去掉,即得
r
r
irp ′

=′
v
v
h
vv
值得注意的是,等式左边的是量子力学的动量算符,而等式右边的
v
$
p i
r
h
v

只是对右矢
r′
v
中本征值
v
′r的微商运算,它已不是算符,不对任何态矢作用。这从上面运算过程可以清楚地看出。与此类似,还有另外两个态矢等式


=′


=′
p
p
irp
p
p
ipr
v
v
h
vv
v
v
h
vv
(5.3)
这可以插入坐标表象的完备条件进行类似证明。
下面利用狄拉克符号作一些运算,包括一些表象变换的运算,
计算在态
v
$
p A中的平均值,设A为归一的。在具体计算这个平均值时,可以选取任何表象进行。例如,可取坐标表象(即在态矢空间中取定连续基{ }r
v
)来表述这个内积表达式,
办法是在适当地方插入坐标基矢的完备条件,如同上面所做的那样。即
∫∫
′′′′′′′′′== rdrdArrprrAApAp
vvvvvvvvv

∫∫
′′′′′?′′′
′′
′=
rdrdrrr
r
ir
AA
vvvvv
v
h
v
)()( ψ
ψ
这一步利用了上面r
r
irp ′′
′′
=′′
v
v
h
vv
公式,于是
vvv
h
v
vvvv
$
() ( ) ( )prri
r
r r dr dr
AA
= ′′′
′′
′? ′′ ′ ′′
∫∫
ψψ
δ
= ′′? ′′?
′′
′′ ′ ′′
∫∫
ψδ
ψrrr i
r
rdrdr()( )( ( )
vvv
h
v
vvv
这里作了分部积分并用到态ψ
A
为束缚态的边条件。
vv
h
v
vv
$
()( ) ()pri
r
rdr
AA
= ′?

′′

ψ
ψ
这就是坐标表象里的表示式,和以前给出的结果完全相同。当然,也可以采用动量表象进行计算,即取连续基{ }p
v
来表述这个平均值的计算过程
∫∫
′′′′′′′′′= pdpdApppppAp
vvvvvvvv

= ′′′ ′? ′′ ′′ ′ ′′
∫∫
ψδψ
AA
pp p p pdpdp() ( ) ( )
v v v v v v v
= ′′ ′

ψψ
AA
pp pdp() ()′
v v v v
这是动量表象中这一平均值的含义。总括起来,对于
v
$
p在态A中的平均值可以有许多种表达式,这里给出三种,比较如下
vv
h
v v
$
()( ) ()prir
AA
=

ψψdr 坐标表象
vvv v v
$
() ()pppp
AA
=

ψψdp 动量表象
ApAp

vv
=抽象的狄拉克符号表示,无表象
对于ArA
v
的计算,可以利用上面给的另外两个态矢等式作类似的计算,这里不再赘述。
其它一切力学量算符的平均值运算也不难照此进行。
在狄拉克符号下,表象变换也十分清楚。比如前面叙述的从坐标表象向动量表象的变换
—— 傅立叶积分变换
Ud
i
pr
=

1
2
32
()
/
πh
v
re
v v
h
(它将ψ
A
r()
v
变换为ψ
π
ψ
A
i
pr
A
pdre()
()
()
/
v
h
vv
r
v v
h
=

1
2
32
),现在就可以表示为
Udrpr=

v v v
作为变换矩阵,U的矩阵元为rp
vv
,其行脚标
v
p和列脚标
v
r均作连续变化。于是,将ψ
A
r()
v
变为ψ
A
p()
v
的变换过程就表示为
v v v v v
pA prdrr A=

而它的逆变换为
Udpr
= ′′

1
p
v v v
U
1
和U相乘为1,是一个恒等变换(它的一般矩阵元为δ()
v v
pp? ′,和其后的ψ
A
p(
v
′)相乘
——即对积分为
v
′p ψ
A
p()
v
),而U和
1
v
′pA相乘为
v
rA。
这里,对算符的各种矩阵元及算符的表象变换再补充以下例算。以动能算符为例,有
)(
22
22
pp
m
p
p
m
p
p ′′?′
′′
=′′′
vv
v
v
v
v
δ (5.34)
∫∫
′′′′′′′′′′′′=′′′ pdpdrpp
m
p
pprr
m
p
r
vvvvv
v
vvvv
v
v
2
2
22
=
′′
′? ′′ ′ ′′
′? ′? ′′? ′′
∫∫
1
2 2
3
2
()
()
//
π
δ
h
v
vv vv
vv
h
vv
h
e
p
m
ppe dpdp
ip r ip r
=


′? ′? ′′

1
2 2
3
2
()
()/
πh
v
v
v vv
h
p
m
ed
ip r r
p
=? ′′
′? ′? ′′

h
h
v
v vv
h
2
3
2
1
2m
ed
ip r r
()
()/
π
p
=? ′′? ′′
h vv
2
2m
rr? δ() (5.35)
这里,等式左边是抽象的狄拉克符号表示,等式右边分别是在动量表象和坐标表象中将它们具体地表述出来。利用狄拉克符号,很容易将第一个矩阵元(动能算符在动量表象中的矩阵元)转换到第二个矩阵元(动能算符在坐标表象中的矩阵元)。对第一个矩阵元的左方作表象变换
r
m
p
rrppdp
m
p
ppdprp
m
p
p ′′′=′′′′′′?′′′?′′′→′′′
∫∫
v
v
vvvvv
v
vvvvv
v
v
2
2
2
222
这里,等号是利用了动量表象基矢的完备性条件。如此同时,该式右边变为
∫∫
′′′′′′?′′?′
′′
′′′→′′?′
′′
rppdpp
m
p
pdprpp
m
p vvvvv
v
vvvvv
v
)(
2
)(
2
22
δδ
=


′? ′? ′? ′′

1
2 2
3
2
()
//
πh
v
v
v v
h
v v
h
e
p
m
ed
ip r ip r
p
=? ′? ′
′? ′? ′′

h
h
v
v vv
h
2
3
2
1
2m
ed
ip r r
()
()/
π
p
=? ′′? ′′
h vv
2
2m
rr? δ()
这就使动量表象中的第一个等式变换到了坐标表象的第二个等式。对任意态的矩阵元,在坐标表象中计算过程为,
∫∫
′′′′′′′′′= rdrdArr
m
rrAA
m
p
A
vvvvhvv
v
)
2
(
2
22
= ′? ′′ ′? ′′ ′′ ′ ′′
∫∫
ψδψ
AA
r
m
rr rdrdr()( ( ) ( )
v h vv vvv
2
2
= ′′? ′′? ′′ ′′ ′ ′′
∫∫
ψδ ψrrr
m
rdrdr()( )( ( )
vvv h vvv
2
2
= ′? ′′

ψψ
AA
r
m
rdr()( ) ()
v h vv
2
2
′ (5.36)
这就是在坐标表象里对A
m
p
A
2
2
v
的具体解释。推导中第二步等号是利用了前面动能算符在坐标表象中矩阵元的表达式,也可以认为是两次使用了
r
r
irp ′′
′′
=′′
v
v
h
vv
第三步等号利用了分部积分和ψ
A
的束缚态边条件。类似地,这种计算也可以通过插入单位算符
Ipdpp =

vvv
转入动量表象表述,
于是,若不选取任何表象,含时方程便抽象地写为 dingeroSchr &&
v
)())
(
2
()(
2
trV
m
p
tH
t
i ψψ
ψ?
v
h +==
这里,和
v
$
p
v
$
r均为算符。而若取坐标表象,则应将此方程向连续基
{ }
v
r投影,即
+?=+
==
)()]()(
2
1
[)())
(
2
(
),(
)(
)(
2
2
trrV
r
i
m
trV
m
p
p
t
tr
itr
t
i
t
t
ir
ψ
ψ
ψ
ψ
ψ?
vv
v
h
v
v
v
v
h
v
hh
v
=? +[( ) ()](,
1
2
2
m
i
r
Vr rth
v
vv
)
ψ
于是即得以前的坐标表象下的方程,dingeroSchr &&
这里附带指出,算符
AA
A
=π (5.37)
是向态矢A的投影算符。它的功能是
ABAB
A

将B态向A态投影,给出在B中所含A的几率幅。至于
2
ABBB
A

为在B态中找到A态的几率。由此,不难理解前面所用的各类基矢的完备性条件:由于基矢是完备的,则所有基矢投影算符之和当然是个单位算符。因为这表明,对任一归一化态矢,
∑∑∑
====
i
i
i
i
i
ii
bBBBBB
2
2
)(1 ξξξ
3,关于狄拉克符号的一个附注
用狄拉克符号表示的矩阵元BA?
可以有两种理解
{ }BABA?=?

,或{ }BA?
如前面所说,这里的bra?
A应理解为ket A
+
的厄米共轭。若
$
是厄米和幺正这两类算符(更一般地,只要?是线性算符),两种理解结果相同,于是这种含混不会引起问题。因为,不论?是厄米还是幺正,都有
$
$
{} { }
+
++
=?=?=? ABBABABA

{}{ }BAAB?=?=
+
+
)
(
从内积的两种表示相等( )(,
$
)(
$
,)AB AB=
+ a
也可以看出这一点。但是,当为反线性算符时(代表时间反演操作的反幺正算符包括在其中),对用狄拉克符号表示的矩阵元的这两种理解将导致不同的结果。这是因为反线性算符
$
$
π不存在通常定义下的厄米共轭算符(参见前面所述反线性算符的厄米共轭算符定义),
$
π
+
(,
$
)(
$
,)AB ABππ≠
+
因为此式左边关于、A B都是反线性的,而右边(不论怎么取)关于、
$
π
+
A B都是线性的。
同样,对一个反线性算符
$
π,也有
{ } { }BABA ππ ≠
因为,左边内积关于、A B均为反线性的,而右边内积关于、A B均为线性的,甚至,π?A
是无法定义的(因为对应的不存在)。于是,这是必须区分下面两种情况
$
π
+
{ }BA π?和{ }BAπ?
或者返回到更精密的记号
AB AB,
$$
ππ≡,
$
,
$
ππAB AB≡
§5.4 Wigner定理
b
1,Wigner定理
“如果一个使体系在物理上保持不变的变换将体系的每个态矢ψ变为ψ′,则总可以调节相位,使得对所有ψ
a
第二种理解
{}
AB
$
相应于。这是因为 (
$
,)?
+
AB
AA A
$
(
$
)(
$
)
$
===
++ ++ +
A。
b
参见Gottfried,P.226;J.R.Taylor,P.91;Encyclopedia of Math,& its Appli.,Vol.9.,P.160。
不是ψψ U=′,就是ψπψ =′
这里U、π分别是某个幺正或反幺正算符。”
证明:设有一变换使正交归一基{ } { }
nn
αα ′→,并保证对任意两个态矢均有
baba ′′=
于是取?和ψ如下,
m
αα? +=
1


=
n
nn
c αψ
变换后成为
m
αα? ′+′=′
1


′′=′
n
nn
c αψ
按规定应有
ψ?ψ? ′′=
这导致
cc cc
mm11
+=′+ ′
在不影响物理内容情况下,可以选ψ′的相位,使得′ =c。接着,展开这个绝对值等式,
可得
c
1
m
1
cc cc cc cc
mmm11 1 1

+=′ + ′
乘以c,并注意′
m
′ =cc
m
22
m
,得
cc c cc cc cc
mmm m m1
2
11 1
2
0

′? ′ ++ =()
解此二次方程,得
′ =
c
c
c
c
c
m
m
m
1
1
不影响物理内容,还可以进一步选定ψ的相位,使为实数,于是得到 c
1
′ =
c
c
c
m
m
m
若为前者,变换是幺正的;若为后者,

′=′
n
nn
c αψ
变换是反幺正的。
这可以进一步证明,每一个变换只能是二者之一。这里不再赘述。
2,讨论
i,“使体系在物理上保持不变”的变换均称为体系的对称变换。其含义是这样一种变换,它保持体系的全部可观测几率、全部力学量的期望值不变。一句话,凡有物理意义的、
可在实验上观测到的量均不变。
ii,Wigner定理也可以换一种说法,
“微观力学体系之间如果是物理上完全等价的,充要条件是在它们之间以一个幺正(反幺正)变换相联系”。
也可以叙述成态矢Hilbert空间的一条定理,参见文献
a

iii,要补充指出,在幺正变换下,原先表象的任何代数关系均不变;而在反幺正变换下,
原先表象的任何代数关系中的常数均应代以复数共轭数。特别地,基本对易规则中的i应h
a
P.Roman,Advanced Quantum Theory,P.634。
代以?。 ih
t(; )
0
§5.5 量子力学的路径积分表示
1,传播子与费曼公设
先引入态矢的时间演化算符Ut,t(; )
0
)();()(
00
tttUt ψψ = (5.38)
Ut的物理意义是把体系的t时刻态矢演化为时刻的态矢。这种演化当然由体系的哈密顿量所决定,是一种动力学演化。如果
0
t
H不显含,则从态矢方程 t dingeroSchr &&
)(
)(
tH
t
t
i ψ
ψ?
=h
可得
)()(
0
)(
0
tet
Htti
ψψ
h
=
就是说
Utt e
it t H
(; )
()/
0
0
=
h
在坐标表象中,上面态矢的时间演化具体表述为

′′′= rdtrrttUrtr
vvvvv
)();()(
00
ψψ
或为
ψψ(,) ( ; ) ( )
v v v v v
rt Urtrt rt dr= ′′

00

这里的积分核
rttUrtrtrU ′=′
vvvv
);();(
00
v
被称为“传播子”。它的意思十分清楚地表现在ψ(,)r t的积分表示式里,说明ψ(,)
v
r t从何处传播来以及怎样传播来
a
。若取ψ δ(,) )(
v v v
′ = ′?rt r r
00
,这时
v v v
ψ(,) ( ; )rt Urtrt=
00
这是说,传播子Urtrt(; )
v v
00
是这样一种几率幅,它是当时刻粒子位于t
0
v
r
0
处,演化到时刻在
t
v
r处粒子的几率幅。于是,对任意初始的ψ ()
v
′rt
00
,在后来的时刻t
v
r处该粒子的几率幅便可以乘以ψ ()
v
′rt
00
t>
并对全部r积分得到。这里应当强调指出,上面这个传播子Ur
按其物理含义可知,只当适用。为了这个物理的理由以及数学的理由(见下面第三节),
方便的是将其乘以单位阶跃函数
v

0
t
0
tr(;
vv
t)
00
θ(tt)?
0
,就是说,当tt<
0
时令其为零。这便是系统的推迟格林函数(第三节叙述),或称推迟传播子。所以严格地说应有
)();(
00
)(
00
0
ttrertrtrU
Htt
i
=

θ
vvvv
h
(5.40)
下面在不产生误会的情况下常常略写θ(tt)?
0
因子。
例如,对自由粒子,H
p
m
=
2
2
,它的时间演化算符为


= pdppettU
m
p
tti
vvv
h2
)(
0
2
0
),(
而它的传播子为

== pdrpprerttUrtrtrU
m
ptt
i
vvvvvvvvv
h
0
2
)(
0000
2
0
),();(
a
严格说来,应有Urtrt rUtt r t t(; ) (,) ( )
v vv v
00 0 0 0

eLalo&&.
,是“推迟传播子”,详见下面叙述,亦见
C.Cohen-Tannoudji,B.Diu,F,“Quantum Mechanics”,Vol.1,P.331,John Wiley & Sons,1977。
=
+


1
2
3
2
0
2
0
()
() ( )
πh
v
h
v v v
h
ed
i
tt p
m
i
pr r
p
=

(
()
) exp(
()
())(
/
m
itt
im
tt
rr tt
22
0
32
0
0
2
0
π
θ
hh
vv
)
可以直接检验,当时,这个传播子满足方程。 tt>
0
dingeroSchr &&
在路径积分框架里,对有相互作用的一般量子体系,它的传播子Ur是根据下面费曼公设
trt(; )
vv
00
a
给出的,
i,查出连接()
v
rt
00
和()
v
rt的全部
..
路径( ); tt>
0
ii,对每条路径
v
r t(),算出作用量; SLrtd
t
t
=

(())
v
τ
0
iii,即为相因子对全部
..
路径求和 Urtrt(; )
vv
00
e
iS /h
Urtrt N e
iS
(; )
/
v v
h
00
=

全部路径
(5.41)
这里N是归一化因子。
这里需要对这个公设本身和“全部路径”的含义作一些解释:设粒子在时刻处于的几率幅为
t
0
v
r
0
ψ (
v
rt
00
),经过演化,到时刻位于t
v
r处的几率幅为ψ()
v
rt,费曼公设主张ψ()
v
rt
按下式决定
ψψ() (; )( )
v v v v v
rt U rt r t r t dr=

00 00 0
(5.42)
并且Ur由上面式子计算。至于“全部路径”的含义可参见下图。将t时间分为个等间隔
trt(; )
vv
00
t
0
n ε =?
+
t
i1
t
i
,对
每一时刻(任意)指定一个位置,t
i
v
r
i
这些指定的
v
ri n
i
(,,...,)=?12 1
v
的集合(加上两个固定的端点,r
i00
|
=
v
r
in
|
=
)便构成一条路径。重新指定
一个、数个或全体
v
r
i
便又构成了另
一条路径。全部路径的含义是,
每一时刻所对应的r均可独立地,t
i
v
i
任意地改变。形象地说,粒子在
(tt
0
v
)内将以一切可能画出的路径
从点到达点。费曼公设认为,每条路径对最终几率幅都只贡献一个相因子。严格说
r
0
v
r e
iS /h
来,由于这里的划分是有限区间的划分,仍然不能算是穷尽了全部路径。只当令ε → 0,
,取极限,才算是考虑了全部可能的路径。于是将传播子Urn →∞ trt(; )
v v
00
写出来便成为
v v
Urtrt
A
i
L
rrrrt tdr
A
dr
A
n
iiiiii n
i
n
( ; ) lim exp{ (,,)}
vv
L
h
v v v v
K
00
0
3
111 1
3
1
3
0
1
1
22
=
+ +

→∞
++
=



ε
ε
ε

∫∫
exp{ (
&
(),(),) } ()
i
LrtrttdtDrt
t
t
h
vv v
0
v
(5.43)
v
这里,tt
n
= r
n
= r,并且对取极限的无穷多重数的重积分(泛函积分)采用了特别的标记
Dr t()
v
。后一表达式的含义即为它前一道表示式。是每重A
3
dr
i
v
积分的归一化系数,它由
a
R.P,费曼,A.R,希布斯,量子力学与路径积分,科学出版社,1986。
体系拉氏量的具体形式决定。当L L
m
rVrt=?
2
2
vv
&
()时,的表达式在下节中给出。A U中多出的一个
1
3
A
是考虑到
)
v
dr
dr
v
0
0
v
(r
v
tt
ii
v
r
i+1
+
)
1
(d
i
r
v
)
i
p
v
(
(
i
rd
v
2
(
i
r
h
v
π
i
i
e
h
++
dp
v
h
v
}
2
()π
11
3
()
}
h
i
()
1
πh
++
(
i
22
1
v

v
rt
ε
)
v
rt
i
+
()rt
ψψ() (; )(
v v v v
rt U rt r t r t=

00 00 0
表示式里对积分归一化的需要。
下面首先来证明,在H不显含时间情况下,前面的
)(
00
0
); rertrt
Htt
i
vv
h

=
tt
ii

+1
U表示式和现在用作用量相因子路径积分的表示式是一致的。为此考虑这个小区间。
注意,在此小区间中路径仍为任意可能的,因此即便
+
=
1
ε很小,
v
r也不一定和很接近。这时
i

++
= )(,(
11 iii
H
i
iiii
trrerrdtr
vvvvv
h
ψψ
ε
∫∫∫
++++
= )()
1111 iiii
H
i
iiiiii
trppepprppr
vvvvvvvvv
h
ψ
ε
∫∫∫
=
+
+

+
+
++
)((
)2
)
1
)
2
,()(
3
1
1
11
iii
rr
pH
i
rprp
i
ii
trpee
pp
ii
iiiii vv
h
vv
vv
v
h
vvvv
h
ψδ
π
ε
∫∫
+

+
+
= )(
)(
)
)
2
,()(
3
1
1
ii
rr
pH
i
rrp
i
i
tre
pd
ii
iiii v
v
vv
v
h
vvv
h
ψ
ε
∫∫

+
+
+
=
m
pi
rrp
i
i
ii
rr
V
i
iii
ii
e
pd
trrd
2
)(
3
)
2
(
2
1
1
)2(
)(
v
h
vvv
h
vv
h
v
vv
εε
π
ψ
v
其中后一关于的积分 p
i
=+?+
+

im p
m
p
m
rr rr
im
rr
iii
ii ii ii
h
v v
vvvv
h
v
exp{ [ ( ) ( ) ] ( )
2 2
2
2
3
22
2
2
1
ε
εε
ε
=?
+

1
2 22
321 2
exp{ }exp{ ( )
π
η
ε
η
ε
εhh
v v
d
im i m rr
i
=
+
(,)
/
( ) exp{ ( ) }
544
3
32
3
3
1 2
2
2
2
πε
ε
ε
ε
h
h
v v
im
mimrr
ii
故整个积分
=
+

() (
/
[( ) )]
m
i
dr e r t
i
imrr
V
rr
ii
ii i
2
32
1 2
πε
ψ
ε
ε
h
vv
h
v v v
)
=
++
+
dr
A
er
i
i
L
rrrr
ii
iiii
v
v
h
vvv v
3
2
11
ε
ε
ψ
(,)
()
,
v
t
由这个结果出发,向过去反推至(rt
00
、向将来推至(),并取极限,即得
v v v
)
ψ
ε
ψ
ε
( lim exp{ (,)} ( )K
v
K
h
v v v
v
dr
A
dr
A
dr
A
i
L
rrrr
rt
n
ni
i
n
=
+

→∞
=



0
1
3
1
3
0
3
11
0
1
00
2
与前面ψψ(; )( )
vvvvv
U rt r t r t dr=

00 00 0
相比较可得
Urtrt
A
dr
A
dr
A
e
n
n
i
L
i
n
( ; ) lim
vv
L
v
L
v
h
00
0
3
1
3
1
3
1
0
1
=


→∞
∫∫
=
ε
ε
=
∫∫
exp{ ( ( )) } ( )
i
Lr d Drt
t
t
h
vv
ττ
0
=

vv
h
re r
i
tt H()
$
0
0
应当指出,传播子的e路径积分表示式比
iS /h
e
i
tt H
h
()
$
0
坐标表象矩阵元表示式更为普遍,因为前者对含时情况仍适用。
$
()Ht
2,和方程的等价性 dingeroSchr &&
经证上述表示下的ψ()
v
rt满足方程。为将问题阐述得更清楚,这里用两种办法来证明。第一种办法是从路径积分表示式出发,
dingeroSchr &&
ψψ() (;)(
v
)
v v v v
rt dr U rt r t r t′ = ′′′ ′

v v
这里U用路径积分表示式。假设′ = +ttε,ε → 0,
v
′ = +rrη (注意
v
η不一定很小,如前所说),这时可取一重积分,即有
ψε
ε
ε
ψ(,) exp{ [ ( ) (,)]} ( )
v
v
h
v v v v
v
rt
dr
A
imrr
V
rrtt
rt+=
′? ′
+ ′ + ′

∫ 3
2
222
=?++ +

d
A
im
Vr t r t
v
h
v
v
v
v
vηεη
ε
ηε
ψη
3
2
2
222
exp{ [ (,)]} (,)
由于第一个指数中包含
v
η
ε
2
,当不是很小时,这项指数将快速振荡,从而对
v
η
2
v
η积分贡献非常小;仅当是
v
η
2
εh
m
v
η
2
量级时,第一个指数的位相改变为1弧度的量级,积分的主要贡献正是来源于这个量级的值。于是,将ψ展开,仅需保留ε量级的项,与此相应,对
v
η则要保留到量级的项,
v
η
2
v
ψε
ψ
ηε
ψη
ψ
η
η
ε
()
()
( ( ))[ ( )
()v
v v
h
vv
v
v
v
h
rt
rt
t
d
A
e
i
Vrt rt
rt
im
+=? +?
∫ 3
2
2
1 +
+++
1
2
2
2
2
2
2
2
2
2
2
η
ψ
η
ψ
η
ψ
xyz
]
利用数学公式(仅取η
x
情况,η
y
、η
z
类似)
de A
i
m
x
im
x
η
πε
η
ε
2
2
12
2
h
h

+∞

==()
/

d
A
e
x
x
im
x
η
η
η
ε
2
2
0
h

+∞

=
d
A
e
i
m
x
x
im
x
η
η
ε
η
ε
2
2
2
h
h

+∞

= (对第一个积分等式两边ε作微分,即得此等式) (5.44)
上式变成
ψε
ψ
ψ
ε
ψ
ε
ψ+=?+
t
i
V
i
mh
h
2

i
tm
Vh
h?ψ
ψ=? +()
2
2
第二种办法是从下式

′?
=′
000
/
)(
)()( rdtrrertr
Htti
vvvvv
h
ψψ
出发。现假定′ = +ttε,于是此式成为

=+
000
)(),( rdtrrertr
Hi
vvvvv
h
ψεψ
ε

=
000
)()
1( rdtrrHir
vvv
h
v
ψ
ε
注意到
∫∫
= )()()(
000000
trrr
r
irdtrrprrd
vvv
v
h
vvvvvv
ψ
ψ
= =?

i
r
dr r r r t i
r
rth
v
vvv v
h
v
v?
δψ
ψ
000
()() ()
代入上式即得
ψεψ
ε?
ψ(,) ( )
$
(,)(
vv
h
h
v
vv
rt rt
i
Hi
r
rrt+= )
也即
i
t
Hh
ψ
ψ=
$
值得注意的是,在导出方程的同时,也给出了坐标表象中坐标算符和动量算符的表达式,就是说也给出了它们之间的对易规则。
dingeroSchr &&
3,传播子Urtrt(; )
v v
00
的再研究
其实,上述传播子即为方程的格林函数。为此这里导出它所满足的微分方程。
dingeroSchr &&
+=?


)(
)}({
121
/
)(
2121
/
)(
2
2
12
12
ttreHr
i
ttrer
t
Htt
i
Htti
θθ
vv
h
vv
h
h
h
)(
121
/
)(
2
12
ttrer
Htti
+

δ
vv
h
)()(
121
)(
2121
)(
22
1212
ttrerttrerH
i
Htt
i
Htt
i
+=

δθ
vvvv
h
hh
这里已经用了
222
ripr
v
h
vv
=、
$
(
$
,
$
)
$
(,Hpr H i r
22
)
2
v v
h
v
=将算符转化为对的作用。同时,由于第二项中含有
v
$
p
v
r
2
δ(t )t
12
因子,因此可令该项矩阵元内指数上,于是可得
t
2
= t
1
i
U
t
HU i r r t thh
vv?
δδ
(;)
$
(;) ( )( )
21
21
2
221
=?
21
)
(5.45)
这里已简记Urt rt U(;) (;
v v
22 11
21=。这就是U作为量子体系推迟格林函数所满足的微分方程。严格地说,此方程再加辅助条件
(;)21
Urt rt t t(;),
v v
22 11 2 1
0= <
才可以完全决定所要的推迟格林函数。
同样,若H不显含时间,将有定态解完备集合
=
=

1
n
nn
nnn
EH


于是这时传播子可明显地写为
)()1;2(
121
)(
2
12
ttrer
Htt
i
=

θ
vv
h
U
)(
121
)(
2
12
ttrer
mn
nn
Htt
i
mm
=


θ
vv
h


θ
nn
i
ttE
n
rre tt
n
() () ( )
()
vv
h
12 2
21
1
dr
(5.46)
另外,由于时间上相继发生的事件,其几率幅将相乘,于是将所有的路径进行两次或多次分割是完全可能的。比如,这时将有
UUU(;) (;) (;)31 32 21
2
=

v
这从路径积分表示式的思想来看或直接在上面式子里插入坐标表象基矢完备性条件来检验均可看出。
4,路径积分计算举例——自由粒子情况
对自由粒子情况,前面已经得到了传播子的表达式。这里再次用路径积分方法直接给出它。这时L
m
r=
2
2
v
&
,



=
+

∞→
= })(
2
exp{
)
2
()
2
(
)
2
(lim);(
1
0
2
1
2/3
1
2/3
1
2
3
0
00
n
i
ii
n
n
rr
im
m
i
rd
m
i
rd
m
i
trtrU
vv
h
h
v
L
h
v
L
hvv
ε
επεπ
επ
ε
vv
(5.47)
这里rr,nt
n
= tε =?
0
。由于这时x、、三个自由度彼此独立,计算完全相同,为书写简单只对
y z
x自由度进行这个多重积分计算。先从对的积分开始,将这个积分乘以x
1
(
2
)
1
2
πεi
m
h
(此因子原在重积分号之外),即得
() exp{[( )( )
2
2
1
121
2
10
2
]}
π ε
ε
i
m
dx
im
xx xx
h
h

+∞
+?

( ) exp{ [( )
()
]}
2
24
1
11
202 20
2
πε
ε
i
m
dx
im
x
xx xxh
h

+∞
+
+

利用积分公式
1
2
1
0
11
0
2
2
π
α
α
α
α
α
edy
i
i
iy

+∞

=
>
<
()
()
上式即为
( ) exp{ ( ) }
πε
ε
i
m
im
xx

22
22
1
2
20
2
h
h
接着将此结果乘以
( ) exp{ ( ) }
2
2
1
2
32
2
πε
ε
i
m
im
xx
h
h
再进行积分,x
2
m
i
dx
im
xx
im
xx
πε
εε
22
42
220
2
32
2
h
hh
+?

+∞

exp{ ( ) ( ) }

∞+
∞?
++?
= ]})(
3
2
))2(
3
1
3[(
4
exp{
22
2
03
2
0322
xxxxx
im
dx
i
m
ε
επ
h
h
=

( ) exp{ ( ) }
πε
ε
i
m
im
xx
23
23
1
2
30
2
h
h
由此重复并递推下去,直至完成对的积分之后,将得到 x
n?1
( ) exp{ ( ) }
πε
ε
in
m
im
n
xx

2
2
1
2
0
2
h
h
v v
注意nttε =?
0
,于是得到Urtrt(; )
00
的x方向部分。、部分计算及结果类似。证毕。 y z
也可以采用维重积分下述公式一次性求积给出上面的结果。已知公式 n
exp{ }
det
,
~
+ =
==

+∞
∑∑

M x x x dx dx
M
e
ij i j
ij
n
ii
i
n
n
n
M
11
1
1
4
1
α
π
αα
L (5.48)
a
这里积分收敛的充分条件要求复对称矩阵M满秩并且实部至少为半正定。对自由粒子情况,

()
xx x
n
=
11
,,L M
im
M=? ′
2hε
为()n?1维对称矩阵,而
′ =

M
210 00
12 1 0
012
000 12
,,,,,
,,,,
,,,,
,,,,
,,,,,
O
O
OO
OOO
O

~
(,,,,)
()
α
ε
=?
im
xx
n
h
L
1244344
0
1
00

可用归纳法验算,det ( )M
im
n
n
=?
2
1
hε;其逆矩阵为
M
M
M
ij ij T?+
=?
1
1
1
det
(( ) det )
(,)
这里M
ij(,)
是从()n?1阶M矩阵中删除第行、第i j列之后的(n )? 2阶矩阵,T是对括号中矩阵元为的(()?1 Mdet
+ij (,)ij
)n?1阶矩阵的转置。于是可得M
1
的4个有关的矩阵元为
()() ( )
,,
MM
im n
n
nn


==?
1
11
1
11
1
2
1

() () ( )
,,
MM
im
n
nn
==?
1
11
1
11
1
2
1

后面两个矩阵元相等是由于对称矩阵的逆仍是对称矩阵。注意,除去第1行第列后的阶矩阵
(n?1)
)(n? 2 ′
M
n(,)11
的行列式为。将以上结果代入重积分公式,得 ()?
1
2n
++
=

∞→
22
0
2
1
1
2/
0
00
)(
4
1
)(
2
exp{
)
2
()
2
(
1
lim);(
εε
ε
π
επ
ε
hh
h
h
im
xx
im
n
m
i
m
i
txxtU
n
n
n
n
()?
xxM
x
x
0
1
0
00
0
0
,,,,}LM
=+?+
+
→∞

lim( ) exp{ ( ) [( ) ]}
/
n
m
in
im
xx
im
xx
n
n
xx
n
ε
πε ε ε
0
12 2
0
2
0
22 0
22
1 2
hh h
a
此公式可如下证明:先作维的矢量平移变换,将指数化为齐次形式,然后作变换n xM=
1
2
y,化指数为平方和形式即可积出。
=

(
()
) exp{
()
()
/
m
itt
im
tt
xx
22
0
12
0
0
2
πhh
}
5,路径积分计算举例——简谐振子情况
这时Lr t mr m r(())
&
vv
=+
1
2
1
2
2
ω
v
2
,仍可分别对x、、计算。下面以y z x方向为
例。由于这时ω ≠ 0,已很难如上面自由粒子的第一种方法那样逐次积分再递推去得到最后结果。这里采用上例中的第二种方法,即一次性对(
x
i
)n?1重积分求积的办法。这时
Uxtxt
i
m
dx dx
im
xx x x
n
n
nnn
(; ) lim
()
exp{ [( ) ( )
/
00
0
2
11 1
2
12
2
1
2
2
=?
→∞


∫∫
ε
πε
ε
h
LL
h
n
++
)]}
++ + ++ +

LL() (xx x x x x
nn10
222
1
2
2
2
1
2
0
2
εω
=
→∞


∫∫
lim( ) exp{
~~
[( ) ]}
/
n
n
nn
m
i
dx dx M
im
xx
ε
πε
α
ε
εω
0
2
11 1
22
0
22
22
1
h
LL
h
ΧΧΧ +
这里,
()Χ=
xx
n11
,,L ()α
ε
=?
im
xx
h
L
0
0,,,,,为M
n?1
(n )?1阶对称矩阵,
M
im
M
nn
= ′
1
22
1
2
2

εω()
′ =

M
n 1
1000
10
01
0
000 1
,,,,,
,,,,
,,,,
,,,,
,,,,,
δ
δδ
δ
δ
δ
O
O
OO
OOO
O
,δ
ε ω
=
>
1
2
1
2
22
,( )
记阶的的行列式,按对k ′M
k
det ′ =MD
kk
′M行列式第1行展开可知
DD D
nn n
=?
1
2
2
δ
于是可得
D
D
D
D
n
n
n
n?
=

1
2
1
2
1
10
δ
递推下去,并注意,即得 DD
10
1==
D
D
n
n
n
=

1
2
1
1
10
1
1
δ
矩阵?的两个特征根及特征向量为
1
10
2

δ
λδ
ε
εω
±

→∞
±
=±?

1
2
14 1
1
2
2
0
()ie
n
i
λ
+
1

λ
1
将按这两个特征向量展开,
1
1
1
11
=
+
+?
+
+?
λ
λλ
λ
λ
λλ
λ
1
代入递推公式即得
det
sin( )
′ ==


+?
+?
→∞
MD
n
nn
nn
n
n
11
1
1
2
λλ
λλ
ωε
ωε
于是
det ( ) ( ) ( )
sin( )
M
im
D
im n
n
nn
n
n n

→∞?
= →
1
1221
1
1
2
2
2hhε
εω
ε
ω ε
ωε
此外,由逆矩阵的表达式,的4个有关矩阵元为 M
n?
1
1
()()
det
det
sin( ( ) )
sin( )
,,
MM
M
Mim
n
n
nnn
n
n
n

→∞
==?→
1
1
11 1
1
11
2
1
21hε ω ε
ωε
= +
2
1
2

ωε ω ε ε
im
nO(cot()()
() ()
()det
det sin( )
,,
(,)
MM
M
Mim
nn nn
n
n
n
n
n

→∞
==

1
1
11 1
1
11
1
11
1
1 2hεωε
ωε
)
n
这里O代表量级为的剩余项,且。将以上这些结果代入中并利用重积分公式,得

2
txt(;
00
ε
2
det ( )
(,)
′ =?
M
n
n
1
11 2 2
1 δ
Ux )
Uxtxt
m
i im n
im
xx
im
n
n
nn
n
(; ) lim( ) (
()
()sin()
) exp{ ( ) [( ) (
//
00
0
2
11
1
12 2
0
22
2
21
4
2
1=
+
→∞


ε
πε
πεωε
ωε ε
ε
h
h
h
h
+
+? +ωε ω ε
ε ωε
ωε ε
εω
ε
cot( ))
sin( )
]( )nxx
im n
im
x
im
x2
2
2
1
2
0
22
0
22
h
hh
}
=

+(
sin ( )
) exp{
sin ( )
[( ) cos ( ) ]}
/
m
itt
im
tt
xx tt xx
ω
πω
ω
ω
ω
22
2
0
12
0
0
22
00
hh
(5.49)
用这种方法也可以计算外力驱动下的受迫振子的传播子。拉氏量为
L
m
xxft=?+
2
222
(& )()ω x,这时只要添加逆矩阵的矩阵元()
的表达式,计算过程类似,结果可见文献
M
n?
1
1
()
,,
MM
nj nj?
=
1
1
11
1
1
a
,这里从略。
如果假设已经知道了谐振子问题的本征值和本征函数,也可以采用前面的另一公式更简捷地计算出传播子。为了再次演示量子力学的路径积分框架和通常的表述是等价的,下面给出这个计算。由前面叙述可知
Uxtxt x x e t t
nn
i
tt E
n
n
(; ) () ( ),( )
()
00 0 0
0
=>



h
这时En
n
=+()
1
2
hω,并且
ω
π
ω
ω
n
n
n
m
x
xn
m
Hx
m
e() ( !) ( ) ( )
//
=
2
12 14
2
2
hh
h
于是
Uxtxt
m
enHx
m
Hx
m
e
m
xx
i
tt
n
nn
in t t
n
(; ) ( !) ( ) ( )
()()
()
00
22
1
0
0
2
0
2
0
0
2=
+

=


ω
π
ωω
ω
ω
ω
hh
h
利用厄米多项式的积分表示
a
结果见费曼和希布斯所著“量子力学与路径积分”;计算过程可见许多量子场论方面的书籍,比如,戴元本,“相互作用的规范理论”,第77页,科学出版社,1987年。
He
i
ed
n
nin
() ( )ξ
π
ττ
ξτξτ
=
+

+∞

1222
2

Uxtxt
m
edd x
m
i
m
xx
i
tt
( ; ) exp{
()()
00
3
22
22
2
0
2
0
2=∫∫′′ ++
+

+∞
ω
π
ττ τ τ
ω
τ
ω
ω
h h
h
+ ′


=


2
1
2
0
0
0
x
m
i
n
e
n
nintt
n
ω
τττ
ω
h
}
()
!
()
()
=?′? ′ + ′? ′
++

+∞

+∞

∫∫
m
ededx
m
ie
m
xx
i
tt x
m
i
itt
ω
π
τττ
ω
τττ
ω
ω τ
ω
τ
ω
3
22
2
2
0
2
0
2
0
2
0
22
h h
h h
()()
()
exp{ }
对′τ和τ的积分均形如下面积分
edy
y?

+∞

=
2
π
积分完成之后得
Uxtxt
m
ee
m
xx
i
tt
itt
(; ) ( )
()()
()
00
22
2
1
2
2
0
2
0
0
1=?


ω
π
ω
ω
ω
h
h

+


exp{ [ ]}
() ()
()
m xxxe xe
e
itt itt
itt
ω
ωω
ω
h
2
00
22
2
2
1
00
0
=

+(
sin ( )
) exp{
sin ( )
[( ) cos ( ) ]}
/
m
itt
im
tt
xx tt xx
ω
πω
ω
ω
ω
22
2
0
12
0
2
0
2
00
hh
如果在用路径积分方法直接得到这个最后结果之后,再继以上面计算的逆过程,便成为在路径积分框架下计算简谐振子本征值和本征函数的过程,正如费曼所做的那样。
最后,应当指出,还有另一种很常用的求积路径积分的办法,这就是将端点在和()xt
00
()xt的任意路径分解为经典路径及量子涨落两部分
xt x t yt
C
() () ()= +
相应地,作用量也被区分开来。可以证明,在这时
Uxtxt
i
Sxtxt Ftt
C
( ; ) exp{ ( ; )} ( ; )
00 00 0
=
h
(5.0)
这里指数中的S是拉氏量沿着经典允许的轨道从()到
C
L xt
00
()xt的时间积分。而只依赖于两端点的时间,与空间变数无关,它是由对的路径积分而得,常称之为涨落因子。
详见文献
F
yt()
a

§5.6 非惯性系中的量子力学
1,等效原理
b
众所周知,在经典力学中等效原理可分为强等效原理和弱等效原理。强等效原理说,如就局部所观察到的物理规律而言,引力的物理效应可用适当的加速度来代替,就是说,如果知道了一个体系在无引力场时的运动规律,根据此原理便可找出在有引力场时的运动规律,
办法是把它转移到适当的加速度坐标系中去便成了,因为这就等于给该体系引入了引力场。
假如这个强等效原理在量子力学中依然成立,那就知道了引力势进入方程的方式,办法是把方程写入加速度坐标系。后面的推导表明,方
dingeroSchr &&
oSchr &&dingeroSchr && dinger
a
量子力学与路径积分,科学出版社,1986;Path Integrals in Quantum Mechanics,Statics,and Polymer Physics,
World Scientific,1990。
b
D.M.Greenberger,Rev,of Mod,Phys.,Vol.55,No.4,875(1983)。或见张永德,裴寿镛,量子力学与引力,大学物理,第11卷,第4期,1992。
程的结构并不排斥强等效原理。
弱等效原理说,在外部引力场中运动的一个粒子,它的运动和它的质量无关,即它的质量不进入问题。正由于此,引力场中粒子的运动便成为纯几何问题。显然,Schr方程的结构是排斥经典的弱等效原理的。至少在低量子数的量子态中,弱等效原理已被破坏,
只在大量子数的经典极限下才成立。例如,对重力势的方程,当量子数n
足够大时(参见前面第三章一维问题的均匀场中运动)。
dingero&&
mgx dingeroSchr &&
粒子的几率分布和能量本征值为(当n,>>1? →?∞x
0
),
Px() () sin( )
/
=→ +ψξ
β
πξ
ξ
π2
2
12
2
3
2
2
34
(5.1)
E
F
m
nxF
n
=
1
2
91
4
22 21
3
23
0
()(),
/
π h
连续→ (5.52)
这里,而 Fmg=
ξ =?()(
/
2
2
13
mF
x
E
Fh
),β
π
=
()
/
///
2
13
12 16 23
m
F h
代入后可知,在向经典过渡,振动因子被抹平之后,P x()将与无关。 m
2,方程的广义伽利略变换dingeroSchr &&
a
前面提过,量子力学是支持强等效原理的。这种支持分两个内容:强等效原理和方程的兼容性,即引力势和非惯性力势是否以惯常的相同方式进入方程;量子引力实验是否支持这种进入方式。这里先讲第一方面。
dingeroSchr &&
dingeroSchr &&
引入“广义伽利略变换”,
=′→
=′→
ttt
trrr )(ξ
v
vvv
(5.3)
由于这时
= ′?
=

′?
ξ
tt
v
&
dingeroSchr &&方程
+=
h
h
2
2 m
Vi
t
ψψ
ψ
变换为
′′+ ′ =

′? ′
h
h
v
2
2m
Vi
t
ψψ
ψ
ξψ[
&
],′ ′ ′ =ψ ψ() ()
v v
rt rt
作变换
′′′= ′ = ′
′ =?′ +∫

ψ
ξξ
() (),(
() [
&
&
]
()
v
)
v
v
h
v
v
v
rt urte t t
Srt
m
rd
iS r t
t
1
2
2
t
则关于的Schr方程为 u dingero&&
′?++?′ =
h
v
v
h
2
2
2m
uVmrui(
&&
) &ξ u (5.4)
于是,在这个广义伽利略变换下,量子体系由惯性系进入非惯性系,方程将因此而多出一项等效位势项:
dingeroSchr &&
mrurt
v
vv&&
(ξ? ′′)
。这相当于有效引力
a
D.M.Greenberger,Rev,of Mod,Phys.,Vol.55,No.4,875(1983)。
F
eff
=?m
v
&&
ξ (5.5)
所带来的引力势项。显然,这就是量子理论中的强等效原理。它表示,量子理论允许引力势以经典的惯常的方式进入。由于微观粒子的质量总是十分小,一般情况下这项(以及重力势)
的影响不易觉察(是个“势微扰”,参见第十章)。但并不排除在某种特殊情况下(如黑洞附近)、
特殊安排下(见下)表现出可觉察的物理效应。
3,COW实验
1975年Colella、Overhauser和Werner第一次完成了量子引力实验,这就是著名的COW
实验。办法是在地球重力场中,通过转动中子干涉仪来检验不同重力势对中子干涉极
值的影响。1980年Staudenmann等人完成了一个更精确的实验
a
。详细请参见原来文献或本书第十章第二节。
COW实验在量子理论的历史上是十分重要的。它第一次测定了引力对量子体系波函数的影响,第一次实验验证了引力进入方程的方式(尽管电磁力、核力进入的方式均已得到验证,但引力毕竟是另一种基本力),第一次实验验证了在弱引力势下强等效原理在量子理论中仍然成立。
dingeroSchr &&
4,引力红移
在地球范围实验条件下,另一个可行的引力量子效应实验是重力场中光子的频率红移实验。
若用平均观点来看上面的非惯性系中方程,该方程所代表的能量关系式为
dingeroSchr &&
<动能>+<势能>+<克服等效引力作功所增加的势能>=<总能>
这一体现能量守恒观点的等式虽然是针对粒子( m ≠ 0 )导出的,显然对光子( m )也成立。
就是说,如果设想一个光子(
= 0
)hω在重力场中垂直向上飞行,应当有如下关系式成立
mgD
=?′hhωω
这里m
c
=

2
是光子的等效质量,为向上飞行的距离,hD ′ω是向上飞行距离后光子的能量,于是光子频率将产生红移
D
ω
ω
=?
gD
c
2
(5.6)
这一理论结果已于1960年由Pound等在哈佛大学校园一座塔内用穆斯堡尔效应测量所证实
b

a
COW实验:Phys。Rev。Lett.,34,(1975)1472;Staudenmann实验:Phys,Rev,A21,(1980)1419。
b
比如,参见E.赫克特,A.赞斯著《光学》,詹达三等译,高等教育出版社,1980年,下册第930页。
第六章 对称性及其应用
§6.1 一般叙述
1,对称性的含义
对称性的含义有广义和狭义两种。从广义来说,对称性是意味着和谐、规律、秩序,它和偶然杂乱无章相对立。事实上,对某种基本的对称性的信念,常驱动着人们去发展科学。
H,Weyl说:“对称性是这样一种意念,人们长年累月地试图以它去理解并创造秩序、美和完善。”就狭义来说,系统的某种对称性是指某种不可分辨性,对某种属性的不可观测,也就是在某种操作、变换下,系统仍然保持不变。比如,体系的空间绝对位置是不可观测的,这导致体系的哈密顿量在空间平移操作、变换下是不变的;立方体相对它的某根对称轴所做的几个特殊的转动之后,将和转动之前相同,或不可分辨等等。
这里应当指出,不同的体系所具有的对称性可能不同。但是同一个体系,所有使它保持不变的对称变换必定构成一个群。因为,接连两次或两次以上施加对称变换,其结果依然使体系不变,也就是说依然是个对称变换;对称变换的逆变换依然是对称变换;存在恒等变换
(“不变”这种特殊的对称变换)。这些已是使对称变换构成群的充要条件了。
研究对称性的意义在于,第一,检验和发展理论,按海森堡的观点,“应当用基本对称性概念去代替已成问题的基本粒子的概念”,“……,必须寻找的不是基本粒子,而是基本对称性”。第二,可以增强人们的物理直觉、简化问题、有利于迅速抓住问题的要点,可以不去求解方程而得到关于态和本征值的一些知识。第三,可以简化一些计算,
比如简化一些矩阵元的计算等等。
dingeroSchr &&
2,量子力学中的对称性
就基本的对称性来说,量子力学中存在的对称性要强于经典力学中存在的对称性。这就是说,经典力学中存在的对称性,量子力学中也对应存在,同时,量子力学中还存在一些在经典力学中不存在的对称性。前者如时空均匀各向同性的对称性,后者如全同性原理。然而,
个别对称性除外,弱等效原理(在引力场中运动的物体,其质量不进入问题,以致运动轨迹的研究成为一个纯几何问题)在经典力学中存在,但在量子力学中被破坏,只当向经典过渡时才又表现出这种对称性逐渐被遵守起来。
量子力学中常见的对称性有:时间的均匀性、空间的均匀性、空间各向同性、同类粒子的全同性原理(或交换对称性),这些对称性是普适的、严格成立的。空间反射对称性、时间反演对称性,这对许多体系严格成立,但终究不是普适的。同位旋对称性,这只是个近似的对称性。此外,还有各种晶体的各种旋转、反射对称性,这些也是属于部分体系(晶体)
的特殊的对称性。
以上这些对称性可以接下面两种观点予以分类:第一,按变换是连续的或分立的来分类。
比如,晶体的对称变换、空间反射、时间反演变换、全同粒子置换等属于分立的变换,其余的属于连续变换。第二,按这些对称性是属于体系的内禀属性还是外在属性分类。空间和时间的平移、空间旋转对称性是体系新处的时空性质对体系运动方式提出的要求(即时空特性对体系哈密顿量的要求)。严格说,由此得出的对称性并不是系统的内在固有的禀性,是时
1
间空间固有属性在体系运动上的体现。与此相反,全同粒子置换对称性和同位旋空间旋转对称性等,是体系内在对称性、是体系的内禀属性。而空间反射、时间反演对称性,也根源于体系内部的动力学性质,因而也应当认为是属于体系的内禀属性。值得强调指出的是,这种观点能深刻地揭示出,为什么从经典力学发展到量子力学时,许多观念、规律都遭到根本性的变换,但和时空属性相关联的能量守恒、动量守恒和角动量守恒这三大守恒定律却几乎原封不动的被量子力学继承下来。这说明,量子体系的行为虽然“乖张”(从经典观点看),但毕竟也运行在和经典力学同一时空之中,从而这个时空的属性也必定“烙印”或“体现”在量子体系的运动行为上,使之表现出(如同宏观体系已经表现出那样)这三个守恒定律的存在。
3,对称性与守恒律及守恒量
前面已部分地涉及了对称性和守恒律的关系。这里集中简要地论证这个问题。
由Wigner定理,可以断定,对称变换是幺正变换(或反幺正变换)。而按以前所说,一个幺正变换U总可以表示为
a

=
αi
eU
这里为厄米算符,? α对连续变换为连续变化的参数,对断续变换为分立取值的参数。这里暂时只研究连续变换。由于U是对称变换,必定使体系的哈密顿量不变,即有
HUHU =
1
也即
[ ] 0,=UH
或写为
[]


=
=?
0
0,
!
)(
n
n
n
H
n

由于α可取连续值(包括零值),可取α足够小,从而得到
[ ] 0,=?H
这就是说,如果体系在(连续变化的)对称变换U的变换下是不变的,则此对称变换U的生成元(厄密算子?)是个守恒量。或者说,此体系存在一种(连续变化的)对称性,就相应地存在一个守恒律及其守恒量。若U是分立变换,同样有
[ ] 0,=UH
对空间反射变换,这个算符既是幺正的又是厄密的,从而U直接就是守恒的力学量。
总之,当体系存在一种对称性时,必定伴随着它必须服从的一条守恒定律,从而存在一
a
这里只讨论幺正变换,反幺正变换参见附录。
2
个守恒的力学量。
a
§6.2 时空对称性及其应用
1,时间均匀和能量守恒定律
时间轴是均匀的,就是说,时间轴上不存在绝对的与众不同的点,因此一个孤立量子体系(孤立经典力学体系也一样)的哈密顿量中不可能显含时间,从而沿时轴平移这个孤立量子体系是不会造成任何物理上可察觉的变化。
当H不显含时,可求得时间平移算符t )(τU,它把体系在时间轴上向前平移τ。按
)(τU定义,应有)()()( ττ?Ψ=Ψ ttU,因为)(τU把体系t
0
t=时刻发生的事件推迟到
τ+=
0
tt时刻发生。即,原先t时体系处于
0
t= )(
0
tΨ。变换后τ+=
0
tt时,体系处于
)(
0
tΨ。这时按方程 dingeroSchr &&
)()( t
i
H
t
dt
d
Ψ=Ψ
h
(6.1)
于是
)()()()(
22
t
i
H
t
dt
d
i
H
t
i
H
dt
d
t
dt
d
Ψ
=Ψ=Ψ

hhh
或一般地有
)()( t
i
H
t
dt
d
nn
Ψ

h
于是
() ()
( )
() ( )
∑∑

=

=
+
=
=
=
00
!!
1
n
n
n
n
n
H
i
tt
dt
d
n
tH
i
n
te τψψ
τ
ψ
τ
ψ
τ
h
h

()tet
H
i
ψτψ
τ
h
=? )(
所以主动地将体系沿时间轴平移τ的算符或幺正变换为
h
τ
τ
H
i
eU =)( (6.2)
a
为上所申明的.这里只研究幺正变换的对称性.至于反幺正变换,对称性存
在并不意味着守恒的力学量存在。
3
按)(τU的定义,它和时间发展算符是反向的。并且显然)(τU是个幺正变换,它不改变体系的一切可观察物理效应。因为在这个变换下
态,)()()()()(
'
τψψτψψ?==→ ttUtt
算符,
1'
)()(
=?→? ττ UU
于是对任给的两个态矢)(t?,)(tψ有
)()()()()()()()(
1''
tttUUttt ψ?ψττ?τψτ?ψ? =?==
(6.3)

)()()()()()()()(
111'''
tttUUUUttUUt ψ?ψ?τψτττ?ψ=?==?

(6.4)
在经受)(τU变换的前后这些几率幅和矩阵元均不变。这说明H不显含t时,体系的确是时间平移不变的。或者说,这时采用时间轴上任何一点作为计算起点都不会产生可观察的物理效应。作为H不显含时间的一个例子,一个孤立系的情况就应当是这样的。
上面的论证也可以换一种说法.对任意两个态矢?,ψ计算如下
ψ?ψ?ψ
ψ?
dt
d
H
t
H
H
dt
d
H
dt
d
+
+?=
ψ?ψ?ψ? H
i
H
t
H
HH
i hh
11
+
+?
= (6.5)
[]ψ? HH
it
H
,
1
h
+
=
根据第一章关于微商算符
dt
dH
的定义
ψ?ψ?
dt
dH
H
dt
d

则有(由于?,ψ的任意性),
[]
t
H
HH
it
H
dt
dH
=+
=,
1
h
当H不显含时(孤立系就应如此),应有t 0=
t
H
和[ ] 0,=HH,即得
0=
dt
dH
(6.)
4
这意味着对任意?,ψ,ψ? H均不随时间变化
a
这显然意味着:第一,在任意态中H的平均值不随时间变化(取?为ψ即可);第二,在任意态中,H取某个本征值的几率不随时间变化,这只要取?为ψ并把ψ按H
的本征态
hh
r
tiEt
n
n
enr
=)(ψ
iE
n
e
展开即可看出,
hh
rr
tiE
n
n
tiE
n
n
n
enberbtr

∑∑
== )()( ψψ
n
n
n
t
EE
i
n
nm
m
EbenHmbbH
nm
2
*
∑∑
==
h
ψ?
这里,在),( tr
v
ψ中测H时,取本征值的几率
n
E
2
n
b不随时间变化。所以H是个守恒量。
这里应指出,说体系H守恒,并不等于体系一定处在H的本征态上。这时体系可以处在H本征态的叠加态上,这要视初条件决定,但不论处在何种状态,必须满足上述两点。
2,空间均匀性和动量守恒定律
类似可求得空间平移幺正算符U )(a
r
,即坐标系不动而把体系平移一段有限距离的变换。这时,按U的定义,对任意态矢均应有
a
r
)(a
v
),(),()()( tartraUtr
rrrrr
==′ ψψψ (6.7)
这里右边的“-”号可以这样理解:设体系为一团几率云,变换前
0
rr
vr
=处的几率云小片,
在变换后将位于
0
rar
vvr
=?,即arr
rrr
+=
0
处。这说明这团几率云移动了距离。于是 a
r
),()(
!
1
),()()()(
3
10
tr
x
a
n
tartraUtr
n
i
i
i
n
rrrrrr
ψψψψ
=?==′
∑∑
=

=
a
这也可直接按本征态展开检验 设
=
==

∑∑
h
h
rr
rrr
tiE
n
nn
tiE
m
mm
m
mm
n
m
erbtr
eratratr
)(),(
)(),(),(
ψψ
ψψ?

∑∑

==
n
nnn
tEEi
mnnn
mn
m
EbaeEbatrHtr
nm
h
rr
)(
),(),( δψ?,与无关。 t
5
)()exp()()exp( trpa
i
tra
rrr
h
rr
ψψ==
r
所以把体系作空间平移的算符 a
pa
i
eaU
rr
h
r

=)( (6.8)
按前面所说,这个幺正变换的生成元动量算符p
r
是个守恒量。就是说,如果一个体系具有平移不变的对称性质(对孤立系这是必定的),这个体系的总动量是守恒的。
和前面能量守恒情况类似,说动量守恒,并不等于体系一定处在动量的本征态上,这要看初条件如何而定。如初条件是一系列动量本征态的叠加(形成自由运动波包),则其后将一直保持如此。
3,空间各向同性和角动量守恒
由于我们所处的空间是各向同性的,并无特殊方向可言(有定向的外加场,如重力场,
破坏这种各向同性的情况除外。现在这里是讨论未遭任何破坏前的空间本身性质。此观点对前面的讨论也适用),所以把一个孤立体系绕任何轴旋转一个任意角度均不应当影响体系的任何物理性质。设该体系绕
n
e
r
轴转?角的变换所对应的算符为U ),(
n
e
v
,则有
),(),()()( trtreUtr
n
ρψψ?ψ
rrrrr
==′
这里?ρ )( re
n
vvv
×=。于是
),(),()()( trtreUtr
n
ρψψ?ψ
rrrrr
==′
),()(
!
1
0
tr
n
n
n
rr
ψρ=


=
[])()()(
!
1
trre
n
n
n
n
n
rrr
ψ×?=

[])()(
!
1
0
trre
n
n
n
n
n
rrr
ψ×=


=
)()exp( trLe
i
n
v
)
r
r
h
ψ
=
所以将体系绕
n
e
v
转?角的变换算符
()
Le
i
n
n
eeU
r
r
h
v

=
ψ
(6.9)
6
如果这些幺正变换对某个体系来说是使体系不变的对称变换,则生成元角动量L
)
v
守恒。这时三个分量均是守恒量,但由于它们彼此均不对易,不能同时测量,故不可能同时取各自的确定的本征值,就是说,比如当取某个本征值(也即这个态是的本征态)时,L将一定的几率分布各种可能的(共2
x
L hm
x
L
y
1+l个)hm′值,这时在单次测量时显然呈现取值的不碓定性,情况也和类似。但尽管如此,的取值和、的不同取值的几率分布都不随时间变化,后者导致
y
L
zz
L
y
L
x
L hm
y
L L
zy
LL,不随时间变化。
这里举一个例子,粒子在均匀无限圆柱形螺旋线场中运动,除能量之外求守恒量。
选柱坐标()z,,?ρ,由这种场的势的形式为
()αρ?tgzV?V =,
由于绕轴旋转z和沿轴平移z的算符分别为
()
()
=?
=?
)exp(
)exp(
z
zz
pz
i
zB
L
i
eA
h
h
v

令απρ
π
α?ρ tgddtgz 2,
2
=
≡=?为螺旋线的螺距。构造新算符
()( )
+==
zzz
p
d
L
i
tgBeAC
π
αρ?
2
exp
h
v
(6.10)
由于
()
=
=
=
+?
0,
2
,
2
.0
2
,
22
zz
zz
p
m
p
L
m
p
p
d
LtgzV
π
αρ?
所以
[ ] 0,=HC
于是在此等螺距的螺旋线场中,下面算符及其对应的物理量
zz
Lp
d
+
π2
7
守恒。
以上讨论的是关于时空的连续对称变换。由于时空内禀地具有均匀各向同性的性质,在这些属性不遭受外来破坏时就会表现出来,并构成三个普通的守恒定律。对孤立系,不论其中发生的是什么过程,均会因为体系所在的时空固有这些属性而必定遵守这三个定律。于是,
如其说这三个守恒定律是体系本身的性质,不如说是时空均匀各向同性性质在体系运动行为上的反映。
4,空间反对称性和宇称守恒
在非相对论量子力学范围内,有关时空的变换除上述三个连续变换之外,还有两个分立的变换,这就是空间反射变换和时间反演变换。时间反演变换在附录中叙述,这里只讨论空间反射变换。
空间反射变换,在经典力学中的定义是
,rr
vv
→ pp
vv

在量子力学中类似可引入宇称算符P,定义是
rrP
vv
=
由于定义首先可得
∫∫∫
+∞
∞?
∞+
∞?
∞+
∞?
=?== |''|| rdrrrdrrrdrrPP
rrrvvvvvv
ψψψψ ' (6.11)
由于()rr
vv
ψψ =|,于是
()

+∞
∞?
=?= rrdrrrPr
vrrvvv
ψψψ '|'`|
这是()r
v
ψ相对原点的镜像反射。其次还可得
rdprrrdrrrPpP
vvvvvvvrv
|`|
_
==
∫∫

∞?
+∞

prdprrrdprr
vvvvvvvv
=′?′′=′?′
∫∫
+∞
∞?
+∞
∞?
`|=,
显然,两次接连的空间反射变换等于一个恒等变换
( ) rrrPrP
rvvv
==?=
2

1
2
=P
这说明宇称算符是“自逆”的,即
1?
= PP (6.12)
另外,对任意两个态矢(为下面表述明确,这里按上面结果,将变换前后的态矢用它们
8
的波函数标记)
( ) ( )rrP
vv
=
( ) ( )rrP
vv
= ψψ
按内积定义总有
()( ) ( ) ( )rrrr
vvvv
ψ?ψ? =

() ( ) ( )rrrPPr
vvrv
ψ?ψ? =
+
)(
从而
1=
+
PP
这说明宇称算符又是幺正的,即
1?+
= PP (6.13)
总之,P既是原来厄米的又是幺正的,
1?+
== PPP (6.14)
而这又只当算符是自逆的才能办到。由1
2
=P 知P的本值只有两个:1±,我们已经知为,
若一个厄米算符满足一个最低、有限幂次的实代数方程? n
( ) 0=?f
(这里的“最低”是指无重根,这个代数方程的含义是它作用到任意态矢上所得结果均为零)。
则这个的本征值为个,对应的n个本征矢构成完备集,可用于展开任意态矢。 n
a
用到宇称算符上,比如在坐标表象里叙述出来,这就成为,任意波函数总可以分解为空间反射对称部分和空间反射反称部份之和,即
( ) ( ) ( )rrr
As
vvv
ψψψ += (6.15)
这里U() () (),rrP
ss
vv
ψψ = () ( ) ( ),rrP
AA
U
vv
ψψ?= ( )PU 是宇称算符在波函数上的表现式。实际上这总是可以做到的,只要令
() () ( ))(
2
1
rrr
s
vvv
+= ψψψ
() () ( ))(
2
1
rrr
A
vvv
= ψψψ
a
此定理的证明见陈威亨译的 狄拉克,量子力学原理”,第31页,逆定理见第37页,科学出版社.1965年。
9
可以证明,宇称算符是个纯量子力学算符,它不能用经典的形式表示出来,(或者说,
它作为力学量不存在经典的对应物
a
),这就是说,P
不能用pr
,
vv
等力学量的任意函数来表示。
这可用反证法证明如下。
假定可以用pr
vv
,的某个函数( )prf
,
vv
把宇称算符P
表示出来,即
( )prfP
,

vv
= (6.16)
于是,向经典过渡h时,0→ (prf )
vv
,
。将趋于它的经典对应物 ( )prf
vv
,。这时对动量算符第i个分量有
i
p?
[ ] 0
lim?,
lim
00
=
=
→→
i
i
p
f
ipf h
hh
即,的经典对应物与对易 f
f
i
p
fppf
ii
=
但一方面,按的定义有 f
fppf
ii

=
这两种结果对时才行,这显然是荒谬的。这说明宇称算符不能被表达成为0=f pr
,
vv
的函数
,否则将有经典对应物,并且必定为零,反推回去宇称算符也为零。 f
f f f
可以得到宇称算符的明确表示式。比如取坐标表象,这时基矢为{ }r
v
。由P的定义并考虑基矢的完备性条件,即得
∫∫
+∞
∞?
+∞
∞?
=?= rdrrrdrrP
vvvvvv
(6.17)
而P
的矩阵元为
()() (rrrdrrrrrdrrrrrPr ′′+′=′′?+′=′′?′=′′′
∫∫
+∞
∞?
+∞
∞?
vvvvvvvvvvvvvv
δδδ )
a
这并不是说经典力学中无对应的变换,而是说不存在对应的力学量。从而,一个经典系统即使具有这种
10
宇称算符的本征值是可乘的。这是当多粒子波函数(态矢)(或同一粒子的几部分波函数或几一部份态矢)总是相乘的,经宇称算符的作用,新得的本征值总是相乘的。与此同时,
连续变换所对应的(力学量的)本征值是相加的。因为连续变换所对应的守恒量均在指数上,
故其各部份波函数(态矢)所属的本征值是相加的。比如,在核和粒子物理的粒子反应中
dcba +→+
这里,相对运动初态abba=其中相对运动ab表示两粒子之间相对运动的即部份波函数,而
ba,
a为a粒子内部状态。于是空间反射要作用于每一部份
相对运动初abPbPaPP

=

aPaP
a
=
bPbP
b
=
相对运动相对运动相对运动
abPabP
ab
=
于是总宇称量子数为,
abba
PPPP = (6.18)
下面将证明,l是两粒子相对运动轨道角动量量子数,故又称为轨道宇称。这样,假如这个过程中宇称是守恒的,就有反应前后总宇称量子数不变
()
l
1?=
相对运动ab
P ba,()
l
1?
() ( )
dcba
PPPP
ll ′
=? 11 (6.19)
这里l之间相对轨道角动量量子数.如果控制这个反射过程的作用力弱作用不保证宇称守恒,则无此等式。
dc
`
为′
现在补充说明,如果两个粒子相对运动的轨道角动量量子数
。这是因为,这时描述它们之间相对运动的波函数为()
l
l 1?,则它们的轨道宇称为
()?θ,
lm
Y,它的空间反演为
() ( ) ( ) ( )?θ?πθπ?θ,,,
lmlmlm
YYYP
l
=+?=
() ( )()θπθπ?θ
lm
Yrr态。故轨道角动量的本征,,在球坐标中空间反演
),,,( +→?
11
反对称性,也不导致守恒的力学量存在
也为宇称算符的本征态,其本征值为( )
l
1?
()

r
v
( )
0
1
r
v
0
1
2
2
n
τ
τ
()
=+=
+Θ=
00
01
1
22
1
33
e
e
eQ τ
§6.3 内禀对称性
1,同位旋空间旋转对称性和同位旋守恒
核力是一种强相互作用,它大体上与电荷无关,这是一个虽然并不十分准确但却是普遍成立的实验事实。据此,可以把质子和中子考虑成是同一个粒子(核子)的两个不同状态。
于是(不计自旋)一个核子的波函数可以记成
( )
()
=
r
r
v
v
2
1
(6.20)
这样,质子和中子的波函数使分别成为
(),
=r
p
v
()
()
=
r
r
n
v
v
2
0
i
τ取下面作为同位旋算符
=
1
0
1
τ,,(6.21)

=
0
0
2
i
i
τ
=
10
01
3
τ
则在这些算符作用下,产生如下变换
=
=
,
1
1
pn
np
τ
τ
=
=
,
p
pp
i
i
=
=
nn
pp
τ
τ
3
3
如果核子之间的相互作用仅仅是强相互作用,根据核力与电荷(是质子还是中子)无关,
从而在上述这些变换下核子体系的能级相同,这将构成能级的简并。这时,由同位旋算符
i
τ
组成的矢量算符
τ
v
v
2
1
=Θ (6.22)
称为核子的同位旋.而质子和中子只是同一个称为“核子”的粒子的同位旋第三分量不同的两个状态.出现了同位旋多重态的简并.还可以附带指出,由同位旋第三分量,可以组成所谓“电荷算符”,
Θ
Q
(6.23)
显然
12
pp
Q =,0=
n
Q?
2,微观粒子全同性原理
在经典力学中,所谓全同粒子在原则上总是可以分辨和追踪的。但在量子力学中,全同粒子,比如两个电子,在原则上就是不可以分辩和不可追踪的,即使设想在某个时刻对两个相邻的全同粒子分别予以定位、鉴别并编号,在无限接近的随后时刻,它的坐标也不再具有定值(由于粒子的波动性,在无限小时间间隔内,可以“跑”出有限大的距离,这种几率并不为零!)。就是说,某时刻的定位对追踪毫无帮助。也就是说,在微观世界里,全同的粒子在原则上都失去了“个别性”、“可以分辨性”。这导致“同类微观粒子原则上完全不可分辨
“。注意,这里所强调的“原则上”是说并非暂时的、技术上的不可分辨。这就是微观粒子的全同性原理。这是微观世界运动所特有的规律,是纯量子效应。
由全同性原理,立即导出两个结论:第一,全同粒子体系的一切力学量,包括系统的哈密顿量,必须相对任何一对粒子的编号的交换是对称的;第二,体系所有可观察几率,相对于任何一对粒子编号的交换也必须是对称的。这是因为,全同粒子体系的任何编号都是人为的、外在强加给体系的,既然在原则上是不能分辨的,那么这种编号的任何改变均不应导致任何可观察的物理效应。关于第二点,需要稍仔细些考察一下。由给出的任何几率都必须对称,所以体系的波函数相对任何一对粒子编号的交换,只能变一个相因子,即
),,,,,,(),,,,,,(
11 nkj
i
nkj
rrrrerrrr
jk
r
L
r
L
r
L
vr
L
r
L
r
L
v
ψψ
δ
=
接着再交换一下,又出一个,还是根据全同性原理,应有
Kj
i
e
δ
kjjk
δδ =。由于这样两次置换已使编号全部还原,故
{ } 12exp =
jk

所以
{ } 1exp ±=
jk

还是根据全同性原理,这个相同子应当与编号无关,从而全同粒子体系的波函数必须是 kj,
( ) ( )
NjkNkj
rrrrrrrr
v
L
v
L
v
L
vv
L
v
L
v
L
v
,,,,,,,,,,,,
11
ψψ ±= (6.24)
总之,从全同性原理可得如下两条重要结论,
i,全同粒子体系的全部可观察力学量算符相对粒子间的交换是完全对称的,
ii,全同粒子体系的全部可能状态相对粒子间的交换不是完全对称的,便是完全反称的。中间的各种状态是不会实现的。
这里预先指出,可以证明,对电子、中子、质子、氚核等具有半整数自旋的费米子,它们所构成的全同粒子体系的波函数必定是相对粒子间交换为反称的;对π介子、光子、氘核、
α粒子等具有整数自旋的玻色子,它们所构成的全同粒子体系的波函数必定是相对粒了间交换为对称的。
13
全同性原理和微观粒子的波动性有深刻的内在联系。可以说,微观粒子的波动性,反映在单个粒子上表现为(正则共轭量对之间的)测不准关系;反映在同类粒子间就是全同性原理。
14
第六章 对称性及其应用
§6.1 一般叙述
1,对称性的含义
对称性的含义有广义和狭义两种。从广义来说,对称性是意味着和谐、规律、秩序,它和偶然杂乱无章相对立。事实上,对某种基本的对称性的信念,常驱动着人们去发展科学。
H,Weyl说:“对称性是这样一种意念,人们长年累月地试图以它去理解并创造秩序、美和完善。”就狭义来说,系统的某种对称性是指某种不可分辨性,对某种属性的不可观测,也就是在某种操作、变换下,系统仍然保持不变。比如,体系的空间绝对位置是不可观测的,这导致体系的哈密顿量在空间平移操作、变换下是不变的;立方体相对它的某根对称轴所做的几个特殊的转动之后,将和转动之前相同,或不可分辨等等。
这里应当指出,不同的体系所具有的对称性可能不同。但是同一个体系,所有使它保持不变的对称变换必定构成一个群。因为,接连两次或两次以上施加对称变换,其结果依然使体系不变,也就是说依然是个对称变换;对称变换的逆变换依然是对称变换;存在恒等变换
(“不变”这种特殊的对称变换)。这些已是使对称变换构成群的充要条件了。
研究对称性的意义在于,第一,检验和发展理论,按海森堡的观点,“应当用基本对称性概念去代替已成问题的基本粒子的概念”,“……,必须寻找的不是基本粒子,而是基本对称性”。第二,可以增强人们的物理直觉、简化问题、有利于迅速抓住问题的要点,可以不去求解方程而得到关于态和本征值的一些知识。第三,可以简化一些计算,
比如简化一些矩阵元的计算等等。
dingeroSchr &&
2,量子力学中的对称性
就基本的对称性来说,量子力学中存在的对称性要强于经典力学中存在的对称性。这就是说,经典力学中存在的对称性,量子力学中也对应存在,同时,量子力学中还存在一些在经典力学中不存在的对称性。前者如时空均匀各向同性的对称性,后者如全同性原理。然而,
个别对称性除外,弱等效原理(在引力场中运动的物体,其质量不进入问题,以致运动轨迹的研究成为一个纯几何问题)在经典力学中存在,但在量子力学中被破坏,只当向经典过渡时才又表现出这种对称性逐渐被遵守起来。
量子力学中常见的对称性有:时间的均匀性、空间的均匀性、空间各向同性、同类粒子的全同性原理(或交换对称性),这些对称性是普适的、严格成立的。空间反射对称性、时间反演对称性,这对许多体系严格成立,但终究不是普适的。同位旋对称性,这只是个近似的对称性。此外,还有各种晶体的各种旋转、反射对称性,这些也是属于部分体系(晶体)
的特殊的对称性。
以上这些对称性可以接下面两种观点予以分类:第一,按变换是连续的或分立的来分类。
比如,晶体的对称变换、空间反射、时间反演变换、全同粒子置换等属于分立的变换,其余的属于连续变换。第二,按这些对称性是属于体系的内禀属性还是外在属性分类。空间和时间的平移、空间旋转对称性是体系新处的时空性质对体系运动方式提出的要求(即时空特性对体系哈密顿量的要求)。严格说,由此得出的对称性并不是系统的内在固有的禀性,是时
1
间空间固有属性在体系运动上的体现。与此相反,全同粒子置换对称性和同位旋空间旋转对称性等,是体系内在对称性、是体系的内禀属性。而空间反射、时间反演对称性,也根源于体系内部的动力学性质,因而也应当认为是属于体系的内禀属性。值得强调指出的是,这种观点能深刻地揭示出,为什么从经典力学发展到量子力学时,许多观念、规律都遭到根本性的变换,但和时空属性相关联的能量守恒、动量守恒和角动量守恒这三大守恒定律却几乎原封不动的被量子力学继承下来。这说明,量子体系的行为虽然“乖张”(从经典观点看),但毕竟也运行在和经典力学同一时空之中,从而这个时空的属性也必定“烙印”或“体现”在量子体系的运动行为上,使之表现出(如同宏观体系已经表现出那样)这三个守恒定律的存在。
3,对称性与守恒律及守恒量
前面已部分地涉及了对称性和守恒律的关系。这里集中简要地论证这个问题。
由Wigner定理,可以断定,对称变换是幺正变换(或反幺正变换)。而按以前所说,一个幺正变换U总可以表示为
a

=
αi
eU
这里为厄米算符,? α对连续变换为连续变化的参数,对断续变换为分立取值的参数。这里暂时只研究连续变换。由于U是对称变换,必定使体系的哈密顿量不变,即有
HUHU =
1
也即
[ ] 0,=UH
或写为
[]


=
=?
0
0,
!
)(
n
n
n
H
n

由于α可取连续值(包括零值),可取α足够小,从而得到
[ ] 0,=?H
这就是说,如果体系在(连续变化的)对称变换U的变换下是不变的,则此对称变换U的生成元(厄密算子?)是个守恒量。或者说,此体系存在一种(连续变化的)对称性,就相应地存在一个守恒律及其守恒量。若U是分立变换,同样有
[ ] 0,=UH
对空间反射变换,这个算符既是幺正的又是厄密的,从而U直接就是守恒的力学量。
总之,当体系存在一种对称性时,必定伴随着它必须服从的一条守恒定律,从而存在一
a
这里只讨论幺正变换,反幺正变换参见附录。
2
个守恒的力学量。
a
§6.2 时空对称性及其应用
1,时间均匀和能量守恒定律
时间轴是均匀的,就是说,时间轴上不存在绝对的与众不同的点,因此一个孤立量子体系(孤立经典力学体系也一样)的哈密顿量中不可能显含时间,从而沿时轴平移这个孤立量子体系是不会造成任何物理上可察觉的变化。
当H不显含时,可求得时间平移算符t )(τU,它把体系在时间轴上向前平移τ。按
)(τU定义,应有)()()( ττ?Ψ=Ψ ttU,因为)(τU把体系t
0
t=时刻发生的事件推迟到
τ+=
0
tt时刻发生。即,原先t时体系处于
0
t= )(
0
tΨ。变换后τ+=
0
tt时,体系处于
)(
0
tΨ。这时按方程 dingeroSchr &&
)()( t
i
H
t
dt
d
Ψ=Ψ
h
(6.1)
于是
)()()()(
22
t
i
H
t
dt
d
i
H
t
i
H
dt
d
t
dt
d
Ψ
=Ψ=Ψ

hhh
或一般地有
)()( t
i
H
t
dt
d
nn
Ψ

h
于是
() ()
( )
() ( )
∑∑

=

=
+
=
=
=
00
!!
1
n
n
n
n
n
H
i
tt
dt
d
n
tH
i
n
te τψψ
τ
ψ
τ
ψ
τ
h
h

()tet
H
i
ψτψ
τ
h
=? )(
所以主动地将体系沿时间轴平移τ的算符或幺正变换为
h
τ
τ
H
i
eU =)( (6.2)
a
为上所申明的.这里只研究幺正变换的对称性.至于反幺正变换,对称性存
在并不意味着守恒的力学量存在。
3
按)(τU的定义,它和时间发展算符是反向的。并且显然)(τU是个幺正变换,它不改变体系的一切可观察物理效应。因为在这个变换下
态,)()()()()(
'
τψψτψψ?==→ ttUtt
算符,
1'
)()(
=?→? ττ UU
于是对任给的两个态矢)(t?,)(tψ有
)()()()()()()()(
1''
tttUUttt ψ?ψττ?τψτ?ψ? =?==
(6.3)

)()()()()()()()(
111'''
tttUUUUttUUt ψ?ψ?τψτττ?ψ=?==?

(6.4)
在经受)(τU变换的前后这些几率幅和矩阵元均不变。这说明H不显含t时,体系的确是时间平移不变的。或者说,这时采用时间轴上任何一点作为计算起点都不会产生可观察的物理效应。作为H不显含时间的一个例子,一个孤立系的情况就应当是这样的。
上面的论证也可以换一种说法.对任意两个态矢?,ψ计算如下
ψ?ψ?ψ
ψ?
dt
d
H
t
H
H
dt
d
H
dt
d
+
+?=
ψ?ψ?ψ? H
i
H
t
H
HH
i hh
11
+
+?
= (6.5)
[]ψ? HH
it
H
,
1
h
+
=
根据第一章关于微商算符
dt
dH
的定义
ψ?ψ?
dt
dH
H
dt
d

则有(由于?,ψ的任意性),
[]
t
H
HH
it
H
dt
dH
=+
=,
1
h
当H不显含时(孤立系就应如此),应有t 0=
t
H
和[ ] 0,=HH,即得
0=
dt
dH
(6.)
4
这意味着对任意?,ψ,ψ? H均不随时间变化
a
这显然意味着:第一,在任意态中H的平均值不随时间变化(取?为ψ即可);第二,在任意态中,H取某个本征值的几率不随时间变化,这只要取?为ψ并把ψ按H
的本征态
hh
r
tiEt
n
n
enr
=)(ψ
iE
n
e
展开即可看出,
hh
rr
tiE
n
n
tiE
n
n
n
enberbtr

∑∑
== )()( ψψ
n
n
n
t
EE
i
n
nm
m
EbenHmbbH
nm
2
*
∑∑
==
h
ψ?
这里,在),( tr
v
ψ中测H时,取本征值的几率
n
E
2
n
b不随时间变化。所以H是个守恒量。
这里应指出,说体系H守恒,并不等于体系一定处在H的本征态上。这时体系可以处在H本征态的叠加态上,这要视初条件决定,但不论处在何种状态,必须满足上述两点。
2,空间均匀性和动量守恒定律
类似可求得空间平移幺正算符U )(a
r
,即坐标系不动而把体系平移一段有限距离的变换。这时,按U的定义,对任意态矢均应有
a
r
)(a
v
),(),()()( tartraUtr
rrrrr
==′ ψψψ (6.7)
这里右边的“-”号可以这样理解:设体系为一团几率云,变换前
0
rr
vr
=处的几率云小片,
在变换后将位于
0
rar
vvr
=?,即arr
rrr
+=
0
处。这说明这团几率云移动了距离。于是 a
r
),()(
!
1
),()()()(
3
10
tr
x
a
n
tartraUtr
n
i
i
i
n
rrrrrr
ψψψψ
=?==′
∑∑
=

=
a
这也可直接按本征态展开检验 设
=
==

∑∑
h
h
rr
rrr
tiE
n
nn
tiE
m
mm
m
mm
n
m
erbtr
eratratr
)(),(
)(),(),(
ψψ
ψψ?

∑∑

==
n
nnn
tEEi
mnnn
mn
m
EbaeEbatrHtr
nm
h
rr
)(
),(),( δψ?,与无关。 t
5
)()exp()()exp( trpa
i
tra
rrr
h
rr
ψψ==
r
所以把体系作空间平移的算符 a
pa
i
eaU
rr
h
r

=)( (6.8)
按前面所说,这个幺正变换的生成元动量算符p
r
是个守恒量。就是说,如果一个体系具有平移不变的对称性质(对孤立系这是必定的),这个体系的总动量是守恒的。
和前面能量守恒情况类似,说动量守恒,并不等于体系一定处在动量的本征态上,这要看初条件如何而定。如初条件是一系列动量本征态的叠加(形成自由运动波包),则其后将一直保持如此。
3,空间各向同性和角动量守恒
由于我们所处的空间是各向同性的,并无特殊方向可言(有定向的外加场,如重力场,
破坏这种各向同性的情况除外。现在这里是讨论未遭任何破坏前的空间本身性质。此观点对前面的讨论也适用),所以把一个孤立体系绕任何轴旋转一个任意角度均不应当影响体系的任何物理性质。设该体系绕
n
e
r
轴转?角的变换所对应的算符为U ),(
n
e
v
,则有
),(),()()( trtreUtr
n
ρψψ?ψ
rrrrr
==′
这里?ρ )( re
n
vvv
×=。于是
),(),()()( trtreUtr
n
ρψψ?ψ
rrrrr
==′
),()(
!
1
0
tr
n
n
n
rr
ψρ=


=
[])()()(
!
1
trre
n
n
n
n
n
rrr
ψ×?=

[])()(
!
1
0
trre
n
n
n
n
n
rrr
ψ×=


=
)()exp( trLe
i
n
v
)
r
r
h
ψ
=
所以将体系绕
n
e
v
转?角的变换算符
()
Le
i
n
n
eeU
r
r
h
v

=
ψ
(6.9)
6
如果这些幺正变换对某个体系来说是使体系不变的对称变换,则生成元角动量L
)
v
守恒。这时三个分量均是守恒量,但由于它们彼此均不对易,不能同时测量,故不可能同时取各自的确定的本征值,就是说,比如当取某个本征值(也即这个态是的本征态)时,L将一定的几率分布各种可能的(共2
x
L hm
x
L
y
1+l个)hm′值,这时在单次测量时显然呈现取值的不碓定性,情况也和类似。但尽管如此,的取值和、的不同取值的几率分布都不随时间变化,后者导致
y
L
zz
L
y
L
x
L hm
y
L L
zy
LL,不随时间变化。
这里举一个例子,粒子在均匀无限圆柱形螺旋线场中运动,除能量之外求守恒量。
选柱坐标()z,,?ρ,由这种场的势的形式为
()αρ?tgzV?V =,
由于绕轴旋转z和沿轴平移z的算符分别为
()
()
=?
=?
)exp(
)exp(
z
zz
pz
i
zB
L
i
eA
h
h
v

令απρ
π
α?ρ tgddtgz 2,
2
=
≡=?为螺旋线的螺距。构造新算符
()( )
+==
zzz
p
d
L
i
tgBeAC
π
αρ?
2
exp
h
v
(6.10)
由于
()
=
=
=
+?
0,
2
,
2
.0
2
,
22
zz
zz
p
m
p
L
m
p
p
d
LtgzV
π
αρ?
所以
[ ] 0,=HC
于是在此等螺距的螺旋线场中,下面算符及其对应的物理量
zz
Lp
d
+
π2
7
守恒。
以上讨论的是关于时空的连续对称变换。由于时空内禀地具有均匀各向同性的性质,在这些属性不遭受外来破坏时就会表现出来,并构成三个普通的守恒定律。对孤立系,不论其中发生的是什么过程,均会因为体系所在的时空固有这些属性而必定遵守这三个定律。于是,
如其说这三个守恒定律是体系本身的性质,不如说是时空均匀各向同性性质在体系运动行为上的反映。
4,空间反对称性和宇称守恒
在非相对论量子力学范围内,有关时空的变换除上述三个连续变换之外,还有两个分立的变换,这就是空间反射变换和时间反演变换。时间反演变换在附录中叙述,这里只讨论空间反射变换。
空间反射变换,在经典力学中的定义是
,rr
vv
→ pp
vv

在量子力学中类似可引入宇称算符P,定义是
rrP
vv
=
由于定义首先可得
∫∫∫
+∞
∞?
∞+
∞?
∞+
∞?
=?== |''|| rdrrrdrrrdrrPP
rrrvvvvvv
ψψψψ ' (6.11)
由于()rr
vv
ψψ =|,于是
()

+∞
∞?
=?= rrdrrrPr
vrrvvv
ψψψ '|'`|
这是()r
v
ψ相对原点的镜像反射。其次还可得
rdprrrdrrrPpP
vvvvvvvrv
|`|
_
==
∫∫

∞?
+∞

prdprrrdprr
vvvvvvvv
=′?′′=′?′
∫∫
+∞
∞?
+∞
∞?
`|=,
显然,两次接连的空间反射变换等于一个恒等变换
( ) rrrPrP
rvvv
==?=
2

1
2
=P
这说明宇称算符是“自逆”的,即
1?
= PP (6.12)
另外,对任意两个态矢(为下面表述明确,这里按上面结果,将变换前后的态矢用它们
8
的波函数标记)
( ) ( )rrP
vv
=
( ) ( )rrP
vv
= ψψ
按内积定义总有
()( ) ( ) ( )rrrr
vvvv
ψ?ψ? =

() ( ) ( )rrrPPr
vvrv
ψ?ψ? =
+
)(
从而
1=
+
PP
这说明宇称算符又是幺正的,即
1?+
= PP (6.13)
总之,P既是原来厄米的又是幺正的,
1?+
== PPP (6.14)
而这又只当算符是自逆的才能办到。由1
2
=P 知P的本值只有两个:1±,我们已经知为,
若一个厄米算符满足一个最低、有限幂次的实代数方程? n
( ) 0=?f
(这里的“最低”是指无重根,这个代数方程的含义是它作用到任意态矢上所得结果均为零)。
则这个的本征值为个,对应的n个本征矢构成完备集,可用于展开任意态矢。 n
a
用到宇称算符上,比如在坐标表象里叙述出来,这就成为,任意波函数总可以分解为空间反射对称部分和空间反射反称部份之和,即
( ) ( ) ( )rrr
As
vvv
ψψψ += (6.15)
这里U() () (),rrP
ss
vv
ψψ = () ( ) ( ),rrP
AA
U
vv
ψψ?= ( )PU 是宇称算符在波函数上的表现式。实际上这总是可以做到的,只要令
() () ( ))(
2
1
rrr
s
vvv
+= ψψψ
() () ( ))(
2
1
rrr
A
vvv
= ψψψ
a
此定理的证明见陈威亨译的 狄拉克,量子力学原理”,第31页,逆定理见第37页,科学出版社.1965年。
9
可以证明,宇称算符是个纯量子力学算符,它不能用经典的形式表示出来,(或者说,
它作为力学量不存在经典的对应物
a
),这就是说,P
不能用pr
,
vv
等力学量的任意函数来表示。
这可用反证法证明如下。
假定可以用pr
vv
,的某个函数( )prf
,
vv
把宇称算符P
表示出来,即
( )prfP
,

vv
= (6.16)
于是,向经典过渡h时,0→ (prf )
vv
,
。将趋于它的经典对应物 ( )prf
vv
,。这时对动量算符第i个分量有
i
p?
[ ] 0
lim?,
lim
00
=
=
→→
i
i
p
f
ipf h
hh
即,的经典对应物与对易 f
f
i
p
fppf
ii
=
但一方面,按的定义有 f
fppf
ii

=
这两种结果对时才行,这显然是荒谬的。这说明宇称算符不能被表达成为0=f pr
,
vv
的函数
,否则将有经典对应物,并且必定为零,反推回去宇称算符也为零。 f
f f f
可以得到宇称算符的明确表示式。比如取坐标表象,这时基矢为{ }r
v
。由P的定义并考虑基矢的完备性条件,即得
∫∫
+∞
∞?
+∞
∞?
=?= rdrrrdrrP
vvvvvv
(6.17)
而P
的矩阵元为
()() (rrrdrrrrrdrrrrrPr ′′+′=′′?+′=′′?′=′′′
∫∫
+∞
∞?
+∞
∞?
vvvvvvvvvvvvvv
δδδ )
a
这并不是说经典力学中无对应的变换,而是说不存在对应的力学量。从而,一个经典系统即使具有这种
10
宇称算符的本征值是可乘的。这是当多粒子波函数(态矢)(或同一粒子的几部分波函数或几一部份态矢)总是相乘的,经宇称算符的作用,新得的本征值总是相乘的。与此同时,
连续变换所对应的(力学量的)本征值是相加的。因为连续变换所对应的守恒量均在指数上,
故其各部份波函数(态矢)所属的本征值是相加的。比如,在核和粒子物理的粒子反应中
dcba +→+
这里,相对运动初态abba=其中相对运动ab表示两粒子之间相对运动的即部份波函数,而
ba,
a为a粒子内部状态。于是空间反射要作用于每一部份
相对运动初abPbPaPP

=

aPaP
a
=
bPbP
b
=
相对运动相对运动相对运动
abPabP
ab
=
于是总宇称量子数为,
abba
PPPP = (6.18)
下面将证明,l是两粒子相对运动轨道角动量量子数,故又称为轨道宇称。这样,假如这个过程中宇称是守恒的,就有反应前后总宇称量子数不变
()
l
1?=
相对运动ab
P ba,()
l
1?
() ( )
dcba
PPPP
ll ′
=? 11 (6.19)
这里l之间相对轨道角动量量子数.如果控制这个反射过程的作用力弱作用不保证宇称守恒,则无此等式。
dc
`
为′
现在补充说明,如果两个粒子相对运动的轨道角动量量子数
。这是因为,这时描述它们之间相对运动的波函数为()
l
l 1?,则它们的轨道宇称为
()?θ,
lm
Y,它的空间反演为
() ( ) ( ) ( )?θ?πθπ?θ,,,
lmlmlm
YYYP
l
=+?=
() ( )()θπθπ?θ
lm
Yrr态。故轨道角动量的本征,,在球坐标中空间反演
),,,( +→?
11
反对称性,也不导致守恒的力学量存在
也为宇称算符的本征态,其本征值为( )
l
1?
()

r
v
( )
0
1
r
v
0
1
2
2
n
τ
τ
()
=+=
+Θ=
00
01
1
22
1
33
e
e
eQ τ
§6.3 内禀对称性
1,同位旋空间旋转对称性和同位旋守恒
核力是一种强相互作用,它大体上与电荷无关,这是一个虽然并不十分准确但却是普遍成立的实验事实。据此,可以把质子和中子考虑成是同一个粒子(核子)的两个不同状态。
于是(不计自旋)一个核子的波函数可以记成
( )
()
=
r
r
v
v
2
1
(6.20)
这样,质子和中子的波函数使分别成为
(),
=r
p
v
()
()
=
r
r
n
v
v
2
0
i
τ取下面作为同位旋算符
=
1
0
1
τ,,(6.21)

=
0
0
2
i
i
τ
=
10
01
3
τ
则在这些算符作用下,产生如下变换
=
=
,
1
1
pn
np
τ
τ
=
=
,
p
pp
i
i
=
=
nn
pp
τ
τ
3
3
如果核子之间的相互作用仅仅是强相互作用,根据核力与电荷(是质子还是中子)无关,
从而在上述这些变换下核子体系的能级相同,这将构成能级的简并。这时,由同位旋算符
i
τ
组成的矢量算符
τ
v
v
2
1
=Θ (6.22)
称为核子的同位旋.而质子和中子只是同一个称为“核子”的粒子的同位旋第三分量不同的两个状态.出现了同位旋多重态的简并.还可以附带指出,由同位旋第三分量,可以组成所谓“电荷算符”,
Θ
Q
(6.23)
显然
12
pp
Q =,0=
n
Q?
2,微观粒子全同性原理
在经典力学中,所谓全同粒子在原则上总是可以分辨和追踪的。但在量子力学中,全同粒子,比如两个电子,在原则上就是不可以分辩和不可追踪的,即使设想在某个时刻对两个相邻的全同粒子分别予以定位、鉴别并编号,在无限接近的随后时刻,它的坐标也不再具有定值(由于粒子的波动性,在无限小时间间隔内,可以“跑”出有限大的距离,这种几率并不为零!)。就是说,某时刻的定位对追踪毫无帮助。也就是说,在微观世界里,全同的粒子在原则上都失去了“个别性”、“可以分辨性”。这导致“同类微观粒子原则上完全不可分辨
“。注意,这里所强调的“原则上”是说并非暂时的、技术上的不可分辨。这就是微观粒子的全同性原理。这是微观世界运动所特有的规律,是纯量子效应。
由全同性原理,立即导出两个结论:第一,全同粒子体系的一切力学量,包括系统的哈密顿量,必须相对任何一对粒子的编号的交换是对称的;第二,体系所有可观察几率,相对于任何一对粒子编号的交换也必须是对称的。这是因为,全同粒子体系的任何编号都是人为的、外在强加给体系的,既然在原则上是不能分辨的,那么这种编号的任何改变均不应导致任何可观察的物理效应。关于第二点,需要稍仔细些考察一下。由给出的任何几率都必须对称,所以体系的波函数相对任何一对粒子编号的交换,只能变一个相因子,即
),,,,,,(),,,,,,(
11 nkj
i
nkj
rrrrerrrr
jk
r
L
r
L
r
L
vr
L
r
L
r
L
v
ψψ
δ
=
接着再交换一下,又出一个,还是根据全同性原理,应有
Kj
i
e
δ
kjjk
δδ =。由于这样两次置换已使编号全部还原,故
{ } 12exp =
jk

所以
{ } 1exp ±=
jk

还是根据全同性原理,这个相同子应当与编号无关,从而全同粒子体系的波函数必须是 kj,
( ) ( )
NjkNkj
rrrrrrrr
v
L
v
L
v
L
vv
L
v
L
v
L
v
,,,,,,,,,,,,
11
ψψ ±= (6.24)
总之,从全同性原理可得如下两条重要结论,
i,全同粒子体系的全部可观察力学量算符相对粒子间的交换是完全对称的,
ii,全同粒子体系的全部可能状态相对粒子间的交换不是完全对称的,便是完全反称的。中间的各种状态是不会实现的。
这里预先指出,可以证明,对电子、中子、质子、氚核等具有半整数自旋的费米子,它们所构成的全同粒子体系的波函数必定是相对粒子间交换为反称的;对π介子、光子、氘核、
α粒子等具有整数自旋的玻色子,它们所构成的全同粒子体系的波函数必定是相对粒了间交换为对称的。
13
全同性原理和微观粒子的波动性有深刻的内在联系。可以说,微观粒子的波动性,反映在单个粒子上表现为(正则共轭量对之间的)测不准关系;反映在同类粒子间就是全同性原理。
14
第七章 电子自旋角动量
实验发现,电子有一种内禀的角动量,称为自旋角动量,它源自电子的内禀性质。在非相对论性情况下,它与电子在外在空间中的运动无关;在狄拉克的电子相对论性方程中,这个内禀角动量由方程的旋量结构所自然体现,使人们对它与相对论的关系获得了较深的理论了解。但是,到目前为止,对这个内禀角动量(以及与之伴随的内禀磁矩)的物理本质仍然不能说有了十分的了解
a
。不过,根据实验测量所拟定的一套关于它的量子力学计算方法,
使我们能够毫无困难地从理论上予计实验测量结果并计算它在各种场合下的运动和变化。
§7.1 电子自旋角动量
1,电子自旋的实验基础和其特点
早期发现的与电子自旋有关的实验有:原子光谱的精细结构(比如,对应于氢原子的跃迁存在两条彼此很靠近的两条谱线,碱金属原子光谱也存在双线结构等);
1912年反常塞曼效应,特别是氢原子的偶数重磁场谱线分裂,无法用轨道磁矩与外磁场相互作用来解释,因为这只能分裂谱线为
sp 12 →
( )12 +l重,即奇数重;1922年斯特恩——盖拉赫实验,实验中使用的是中性顺磁的银原子来,通过一个十分不均匀的磁场,按经典理论,由于束是中性的,不受洛仑茨力的作用,又由于银原子具有一个永久的磁矩并且从高温下蒸发出来成束时其磁矩方向必为随机指向各向同性的。于是在穿过非均匀磁场时,磁矩和磁场方向夹角也是随机的,从而银原子束在通过磁场并接受非均匀磁场力的作用之后,应当在接受屏上相对于平衡位置散开成一个宽峰,但实验却给出彼此明显对称分开的两个峰,根据分裂情况的实测结果为,即数值为玻尔磁子。
B

针对以上难以解释的实验现象,1925年乌仑贝克和高德施密特提出假设:电子在旋转着,因而表现出称之为自旋的内禀角动量,s
v
它在任意方向的取值只能如
2
h
±两个值。为使这个假设与实验一致。可假定电子的固有磁矩μ
r
与其自旋角动量s
v
之间的关系为
s
mc
e vr

这表明,电子自旋的旋磁比是轨道旋磁比的两倍。于是,电子便具有了μ
rv
,,,sem共四个内禀的物理量。在根据实验事实外在地引入电子自旋这一内禀自由度之后,不仅原子的磁性质,
而且原子光谱曲本身的一些精细结构以及在外场下的多重分裂也都得到了很好的解释。
然而,将电子自旋角动量的来源解释为电子旋转造成的这一经典图象却立即遭到困难。
a
杨振宁讲演集,南开大学出版社,1989年
15
因为,可以作估算。设电子半径为,于是作为定性的估算可以合理地假定
e
r
h~,~
2
2
prmc
r
e
e
e
∴,137~
2
cc
e
c
mrem
p
=
==
hh
υ
这就是说,为了要在的半径下旋转得出h的角动量,电子必须以137倍的光速转动才行。
显然这是一个不能接受的图象。这说明,电子的自旋角动量有着另外的更深刻的内禀原因。
虽然相对论性的电子狄拉克方程表明电子自旋已蕴含在方程的旋量结构中,并且我们能进行有关自旋电子磁矩的各种计算,但仍然还不能说对自旋的物理本质有了透澈了解。
e
r
2,电子自旋态的表示法
由于电子自旋是一个新的自由度,并且相应于这个新自由度的新变数s只能取两个值
z
2
h
±,于是电子的状态波函数应为两分量两列矢量,
()
( )
()
()βψαψ
ψ
ψ
ψ trtr
tr
tr
tsr
z
vr
v
v
v
21
2
1
)(
,
,
,+=
= (7.1)
这里分别代表自旋角动量第三分量
=
=
1
0
,
0
1
βα
22
hh
和朝下取朝上
z
s的状态。于是
自旋朝上的几率=

rd
v
2
1
ψ

=自旋朝下的几率rd
v2
2
ψ
总的归一化表示为
( )
∫∫
=+=
+
1
2
2
2
1
ψψψψ rdrd
vv
(7.2)
如果系统哈密顿H中不含自旋角动量,或是自旋和空间两部分可分开即,则自旋波函数和空间波函数可分离,
s
HHH +=
0
()( ) ( )
()
()
()
() ()
+=
=
=
βχαχ
χ
χ
χ
χ?ψ
tt
t
t
ts
tstrtsr
z
zz
21
2
1
,
,,,
vv
考虑电子自旋角动量之后,薛定谔方程便由单分量的方程扩充为两分量的方程,原者常称为
16
泡利方程。
3,自旋算符与泡利矩阵
一方 面,自旋既是角动量就应当满足角动量的对立规则,设
[ ]
kijkji
siss εh=,,这里zyxi,,= 等
另一方面,既然自旋变数取值只有两个,
2
1
±,并且波函数相应为两分量的列矢量,则自旋角动量的三个分量算符自应是3个
2
,22
h
抽出绝对数的厄米矩阵×,引入三个二阶厄米矩阵
i
σ,令
ii
s σ
2
h
=,),,( zyxi = (7.3)
于是,,1就是说的本征值只能为±
i
σ
i
σ为自逆矩阵,再根据上面的对易规则,即得到关于
i
σ的下列关系
[ ]
=
=
0
2
2,
σσ
σεσσ
i
kijkji
i
这里为二阶单位矩阵。由这些关系还可以得出
=
10
01
0
σ
i
σ之间的反对易规则,因为
[][ ] [ ] [ ]
ijijiijij
σσσσσσσσσσ,,,,0
2
0
+===
() }{,,22
kiijkikkiijk
ii σσεσσσσε =+=
jkij ≠≠总可以取对给定的,于是即得
}{,0,=
ji
σσ ji ≠
再考虑到结合起来即有,1
2
=
i
σ
}{
ijji
δσσ 2,=,( ),,,zyxji = (7.4)
当然,由此处的反对易关系亦可推出上面的对易关系,因而是等价的。这便是下面决定
i
σ表达式的出发点。这组关系表明,
i
σ是自逆的、反对易的和零迹的。关于最后一点是由于
17
[ ]
kijkji
tritr σεσσ 2,0 ==
∴ tr 0=
k
σ,()zyxk,,=
下面往求这三个厄米矩阵。这里,应当预先指出,由上面这组反对易关系,并不能完全确定这组厄米矩阵。要完全确定它们,必需另外附加规定。而不同附加规定所得的三个
i
σ也将不同,但这些不同
i
σ的组均能满足上面的全部物理要求,因而是等价的。同组之间相差一个的幺正变换。这就出现选择22×
i
σ的表象的问题。这里只给出
i
σ的一个常用的表象。
为此选定
z
σ为对角的,由于
z
σ的本征值为± 1。于是
=
10
01
z
σ
进一步,根据
x
σ必须是零迹的厄米矩阵令,a为两个待定的复数。根据
=
ab
ba
x
σ b,
和代入
zxxz z
σσσσσ?=
=
10
01
x
σ的表达式后得
x
σ
= 0a,再考虑
2
x
σ
0
,又得为任一因子。至此,仍不能完全决定,再进一步约定
αi
eb =
=α,于是有
=
01
10
x
σ
接着有
=?=
0
0
i
i
i
xzy
σσσ
最后,在的一种位相约定之后为对角的并取定
xz
σσ,得到如下这一组2的自逆、
反对易、零迹的厄米矩阵 —— 泡利矩阵,具体地实现了自旋角动量对易规则,

=
01
10
x
σ,,(7.5)
=
0
0
i
i
y
σ
=
10
01
z
σ
简单考察即可相信,这三个矩阵再加上
0
σ即组成一组完全基,用它们可以分解(展开)
18
任何的复矩阵。应当说,由于它们的自逆性质和22×
i
σ之间的反对易性质,用它们作分解
(展开)并随之而来的乘法运算中将会表明这是最便于使用的一组基,类似于在通常矢量展开中选用了一组正交归一基矢时那样。
A
v
,
i
σσ
j
b σ
σ
v
的本征值为
=
sin
cos
4,例算
[例1] ( )( ) ( ) σσσ
v
vvvv
v
v
v
v
×+?= BAiBABA
这里,B
v
是两个三维矢量,BA
vv
项中已略写
0
σ
()() jbaBA
iji
ij
σσσσ

=
=
3
1
v
v
v
v
()
jji
ji
ji
ii
i
baba
∑∑

=
=
+=
3
1,
3
1
kiijk
ji
ji
aiBA ε


=
+?=
3
1,
vv
( )
vvvv
×+?= BAiBA
[例算2] 求n
vv
σ的本征态
按例算1,() 1,1
2
±?=?因此厄米矩阵nn
vvvv
σσ,写出它的本征方程
±=
b
a
b
a
n
vv
σ
也即
±
b
a
b
a
e
e
i
i
θθ
θθ
cossin
sincos
解之即得
2
2
2
2
2
cos
2
sin
1
2
2
1
θ
θ
θ
θ
i
i
i
i
e
e
e
e
的本征态为对应本征值为的本征态为对应本征值为
19
[例算3] 证明 ( ),sin.cos
.
ασα
α
σα
vv
vr
ei
i
+=e
这里的模长为方向单位矢量,为ααα
α
α
α
vv
v
v
=e。
()
()
()
()
12
12
0
2
2
0
.
.
!12
.
!2
+
+∞
=

=
+
+=
∑∑
n
n
n
n
n
n
i
n
i
n
i
e σασα
σα
vvvv
vv
利用例算1得 (),
22
ασα =?
vr
于是
()
()
()
( )
()
∑∑

=

=
+
+
=
0
2
0
2.
!12
1
!2
1
n
n
n
n
n
n
i
n
i
n
e ασαα
σα
vv
vv
ασα
α
sin)(cos
vv
+= ei
这里附带指出以下三种情况,
( ) ( )
()
Τ?+=Τ
Τ+=
Τ+Τ=
Τ
Τ
Τ
.sincos
sincos
sincos
.
αα
αα
αα
α
α
α
α
v
v
v
r
r
eie
ieT
ieT
i
i
i
为自逆反对易应矩阵组若为自逆矩阵若为任意矩阵若
[例算4] 求 ()
1
0
2
+
x
σσ
根据上面所说,总可以把所求结果按}{
zyx
σσσσ
,0
,,展开,即总有
()
0
1
0
2 δαγσβσασσσ +++=+
zyxx
这里δγβα,,,为待定系数。于是
( )( )
xzyx
σσδσγσβσασσ ++++=
000
2
( )( )( ) ( )
0
2222 σαδσβγσγβσδα ++?++++=
zyx
ii
于是
zyx
σσσ和,,前的系数必须为零,而解之即得,12 =+αδ,0,
3
1
==?= γβα
,
3
2
=δ即
()
xx
σσσσ
3
1
3
2
2
0
1
0
=+
5,
2
1
自旋态的极化矢量与投影算符
20
一个自旋态δ的板化矢量
δ
p
v
定义为
δσδ
δ
vv
=p (7.6)
注意,由于σ
r
已经过了态平均,
δ
p
v
是一个具有经典性质的普通矢量,可以对之作普通矢量的图解分解。
向一个自旋态λ投影的投影算子
λ
π定义为
λλπ
λ
= (7.7)
于是,在自旋态δ中找到自旋态λ的几率为
2
| λδδπδ
λλδ
==p (7.8)
注意。
δλλδ
pp =
就电子的任意自旋态λ而言,它的
λλ
π和p
v
之间有一个有用的关系式,

α
π
λλ
vv
+= p1
1
) (7.9)
证明:如此定义的算符
λ
π的是个投影算子,因为
()()[]=++= σσσπ
λλλλ
rvvvvv
ppp,21
4
1
2
()[ ]...21
4
1
2
.
σσ
λλλλ
vvvvvv
ppipp ×+++
()
λλ
πσ =+=
vv
.1
2
1
p
实际上,可以直接验算该关系式,这只要注意
()
{ }?
=
=
==
λσλλσλλσλ
λλλ
λλλ
λλ
λ
λ
λλπ
λ
321
2
22
*
1
*
21
2
1*
2
*
1
2
1
,,
x
p
v
{ ()( ) }
2
2
2
12
*
12
*
1
,Im2,2 αααααα?=
e
R
即可。
利用极化矢量和报影算符间的这一关系式可以方更地进行如下运算:i,求态中自旋和取向;ii,对给定的自旋态进行几率分解;iii,求给定态中自旋在某一方向的测量平均值等。
21
例如,在
=
0
1
α态中,测得自旋在( )?θ,n
v
方向(即极化矢在方向的态n
v
)的几率为
()ασααπα
α
vv
+== np
nn
1
2
1
= }{ αββα
zyx
ninn +++
2
1
2
1
()()
2
coscos1
2
1
1
2
1
2
θ
θ =+=+=
z
n
类似地,在n
v
态中测到自旋在
=
1
0
β方向的几率为
πβ
β
=
n
p ()()
2
sincos1
2
1
1
2
1
2
θ
θβ =?=?=
zn
n
又例如,在λ态中测得沿n
v
方向取的自旋平均值为
λσλλλ
λ
vvhvv
v
.
2
.
,
nsnS
n
==
() ()λσλλσλ
vvhvvh
.1
2
1
2
.1
2
1
2
nn?
++=
λπλλπλ
nn?
=
22
hh
于是比如,在α态中自旋沿n
v
方向的平均值为
=
2
sin
2
cos
2
22
,
θθ
α
h
v
n
S θcos
2
h
=
而在β态中此值为
θ
θθ
β
cos
22
cos
2
sin
2
22
,
hh
v
=
=
n
S
这两个结果均符合经典的几何图象。
§7.2 自旋角动量与轨道角动量耦合
22
1,与S
r
L
v
的合成
如前所说,LS
vr
与代表了两种不同的运动自由度,因此它们之间彼此对易,即有
[ ],0,=
ji
LS zyxji,,,= (7.10)
从而它们合成的结果仍为一角动量,即有
[] ()
[]
=
=×=
=+
0,
,
2
i
kijkji
JJ
JiJJJiJJ
JSL
r
h
vv
h
vvv
或简记为ε
此外,可以验证还有
[][ ]
[][ ][ ][ ]
=?=?=?=
==
0,,,,.
0,,
222
22
z
ii
JSLJSLSSLLSL
SJLJ
rvrvrrrv (7.11)
最后一个对易子为零的公式是由于
( )
222
2
1
SLJSL=?
vv
总之可得两组关于角动量的完备力学量组,它们是
( ) ( )
zz
SSLLSLJJ,,,,,,
22222

v
(7.12)
在每一组的组内四个角动量彼此对易,存在共同的本征态,由这些本征态便构成一个关于角动量状态的表象,于是可得如下两个表象,
它们构成耦合表象基矢作为——,
2
1
,,,
l
j
mj
它们构成无耦合表象作为基矢——,,
2
1
,,
s
mml
前者称为耦合表象是因为,如果哈密顿墨H中含有SL
vv
的项,即旋—轨耦合项时,在此表象中仍能将H对角化,而后者则不能,因为这时已不守恒。下面叙述中,如不会产生混乱,就是说如果计算是在量子数为确定值的子空间中进行,也常将耦合表象基矢简记为
zz
S和L
S和l
,
j
m,j而无耦合表象基矢则简记为
s
mm。
2,角动量的升降算符
如同轨道角动量那样,可以引入关于自旋算符的升降算符
23
yxyx
iSSSiSSS?=+=
+
,(7.13)
并有以下封闭对易规则(已令) 1=h
[] [ ] [ ] 0,,,,2,
2
=±==
±±±?+
SSSSSSSS
zz
(7.14)
此外还可得对无耦合表象基矢的作用,
±
S
==
=?+=

+
0
2
1
,
2
1
,
2
1
,
2
1
2
1
,
2
1
2
1
,
2
1
2
1
,
2
1
,0
2
1
,
2
1
SS
SS
为证明这些作用式,先证明关于轨道角动量升降算符的如下两个作用式
( ) ( ) 1,11 ±±?+=
±
mmmmL llll (7.15)
证:根据对易子[ ] [ ],,,0,
2
±±±
±== LLLLL
z
和可得
()( )( )
()()()
±=
+=
±
±
±±
mLmmLL
mLmLL
z
ll
llll
1
1
2
于是可记
,1,+=
+
mmL
m
ll
l
α 1,?=
mmL
m
ll
l
β
这里
ml
α和
ml
β是两个待定的实系数
a
,至此,我们再根据
( ) ( )[ ] mmmmLLLLLLL
zz
llll 11,
22
+?+==
+?+?
可得
结合上面结果,可得
( ) ( )11
1,
+?+=
+
mm
mlm
ll
l
βα
满是这个等式的最简单的取法是
( ) ( ).11
1,
+?+==
+
mm
mm
ll
ll
βα
因此又得
( ) ( )11
,
+= mm
m
ll
l
β
最后即得
a
因为总可以略去相因子而不会影响这个态的归一性质以及它与别的态的正交性质。
24
( ) ( ) 1,11 ±±?+=
±
mmmmL llll
由于在上面推导中只使用了角动量的对易规则,而这些规则对轨道角动量和自旋角动量是相同的,因此所得结果对自旋升降算符也适用。由于量子数
±
S
代入上面公式即知
2
1
,
2
1
±==
s
mS
=?=
++
2
1
,2
1
,0
2
1
,2
1
SS 0
2
1
,
2
1
,
2
1
,
2
1
2
1
,
2
1
,
2
1
,
2
1
==

SS
3,自旋—轨道耦合作用与碱金属原子光谱的双线结构
原子中的电子绕原子核作空间运动时将产生磁场,这个磁场必定与电子固有磁矩发生作用,使原有能极劈裂并产生附加能移。这就是自旋—轨道耦合作用。哈密顿量中考虑这一作用所引入的项通常称之为旋—轨耦合项,又称为托马斯项,可以按下面经典图象推导出来,
然后将其算符化并化引入薛定谔方程中。
设电子回旋在电子所处位置产生的磁场为H
v
,电子的磁矩为μ
v
,则此附加项为
H
v
v
μ。磁场H
v
可这样来计算:电子回旋等效于原子核绕电子回旋,于是H
r
是带正电的原子核绕电子回旋时按毕奥—萨瓦特定律所产生的,
υ
v
v
v
r
v
×=×?= E
cr
rZ
v
c
H
e
11
3
(7.16)
这里υ
v
为原子核绕电子的速度,也即电子绕原子的速度,E
v
为电子处库仑场场强。于是
)0( >e
×?
= υσμ
v
v
rh
v
v
E
cmc
e
H
1
2
SL
dr
dV
rcm
rv
=
11
22
这里,prL
vv
v
×=,υ
vv
mp =,
2
σ
vhv
=s
r
Z
e
2
=V。这就是旋—轨耦合项,但它比正确表达式少
2
1
因子,托马斯于1926年将上面推导中使用的电子静止参照系用洛仑茨变换转到合理的原子核静止的参照系,作了所谓的托马斯进动修正,给出了这个
2
1
因子,从而正确的旋
25
—旋耦合项成为
SL
dr
dV
rcm
rv
1
2
1
22
(7.17)
下面对这项的量级进行估算。为此,将r替换为SLa
Z
Bz
和,
1
=a均替换为a即有,p
z
该项()
22
2
22
2
3
2
22
~)(
2
~
2
1
~ β库仑能
c
v
a
Ze
pa
a
ze
cm
B
z
z
.于是,旋—轨耦合效应的量级是个相对论性修正。
碱金属原子光谱双线结构(如钠黄光)的物理根源正在于,最外层价电子自旋与轨道角动量之间平行反平行耦合使能级出现双重劈裂。这时哈密顿量H为
() (),
2
2
SLrrV
m
H
vv
h
++= ξ ()
dr
dV
rcm
r
1
2
22
2
h
=ξ (7.18)
这里,( VSLJSL,
2
1
222
=? )
rv
为等效的屏蔽库仑势,是考虑到碱金属原子的内层电子对核库仑场的屏蔽作用,并且0?)(rξ。取耦合表象基矢(计入主量子数)来计算旋—轨耦合项所造成的能移,
n
() ()()
+?+=?=?
4
3
11
2
1
2
1
2
1
lll
vv
l
l
jjnjmSLrnjmE
njj
ξξ
这里
[]() drrrrR
nln
2
2
0
ζζ


=)(
l
()1
2
12
3
4
3
0
22
22
+
+
=
lll
h
n
Z
acm
e
eff
于是
()
=+?=?
+==?
2
1
1
2
1
2
1
2
ll
l
l
jE
jE
nl
nl
ζ
ξ
由于V为吸引势,是负的,所以()rξ总是正的,也即
nl
ξ是正的,自旋—轨道耦合的结果使
j较大的态有较高的能量,即。对S态反平行
E
平行
E? 0=l,不存在旋—轨耦合造成的双线分裂。对钠原子,外层价电子处于
21
3s态,其上面态由于旋轨耦合而出现双重劈裂:p3
26
,33
2/3,2/1
pp
5896分别为
受激后由这两个3p态向态的跃进产生了钠的双线黄光

2/1
3s
oo
AA 5890和( )作粗略计算对可以引入r
nl
Z
eff
ξ,这时
()r
nl
表达式后可得,于是代入R
r
eZ
V
eff
2
=
()1
2
1
4
3
0
22
22
+
l
h
n
acm
e
eff
eff
ZZ而均极有关系,
0
,0,0
n
3
+ll
Z
eff
,l
ξ时当=> ll
l ±=
1
2
1
2
+
+l
2
2
=
ln
ξ (7.19)
由此可以看出,双线劈裂数值与成反比,和又由于原子实屏蔽和轨道贯穿而依赖于。注意这个粗略计算只适用于
3
n
l积分发散,不能用这里的微扰论办法计算。
4,耦合表象与无耦合表象基矢的相互展开
耦合表象基矢),,,
2
1
(
2
1
jjmjjm
jj
Ll?=,总数为
(1221
2
1
2 +=
+
+
ll;无耦合表象基矢
,
2
1
,
2
1;,,,
2
1
=?=
ss
mmmm lLll总数也为( )12 +l个。这两个表象的基矢数目相等,它们相对于
=
2
1
,sl量子数固定的全体角动量状态而言,各自都构成了完备基。于是这两套基矢之间也应能彼此相互展开。
)
下面寻找任一耦合表象矢在无耦合表象中的展开式。由于自旋角动量在任何方向都只能取两个值
2
h
±,所以自旋—轨道耦合产生的总角动量,其量子数只能有两个数值
2
1
±= lj
===
2
1
0 sj时当l,正号表示两者的平行耦合,负号表示反平行耦合。首先考虑平行耦
27
合情况下的任一耦合基矢[jjmmj
jj
,,,
2
1
,,,
2
1
Lll?+=为这里]中某一既定值。用检查即知,这一展式只涉及无耦合表象中的如下两个基矢,
zzz
SLJ +=
+=j l
m
2
1
,
2
1
,1,
2
1
,
2
1
,,
2
1
2
1
,,,
2
1
21
++==

mmmmm
s
s
mm
smmj
llll ααα
这里的取值应满足
2
1
+= mm
j
。由slmlj
j
,,,
2
1
+=的归一化条件知。
于是只需决定
1
2
2
2
1
=+αα
2
1
α
α
比值即得展开式,为此用SLSLJ
rv
++= 2
222
作用于这个展开式的两边,并注意
()
()( )
() ()( )
+?++?++?=
++?+=?+?
++?++=?+=
++=?
+?
+
++?
2
1
,11
2
1
,11
2
1
,1
2
1
,12
2
1
,12
2
1
,111
2
1
,
2
1
,
2
1
,
2
1
,2
2
1
,2
mmmllmm
mSLmSLmSL
mmmmmmLmm
mSLSLSLmSL
zz
zz
vv
ll
rv
于是可得
{() ()( )}
()( ) () ()
2
1
,11
4
3
111
2
1
,11
4
3
1
2
1
,,,
2
1
2
3
2
1
21
21
+
+?++++?++
+?++
+++=
+=
+
+
mmmm
mmmm
mjl
j
llll
llll
lll
αα
αα
将原先展开式两边乘以
+
+
2
3
2
1
ll并和此式相比较,即得关于系数
21
,αα的两个方程,
化简后,这两个方程即为
28
( ) ( ) ( )
()( ) ()?
++=+?+
=+?+
111
11
21
12
mmm
mmm
lll
lll
αα
αα
(7.20)
这两个方程其实是一个,于是即得比值
m
m
++
=
l
l 1
2
1
α
α
再计入归一化条件,即得,
12
,
12
1
21
+
=
+
++
=
l
l
l
l mm
αα于是最后得展开式为
=
2
1
j
mm
2
1
,
2
1
,1,
122
1
,
2
1
,,
12
1
2
1
,,,
2
1
+
+
+
+
++
=+= m
m
m
m
mj
j
l
l
l
l
l
l
ll (7.21)
类似地,对于反平行耦合情况,
2
1
,
2
1
,1,
2
1
,
2
1
,,
2
1
,,,
2
1
21
++=?= mmmj
j
llll ββ (7.22)
用作用于两边后,得
2
J
() ()( )
2
1
,11
4
3
1
2
1
,,,
2
1
2
1
2
1
21
mmmmmj
j
+?++
+++=?=
+
llllllll ββ
()( ) () ()
2
1
,11
4
3
111
21
+
+?++++?++ mmmm llll ββ
和上面原展开式相比较得
() ()( )
()( ) () ()
+
=
+?++++?+
+
=+?++
+++
2
1
2
1
1
4
3
111
2
1
2
1
11
4
3
1
221
121
llllll
llllll
βββ
βββ
mmm
mmm
化简后即发现这两方程相关,于是得比值
1
2
1
++
=
m
m
l
l
β
β
29
考虑到,即得1
2
2
2
1
=+ββ
12
1
,
12
21
+
++
=
+
=
l
l
l
l mm
ββ于是
=
2
1
j
mm
2
1
,
2
1
,1,
12
1
2
1
,
2
1
,,
122
1
,,,
2
1
+
+
++
+
+
=?= m
m
m
m
mj
j
l
l
l
l
l
l
ll
或者,将这两个态矢等式向坐标表象基矢?θ,,r投影,得到波函数中与角动量有关的展开式
=
++
+
+
=
++

+

+=
+
=
++
+

++
+
+


+
+
)
2
1
(,
),(1
),(1
22
1
),(1
),(
12
1
),,(
)
2
1
(,
),(
),(
2
1
),(
),(1
12
1
),,(
2
1
,
2
1
2
1
,
2
1
1,
)(
2
1
,
2
1
2
1
,
2
1
1,
)(
lj
Ymj
Ymj
j
Yml
Yml
l
S
lj
Ymj
Ymj
j
Yml
Yml
l
S
j
j
j
j
j
j
mj
j
mj
j
ml
lm
zjm
mj
j
mj
j
ml
lm
zjm
θ
θ
θ
θ
θ
θ
θ
θ
θ
θ
逆过来,也可以将无耦合表象的基矢用耦合表象基矢展开,这里只写出结果
=
+
++
++=
+
=?+
=
+
+=
+
++
=
2
1
,,,
2
1
12
1
2
1
,,,
2
1
122
1
,
2
1
,1,
2
1
,,
2
1
122
1
,,,
2
1
12
1
2
1
2
1
ll
l
l
ll
l
l
l
ll
l
l
ll
l
l
l
mjj
m
mj
m
m
j
m
mj
m
m
j
j
作为一个计算例子,下面往算
jzj
jmjm σ在平行耦合
+=
2
1
lj
和反平行耦合
2
1
= lj情况下的数值。
30
()()[]
()()[]
=
+
=++?+?
+
=
++
+
++++?
+
=
+==+=
+
+?=
++
+
++
+
+?

+?



)
2
1
(,
1
11
22
1
1
1
11
22
1
,,
)
2
1
(,
2
1
2
1
,,
2
1
,
2
1
2
1
,
2
1
2
1
,
2
1
2
1
,
2
1
2
1
,
2
1
2
1
,
2
1
2
1
,
2
1
2
1
,
2
1
lj
j
m
mjmj
j
Ymj
Ymj
YmjYmj
j
dmjmj
lj
j
m
mjmj
j
Ymj
Ymj
YmjYmj
j
dmjmj
j
jj
mj
j
mj
j
z
mj
j
mj
jjzj
j
jj
mj
j
mj
j
z
mj
j
mj
jjzj
j
j
jj
j
j
jj
σσ
σσ
§7.3 两个
2
h
自旋角动量的耦合
1,自旋单态和自旋三重态
和前面自旋 —— 轨道耦合相似,由于
21
ss
vv
和是两个不同的自由度,所以
[ ],0,
21
=
ji
SS ( )zyxji,,,= (7.23)
耦合而成的总自旋角动量
21
SSS
rrv
+=
的对易规则一如以前,
SiSS
v
h
vv

.0],[
2
=
i
SS ( )zyxi,,=
这里。
iii
SSS
21
+=
==
==+=
构成自旋单态反平行耦合结果构成自旋三重态平行耦合结果
,0,0
,1,0,1,1
2
1
2
1
s
s
mS
mS
2,两套基矢—耦合基和无耦合基
无耦合基
21 ss
mm 耦合基
s
sm
31

2
1
,
2
1
2
1
,
2
1
2
1
,
2
1
2
1
,
2
1
=
=
+?=
=?
2
1
,
2
1
2
1
,
2
1
2
1
0,0
2
1
,
2
1
1,1
2
1
,
2
1
2
1
,
2
1
2
1
0,1
,
2
1
,
2
1
1,1
注意,耦合基矢中,平行耦合所形成的三重态关于两粒子自旋交换均为对称的;反平行耦合的单态关于两粒子自旋交换为反对称的。这些展开式可以直接验证
,
4
1
1(
2121
σσ
vv
vv
h?=?= SS并注意设脚标为粒子的编号)。比如,
()
+++=
2
1
,
2
1
2
1
,
2
1
2
1
20,1
21
2
2
2
1
2
SSSSS
rr
+
+?
+=
2
1
,
2
1
2
1
2
1
2
1
,
2
1
2
1
,
2
1
2
1
4
3
4
3
() ()
+
++
2
1
,
2
1
2
1
,
2
1
2
1
,
2
1
2
1
,
2
1
iiii
012
2
1
,
2
1
2
1
,
2
1
2
1
2,=
+=
在这些计算中注意,对两粒子中任一粒子分别均有
jj
xj
2
1
2
1
m=±σ,
jj
yj
i
2
1
2
1
m±=±σ,
jj
zj
2
1
2
1
±±=±σ
而且
j
j
j
j
=?
=
1
0
2
1
,
0
1
2
1
,21,=j为粒子的编号。
3,运算举例
[例1] 1,1
2
1
1,1,0,0
2
1
0,1
1,1,
==
zz
ss
,1,1
2
1
1,1,0,1
2
1
0,0
1,1,
=?=
zz
ss
以第一式为例直接验算如下
32
0,0
2
1
2
1
2
1
2
1
,
2
1
2
1
2
1
2
1
,
2
1
2
1
,
2
1
2
1
0,1
1,1,
=
=
+?=

zz
ss
[例2]
()
0,1
2
1
1,1,0,1
2
1
1,1
1,11,1
2
1
0,1,00,0
==?
+==
xx
xx
ss
ss
验证第二式如下
()
+?+=
2
1
,
2
1
2
1
,
2
1
2
1
0,1
2,1,xxx
sss
()1,11,1
2
1
2
1
,
2
1
2
1
,
2
1
2
1
2
1
2
1
,
2
1
2
1
,
2
1
2
1
2
1
+=
++
+=
[例3] 设()= 0,0,cos,sinsin,.sin
1
σθ?θ?θ
vvv
nxosn计算
Q 于是
1
1
cossin
sincos

=?
θθ
θθ
σ
i
i
e
e
n
vv


=?
2
1
2
11
1
2
1
1
0
2
1
0
1
2
1
cossin
sincos
0,0
θθ
θθ
σ
i
i
e
e
n
vv
++?=

2
1
,
2
1
cos
2
1
,
2
1
sin
2
1
,
2
1
sin
2
1
,
2
1
cos
2
1
θθθθ
ii
ee
1,1sin
2
1
1,1sin
2
1
0,1cos

θθθ
ii
ee
+=
4,自旋交换算符
在涉及两个
2
1
自旋粒子自旋态的运算中,利用自旋交换算符常常会带来方便,它可表示为
12
P
33
(
2112
1
2
1
σσ
rr
+=P ) (7.24)
由于
() ( )
∑∑∑
===
+==?
zyxijk
kkijk
zyxi
ii
zyxji
jiji
i
,,
21
2
,,
2
2
2
1
,,,
2211
2
21
σσεσσσσσσσσ
vv
( )
21
,,
21
2
233 σσσσε
vv
=?=


=
ji
zyxijk
kkijk
于是可得
1
2
12
=P (7.25)
同时还有 ()1=h
1
,
4
1
2
1
,12
12
2
12211221
+=
==?
PS
PssP
vvvv
σσ
可以直接验证,这个算符的作用是将来后面态矢中两粒子的自旋第3分量取值交换(所以称它为自旋交换算符)。就是说,对无耦合表象基矢有
12
P
2,1 ss
mm
,
2
1
,
2
1
2
1
,
2
1
12
=?P
2
1
,
2
1
2
1
,
2
1
12
=?P
比如对第一个等式,
()
21
21212112
1
0
0
1
1
2
1
2
1
,
2
1
++=?
zzyyxx
P σσσσσσ
()
+
+
=
21212121
1
0
0
1
0
1
1
0
0
1
1
0
1
0
0
1
2
1
ii
2
1
,
2
1
=
此外还有
,
2
1
12112
σσ
vv
=
PP
1
1
12212
σσ
vv
=
PP
这是因为对比如
1x
σ,有
34
()()
21121
1
2
1
1
2
1
σσσσσ
vvvv
+?+
x
(){ }()[]
121211121
,1
4
1
xxx
σσσσσσσσσ
vvvvvv
+?+=
() ()[ ]
1
2
212121
21
4
1
xxx
σσσσσσσ
vvvv
+?+=
()()[ ]
1
2
2122121
22
4
1
xxxx
σσσσσσσσ
vvvv
++=
= () ()[]
121122121
2322
4
1
xxxxx
σσσσσσσσσ
vvvv
+++
2
212121212
22224
4
1
x
zyyzyzzyx
iiii
σ
σσσσσσσσσ
=
+?+?=
在无耦合表象中,使用代替
12
P
21
2
SSS
rr
和作运算是方便的。比如
2
1
2
1
,
2
1
2
1
2
1
,
2
1
2
1
2
1
,
2
1
2
1
,
2
1
1221
=?
= PSS
rr
最后再举一个同时使用自旋投影算符和自旋交换算符作运算的例子。设算符为
12
S
( )( )
()
21
2
21
12
3
σσ
σσ vv
vvvv


=
r
rr
S
往求。注意
2
12
S
12
11
=?
r
r
r
πσ
v
v
这里
1r
π是第一个粒子自旋态向
r
r
r
e
v
v
=方向投影的投影算符。于是可将改写为
12
S
()( ) ( )1212123
122112
= pS
rr
ππ
∴ ()()( )( )( )( )
2
121221
2
2
2
1
2
12
12121212612123?+= ppS
rrrr
ππππ
()()( ) ( )
2
121221
1212121269?+= pp
rr
ππ
()[]( ) ( )
2
12121212
12121229?++?= pppS
()( )
2
121212
121229= ppS
35
()( )
121212
451229 pSp=
()
122121
226 Sσσσσ
vvvv
+=
计算中用到了投影算符性质.以及
ii λλ
ππ =
2
1
2
12
=P
12121212
SPPS =(这是因为为关于脚标交换为对称的)。
12
S
36
第八章 电磁作用问题
电磁作用和弱作用是迄今了解得最为清楚的基本作用力。特别是电磁作用,在经典力学中对其基本规律早已有很好的研究和阐述。因此,量子力学对于电磁作用下的单体、两体等可解问题的解答成为检验并显示量子力学正确性的试金石和支撑点。本章叙述,除已叙述过的中心场库仑场作用问题之外,电磁场作用下粒子的定态问题和某些含时问题。量子力学的确不负所望,继中心库仑场之后,在这类问题上再次给出了微观粒子电磁现象的正确的统一的理论描述。不仅如此,根据AB效应,量子力学还指出了经典电磁理论用场强表述的局限性,并以浅显的方式丰富了规范理论关于位相物理学的内容。
§8.1 有电磁场的方程 dingeroSchr &&
1,最小电磁耦合原理及有电磁场下的Schrodinger方程
如前所说,在建立Schrodinger方程的一次量子化中,我们使用了以下对应


h
h
ip
t
iE
(8.1)
在存在电磁的情况下,按经典规则,( )?qH?和
A
c
q
p
之间的关系如同无场时(H和p
的)一样
a
,这里电磁场势表示为( )
Au
=( )?iA,,p为正则动量(广义动量)。于是可设无电磁场到有电磁场的速度


A
c
q
ii
q
t
i
t
i
hh
hh?
以这种方式将磁场引入方程,称之为“最小电磁耦合原理”。原则上,这仍是一个假设,其正确性由所导出的全部结论是否与实验相符合所决定。迄今尚未发现不遵守这一原理的电磁相互作用。
dingeroSchr &&
于是,有电磁场存在时方程为 dingeroSchr &&
a
Л.Д,Ланgау,Е.М,Лuфuзuу,Тeорus,Госу,Изgамельсмьо Фuзuко-Мамеuамurеско? Лuмерамурн,Москьа
1960,смр,60。
37
ψ?
ψ
++

=
qV
A
c
q
i
mt
i
h
h
2
2
1
(8.2)
这里,V为其它(如引力势等)势能项,A
c
q
i= hp是机械(普通)动量算符,= hip
为正则动量算符(将x和正则动量p量子化为满足对易字等于i的算符,称为正则量子化方法
h
a
)。现在需要注意粒子的
机械动量≠p正则动量)(= hip
与此同时,
粒子的速度算符)(
1
A
c
q
i
m
v +?
= h
(作为对照,在原先无电磁场时,粒子的机械动量=正则动量,从而仍为正则量子化。)
2,方程的某些考察
将方程展开,为此先计算
{}
AA
c
q
pA
c
q
p
c
q
Ap
c
q
pA
c
q
p
2
2
2
22
2
2
2
2
,+=+?=
这里{,}为反对易子符号,同时,已取定了横向规范条件0=? A?。于是得方程的展开形式
dingeroSchr &&
ψ?
ψ
+++=
VqA
mc
q
pA
mc
q
m
p
t
i
2
2
2
2
22
h (8.3)
其次,往求几率流密度的表达式并考察几率守恒问题。由上面电磁场下方程,取复共轭,得
dingeroSchr &&
++
=
ψ?
ψ
VqA
c
q
i
mt
i
2
2
1
hh
将这个方程和原先方程分别乘以ψ和
ψ
并相减,即得
a
设L为有电磁场下粒子的拉氏量,按Legendre变换,得哈密顿量H= L
v
L
v
,这里
v
L
p
=
38
()
()0
2
=
+

ψψψψψψ
ψψ
A
mc
q
mit
h
令,
=ψψρ ()
=

ψψψψψψ A
mc
q
mi
j
2
h
,前者为ψ态中( )tx,处的几率密度,
后者为(存在电磁场下)ψ态中的几率流密度。于是仍存在表征几率守恒的连续性方程
0=+
j
t
ρ
(8.4)
这里,j中含矢量A的第二项是电磁场影响粒子的机械动量从而使几率流密度改变。
其三,考察一下电磁场下方程的规范不变性问题。 dingeroSchr &&
对任意可微函数( )trf (它具有磁通的量纲),可引导出对电磁势的一个规范变换
=→
+=→
t
f
c
fAAA
1
'
'

(8.5)
可以证明,在方程中,当势dingeroSchr && ( ) ( )',', iAiA →,即经受上述规范变化时,只要波函数也经受一(定域的)位相变换ψ
e
c
qf
i
h
=' oSchr &&ψψ →,方程就保持形式不变。这就是电磁场下方程的规范变换不变性。
dinger
dingeroSchr &&
证明:注意有
() ()[]
=
++=
=?
ψψψ
ψψ
)(' A
c
q
pfA
c
q
f
c
q
pA
c
q
p
t
f
c
q
t
i
t
i
eee
ee
c
qf
i
c
qf
i
c
qf
i
c
qf
i
c
qf
i
hhh
hh
hh
∴ ψ?ψψψ
++?=
t
f
c
qVA
c
q
p
mt
f
c
q
t
i
1
)(
2
1
2
h
于是,
ψψψ
ψ
qVA
c
q
p
mt
i ++?=
2
)(
2
1
h

39
'''')'(
2
1'
2
ψ?ψψ
ψ
qVA
c
q
p
mt
i ++?=
h
是同时成立的.这里',','?ψ A分别由上面变换式表示。
于是,在电磁场中,由于电磁势不是唯一的,可以相差任一规范变换,因而粒子的波函数也不是唯一的,可以相差一个局域的(即随空间点改变而变化的)位相。
最后,再考察一下时间反演问题。对于一个定态问题,
ψψ? EVqA
mc
q
pA
mc
q
m
p
=
+++
2
2
2
2
22
(8.6)
在时间反演下,pp?→,于是只当与此同时,改变磁场,既令AA?→ (由于AB ×?=,所以也即BB?→ ),才可以不变,这与宏观情况相同。
§8.2 均匀磁场下粒子的运动—无自旋情况
1,均匀磁场下不考虑自旋的中心场方程 dingeroSchr &&
这里将上面方程用到处于均匀磁场中的单电子原子问题中。于是
() rBA
r
Ze
rV ×=?=
2
1
,
2
( )AB ×?=
2
2
2
2
2
)(
2
A
mc
e
pA
mc
e
rV
m
p
H +?++=
2
2
2
0
2
A
mc
e
pA
mc
e
H +?+=
2
2
2
0
)(
8
)(
2
rB
mc
e
prB
mc
e
H ×+?×+=
( )[ ]
()
()rBrB
mc
e
LB
mc
e
H
rBrB
mc
e
LB
mc
e
H
rBrB
mc
e
prB
mc
e
H
222
2
2
0
222
2
2
0
2
2
0
sin
82
82
)(
8
)(
2
+?+=
+?+=
××?+×?+=
40
若取
z
BB ε=,则
()
222
2
22
2
822
yxB
mc
e
L
mc
eB
r
Ze
m
p
H
z
+++?=
现来估算一下
2
B项和B项的比值。对磁场中的原子( )
2
822
10~~ cmayx
B
+,于是
4
2
2
22
10
2
10~
1374
4
4
~
5
=
×
=
=
e
Ba
e
c
e
Ba
c
eBa
B
B
BBB
B
hh
高斯项含项含
可知,和含B一次幂的项相比,经常可以略去含
2
B的项.这意味着对磁场作线性近似.在这种近似下,哈密顿量为
z
L
mc
eB
r
e
m
p
H
22
2
2
+
= (8.7)
2,正常塞曼效应
考虑上面哈密顿量的定态问题,也既在均匀外磁场下的氢原子问题。
+
=
=
z
L
mc
eB
r
e
m
p
H
EH
22
2
2
ψψ
(8.)
这时,守恒量为( )
z
LLH,,
2
,于是好量子数为(nlm)。波函数和能谱分别为
() ( ) ( )
+=
=
.
2
,,,
hm
mc
eB
EE
YrRNr
nlnlm
lmnlnlnlm
θ?θψ
+=
=
=
lllm
l
n
,...,1,
,...2,1,0
,...3,2,1
由此可知,一旦加上外磁场后,系统的球对称性遭到破坏,原有的能级关于磁量子数m的简并
(2l+1重)即被解除,一条能级等间距地分裂为(2l+1)条,间距为hh
拉摩
ω=
mc
eB
2

由于能级分裂,相应光谱线也发生分裂.但由于存在选择定则(参见后面跃迁章)Δm=0,
±1,故光谱线呈现
三分裂(⊥B方向关察)
二分裂(∥B方向观察)
41
l=2 d



l=1 p
图,这里谱线上±1,0等数字表示,相对于原先无磁场时的谱线频率,移动
(+1表示增加,-1表示减少,0表示不变)了一个拉莫(Larmor)频率。于
是,这里总共出现了三条不同频率的光谱
LL
ωωωωω?+,,
但不变的那条对应于Δm=0,在‖B方向观察不到。
3,均匀磁场下自由带电粒子运动
这时V(r)=0。取B在
z
e方向,矢势( )0,0,ByA?=,哈密顿量H为
++
+=
22
2
2
1
yxx
PPy
c
qB
P
m
H (8.9)
注意,这里并没有略去
2
A项,即未做磁场的线性近似。
由于H中不显含x、z,故守恒.但这里的是正则动量,而不是机械动量
zx
PP,
x
P
,y
c
qB
PP
xx
+= 并不守恒,只有是机械动量,于是只有z方向的速度恒定并可连续变化,但x方向的速度并不恒定。因此,虽然波函数可写为
x
P
z
p
()
( )
( )yxezyx
yPxPi
yx
h/
,,
+

但能量却为
mc
Bq
n
m
P
E
z
h)
2
1
(
2
2
++=
并不含。这里的能量表达式是将
x
P ( )zyx,,ψ代入上面定态方程中,化为关于
y的谐振子方程而求得。由这里的能量表达式可知,磁场中的自由带电粒子其磁附加能为正值,因此具有反磁性。
dingeroSchr &&
42
§7.4 均匀磁场下束缚粒子的运动 —— 1/2自旋情况之二
1,求解中心场加均匀磁场下
2
1
自旋粒子Schrodinger方程。
这时,存在自旋及轨道磁矩向外磁场取向的附加能以及自旋—轨道耦合能这三项附加能量。于是,再略去
2
A项即对磁场影响作线性近似之后氢原子的哈密顿量为
() (
SL
e
p
zz
mc
eB
SLr
rm
H 2
22
2
2
++?+
= ξ ) (8.10)
这里已假设了外磁场在z轴方向,()
rcm
e
r
322
2
1
2
=ξ,如果研究只限于量子数n、l均为给定值的状态子空间中(如下面所进行的),则可代以常数( )nlrnl
nl
ξ
ξ
=。记
βα
ξ
===
mc
eB
rm
nl
H
e
p
2
,,
2
2
0
2
2
h
h
即得
)2(
0 SLH zz
SLH ++?+= βα (8.1)
这里,角动量SL,均已无量纲化,而α,β的量纲均为能量。这一定态问题(也包括给定初态的含时问题)其实不必近似求解,从而对以往常在微扰近似下分别求解的塞曼效应、反常塞曼效应、帕刑-巴克效应等(磁场下谱线分裂的各种)现象作一统一的叙述。下面根据文献
a
中思想给出这个统一的求解过程和结果。
为此,引入升降算符和
yx
iLLL ±=
±
()
±±
=±= σσσ
yx
iS
2
1
,并注意
( )
yyxxzz
LLLLSL σσσσ ++=?=?
2
1
2
1
2
1
()
++?
++= σσσ LLL
zz
2
1
2
1
这里为1/2自旋的升降算符。于是可将H改写为如下形式
=
=
+
01
00
,
00
10
σσ
43
a
Sixia Yu,H.Rauch,Yongde Zhang,Phys,Rev,A,vol,52,No,4,2585 (1995)。
(
++?
++
+++= σσ
α
σβ
α
β LLLLHH
zzz
22
0
) (8.12)
由于H中包含
zz
SJSL ββα 2++?,于是,除能量之外,L及守恒,但
22
,S
z
J [ ] 0,
2

z
SJ,
故不守恒。这时好量子数为
2
J ( )
j
nlsm但不包含j.这允许我们在这几个好量子数分别取确定值的任一子空间中考虑问题(这时j=l±1/2,取两个可能的值)。这时定态方程为
dingero&&Schr
jj
zz
zz
nlmEnlm
LLHL
LLLH
=
+?+
+++
+
β
α
β
α
α
β
α
β
22
22
0
0
由于为守恒量子数,现取为固定值,于是应有
j
m
,
1,
2
1
+
=
mnlr
nlmr
nlm
j
=
2
1
j
mm
这里为两个待定系数。这个方程左边为耦合表象基矢的一个叠加态(j=l±1/2),右边则为无耦合表象基矢的叠加。代入方程并令,得特征方程
21
,rr
nl
EEE?=
'
()( )
()( ) ()
=
+
++?+
=+?++
+
+
01
2
11
2
011
22
2
'
1
21
'
rEmrmmll
rmmllrEm
β
α
β
α
α
β
α
β
令此联立方程的行列式为零,即得决定系统能谱的公式
()
()
2
2
2
12
2
1
2
1
42
βαβα
αβ
β +++
+±?++=
±
mlmEE
nl
(8.13)
随后,系数的表达式(考虑)即为
21
,rr 1
2
2
2
1
=+ rr
44
()
()
+++
+
+
+
=
+++
+
+
+
+=
+
+
2/1
2
2
2
2
2/1
2
2
2
1
2
1
2
2
1
2
2
1
2
1
2
1
2
2
1
2
2
1
2
1
ml
m
r
ml
m
r
αββα
βα
αββα
βα
(对于
)(+
E)
以及
( ) ( )
() ()
=
=
+?
+?
12
21
rr
rr
(对于
()?
E)
这里的推算中注意下面的等式
()
+?+++
+=+?+ βαβαβα
2
1
12
2
1
)1()1(2
2
2
2
mmlmmll
()
2/1
2
2
2
2
1
12
2
1
+
+++++
+? βαβαβα mml
于是,给定( )
j
nlm的定态解为
()
()
()
()
()
()
()
()
+
=
+
=
+
+
+
+
,
1,,
,,
,
1,,
,,
/
2
1
/
2
1
h
h
tiE
nlm
tiE
nlm
e
mlnr
mlnr
e
mlnr
mlnr
j
j
ψ
ψ
=
=
2
1
2
1
j
j
mm
mm
(8.14)
2,讨论
分别以下几种情况对上面结果进行讨论。
i,β=0情况。这时无外磁场,只存在电子的旋-轨耦合项。上面
()±
E表达式简化为
45
()
+
+=
±
,
2
1
,
2
α
α
l
l
EE
nl
=
+=
2
1
2
1
lj
lj

()
()
+
=
+
++
=
+
+
,
12
12
1
2
1
l
ml
r
l
ml
r
()
()
+
++
=
+
=
.
12
1
12
2
1
l
ml
r
l
ml
r
于是这两组系数退化为
j
nljm态在无耦合表象中展开时的C.G.系数。这时由于,(这时和H已是对易的了)定态解已是量子数j的本征态。
2
J
ii,β《α,即外磁场很弱,以致于L和S对外磁场取向的附加能量很小于旋—
轨耦合能。这就是考虑旋—轨耦合的反常塞曼效应。将
( )±
E表达式对β展开,保留到
β的一阶项,
()
+
+
+
+
+
+
+
+
+=
±
,)
2
1
(
12
2
2
1
,)
2
1
(
12
22
2
βα
βα
m
l
ll
m
l
ll
EE
nl
=
+=
2
1
2
1
lj
lj
这时,含β的项相当于在上面i 中β=0结果的基础上,取
j
nljm耦合基矢对外磁场
附加一阶微扰计算所得。就是说这时
( )
jjzzj
gmjmSJjm ββ =+
这里,.
)1(2
)1()1()1(
1,
2
1
+
+++?+
+=+=
jj
sslljj
gmm
j
其中用到上一章的计算结果
+
=
,
1
,
j
m
j
m
jmjm
j
j
jzj
σ
=
+=
2
1
2
1
lj
lj
简单验算即知这些结果都是一致的。
iii,磁场很强,即β》α情况。
46

()±
E对α展开,保留到α的一阶项,得
()
+?
++
+=
±
,)1(
2
1
,
2
1
)1(
αβ
αβ
mm
mm
EE
nl
这就是在α =0(即,略去旋-轨耦合,只剩下( )
zz
SL 2+β项)基础上,选取无耦合表象基矢对旋-轨耦合效应作一阶微扰计算的结果,即
+?=?++
=?
,)1(
2
1
2
1
,1)(
2
1
,1
,
2
1
2
1
,)(
2
1
,
αα
αα
mmSLm
mmSLm
+=?=
+=+=
2
1
,
2
1
2
1
,
2
1
mmlj
mmlj
j
j
注意,这时情况仍为守恒,而并不守恒。
z
J
2
J
而α=0情况即为Paschen-Back效应。这是当磁场十分强,电子的轨道磁矩及自旋磁矩均分别独立的受外磁场作用而略去自旋-轨道之间的相互作用的的情况。值得注意的是,
这时,
( ) ( )
0,1
0201
==
=
+
=
+
αα
rr
于是对应
()+
E能级的态为
0
nlm
;对
( )?
E为
+1,
0
mnl

iv,在一般磁场情况下,对于m=l的能级,由
( )±
i
r的表达式知,
() ()
( )
( ) ( )
( ).1,0;0,1
2121
====
=
=
=
+
=
+
lmlmlmlm
rrrr
并且这时
()±
E分别为,
()
+?
++
+=
±
,)1(
2
1
,
2
1
)1(
αβ
αβ
lm
lm
EE
nl
表现出能移和α及β呈严格线性关系。而对于m=-l,则无此现象。总起来计算,在任意磁场下,能级分裂总数为2(2l+1).(对每一个m值存在
( )±
E两个能量值,不同m值有2l+1个)。
47
§8.3 均匀磁场下自由中子的运动——1/2自旋情况之一
1,中子极化矢量在磁场中的运动。
中子自旋为1/2,并有一反常磁矩σμμ
n
=,91314.1?=
n
μ核磁子。于是,磁场中自由中子哈密顿量H(不计平动部分)为
BBH
n
== σμμ (8.15)
设中子的自旋为|λ>,则中子极化矢量的运动方程为
+
== λσλλσλλσλ
λ
ttdt
d
dt
Pd
[ ]λσλλσλλσλ H
i
H
i
H
i
,
111
hhh
=+?=
由于等式[ ] ( )σσσ ×=? AiA 2,,现在矢量BA
n
μ=,即得
λ
P在磁场中的进动方程
( )
λ
λ
Pe
dt
Pd
BL
×= (8.16)
这里B
nL
μ?
h
2
=是Larmor频率,
B
B
=
B
e为磁场方向的单位矢量.由这个方程可知,磁场中中子极化矢量
λ
P将绕磁场方向作右手进动运动,其进动频率为Larmor频率。
2,中子的旋量叠加与干涉——中子干涉量度学。
讨论中子通过板状均匀磁场的问题。这时
B
m
H
n
+= σμ
2
2
h
(8.17)
设()
in
rs,ψ和()
out
rs,ψ分别代表射入和透出板状磁场时中子的状态矢量。若不记板状磁场界面上的中子波反射损失,态矢模长不变,可得
in
B
m
H
i
in
H
i
out
n
ee ψψψ
σμττ )
2
(
2
+
==
h
hh
由于H中空间部分和自旋部分可交换,可以将态矢中空间部分分离掉,于是可得自旋部分
()
in
i
in
B
i
out
seses
n
)()(
2
ψψψ
ρσσμτ
==
h
这里
BL
eτ?ρ =,τ为中子在板状磁场中穿行的时间,于是ρ即为在磁场期间中子极化矢
48
量进动转过的总角度。利用前一章中有关公式,上式即为
()
inBLLout
seis )()])(
2
1
sin()
2
1
[cos( ψστ?τ?ψ=
值得注意的是这里的表达式中有个1/2因子。在自旋态矢如上变化的同时,极化矢量P的变化可如下求得。因为
() ( )
outoutoutininin
ssPssP )(,)( ψσψψσψ == (8.18)
于是
()
in
ii
inout
seesP )(
22
ψσψ
σρσρ
=
.
100
0cossin
0sincos
},sincos,sincos{
)
2
sin
2
(cos)
2
sin
2
(cos
,
,
,
,,,,,

=
+?=
+=
zin
yin
xin
zinxinyinyinxin
inBBin
P
P
P
PPPPP
eiei
ρρ
ρρ
ρρρρ
ψ
ρ
σ
ρ
σ
ρ
σ
ρ
ψ
于是
inBout
PeRP ),( ρ= (8.19)
这里),( ρ
B
eR表示绕
B
e方向转过ρ角的空间转动变换,在此是个3×3的正交矩阵。
举两个例子说明。其一为一单色热中子束,再中子干涉仪(由整块硅(挖成“山”字型)
作成)的A点由于Laue散射而被分解成透射和衍射的两束,然后又分别在B和C点经过反射,交汇于D点.其中一束穿过一个横向板状均匀磁场B区域,距离为l。假定从A到D的这两条路径除磁场外完全对称,在中子极化方向平行于磁场情况下,求出点D的强度依赖于B、l和中子波长λ的关系。
解:设AC束前进方向为y,
x
eBB =,
按题设{ }0,0,1==
outin
PP,于是
()
=
1
1
2
1
in
sψ。由于两条空
间路径相同,且中子不带电荷磁
场对中子空间波函数不起作用,故
49
空间波函数对D点的干涉不起作
用,D点干涉强度只决定于自旋波
函数的相干叠加,并正比于
2
0
)1(
2
0
)1(
2
)2()1(
),(),(),(),(
n
D
i
DDD
tsetststs
x
ψψψψ
ρσ?
+=+
()
4
cos4
1
1
)1)(1(1,1
2
1
2
22
ρ
ρσρσ
=
+=
xx
ii
ee

).
4
(cos)0()(
2
2

λμ Blm
IBI
n
DD
=
值得注意的是,当ACD分支穿过这个磁场区时,若l(或B)选择得使极化矢量P转过的总角度ρ=2π,就是说
inout
PP =时,这一分支的自旋波函数仍未完全还原,而是出一负?的相因子,以致使D点的相干叠加呈现极小。这就是著名的自旋1/2粒子波函数的旋量性质:
波函数转2π时出负号,只当P绕磁场方向转过4π时才完全还原。中子干涉量度学的这个实验证实了这一点:中子的波函数是个旋量波函数。
例2.非相对论中子的自旋回波共振(echo).在上例的中子干涉仪中,如图安置两个磁场( )( )
2211
,,,lBlB,
则D点的态矢为
( )
()0
2
21
1
2
1
D
i
IIID
e ψψψψ
σρρ
+=+=
+?
50
这里
Biiii
eτωρ =
()
,(i=1,2),为两个磁场区域中极化矢量的转动角矢量(方向分别为两个磁场方向),
0
D
ψ为无磁场时D点的态矢。于是D点的强度为
( ) ( )
()
2
0
22
2
2121
11
4
1
DD
ii
DDD
reerI ψψ
σρρσρρ
+
+==
++

()
+
=
4
cos
21
20
ρρ
DD
II
如果两个磁场的强度、长度相同,但方向相反,就成为中子自旋回波共振装置(实践中,如
l、B不等,总可以调整产生磁场的线圈电流强度,使D点达到中子计数率的极大值即可),
好象天平的两臂,达到了平衡。这时()π
π
λμ
ρρ
klBlB
m
n
2
44
1122
2
21
=?=
+
h
。由于这是位相的平衡,十分灵敏,一旦在两臂之一施加某种影响(如在一段路径上加入物质薄层,相当于加入移动位相的相移器),平衡极易遭破坏,D点中子计数率呈明显变化。由这种(以及类似的)安排,在中子干涉仪上完成了大量有关检验基本原理的实验研究和实际测量,构成了具有高精密度的中子干涉度量学。详细情况可参见有关文献
a

§8.5 Aharonov-Bohm(AB)效应
经典力学中,描述电磁场和带电粒子运动的Maxwell方程和Lorentz力公式,都是用场强表达的.电磁势的引入只为数学上的方便,并不具有物理意义,只有在规范变换下不变的场强才有无论意义.在量子力学中,电磁场下的方程虽然用电磁势来表示,但由于电磁势经规范变换时仅仅导致波函数多一个相因子(即方程的规范变换不变性),
因此人们一直认为在量子力学中,也如同在经典力学中一样,只有电磁场的场强才具有可观测的物理效应,电磁势不具有可观测的物理效应。但是,1959年Aharonor和Bohm提出
dingeroSchr &&
Schr dingero&&
b
,
在量子力学中,电磁势有直接的可观测的物理效应。这里,主要取自
c
1,磁AB效应
为节省篇幅,这里只讲述磁AB效应,关于电AB效应可见上面附注3所引介绍AB效应及
a
U.Bonse,
b
Y.Aharonov,Q.Bohm,Phys,Rev.,115,485 (1959); R.G,Chambers,Phys,Rev,Lett.,5,3 (1960)。
c
张永德,大学物理,第6期,第1页,1992年。
51
有关问题的文章.AB效应是一种表面看来很奇异的量子效应,它表明,在某些电磁过程中,具有局限性质(因为是关于空间坐标微商)的电磁场场强不能有效地描述带电粒子的量子行为。
它可用如图的理想实验来说明。
在电子双缝实验的缝屏后面两
缝之间放置一个细螺线管。通
电后管内H≠0;但管外H =0,
矢势A≠0.这个细螺线管产生
一细束磁力线束,称为磁旋。理
论分析表明,相对于没通电的情
况来说,通电后,接受屏上的干
涉花样在包络(图中虚线所示的轮廓曲线)不变情况下所有极值的位置都发生了移动,电流改变峰值位置跟随改变,电流反?峰值位置也反向移动.下面对此作相应的理论分析,
由于两个缝,a是相干分解,不失一般性,可以假设缝和上电子波函数有相同的位相,并将之简化为下图。
1
a
2 1
a
2
a
通电前,
() ()
() ()
=
=
h/
00
00
2
2
iEt
ertr
rEtr
m
P


C点的合振幅为
() ()
()
()
( )cfcff
c
0
2
0
1
0
+=,通电后,A
c
e
PP?→。于是有,
() ()
() ()
=
=
h/
2
2
1
iEt
ertr
rErA
c
e
P
m


直接验算即知,此方程的解为
() ()rer
r
a
rdrA
c
ie
0
''


=

h
(8.20)
注意,这里的相因子在H≠0的区域与路径有关,因而是不可积的;只在H =0的区域与路径无关(这也说明了,磁场毕竟是一种物理的实在,并不能通过数学变换将其转化为相因子)。
52
于是,通电情况下,C点的合振幅为
()
()
()
()cfcff
c
a
c
a
ldA
c
ie
ldA
c
ie
c
0
2
0
1
2,1,
expexp

+

=

hh
()
()
()
()

+

=

cfecf
ldA
c
ie
ldA
c
ie
c
a 0
2
0
1
1,
exp
hh
这里,指数上线积分的脚标1和2表示积分分别沿路径1和2进行。大括号外的相因子是新增加的外部相因子,没有可观测的物理效应,可以略去;大括号内
( )
( )cf
0
2
前的相因子为新增的内部相因子,它改变了两束电子在C点的相对位相差,从而改变了双缝干涉的极值位置.这个内部相因子还可改写为
( ) φ
c
ie
dSA
c
ie
ldA
c
ie
eee
hhh
=
∫∫
=

×
(8.21)
这里φ是回路所包围面积内的磁通。由于这个相因子并不改变单缝衍射的强度分布,所以在条纹移动时,诸条纹极值的包络曲线仍不变。这些结论已一再为实验所证实
a

2,向电磁AB效应的推广
众所周知,电磁现象是Lorentz变换不变的,磁的和电的现象经过Lorentz变换可以相互转换。因此上面的电磁AB效应应当扩充为包括电AB效应在内的Lorentz变换协变形式。
这时,由于() ( )?
μ
iAA,=和()( )ictxx,=
μ
,上面关于相因子的路径积分应扩充为
( )
∫∫
=→

dtcxdAdxAldA?
μμ
于是不可积相因子成为
.exp

μμ
dxA
c
ie
h
(8.2)
由于在Lorentz变换下是标量,于是总电磁AB效应在Lorentz变换下是不变的。
μμ
dxA
与此同时,总的电磁AB效应也是规范变换不变的。因为,对于任一可微函数( )txf,所导引出的规范变换
() ( ) ( ) ( )xfxAxAxA
μμμμ
+=→
'
所以上面不可积相因子又是规范变换不变的。
3,几点讨论
a
53
I,关于用场强表述和用势表述的基本问题
AB效应表明,用场强不能完全描述全部可观测的微观电磁现象,或者说,就量子力学而言,BE,所提供的信息不足。但是,势( )?iA,是规范变换可变的,因此它们虽然能描述全部微观电磁现象,但却提供了过多的信息,就是说势( )?iA,中也包括了非物理的信息。
只有在规范条件约束下的势既能描述全部电磁现象,有很少(并非完全没有!)提供多余的非物理的信息。更准确地说,电磁学正是不可积相因子
( )
+?

xdAdtc
c
ie
h
exp (8.23)
的规范不变的表现
a

ii,电磁场的局域性与整体性问题。
按一般理论,一个物理的事物应当是Lorentz变换协变的和规范变换不变的。以前宏观磁场现象的描述中,使用的场强张量
,
μννμμν
AAF= (8.24)
这是一些关于势场的微分量,表征了势场的局域性质,的确是满足上述不变性的要求,可以作为物理量。但是,满足这两个一般要求的结构并非只有这种微分形式,现在AB效应中不可积相因子里的闭合回路积分

μμ
dxA也能满足这两个一般要求,也可以作为一种物理的事物而表现出可观测的物理效应。然而,现在的这种(具有不变性的)形式并非是微分量而是积分量,因而体现了势场的整体性质。所以说,AB效应是电磁势场整体拓扑性质的物理体现,而以前全部宏观电磁现象只是电磁势场局域性质的物理体现。
iii,AB效应并不证明微观世界有超距作用存在。(以下推导可借鉴MergGacher,
P.596)
以磁场中的自由电子为例,此时哈密顿量为
2
2
1
= A
c
e
p
m
H (8.25)
于是速度算符为
[ ],
1
,
1
== A
c
e
P
m
Hx
i
v
h
而量子Lorentz力算符为
a
T.T,Wu,C.N,Yang,Phys,Rev,D,12,3845 (1975)。
54

=
==
A
c
e
PA
c
e
PA
c
e
P
mi
HA
c
e
P
i
vmF,
2
1
,
1
hh
其中,第i分量方程为(重复指标求和)
=
jjjjiii
A
c
e
PA
c
e
PA
c
e
P
mi
vm,
2
1
h
+
=
jjiijjjjjjii
A
c
e
PA
c
e
PA
c
e
PA
c
e
PA
c
e
PA
c
e
P
mi
,,
2
1
h
这里
()()()()
ijjijjii
Ai
c
e
Ai
c
e
A
c
e
PA
c
e
P+=
hh,
()()(){}(),
kijkijji
Bi
c
e
AAi
c
e
εhh == ( ).AB ×?=

+
=
kjjijkjjkijki
BA
c
e
Pi
c
e
A
c
e
PBi
c
e
mi
vm εε hh
h2
1
+
=
kjjijkjjkikj
BA
c
e
PA
c
e
PB
c
e
εε
2
最后得
×?×+×+×?=
BA
c
e
BPAB
c
e
PB
c
e
vm
2
×?×
= A
c
e
PBBA
c
e
P
c
e
2
,

( ).
2
vBBv
c
e
F ×?×= (8.26)
这个关于磁场中Lorentz力的量子化算符公式表明,量子的Lorentz力公式可以直接由经典的
Lorentz力公式经一次量子化得到。和经典物理一样,在此处方程右边的力的表达式中,当地的力只和当地的参量(粒子的v和场的B)有关,即只含物理量在当地的数值,这说明微观世界也不存在超距作用力。
55
§8.6 超导量子理论基础
1,超导体中的流密度与London方程
低温下凝聚态物质中的导带电子大量耦合成Cooper对,构成无相互作用的玻色子气
体,它们的流动形成超导电流,这一电流不服从欧姆定律
a
。对于由Cooper对组成的玻色气体,其几率幅可写为
( ) ( )
( )tri
etrtr
θ
ρψ = (8.27)
这里( )trρ为Cooper对玻色子的密度。将它代入流密度表达式粒
j中,
= ψψψψψψ
***
2
2
1
A
c
q
PP
m
j

这里m为此玻色子的有效质量(q=-2e为它的电荷),即得
= A
c
q
m
j θ
ρ
h

于是超导电流密度为
== A
c
q
m
q
jqj θ
ρ
h
粒电
对此式取旋度,注意θ?的旋度为零,得
B
mc
q
j
ρ
2
=×?

(8.28)
这里BA=×?。这就是早期超导唯象理论中著名的London方程。
2,Meissner效应
这个实验效应是说:块形超导体在外磁场中是个相当理想的抗磁体,其内部(除表面
薄层以外)0=B。
由London方程出发,用Maxwell方程消去电
j即可解释这一效应。因为根据Maxwell
方程,对于现在稳定情况,有
电电
j
c
j
ct
E
c
B
ππ 441
=+
=×?
a
当然,一般也存在未配对的电子,它们的行为造成Ej
n
σ=,服从欧姆定律。
56
代入London方程,
( ) B
mc
q
B
c
j
ρ
π
2
4
=×?×?=×?

由于( ) ( ) BBBB==×?×?,即得
,
1
2
BB
L
λ
=?
ρπ
λ
2
2
4 q
mc
L
=
L
λ称为London穿透深度,约为( )cm
56
1010

量级。这样称呼
L
λ是因为,对于一维情况,
可得块体外部的磁场穿入超导体时衰减公式
L
x
eBB
λ/
0
= (8.29)
由此,除表面薄层外,0=

j,于是按上面Maxwell方程,0=E,总之,在超导体内部,
0===

jEB。
3,磁通量量子化
设有一细环状超导体,环内有一磁场.H取超导体中离开表面的一条回路C,在这条回
路上有
.
= A
c
q
m
j θ
ρ
h

由于在C上处处有.0=

j于是在C上处处有
A
c
q
=?θh
环绕一圈的位相变化为
ldA
c
q
ld

=?=
h
12
θθθ

由于在现在的
θ
ρψ
i
e=玻色子Cooper对近似中,ρ和θ均成为可观测的量,于是ψ必须为单值函数,即要求
,2
12
πθθ n=? ( )L,2,1,0 ±±=n (8.30)
而同时
57
( ) Φ=?=?×?=?
∫∫ ∫∫∫
SdBSdAldA

0
2
Φ==Φ n
q
c
n

,
e
chπ

0
这说明,超导环中的磁通Φ是量子化的,其最小单位为
e
chπ

由磁通最小数值还可以导出磁单极子μ的强度——假定它存在的话。
设μ位于坐标原点,取μ上方的半个球面,于是通过这半个球面的磁通将为这个最小磁通,即有
( )
0
2
2
4
2
1
Φ=? r
r
π
μ
于是得
.
2
137
2
e
e
c
==
h
μ
这就是Dirac著名的关于磁荷与电荷关系的公式。但迄今实验上并未找到磁荷,所以上面的论断在物理上是存在问题的。
4,超导Josephson结的AB效应
i,直流和交流Josephson效应及单结磁衍射现象。
这是超导电子对从一个超导体穿过一层薄绝缘层进入另一超导体的隧道贯穿现象。这
一薄绝缘层和其两边的超导体便构成一个Josephson结。实验表明,当结的两端不加任何电磁场时,有直流电流通过结,这称之为直流Josephson效应;当结上加了直流电压时,有射频交流电流通过结,这称之为交流Josephson效应;当结上存在恒定磁场时,出现电流强度随磁通变化的单结磁衍射现象。它们的物理机制统一简述如下。
设结的左、右方超导体中的电子对几率幅分别为
21
,ΨΨ;?h唯象地表示结的特征常数,表征结两边电子对耦合作用所造成的隧道效应,由于绝缘层很薄,这种量子作用只与一次方有关,Ω的量纲是频率。若结上加有电压V和磁场
h
B(见图),
58
21
ΨΨ和的方程为
Ψ+Ψ

=
Ψ?
Ψ?Ψ

=
Ψ?

21
2
12
1
2
2
2
1
2
1
qV
e
t
i
qV
e
t
i
ldA
c
iq
ldA
c
iq
h
h
hh
hh
(8.31)
这里q=-2e是电子对的电荷;由于B横穿结介质;绝缘层以及两边超导体相邻的薄层内有A
存在,这使电子对穿过结时获得附加相因子

±
2
1
ldA
c
q
i
e
h
。令
i
i
ii
e
θ
ρ=Ψ,代入第一个方程,并乘以
(2,1=i )
1
1
θ
ρ
i
e
,得
121
1
1
1
22
1
ρρρ
θ
ρ
ρ
δ
h
qV
iei
t
i
t
i
+=
+
这里

+?=
2
112
ldA
c
q
h
θθδ。分开此方程的实部和虚部,得
=
=
δ
ρ
ρθ
δρρ
ρ
cos
2
sin2
1
21
21
1
h
qV
t
t
对第二个方程作类似计算,得
=
=
δ
ρ
ρθ
δρρ
ρ
cos
2
sin2
2
12
21
2
h
qV
t
t
假定结两边的超导体相同,于是
21
ρρ =,得
h
qV
=?

12
θθ
积分即得

=?
2
1012
Vdt
q
h
δθθ,将此代入δ表达式,
()

+

+=
2
1
2
10
1
ldA
c
Vdt
q
t
h
δδ
由于
tt?
21
,
ρρ
分别正比于从2贯穿到1和从1贯穿到2的隧道电流,将两个
t?
ρ
的方程相
59
减,即得流过结的总电流密度和位相差的关系
() () ( )
+==

2
1
000
sinsin cVdtldA
c
q
JtJtJ
h
δδ (8.32)
下面对此式做些讨论
第一,当V= A=0时,
00
sinδJJ =
说明此时结上有直流电流流过。这即是直流Josephson效应。
第二,当结上加了直流电压V时,结上流过交流电流,其频率为
h
qV
=ω。这就是交
流Josephson效应。当 V=1微伏时,ω=3039兆周,是微波频率范围,将伴有相同频率的电磁波从结上辐射。这是因为,Cooper对穿过绝缘层时,势能降低了9V,一对或多对Cooper
对所降低的能量将作为1个(或多个)光子的能量ωh(或n ωh)发射。通过测量直流电压和交流频率,可以得到非常精确的
h
e
数值。
第三,当结上只有磁场B时,流过结的电流密度为
)sin(
2
1
00

+= ldA
c
q
JJ
h
δ
而流过结的总电流是此式对(x-y)截面的积分。由于
x
eBB =,可取{ByA,0,0= },于是
BydzldA =?,流过结的总电流为
00
2/
2/
2
2
0
0
0
sin
2
sin]sin[ δδ
λ
λ
Φ
=+=
∫∫∫
+

c
q
cIBydz
c
q
dydxJI
a
a
d
d
l
hh
这里sinc(f)=sinf/f,并且在对子积分时已考虑了两块超导体中紧邻绝缘层的表面有磁通透入,
透入的深度大约为一个London穿透深度λ;Φ=(d+2λ)αB是结的有效截面积内的磁通。
这公式表明,总电流I对磁通Φ的振荡关系类似于光学中的单缝衍射花样,所以称为单结磁衍射现象。
ii,Josephson结的AB效应。
其实,上面单结磁衍射现象中已含有AB效应。这里再研究两个并联Josephson结中
间有磁旋通过的情况(如图)。
60
按前面AB效应的论述,环绕磁旋一周将出现一相因子
c
q
i
e
h
Φ
,这里q=-2e。于是流过每个结的电流为
)sin(),sin(
0201
c
q
JJ
c
q
JJ
hh
Φ
=
Φ
+= δδ
这里已假定两个结完全一样,从而两个单结的位相差δ相同。由此得到这两个并联结的总电流
)cos(sin2
021
c
e
JJJJ
h
Φ
=+= δ

sinδ代表单结衍射因子,)
c
e
h
Φ
cos(代表双结干涉因子,这已为实验所证实。值得指出,利用Josephson结的AB效应,可以制成各种高灵敏度的超导量子干涉器件(Superconducting
Quantum Interference Device,缩写为SQUID)。
61
第九章 定态微扰论
量子力学中,大部分定态问题都难以精确求解。因此发展了许多关于求解本征值、本征函数的近似方法。本章介绍其中的定态微扰论近似方法。方法的要旨是,从一般难以精确求解的哈密顿量H中,划分出其中数值较小(可用估值办法或视其中所含参数的数值)而又妨碍对H精确求解的部分H′,即有HHH ′+=
0
。划分出H′后剩下的
0
H应能精确求解。
然后,以
0
H的本征态和本征值为基础和出发点,以逐级近似的方法考虑H′的影响,给出H
的本征态和本征值的逐级近似解。方法区分为非简并态微扰论和简并态微扰论两部分。
§9.1 非简并态微扰论
1,基本方程组
假定H可以划分为两部分:
0
H和H′,
0
H为H的基本部分并且其定态问题可精确求解;而H′是妨碍H可精确求定解的部分。并且假H′比
0
H小(以两者的本征值作量级的比较,或对算符直接作数量级的估算后进行比较),以致可将H′看作是对
0
H的一种扰动。
于是有如下定态方程
( )
() () ()
=
=′+=
000
0
0
nnn
EH
EHHH
ψψ
ψψψ
(9.1)
这里上标“
()0
”表示未受
( )
{ }
0
n
E、
()

H
()
{(注意它
H′扰动是系统的物理量。按上面的假设,}
0
n
(下面简记它为
()
{ }
0
n)是已知的。将系统的ψ相对于未受扰动时的参照系}
0
n
应是完备的)展开,
( )
m
m
ψψ EH =中,得
( )
()
( ) ( )
m
E
0
m
0
两边乘以
()0
k,利用}

= mc
0
ψ
代入
∑∑
=′+
m
m
m
mEcmHc
0
()
{
0
m正交归一性质,得
()
( ),
0
′=

kEE
m
(9.2) cHc
mkmkk
( )L,2,1=?
62
其中,
() ()0
mH′
0
k=。当给定的k改变时,将得到一个线性联立方程组。这个方程组就是下面进行微扰近似计算的出发点。注意,在这个方程组中,未知数列是{}
m
c,未知
H
km

的本征值是E,并且至此尚未做任何近似。
下面进行微扰论逐级近似计算。为了下面逐级近似中便于区别各级近似的阶数,引入一个无量纲的常数λ,将HHH ′+=
0
改写为( ) HHH ′+= λλ
0
,在对λ
。于是,先把E、c按微扰级别(也即按参量
m
λ的幂次)展开:
的各级近似完成之后,令1=λ
( ) ( ) ( )
() () ()
+=
+++= L
210
210
mmmm
cccc
EEEE
(9.3)
其中,
()1
+L+
E和
()1
m
c为含有λ一次幂项,即一级小量,
( )2
E和
( )2
m
c具有二级小的量级(含有
2
λ
项)。它们分别表示H′对
()0
E和
( )0
m
c的一级和二级修正。
假定在未受H′微扰之前,
0
H处于定态系统
n→
( ) ( )
{ }
00
,nE
n
上,加上H′后,
()
nn
EE →
0

()
n
0
,下面用逐级近似法求微扰H′对
0
H的第n个本征值
( )0
E
()
n
和本征函数
2,
0
n的修正问题。此时
() ()00
n
EE =,
()
mnm
c δ=
0

一阶微扰论
此时,本征值和本征函数的计算只准确到含λ一次幂,即只令
( )
+
+
0
mn
n
E
δ
( )
()
=
=
1
1
mm
nn
cc
EE
(9.4)
于是
( ) ( )
( )
( )
()
+
0
将E,c展式代入前面基本方程并只保留到一阶,于是
() ()

∑∑
=
+==
m
m
m
mmn
m
m
mcn
mcmcn
01
010
δ
n m
() () ()
( )
( )
( )
( )
( ) ( )L,2,1,
11010
=+′=+?+

kcHcEEE
m
mmnkmknkknn
δδ
成为
63
(E
() ()
)
( )
( )
( )
kn
m
mnkmnknknkkn
HHEcE ′=′=++?

δδδ
1100
当nk =,得
() () () () ()

′=′=′ dHnHnH
nnnn
0000
ψψ=
rE
n
r
1
当nk ≠,得
()
() () 0
00
0
1
kn
kn
E
kH
EE
H
c


=
至此,仍有一个问题未解决。这就是
00
nk
E
k
=
k
( ) ( )
() ()
( )1
n
c尚未确定。它必须如此选取,使得一级近似下的扰动态
m
m
即应有nn =
( ) ( ) ( )

+= mcnn
000
在不含有一级小项的情形下得到归一化。
。于是最后得一级微扰论的公式
( )
2
1 λO+。根据这一点,显然必须令
()
0
1
=
n
c
( )
′+≈ HEE
0
()
() ()
+

k
n
E
H
n
0
'0 ()


k
kn
nnnn
k
E
n
0
0
(9.5)
这里求和号上的撇记号表示求和指标nk ≠。公式表明,在一级微扰论近似下,能量的修正为H′在未受扰动态
()0
n中的平均值,而扰动后的态中别的态
( )
( )k
0
也将混入,
混入的几率幅正比于
nk ≠
H′在
()0
k中的平均值、反比于
( ) ( )
( )
00
kn
EE?值。能量和态矢的公式都只准确到含λ的一阶项。
由n展式可知,
() ( )
作为对n k
0
一级修正的
0
态前的系数的数值应当远小于1。于是可得上述微扰论适用的条件
( ) ( )00
nkkn
EEH?<<′ (9.6)
就是说,微扰算符在混入态(
()
0
k)和被扰动态(
0
)之间的矩阵元必须远小于这两个(未受扰动)态之间的能级间距。
如果未受扰动系统还包括了连续谱,则严格说,
n
( )
0
H n的表达式应修改为
64
()
() ()
()
() ()
()



+

+= ν
ν
EE
H
k
EE
H
nn
nkn 0
00
0
00
'0
ν
ν
d
n
k
kn
(9.7)
这里,ν为一组物理量的数值集合,对于微扰是针对分立谱中的一个态
()
它们用来区分连续谱中的各个态连续变化。
以及大多数H均有
( ) ( )00
EE <
0
n
0 ν
(对所有
n
()0
n
E?
0
n
EH <<′
νν
,而且更是由于积分号下被积函数在ν值全部积分范围内无奇点,积分可略去。
()
只对那些在分立(对另一些自由度的)也即只在
n
E
n来计算物理量
ν值)的情况,鉴于
()
0
n态附近杂有连续态,
( )0
进入了连续谱或带状谱区域内时,才需要考虑这一积分修正项。
还有一个常用的公式,是用一级微扰态?的矩阵元
() () () () () ()
()
() ()
() ()
()()
∑∑
≠≠
+

+?≈
mk
m
nk
nk
kn
kmnk
nm
EEE
H
0
'
00
'0
∑∑

+?
+=?
k
km
k
k
k
k
E
H
kcmkcnmn
0
00
01'001'0
(9.8)
此公式准确到一级微扰近似。
3,二阶微扰论
通常,波函数只作到一阶微扰近似,但有时能量的一阶修正为零或很小(如禁戒等情况)
如上面那样;
,这时能量要作到二阶微扰近似。为此,将
( ) ( ) ( )210
(9.)
代入基本方程组,即,
() ()
() () ()
++=
++=
210
mmmm
cccc
EEEE
( )
() () ()
( )
( ) ( ) ( )
c
1
cE
2
c
0 ( )
( )
( )()()
( )

+′=+++++?
m
mmkmknkknkkkkn
cHcEcccEE
001121000
由于此方程两边的一阶量已相等,取二阶量等式
() ()
( )
( ) ( ) ( ) ( ) (
++?
knknkkn
cEcEcE
0211200 ) ( )

′=
mkm
cHE
1
令nk =,得
()
m
( ) ( ) ( ) ( )

′=+ cHcEcE
10211
() ()
() ()
m
mnmnnnn
两边消去
() ()11
cE项并将
()
()nmc ≠
1
表达式代入,得
nn m

′ HH
2
() ()
∑∑∑
≠≠≠
=
′=′=
nm
mn
mn
nm
mn
mn
nm
nm
mnmn
EEEE
HcHE
00
'
00
'1'2
65
于是二阶微扰近似下的能量表达式为
() () () ()


+′+=++=
nm
nnnnnn
E
HEEEE
'0210
() ()

mn
mn
n
E
H
E
00
2
(9.10)
这里注意,第一,上述近似若成立,要求
( ) ( )00
EEH?<<′
nmmn
(9.1)
n附近有别的能级存在(简并或近简并情况),这里的公式将不成立或不够精确。态能级的二阶修正永远是负的,这一点和系统(
0
H)及扰动(
因此,若在所考虑的能级第二,基H′)
无关。
4,计算举例
i,Lamb
由谐振子计算知道,也存在着零点振动(表现为
移动的量子力学估算
谐振子即便处在自己的基态上,x?虽然为零,但? 0
2
>x,对应的零点振动能为ωh
2
1
)。而电磁场可分解为一系列频率的谐振子集合,于是,即便在真空中,其实电磁场正处于它的基态上,存内),就是说,真空中的电磁场的平均场强虽然为零,但场强的均方差(现在即为均方值)
并不为零,这就是著名的电磁场的“真空涨落”。这个处处存在时时存在的涨落着的量子化电磁场和电子相互作用,导致电子位置坐标的均方涨落。电子位置的看来好象不存在电磁场,
在着这个谐振子集合的零点振动(在氢原子中的库仑场看成是经典的外场,不在此考虑之
(处于库仑外场中的)
这种“弥散”,意味着施加到电子上的外场库仑势( )rV有一个“弥散”,
() () ( )δ
r
r
+= rV。对()rU作Taylor展开,对涨落平均并取到一阶近似,得
rr
→ rUrV
()rU
r
() () HrVrV +=?+≈
0
6
δ
()r
r
H′
22
1 r
r
r
这里微扰VH
r
22
6
1
=′ δ。注意,这种“弥散”是围绕平均值( )rV
r
的涨落,一阶导数项的平均为零。
2
δ
r
是电子位置涨落的均方值。于是对原子中任一态矢nl,这个涨落造成的能移为
()nlrVnlE
nl
=?
2
6
1
δ
r
(12)
由于()
9.
()()rZe
r
Ze
rV
r
πδ4
2
2
=

=?,于是对s2态和p2态,分别有?
66
()
=?
2 p
E ()
=
=
00
3
2
0
3
2
2
2
22
2
2
22
2
p
ss
e
Z
e
Z
E
ψδ
π
ψδ
π
r
r
p2 → Lamb移动。现在的任务是如何计算电子位置的均方差值
二者之差即为s2这条谱线的著名的
2
δ
r
。限于篇幅,这里只给出这个修正的上、下限。显然可以认为
2
δ
r
数值在如下范围之内,
2
22
<<
e
δλ
r
9.13)
这里,
Z
a
B
(
2
mc
e
2
e
=λ为电子的康普顿波长,
2
2
me
h
a
B
=为波尔半径。注意到
()
2
3?
2
1
4
1
0?
=
Z
a
B
s
π
ψ
2
,于是可得
2
2
mc
4
12
Z
<
2
mc
6
α
4
12
Z
α?< E
Lamb
初步计算给出
a
2
22
2
ln
1
ln
e
ZcmZ
λ
απααπ
δ
=

r
2
h 1?22α?
代入得出
Z 1
25
4

≈?
α
α
π Z
mcE
Lamb
ln
6
(9.14)
ii,
原为各向同性的介质,
电介质的极化率
在外电场(设沿x方向)作用下出现极化现象。现在利用微扰论研究这一现象。
无外电场时,x、y、z三个方向离子的谐振子振动相同,加上电场后,x方向为
qExxm
dx
d
m
H?+?=
22
2
22
2
1
2
ω
h
(9.15)
a
J.D,比约肯,S.D,德雷尔,相对论量子力学,p.63,科学出版社,1984。
67

22
2
22
0
2
1
2
xm
dx
d
m
H ω+?=
h
,qExH?=′。于是
() ()
() ()
()
()
=
+=
===
Lh
h
,2,1,0,
2
1
,
0
2
1
00
22
nnE
m
xHeNxnx
n
n
x
nn
ω
ω
ααψ
α
往算H′对第n个能级的影响,注意
()

()
()
()
()
( )
000
111
1
+++?= nnnnnx
这可用x
()
作用后,向坐标表象投影,并利用厄米多项式的递推关系式得知。于是
()
0
00
=nxn。因此需作第二阶的微扰论计算,
() ()
() ()
()
2
22
22
22
1
2
1
2
22
2
'
22
00
2
'20
22
1
2
1
22
1
2
1
2
1
2
1
ω
ω
ααω
ω
χχ
ω
ω
χ
ω
ω
ω
m
Eq
n
nnEq
n
mnmn
Eq
n
mn
Eq
n
EE
H
nEEE
nm
mn
nm
mn
m
mn
m
mn
mn
nnn
+=
+
+
+=
+
+
+=
+
+=

+
+=+=
+=?=


h
h
h
h
h
h
h
h
()
() ()
()
()
() ()
()
()
() ()
()
()
()
()
()
()
()
{}
000
0
0
1
0
,10
0
1
0
,10
0
00
'0
111
2
11
++
+=
+

+?

+=

+=
+
+

nnnn
qE
n
n
EE
H
n
EE
H
n
m
EE
H
nn
nn
nn
nn
nn
m
mn
mn
ωαh
于是,在二阶微扰近似下,每个能级均下降了一个相同的数值并与电场强度
2
E成正比,而每个态又都混入了它的上下紧邻的两个态。混入的几率幅与电场强度E成正比。再往求极化率
E
D
,D为诱导的电偶极矩。为此先计算
68
()
()
() ()
()
() ()
() ()
()
()()
22
2
0
00
'
00
0
'
00
0
'0
1
2
1
2
2
1
ωωα
χ
χχ
χ
χχ
m
qE
nnqE
EE
qE
EE
H
EE
H
nn
k
kn
kn
k
kn
knnk
k
kn
knnk
nn
mn
=+=
=

+

+=
=

∑∑
h
于是在n态中诱导电偶极矩
n
D等于
2
2
2
2
2
,
2
ωω
χ
m
q
E
D
m
Eq
nn
==2qD
n
=
Waals)力
(9.16)
iii,
两个中性的原子(或分子),当它们之间距离>>
范德瓦尔斯(Van der
R它们本身波包尺度时,它们之间表现出一种长程的相互吸引力,并与
6
R成反比,这就是Van der Waals力。下面以两个氢原子为例用二阶微扰论求解。
首先,利用奥本海默近似 —— 在讨论
电子运动时,原子核的动能忽略不计。
()
+=′
+=
′+=
1
2
2
2
12
22
2
2
1
2
21
2
0
0
2
ba
r
e
r
e
r
e
R
e
H
r
e
r
e
m
H
HHH
h
这就是说,在两个基本上是相互独立的氢原子a和b(各自带有一个电子1和2)的基础上,
将它们之间的相互作用考虑作为微扰。这里( )
1212
rrRr
rr
r
r
+=,
22
rRr
a
r
r
r
+=,
11
rRr
b
r
r
r
+?=。
由于
21
rrR、>>,于是
=
R
R
e
R
r
r
()()
+=
+?+=
++=
1
2
1
2
1
2
2
22
2
2
2
1212
2
12
2
2
2
rRrRr
rrRRr
rrrrRRr
b
a
r
r
r
r
rrrr
r
69
()() ()[ ]
()
()
+?

+?


+





2
1
22
2
1111
1
2
2
22
2
2211
2
2
2
12
2
2
121211
12
2
3
2
1
2
3
2
1
2
3
2
1
re
RR
r
R
re
Rr
re
RR
r
R
re
Rr
R
rre
R
rr
R
rre
Rr
R
R
b
R
R
a
RR
rr
rr
rr
rr
rrrrrrrr
H′可表示为
()
()[] () ()()
()?
R
r
r
r
2
3
12
2
3e
R
rre
rr
rr
e
R
r
()()
()([]
+
1212
3
3
12
3
1
DeDeDD
R
re
RR
R
r
rr
rr
由于两个氢原子之间距离已大于它们各自波包的尺寸,
)
+=
=
++++
=′
4
4
42
1
3
2
3
2
1
2
2
2
3
2
3
2
2
2
2
12
3
2
3
2
12
2
1
1
2
3
22
3
22
3
2
R
O
R
O
ROre
R
e
R
re
re
R
e
R
re
rre
R
e
R
rre
H
RRR
rrrrrrr
rr
两氢原子波包之间已不交叠,可
略去两原子的交换对称性。于是此双原子体系的基态的零级波函数为
( )
( ) ( )
21001
rr
10021
0
,rr
rr
ψ 9.17) ψψ = (
由于
100
ψ是关于()
21
rr
rr
的偶函数,而H′对( )
21
rr
rr
是奇宇称,故
( ) ()
0
00
=′ψψ H,即一阶
微扰无贡献。二阶微扰修正为
) (
rr
()
() ()


=
k
k
EE
kH
E
00
0
'2
0
2
18) (9.
由此可知以下两点,
第一,由于
3
1
R
H ∝′,R为常数,因此
()
6
2
1
R
E ∝。
,第二,由于
() ()0
0
0
EE
k
>
总之,可得两个相距较远氢原子相互作用附加能为
()
0
2
<E。
()
6
2
R
A
E
=
§9.2 简并态微扰论
1,简并态微扰论要旨
在被扰动态有简并或近似简并的情况下,上述微扰论不适用。因为这时在简并能级(或近简并能级)之间,下面不等式不成立
70
( ) ( )00
mmn
EH <<′ (9.19)
现在,
0
H的第下面构造关于这种存在简并情整个办法的核心部分是在
n
f个简并态子空间中,写出微扰项
n
E?
n
f重简并,假定被扰动的态为n个能级,它有况的一阶微扰论。H′的个本征值即为一阶近似
nn
ff ×
将它对角化并找出它的
n
f个本征值及本征矢量。
n
下的能量修正,而这
n
f
2,简并态微扰论

0
H的第n个能级的简并量子数为维厄米矩阵,这f
个本征矢量将构成零阶近似态矢。
( )
n
f,,2,1 L=νν,于是扰动后的态矢展开为
( )
( ) ()
∑ m
mc
0


+=
νν
ν
ν
nm
n
ncn
0
(9.20)
这里右边第一个求和是在简并态子空间内进行,而第二个求和是对此子空间之外所有态进行。基本方程组成为(也区分为简并子空间和其之外)
( )
() ( )′?
∑∑

μ
νμ
νν
μμ
nmcEE
cHc
nk
mn
mmnnn
,,
0
,
0
i,
≠nm
ννμ
cH
nnn,
f
n
,2,1 L
EE
n
()
() ()
≠?
=′+=
′+′=
∑∑ νν
νν
kcHcH
kmknnm
,,
,m
下面由这个基本方程组出发,作近似计算。
态矢零阶近似,能量修正一阶近似
( ) ( )
=
+
,
1
n
c
E
()
=
=
0
n
c
E
n
c
E
0
0
mnn μμ
这时基本方程组和态矢分别为
()
()
()
n
f
01
()
()
()
=

=
n
f
n
nnn
nc
1
00
1
,
ν
ν
ν
=?′

=
n
cEH 0
ν
νμννμ
δ
这里关于未知能移
()1
E
维简并子空间中的本征方程:本征值为本征矢量为L,2
和未知矢量的方程是
n
f个齐次联立代数方程组,实质是微扰H′在这个
n
f
( )
( ){ }
n
fc,1,
0

ν
。上面
( )1
E,
本征方程若有非零解
()
{ }
0
ν
c,系数行列式必须为零,由此即求得本征值
()1
E的方程,
71
( )
0det
1
=?′
μνμν
δEH
这个方程也常称之为关于简并能级一阶修正量的久期方程。由于矩阵元
n
f个本征值
μν
H′全体组成了
n
f
维的厄米矩阵,它的
( )
( )
n
fE L,2,1
1

ν
均是实数,若它们彼此均不相等,说明在H′扰动下n
n
f个本征值能级
n
f重简并完全被解除,否则只是部分地被解除。
按通常代数方法,对应于
( )1
E,可得
n
f个正交归一的
n
f维矢量
()
{ }
0
,μν
c。于是,简并态最低阶微扰论(能量一阶、态矢零阶)的结果写出来便是
( ) ( )
=
=

=
n
ncn
1
00
,
ν
μν
μ
νμ
(9.21)
ii,态矢一阶近似,能量修正二阶近似
有可能发生这样的情况,由于某种禁戒原因,这个第n能级各简并态()
()
n
E
,
+
n
f
nn
EE
1
,
0
μ
()
()
()
0
μn之间的
H′矩阵元全都非常小(甚至是零),这时就需要作进一步近似,计入此空间之外的矩阵元
νnm
H
,
′。
()这时将上面基本方程组的第二个方程做如下近似,以求出一阶小量νnmc
m

()
=?

()
()
()

′+′=?
∑∑
ν
νν
k
mknnmmm
cHcHcE
0
,
0
ν
νnmmn
kn
cHcEE
E
0
于是基本方程组成为
()
()
() ()
′=?
′+=

νnm,


∑∑
ν
νν
μννμ
nnmmmn
m
mnnn
cHcEE
cHcc
,
00
,
0
将第二个方程中
m
c解出,代入第一个方程,得
ν
μ nm
H
,m
nn
EE
() ()
()
()
∑∑
′′
+′
ν
νμ
νμν
δ
00
,,
,
m
mn
nmmn
nnn
EE
HH
Hc =?
μν
0
0
nn
EE
而态矢这时成为,
72
()
()
()
∑∑
∑ n
c
ν
ν
+=
m
n
mc
E
nn
00
ν
ν
ν
计算举例

mn
nm
E
H
00

(9.22)
3,
在z
2
121,211,210,200 9.23)
扰动为cos >==′ eeEreEz
(
H
lm
ml
eEr
22
ψθψ
′′
由于
cos
H′量子数,因此不为零的矩阵元只有两个于是微扰
(
它的四个本征值
()
()
考虑氢原子2=n能级的斯塔克效应,假定电场E方向。电场扰动前,氢原子在能级上有四重简并,相应的态nlm为 =n
()0θ。这时在这个子空间里矩阵元为
是奇宇称,加之H′不改变m
cos

4
r
==
=
=
=
=′
∫∫
∫∫

∞?
为波尔半径
2
0
2
0
2
0
4
4
2
2
2
3
2
2
3
210200200,210
sin2
cossin
2
1
sin
62
1
cos
4
3
cos
2
1
2
1
4
1
cos
me
a
ddr
a
r
ddre
a
r
r
a
eE
ddre
a
r
a
re
a
r
a
eE
reEH
a
r
a
r
a
r
a
r
h
π
π
θθθ
θθθ
θθθ
π
θ
π
ψθψ
e
2
0
4
,3
16
8
aeE
a
eE
=′
210,200
H
H′在此子空间中化为如下44×厄米矩阵
0030 aeE
9.24)
()1
0000
=′
0000
0003aeE
H
E分别为0,0,3,3aeE aeE?,而四个本征矢量分别为
73
1
1
1
1
1
1
0
0
0
0
(9.25)
将它们写出来即为
0
0
2
,
0
0
2
,
1
0
,
0
1
()()210
2
1
,210200
2
1
,121,211?+
这样,在电场作用下,氢原子2=n能级的四重简并被部分地解除,分裂出三个能级
200
aeE3aeE3+EE,
22
(9.2
( ) ( ) ( )
EE,
0
2
00
2
= 6)
这表明,p态氢原子就像一个具有3的电偶极矩,在外场中有三中取向:垂直于外电场的两个态、分别平行和反平行于外电场的两个态。
ae
74
第十章 势散射理论
§10.1 一般描述
1,散射(碰撞)实验的意义及分类
散射(碰撞)实验是指具有一定动量的入射粒子束流,射向处于气、液、固体形态的靶粒子上,和靶粒子相互作用(电-弱作用或强作用)之后,原先的入射粒子(及靶粒子)或新生出的粒子由相互作用区域(统称为散射点)散射飞出。实验测量的对象,除入射粒子的流强和能量之外,主要是出射粒子的种类、能量、角分布(微分截面)、极化状态、角关联等等。散射(碰撞)实验中,入射粒子束流可以被近似看为该粒子的单色平面波,(不一定和入射粒子同类的)
出射粒子束流可以被近似看为该粒子的(渐近自由的)出射球面波,而发生在局域空间中的入射粒子-靶粒子相互作用导致了在这两类粒子的两种自由状态之间的跃迁。这些种种类型的跃迁可以在假定相互作用下由一般散射理论计算。从而理论计算结果可以直接经受实验的检验,这使得散射(碰撞)实验在微观粒子相互作用以及它们内部结构的研究中处于特殊的地位,它们是原子物理、核物理的重要研究手段,是粒子物理的几乎唯一的研究手段。
散射(碰撞)过程可以区分为以下三大种类,
弹性散射过程 AB AB+ → +
非弹性散射过程 AB A B+→ +
碰撞反应过程 AB CD+ → +
这里,弹性散射过程中,不仅不发生粒子种类上的改变,而且不产生弹性散射机械能和粒子内能之间的转化,因此弹性散射中机械能守恒;在非弹性散射中存在散射机械能与粒子内能之间的转化,这将导致,比如,靶粒子的激发或退激发;碰撞反应中A、之间有粒子交换,或是有新粒子产生,以致出射时为不同于入射的新的粒子
B
C和。有时也把除了弹性散射以外的全部散射(碰撞)过程统称之为非弹性散射过程。至于散射(碰撞)中的相互作用也可以将之区分为两大类:可以用一个局域的空间函数——势函数描述的情况,这时的散射称为势散射;不可以(或不完全可以)用一个局域的空间函数来描述的相互作用,它们属于更一般的形式散射理论(未二次量子化的
D
S矩阵理论)以及量子场散射理论(二次量子化的S矩阵理论)处理的范围。本章只研究弹性势散射过程,但其中一些概念对非弹性势散射(仍至碰撞反应过程)也适用。
2,基本描述方法 —— 微分散射截面
设入射粒子束的流密度为,其量纲为(),在散射区域经受和靶粒子的相互作用之后,朝
j
0
厘米秒
2?1
(,)θ?方向散射出去,设j(,)θ? =单位时间中沿(,)θ?方向单位立体角内散射出去的粒子数目,其量纲为,于是,定义朝(,秒
1
)θ?方向散射的微分散射截面为 dd)?σθ? σθ?(,) (,≡
σθ?
θ?
(,)
(,)
d
jd
j
=
0
(10.1)
75
这里σθ的量纲为。如果入射粒子束用平面波描述(如象下面所做的那样),

(,) ()厘米
2
e
ikz
j V
0
=
k
=
h
μ
,这里的量纲不正确是由于入射波波函数(它原应是全空间积分归一的)量纲不正确,但只要在计算时也用这同样的入射波波函数,作为比值的σθ的量纲就将是正确的。
j
0
j(,)θ(,)
总散射截面为
σσ
σ
σθ? θ?
ππ π π
== = =
∫∫ ∫ ∫
d
d
d
dd
j
jd
44 4
0
4
1
(,) (,)? (10.2)
由此可以看出,等于一个靶粒子和每秒每平方厘米一个粒子入射情况下的散射粒子份额。 σ
3,入射波、散射波和散射振幅
假定已对散射这类两体问题取了质心系,并且已经分离掉了质心的平动。于是,这里所研究的势散射总都是入射粒子以折合质量μ在静止势场Vr()
v
中的散射,这里的
v
r为靶粒子到入射(散射)粒子的矢径。
通常,入射粒子束流不可能绝对地单色,入射粒子波函数应当以某种形式的波包来描述,
但这种描述不但本身难以统一和确切化(对不同的入射束流产生装置,非单色情况会不同),
而且会给散射的理论处理带来复杂性、不确定性。因此,下面总是将入射波理想化地看作为平面波,并设它沿z轴入射:。进一步仔细分析表明,只要入射束流足够好地单色(即,
动量波函数足够好地集中于其平均值附近),这里的平面波近似不会带来影响(即,散射结果与的具体形状无关)
e
ikz
ψ()
v
p
)ψ(
v
p
a

远离散射中心的散射粒子,其状态—散射波将是一个渐近形式为f
e
r
ikr
(,)θ?的出射球面波,这里θ为出射粒子相对于入射粒子的偏转角又称为散射角,为探测点到散射中心的距离,
r
k为散射波的波数,由于是弹性散射,k也就是入射波的波数。其中,描述这个球面波不同方向的各向异性性质,常称之为散射振幅。
f (,θ?)
于是,散射问题可表述为:求解散射势Vr()
v
下的定态方程的如此正能解,
这个正能解当
dingeroSchr &&
r →∞时具有下面的渐近形式
ψθ? θ?(,,) (,)ref
e
r
r ikz
ikr
→∞
→+
注意当θ时,此渐近形式中两项之间不存在干涉。这是因为它们的交叉项正比于
,由于
≠ 0
os)θ
e
ikr(c1?
kr足够大,因此当θ≠ 0
0
时此因子快速振荡,但探测器总会有一个小的张角
,所以只要探测器不放置于θ附近,此项因子在?θ =?θ内将由于快速振荡而被抹去。也就是说,(通常均放置在r →∞处的)探测器只要不位于θ= 0附近,是检测不到入射波(以及它与出射波的干涉)的。
e
ikz
现在来计算上面散射波波函数的流密度。为此将流密度表达式写入球坐标,并代入散射
a
J.R,Taylor,Scattering Theory:The Quantum Theory on Non-relativistic Collisions,John Wiley & Sons,Inc.,
1972。
76
波波函数f
e
r
ikr
(,)θ?,简单计算可得
v
v v v
jjejej
r
r散
(,) (,) (,) (,)
() () ( )
θ? θ? θ? θ?
θ
θ
=++e
=++
v h vv
e
k
f
r
eO
r
eO
r
r
μ
θ?
θ?
(,)
() (
2
23
11
)
3
考虑到由球面波流密度矢量求散射粒子数jd(,)θ时,应将上面流密度矢量乘以球面元
dS r d=
2
,于是可知当r →∞时可得散射球面波的流密度矢量为
v
h v
j
k
f
r
e
r
=
μ
θ?(,)
2
2
于是在(,)θ?方向d?立体角元内散射流为
v v
h
jdS
k
fd?=
μ
θ?(,)
2
注意这时入射流密度j
k
0
=
h
μ
,从而微分截面为
σθ?
θ?
θ?(,)
(,)
(,)d
jd
j
jdS
j
fd?
==
=
00
2
v v
就是说
σθ? θ?(,) (,)= f
2
(10.3)
于是,一旦求出对应入射粒子正能量值的散射球面波渐近表达式,其中方向函数的模平方即为所求的微分截面。下面紧接的两节将用不同的方法去求这个正能定态解的渐近表达式,也就是其中的出射球面波渐近表达式,从而求得
(,)θ? f
f (,)θ?表达式。
§10.2 分波法 —— 分波与相移
1,分波法的基本公式
当势场为中心场Vr时,由于、守恒,散射过程可以得到如下一种简单直观的理论描述。
Vr() ()
v
= L
2
L
z
这时散射将是绕z轴旋转对称的,就是说散射分布与?角无关。于是散射问题这时归结为:在渐近条件
ψθ() ()
v
ref
e
r
r ikz
ikr
→∞
→+
之下,求解正能量定态方程 dingeroSchr &&
77
+=
h
2
2
2
2
2
2
1
m
r
r
r
r
L
r
rVrr Er[ ( ) ] (,) () (,) (,)
ψθ ψθ ψθ (10.4)
为解决这样一个问题,可令
ψθ θ(,) (cos ) ()raPR
ll kl
l
=
=


0
r (10.5)
能够事先假定ψ θ(,)r
l
具有这种展开的理由是,这时守恒,l是好量子数,从而如果将入射平面波分解为不同分波的叠加,则每个的分波将会各自独立地散射,可以分开处理;另外,整个问题已与
L
2
l
角无关,并且守恒,也是不变的量子数,可径直取其为零,于是球谐函数
L
z
m
Y
lm
(,)θ?化简为Legendre多项式P
l
(cos )θ。将ψ θ(,r )展式代入定态方程,不同的分波彼此完全分离,得
dingeroSchr &&
l
112
0
2
22
22
r
d
dr
r
dR
dr
k
ll
r
Vr R
kl
kl
()[
()
()]+?
+
=
μ
h
这里k
E
2
2
2
=
μ
h
。作变换χ
kl kl
rrRr() ()=,得
d
dr
k
ll
r
Vr
kl
kl
2
2
2
22
12
0
χ μ
χ+?
+
[
()
()]
h
= (10.6)
下面研究这个方程的渐近行为。当r → ∞ (确切说应为kr → ∞ )时,上面χ
kl
趋向下面方程
dy
dx
ky
2
2
2
0+=
而这一方程的解为yx c kx() sin( )=+α,这里和c α为待定常数。于是可得χ
kl
r()的渐近表达式
χ
π
δ
kl
kr
l
rckr
l
() sin( )
→∞
→ +
2
这里,为了和下面平面波展式相对应,已从α中分离出?

2
,剩下仍称待定常数。由此可得
δ
l
Rr
r
kr
l
kl
kr
l
() sin( )
→∞
→ +
2
2
π
δ
= +
+? +
1
1
12
ir
ie e e
li l ikr ikr
ll
() ( ) )
()δδ
因此
ψθ θ
δ
δ
(,)
()
(( ) ) (cos )
()
ra
ie
ir
ee P
kr
l
li
likrikr
l
l
l
l
→∞
+? +
=


+∑ 1
12
0
=
+ +
+?
=

=

∑∑a
ie
ir
eeP a
ie
ir
eeP
l
li
likrikr
l
l
l
li
iikr
l
l
l l
l
()
[( ) ] (cos )
()
()(co
δδ
δ
θθ11
1
0
2
0
s)
将这里的第一项和下面关于平面波的展开式相比较
78
ee lijkrP
ikz ikr l
ll
l
==+
=


cos
()()(cos
θ
θ21
0
)
kr l
l
l
li
kr
kr
l
P
→∞
=

→ +?∑()sin( )(cos21
1
2
0
π
θ)
=
+
+
+?
=


()
(( ) ) (cos )
21
2
1
1
0
l
ikr
eeP
likrikr
l
l
θ
比较可得a
l
k
ie
l
li
l
=
+21
2
δ
,代入ψ θ(,)r表达式,
ψθ θ(,) ( ) (cos )[( ) ]r
kir
lP e Se
kr
l
likr
l
ikr
l
→∞ +?
=

→ +? +∑
1
2
21 1
1
0
=+
+
=

∑e
e
r
l
ki
SP
ikz
ikr
ll
l
21
2
1
0
()(cosθ)
这里。最后得到散射振幅的分波法表达式 Se
l
i
l
=
2 δ
f
ki
lSP
ll
l
() ( )( ) (cos)θ=+?
=


1
2
21 1
0
θ (10.7)
这个公式表明,中心场散射振幅的计算可归结为各分波在散射中相移的计算。如果某一值的分波相移为零或
l
nπ,则该分波的散射振幅为零,表明该分波不为势所散射。 l
由的模平方可得微分截面f ()θ σ θ?(,),再进一步对()θ?积分可得总截面,
σθ π θθ
δπ
t
i
l
l
fd
k
le P d
l
== +?



=

( ) ( )( ) (cos ) sin
2
2
2
0
2
0
2
1
4
21 1?
由于Legendre多项式有如下正交归一积分关系 P
l
(cos )θ
PP d
l
ll
(cos ) (cos )sinθθθ
π
′′∫
=
+
0
2
21
l
δ
于是得到
σ
π
δ
t
l
l
k
l=+
=


4
21
2
2
0
()sin (10.8)
因此在中心场的分波法中,计算微分截面和总截面的问题都归结为计算各分波的相移δ。
中的每一项代表该分波的分波截面。
l
l
σ
t
l
2,分波法的一些讨论
i,实际计算中对求和不可能也不必要一直计算到无穷大。这里按物理分析给出的一个估算。一般地说,l值越大,该分波对应的角动量就越大,离心倾向也越大,
从而对散射中心的距离b越大,受中心力场的影响就愈小,从而
l
l
max
δ
l
也越小。
79
当增加到对应的δ时,就 l
l
≈ 0
不必要再考虑这个分波(以及值 l
更大的分波)了。这个可如下 l
max
估算。由于b< a~,故得mvb mva<~,
而可代以角动量,可 mvb l
max
h mva
代以,于是得 hka
lk
max
~< a
这说明,入射粒子能量越大,波长越
短,力程越长,则需要考虑的分波数
就越多。反之,对低能粒子入射到
短程势的情况,所要考虑的分波数就很少。事实上,当ka <1时,只需要考虑l的= 0 s分波,
这时由于,于是在质心系中散射角分布是各向同性的。 P
0
1(cos )θ =
ii,相移δ
l
是有Vr时实际径向波函数的渐近形式和无Vr径向函数渐近形式之间的位相差。如上所说,当
() Rr
kl
() ()
jkr
l
() δ
l
= 0或nπ时,该分波不发生散射,仿佛完全从势场中透过。这组相移
{ }δ
l
完全确定了散射。根据实验测得的σθ曲线,用最小二乘法拟合可以定得一组参数
()
δ
l
。这组δ
l
是研究入射粒子与靶粒子之间相互作用的重要资料;根据所得的这些δ,可以进一步近似复原散射势的形状。为复原Vr,原则上只需要知道一个相移(比如
l
()
s波相移δ
0
(k))δ
l
k()的函数形状就可以了,如果还有分立的(负)能级的话,还需要知道分立态波函数渐近式中的a (这里E
n
Rae
n

α
nl
r
n
n
α
n
n
mE
=
2
2
h
.
)
a

中心势Vr正负号与正负号的关系,() δ
l
吸引势负出射波函数被拉向后向散射中心于是在空间上便落后于自由运动排斥势正出射波函数被推离散射中心在空间运动上超前于自由运动
,,,( ),
,,,,.
V
V
l
l
δ
δ
>
<
0
0
这是因为,位相为ckr
l
l
=?+
π
δ
2
,若δ
l
> 0,达到位相所要的较小;而δ,对应同一个所要的r较大。注意,散射截面只依赖于
c r
l
< 0
c δ
l
的数值,并不依赖于δ
l
的符号。
3,光学原理
利用这里的分波法基本公式,可以证明散射理论中的一个普遍规律,即,总截面(包括非弹性散射和吸收截面在内)和弹性散射的朝前散射振幅虚部成正比,为
σ
t
Im ( )f

0
σ
π
t
k
f=
4
0Im ( )

(10.9)
a
Ланg ау,Ⅱ.c m p,252。
80
证明:由上面公式得 f ()θ
f
ki
le P
ki
le
i
l
l
i
l
ll

() ()( )() ()(θ
δδ
== +? = +?
=

=

∑∑0
1
2
21 1 1
1
2
21 1
2
0
2
0
)
于是
Im ( ) ( ( ) ( )) ( )( )f
i
ff
k
le e
ii
l
ll
弹弹弹
0
1
2
00
1
4
21 2
22
0
=?=
++?

=


δδ
=+ = +
=

=

∑∑
1
21
4
4
21
2
0
2
2
0
k
l
k
k
l
l
l
l
l
()sin ()sinδ
π
π
δ
=
k
t

σ
其实,这一关系式相当普适,是个普遍的规律。就是说,不论导致散射的相互作用是否能够用势函数描述,不论入射粒子静质量是否为零,也不论入射粒子能量高低,这一关系式都成立。它的物理解释为,总截面是入射波减弱的一种度量(σ
t
越大,入射波的减弱越大),而这种减弱是由于入射波和(同方向的)朝前散射波相消干涉的结果。于是,朝前散射波的波幅越大,这种相消干涉也越大(它从入射波中移去足够的入射流,以说明吸收反应、非弹性散射以及其余的弹性散射),减弱也越多,总截面也就越大。
4,低能散射的若干一般特征
由于入射粒子能量很低,计算可以只对s分波( l = 0 )进行。这时有方程(带中心边条件
) χ
0
00()=
′′ +? ==χ
μ
χ
μ
0
2
2
0
2
0
2
() ( ()) (),rk Vr r k
E
h
h
(10.10)
我们可以用一般的方式来研究这时情况,而不去仔细地区分Vr的形状。 ()
假设Vr的作用半径(势的有效范围)为r,至少就量级而言,也是可以如此假定 ()
0
的。于是在势场之外方程成为
χχ
0
0 2
0
0
0
() ()

+=k,()rr>
0
其解为
χδ
0
0
0
()
() sin( )rkr=+,()rr>
0
为了描述的方便,按下面的定义引入散射长度,a
0
ar
r
rr
k00
0
0
0
0 0
0
0
0
1=?


(
()
()
)|
()
()
χ
χ
将表达式代入,由于粒子能量低,χ
0
0()
()r kr
0
1<<并且δ
0
1<<,于是,

tan( )kr kr
00 0
+≈+δδ
0
a
k
0
0
=?
δ
,或δ
00
=?ka
这时总截面为( ) rk
0
0→
81
σ
π
δ
π
δπ
t
kk
a===
44
4
2
2
0
2
0
2
0
2
sin
就是说,总截面为等效散射球(半径为散射长度)截面积的4倍,等于散射球表面的面积。 a
0
作一些进一步的讨论,
i,散射长度几何意义
这里以Vr为球方势垒为例说明a的几何含义。 ()
0
这时
Vr V r a
Vr r a
(),
(),
=<
=>
0
0
于是,r
0
= a
aa
a
aa
k0
0
0
0
0
0
1=?


(
()
()
)|
()
()
χ
χ
图示说明了其实即为通常的“有效半径”,而a
0
χ
χ
()
()
)
r
rr
0
00

即导致有效边界条件。对于刚球情况,V,,这时
0
=+∞ χ
0
0
0
()
()a = a
0
a=。
ii,上面求δ和的关系的推导中,利用了
0
a
0
δ
0
1<<,若进一步,设δ不很小,可改进计算如下。这时,由于
0
kr
0
1<<,tan( )δ
00
+ kr可在tanδ
0
处展开,取一阶近似
tan( ) tan
cos sin
cos
tan
cos
δδ
δδ
δ
δ
δ
00 0
2
0
2
0
2
0
00
0
2
0
+? +
+
=+kr kr
kr
这里等式右边的第二项(含)为修正项。于是有 cos
2
0
δ
ar
kr
kr
r
kr
00
00
0
0
0
0
2
0
11
1
=?
+
=(
tan( )
)(
tan
cos
)
δδ
δ
}tan1{)
cos
1
1(tan
1
0
2
0
0
0
0
2
000
δ
δ
δ
a
r
ara
k
=?+?=∴
=? +?
=ar a r
00
0
2
00
2
1
1
1
2
1
2
()
δ
δ
0
2
0
0
2
0
2
0
0
0
0
2
11
2
11
cot kr
aa
r
a
k +?=+?=∴ δδ
iii,对球壳、球方阱、球方垒情况简述
<球壳> <球方阱> <球方垒>
82
Vr r r a() ( )=?δ;; 。
Vr V r a
Vr r a
(),
(),
=? <
=>
0
0
Vr V r a
Vr r a
(),
(),
=><
=>
0
0
0
′′+ =


=
χ
μ
δχ
χχ
μ
χ
0
2
2
0
0
0
0
2
0
0
2
0
2
[()]
() () ()
() () ()
k
r
ra
aa
r
a
i
h
h;
χκχ
χχ
κκ
μ
0
2
0
2
0
0
02
0
0
0
22
0
0
2
0
0
2
() ()
() ()
()
,
ii
kr
k
Kk
V

++ =

+=
=+ =
h;
χκχ
χχ
κκ
μ
0
2
0
2
0
0
02
0
0
0
22
0
0
2
0
0
2
() ()
() ()
()
,
ii
k
k
Kk
V

+? =

+=
=? =
h

χ
χδ
0
0
0
0
()
()
sin
sin( )
i
Akr
kr
=
=+
; ;

χ
χδ
0
0
0
0
()
()
() sin
() sin( )
i
rAKr
rkr
=
=+
χ
χδ
0
0
0
0
()
()
() sinh
() sin( )
i
rA Kr
rkr
=
=+
kka Akka
r
ka
r
Aka
Aka ka
cos( ) cos
sin( )
sin
sin sin( )
+?
=+
=
=+
δ
μ
δ
μ
δ
0
2
0
2
0
2
2
h
h
=; ;

Aka ka
AK Ka k ka
sin sin( )
cos cos( )
=+
=+
δ
δ
0
0
AKa ka
AK Ka k ka
sinh( ) sin( )
cosh( ) cos( )
=+
=+
δ
δ
0
0
χ
χ
μ
χ
χ
χ
0
0
2
0
0
0
0
0
2
()
() ()
()
()
()
[() ()]
()
()
i
ii
a
aa
r
a
a
aa

+
=
=

h;
χ
χ
χ
χ
0
0
0
0
0
0
()
()
()
()
()
()
()
()
i
i
a
aa
a
aa

=
′;
χ
χ
χ
χ
0
0
0
0
0
0
()
()
()
()
()
()
()
()
i
i
a
aa
a
aa

=


Aka
aA k ka
r
ka
ka
ak ka
sin
(cos sin )
sin( )
cos( )
+
=
=
+
+
2
2
0
0
μ
δ
δ
h;
AKa
AaK Ka
ka
ak ka
sin
cos
sin( )
cos( )
=
+
+
δ
δ
0
0;
AKa
AaK Ka
ka
ak ka
sinh( )
cosh( )
sin( )
cos( )
=
+
+
δ
δ
0
0

∴=?
+
=?
+
=+

aa
ka
ka
a
ka
ka
ra
ka
ra ra
k0
0
0
2
0
2
2
2
2
1
1
2
2
1
2
(
tan( )
)|
(
tan
tan
)|
/( )
δ
μ
μμ
h
hh;
aa
ka
ka
a
Ka
Ka
a
a
a
k0
0
0
0
0
0
1
1
1
=?
+
=?
=?

(
tan( )
)|
(
tan
)|
(
tan
)
δ
κ
κ;
aa
ka
ka
a
Ka
Ka
a
a
a
k0
0
0
0
0
0
1
1
1
=?
+
=?
=?

(
tan( )
)|
(
tanh
)|
(
tanh
)
δ
κ
κ

(10.11)
§10.3 格林函数方法与波恩近似
83
1,格林函数方法与势散射基本积分方程
令)(
2
)(
2
rV
m
r
v
h
v
=U,于是这时正能量定态方程成为 dingeroSchr &&
)()()()(
2
rrUrk
vvv
ψψ =+? (10.12)
这里k
mE
=
2
2
h
,E为入射粒子的能量。往求这个方程的如此解ψ()
v
r,它当r →∞时具有下面渐近形式
ψθ? θ?(,,) (,)ref
e
r
ikz
ikr
→+
v v
引入和这个方程相应的格林函数Gr r
k
(? ′方程 )
( )()(?+?′ =?′kGrr rr
k
2
)
v v v v
δ (10.13)
若已知将有助于求解的方程,因为对此方程乘以UrG
k
ψ r()()
v v
′′ψ并对
v
′r积分,得
( ) ( ) ( ) ( ) () ()?+?′′′′=

kGrrUr rdrUr r
k
2
v v v v v v v
ψψ
将这个方程与方程相比较可知ψ()
v
r GU drψ
v


与ψ只相差一个齐次方程(的通解
)()?+=kr
2
0?
v
()
v
r,于是有
v v
ψ? ψ() () ( ) ( ) ( )
v v v v v
rrGrrUrrd
k
= r+? ′′′

v
′ (10.14)
v
由于积分号内只有才含有变数G
k
r,故ψ()r的渐近条件要求(当r → ∞时应趋于出射球面波)
将转嫁到上,对格林函数Gr r
k
(
v
′)
v
Gr r(
k
)
v v
′作这种要求。
现在往求这样的格林函数Gr r
k
(
v
)
v
′,它当kr → ∞时趋于
1
r
e
ikr
。为此,将所满足的方程两边同乘以无奇点的正规算符,于是可得
Gr r
k
()
vv

(+±
ki
2
η)
1
Gr r
ki
e
dk
k
ik r r
( ) lim
()
()
vv
v
v
vv
′ =



′′

η
ηπ
0
2
3
3
1
2?
=
′ +±


′′

lim
()
()
η
ηπ
0
22
3
3
2
e
kki
dk
ik r r
v
v v v
按边条件
1
r
e
ikr
( )的要求,由后面的推导可知这里kr →∞ iη前应取正号(若取?iη,将得出另一种格林函数,当时趋于kr →∞
1
r
e
ikr?
)。现在来计算这个积分,
Gr r
e
kki
dk
k
ik r r
( ) lim
()
()
vv
v
v
v v
′ =
′ ++


′′

η
πη
0
322
1
2
84
=
′′
′ ++

∞′? ′
∫∫
1
2
3
0
2
2204
()
lim sin
cos
πη
θθ?
η
θ
π
kdk
kki
ed
ik r r
vv
d
=

′ ++
′′

′? ′? ′? ′


1
2
1
2
0
220
()
lim
ηir r
ee
kki
kdk
ikrr ikrr
vv
v v v v
=


′ ++


′? ′

+∞

1
4
2
0
22
πη
η
ir r
ke
kki
dk
ik r r
vv
v v
lim
可以将积分变数′k延拓到复平面,利用留数定理计算这个积分。在′k为复数的平面上,被积函数有两个一阶极点
′ =+kki
22
η,也即′′≈± +kk ki
AB
、()
η
2
这里只要求,它的数值并不重要,因为积分完成之后要令η> 0 η→ 0。于是得
Gr r
ir r
i
ke
k
k
A
ik r r
A
A
( ) lim( )
vv
vv
v v
′ =


′→
′? ′
1
4
2
2
2
0
π
π
η
=?


1

e
rr
ik r r
v v
vv
显然,这个表达式满足先前所说的当r → ∞时趋于
e
r
ikr
的边条件。
于是,最后即得在势散射理论中处于中心位置的下述积分方程
ψ
π
ψ() ()()
v
vv
vvv
v v
re
e
rr
Ur r dr
ikz
ik r r
=?

′′


1
4
′ (10.15)
由于作为积分核的格林函数满足事先规定的r → ∞的边条件,故这个ψ()
v
r表达式满足这个边条件。这里已经取?为入射平面波。方程右边第二项为出射球面波(这由它的位相可知)。这是一个积分方程,在势散射下是严格的。它是作近似解或迭代解的出发点。
()
v
re
ikz
=
(kr t?ω )
2,一阶波恩近似
当势Vr较弱,或是相当地局域(Vr()
v
()
v
起作用的基本区域较小),或者当入射粒子的能量足够大时,由后面分析可知,上面积分方程的第二项数值将小于第一项
e
e
rr
Ur r dr
ikz
ik r r
>>

′′


v

v
vv
vvv
()()ψ
,(对任意
v
r值)
在这种情况下,可对积分方程第二项作一级波恩近似:将积分号下的ψ()
v
′r近似代以其零阶
85
近似,格林函数的分母e
ikz
vv
rr? ′取零阶近似为,分子r e
ik r r
v v

取一阶近似,即当
vv
rr>> ′时,

qk=?
ψ()
v

2
v
q
)
σ


0
rV
vv vv
v v
v v
rr r rr r r
rr
r
rer
r
′ =′+ ′ ≈?

=′
22
2
21()
接着,引入两个波矢:入射波波矢
v
v
kke
z0
=,散射波波矢
v
v
kke
r
=,由于是弹性散射,两个波矢的数值相同仅方向不同,并
令为入射粒子动量的
v
vv
k
0
改变,按图可得qk= 2
2
sin
θ

于是得到的如下渐近形式 ψ()
v
r
π
( )
h
vv
vv
re
me
r
eVrdr
ikz
ikr
iq r
→? ′′


2
2
因此,在一阶波恩近似(常称为波恩近似)下散射振幅的表达式为
f
m
eVrdr
iq r
(,) ( )θ?
π
=? ′

2 h
vv
vv
(10.16) ′
注意的模值只依赖于(以及θ k ),和?无关,但
v
q的方向(通过
v
k )依赖于?。这个公式表明,
方向的散射振幅正比于势场Vr(,θ? ()
v
的相关的傅立叶分量。公式还表明,一般地说:i,大动量传递(值较大)的散射截面比较小,因为指数因子的快速振荡减小了积分数值;ii,对高能(
q
k大,这导致的模值增加)入射粒子,若要q θ?(,)不为零,需要θ比较小,这时由于q值仍不大而避免了快速振荡使积分值变得很小,这就是说,高能散射一般集中于朝前方向。
若Vr为中心场,上式的角度积分可以预先算出,于是得 Vr() ()
v
=
ff
m
q
rV r qr dr(,) ( ) ( )sin( )θ? θ==? ′′ ′
2
2
h

σθ θ() () [ ( )sin( ) ]== ′′ ′′


f
m
q
r qr dr
2
2
42 0
2
4
h
(10.17)
注意这里的结果表明,入射粒子的动量k和散射角θ总是通过值的方式进入的。 q
3,波恩近似适用条件分析
a
a
Л.Д,朗道,E.M,栗弗席茨,量子力学(非相对论理论),高等教育出版社,1981年,或 张永德,波恩近似适用条件的推导与讨论,大学物理,1988年,第6期,第11页。
86
如前所说,若要波恩近似成立,充要条件是基本积分方程右边第二项数值上远小于第一项(对任意值),只有这样,对第二项才可以做前述的波恩近似。而若要这个积分项数值小,
至少需要下面三个条件之一成立
r
i,
ii,
iii.,
势足够弱势虽不够弱但其展布的区域足够局域入射粒子能量足够高
Vr
Vr
(),
(),
v
v
当然,联合作用会使近似更好地成立。之所以如此是因为,积分项主要贡献来自Vr不接近于零的基本区域,如果这个区域相当小(和入射粒子波长,即
()
v
1
k
,相比较),就是说势Vr()
v
相当局域,这项积分的数值自然很小;又,若Vr()
v
本身很弱,这项积分也会很小;再就是,
若入射粒子能量很大,k就很大,于是被积函数中的振荡因子e
ik r r
v v

将随积分变数
v
′r (以及参数
v
r ——它的模将足够大,以使kr → ∞ )的变化而快速振荡,积分值将因此急剧下降。
对积分进行详细估值可得如下两个波恩近似适用条件表达式
V
ma
V
v
a
<<
<<
h
h
2
2
(10.18)
这里是势场展布的不显著为零的区域的尺寸,为入射粒子的速度。第一个不等式只涉及势场本身,不涉及入射粒子的能量,它是说,按测不准关系,将粒子局域在中时的动能应大大于势能;第二个不等式表明,只要入射粒子能量足够高,不论势场形状如何波恩近似总能成立。于是,对一个场势,若低能时可对之作微扰,进行波恩近似,则高能时一定更可以;
反之不然。
a v
a
这里应当指出,对库仑场,难以给出一个确定的值,这时,可将第二个不等式右边代以(同时左边的
a a
r V中也是同一个),于是得r
A
r
<<
v
r
h
,也即
A
vh
<<1
这就是库仑场可当作微扰,能进行波恩近似的条件。
4,例算
i,库仑散射
这时Vr,于是
A
r
()=
f
m
q
r
A
r
qr dr
mA
q
qr dr() sin( ) sin( )θ =? ′

′′=? ′′
∞∞
∫∫
22
2 0 2 0
hh
这个积分在′ =∞r处呈现不确定性,这种不确定性在关涉到库仑场的不少积分计算中均可能出现,可利用以下通用的技巧将之避免过去(ε> 0 ),
87
f
mA
q
eqr
r
() lim sin( )θ
ε
ε
=? ′′

′∞

2
2
0
0
h
dr
=?
+
=?

22
2
0
22 2
mA
q
q
q
mA
qhh
lim
ε
ε
2
∴ σθ θ
θ
() ()
sin
==f
A
mv
2
2
24 4
4
2
(10.19)
这正是著名的Rutherford散射公式,它是1909年Rutherford研究α -粒子在金属薄箔上散射时提出的。这里公式表明库仑散射有两个特点:其一,集中于小的θ角,其二,截面反比于入射粒子能量的平方。另外,若假设A Ze=
2
,由上面Av<< h可得(
e
hc
2
1
137
=为精细结构常数)
Zv
c137
<<
于是,当Z ~ 1或入射粒子能量足够高,波恩近似对库仑场也是成立的。
ii,电子与原子的散射—屏蔽效应
电子和多电子原子散射时,入射电子一方面受原子核库仑吸力的作用,另一方面还受核外多电子库仑斥力的作用。严格说,这是个多体问题,但如果将核外诸电子的作用以一个分布电荷作用近似代替,可将这个问题化为一个两体散射问题,也就是电子入射到固定力心的散射问题。这时取
erρ()
v
Vr
Ze
r
e
r
rr
dr()
()v
v
vv
v
=? +




2
2
ρ
∴ f
m
e
Ze
r
e
r
rr
dr dr
iq r
(,)
()
θ?
π
ρ
=

+
′′
′? ′′
′′


∫∫
2
2
2
2
h
v
vv
vv
vv
=?′′ ′′
′′

me Z
qq
re dr
iq r
2
22 2
2
44
π
ππ
ρ
h
vv
vv
()
这里利用了积分公式
e
dr
r
q
iq r′


=

vv
v
4
2
π
,e
dr
rr
q
e
iq r iq r′? ′′

′? ′′
=

vv vv
v
vv
4
2
π
求这些积分时也要用到上例中库仑场技巧。于是令
Fre
iq r
(,) ( )θ? ρ= ′′ ′′
′′

d
v v
v v
称为波恩近似下的弹性散射形状因子,即得
f
me
q
ZF
me
q
Z
eff
(,) ( (,))θ? θ?=?=
22
2
22
2
22
hh
88
∴ σθ (10.20)?
θ
(,)
sin
=
meZ
k
eff
24 2
44
44
1
2
h
对这个例子可以略作讨论如下:首先,由积分估值可知Frdr(,) ( )θ? ρ< ′′ ′′ =

Z
v v;其次,如果是单单被核散射,ρ= 0,转化为上例的库仑散射;再三,核外电子屏蔽势常用Yukawa势,
即ρ,相应的计算很易进行;再四,这里的()′′ =
′′
′′
rc
r
e
r
a
1
ρ()
v
r也可看成是某个波函数的模平方ψ()
v
′′r
2
,其余参见上例计算。
§10.4 全同粒子散射
1,全同性原理在散射问题上的应用
记两粒子体系自旋耦合基矢为|,若将、交换,按角动量耦合理论可得 SMs s
12
> s
1
s
2
|()|SMs s SMs s
ssS
21 12
1
12
>=? >
+?
如果两粒子是全同的,,上式成为 ss
1
=
2
两全同波色子两全同费米子
|()|SMs s SMs s
SMs s SMs s
S
S
21 12
21 12
1
1
>=? >
>= >
说明两全同粒子体系自旋波函数是对称或反对称仅由S奇偶性决定。鉴于全同波色(费米)子体系总波函数必须是对称(反对称)的,于是得到结论:不论两个散射的全同粒子是波色子还是费米子,当体系总自旋S =奇数时,体系的空间波函数为反对称的;当总自旋S =偶数时,
体系的空间波函数为对称的。由于? =? +
v
rr(,,)π θ π?,对称化(反称化)的空间波函数渐近形式为
ψθ? θ? πθπ?(,,) ( ) ((,) (,)ree
r
ef f
ikz ikz ikr
→± + ±?+
1
于是有
σθ? θ? π θπ?
σθ? θ? π θπ?
σθ? θ? π θπ?
(,) (,) (,),
(,) (,) (,),
(,) (,) (,),
s
a
ff S
ff S
ff
=+?+
=+
=+?+
2
2
22
=,
=,
对偶数两全同粒子对奇数两全同粒子对两个可分辨粒子
注意,这里波函数ψ中的是和,而非平均,因此没有f
1
2
因子。这是由于考虑下图的两种过程,一并计算的缘故。而这归根到底又是由于,所测得的在质心系中向角散射的粒子,
无法说清它们是属于自左入射的还是属于自右入射的。
θ
89
2,例算
i,两全同的波色子散射。
这包括散射、散射、
16
αα? ππ
0
0 16
O O?核散射等自旋为零的粒子散射以及其他自旋为整数粒子的散射(如自旋1的、散射等)。这时散射微分截面中应使用空间对称波函数还是反对称波函数,要看耦合总自旋
π?π
++
π
π
S是偶数还是奇数决定。对和αα?
16 16
O O?散射而言,,因此这两个散射所使用的微分截面均为 S = 0
σθ θ π θ() () ( )
s
ff=+?
2
{ }
=+?+?
ff ff() ()Re()()θπθ θπ
22
2 θ
最后一项是干涉项,它正是基于全同性原理表现出的粒子的波动性,是纯量子效应。举个例子,设两个自旋为1的全同粒子散射,求非极化的散射微分截面。这时,总自旋 共
1+3+5=9个自旋态,其中的自旋态有3个,它们的空间波函数(按前面所说)均为反对称的,其余6个自旋态对应总自旋为0或2,空间波函数均为对称的。于是非极化截面为对这些自旋初态取平均(自旋权重问题的进一步叙述见下节),
S = 012,,,
S =1
σθ σθ σθ() () ()

=+
1
3
2
3
as
=++?
1
3
2
3
22
ff ff() ( ) () ( )θπθ θπθ
{ }
=+?+?
ff ff() ( ) Re ()( )θπθ θπ
222
3
θ
+
注意上面截面中的取实部项可正可负,代表由于全同性原理所引入的一种特殊的交换作用,是一种纯量子效应。
Re
ii,两全同费米子散射。
这包括散射、e散射、pee? e
+
p?散射、nn?散射等。这时,相应于自旋三重态(自旋波函数由于平行耦合而是对称的)的微分截面必须使用反对称的空间波函数;而相应于自旋单态(自旋波函数由于反平行耦合而是反对称的)的微分截面必须使用对称的空间波函数。
于是,若散射过程是非极化的,即入射粒子与靶粒子均未极化,它们自旋的取向都是无规的,
则各自旋态出现的几率不仅是相等的而且它们之间是非相干迭加。因此对自旋初态的平均来说,单态和三重态的统计权重分别为
1
4

3
4
。这就是说,非极化散射截面将为
σθ σθ σθ() () ()

=+
1
4
3
4
sa
90
=+?+
1
4
3
4
22
ff ff() ( ) () ( )θπθ θπθ
{ }
=+
ff ff() ()Re()()θπθ θπ
22
θ
当然,上面的分析是在自旋耦合表象中进行的;其实上述分析也可以在无耦合表象中进行。简记无耦合基为|。于是自旋初态有以下四个:mm
SS
12
> |,,
1
2
1
2
> |,,
1
2
1
2
> |,,?>
1
2
1
2
|,>
1
2
1
2
,它们各自的权重均为
1
4
。当自旋初态为|,,即入射粒子和靶粒子自旋态为
,则由于
1
2
1
2
>
1
0
1
0
入射靶
S、M守恒,出射时自旋态不变,但由于无法区分在角处测得的是入射粒子还是靶粒子,并且由于自旋态这时是对称的,所以这种情况的
θ
σθ θ() ()=?π(ffθ)?
2;当自旋初态为|,,即
,情况和第一种类似,
结果为
>
1
2
1
2
0
1
0
1
入射靶
σθ() θ()=?π θ( )?ff
2;当自旋初态为|,,即时,由于自旋第三分量取向不同,入射粒子与靶粒子已可区分,这时在
1
2
1
2
>
1
0
入射
0
1

θ角处的微分截面(不论测得的是入射粒子还是靶粒子)σθ π θ( )?ff
2
θ() ()=+;最后,当自旋初态为|,,即时,
和第三种情况类似,有
>
1
2
1
2
0
1
1
0
入射靶
σθ() θ π θ() ( )=+?ff
2
。总括这四种情况,将它们乘以权重
1
4
相加,
得非极化截面
{}
σθ θ π θ θ π θ() () ( ) () ( )

=++?
1
4
222
22
ff f f
2
{ }
=+
ff ff() ()Re()()θπθ θπ
22
θ
结果和耦合表象中的一致。
§10.5 考虑自旋的散射
1,散射分道的概念。
如果散射时相互作用势和自旋态无关,散射中入射粒子和靶粒子的自旋态都分别保持不变。这相当于前面所考虑的情况。如果相互作用势含自旋相互作用,守恒的自旋量子数当然在散射前后保持不变,但不守恒的自旋量子数将发生变化。这就可能会导致入射粒子和靶粒子自旋状态的改变。记散射前这两粒子系统自旋初态为(称为入射道),散射后的自旋末|x
i
>
91
态为(称为出射道),于是,两粒子系统自旋态从的散射,称为散射分道。由于初态和末态一般均不止一个,于是考虑自旋的散射将会有多个散射分道。原则上,
每个分道的散射振幅(从而微分截面)各不相同,应分别予以计算。这里,自旋态|、|
已取定为自旋耦合表象基矢,分道名称均按自旋初、末态的耦合表象基矢来标定。
< x
f
| ||xx
if
>→ > ()fi
x
i
> x
f
>
)||>=>Eψ
(,)|ψθf x
ifi
+
f
>
fi
)
)
fi
2
→∞
sre
ikr
r
|>
→∞
xxV
ff
| |><
′′
I
|>
,,)|(ψ
vvv v
s s r
12
′′
→∞ ψ >
2,分道散射振幅计算 —— 带自旋的波恩近似。
现在的问题是:求解定态方程 dingeroSchr &&
((,?+
h v vv
2
12
2m
Vrs s?ψ
如此正能量解,它在r →∞时有下面渐近形式
|(,,) |
vv v
rs s e x
e
r
r ikz
ikr
12
>? →> >
→∞
右边渐近形式中,第一项为入射态,带着这个自旋初态,第二项为球面波出射态,带着|这个自旋末态。因此这是分道,相应的分道散射振幅为,分道散射截面(常称为极化截面)为
|x
i
>
)fx (i→ f (,θ?
σθ? θ?(,) (,
fi
f=
和无自旋时类似,由于渐近形式中入射平面波和出射球面波之间的干涉项当r时趋于快速振荡而被抹去,所以可将写为 f
fi
(,)θ?
fxr
fi f
(,) | (,,)θ? ψ=<
v v v
12
重复以前计算,引入
Urs s
m
Vrs s
m
xx
i
if
(,,) (,,) | |
vv v
h
vv
h
12
2
12
2
22
= = ><∑

′′
i′
这里利用了初、末自旋态的完备性条件||xx><∑ =,于是
()|?+>=kU
2
ψψ
注意此方程相应的格林函数方程和无自旋时相同,因此按格林函数方法求得此方程解的积分表示式为
|(,,) | (,,)ψ
π
vv v
vv
vv v
v v
rs s e x
e
rr
Ur s s dr
ikz
i
ik r r
12 12
1
4
>= >?

> ′


当r时对这个表达式作波恩近似,即将积分号内的|代以,对格林函数的指ex
ikz
i
| >
92
数作一级近似而分母作零级近似,并取定出射的自旋态为| (选定出射分道,即用|左乘上式),于是即得分道(的散射振幅为
x
f
> < x
f
)if→
f
fi
(,)θ?
f (,)θ?
x
i
>
}
)m
Vrs
f
,,′
12
vvv
i
>
| x
f
>
f
ml
)
)
fi
)
ml
2
(,
c
l
i()
2
>
σθ?(,
m
ex x
iq r
i
| ( )|
π
=? < > ′


2
2
h
v
vv
(10.21) dr
这个公式和前面无自旋的差别仅在于:相互作用势代为它在自旋初、末态之间的矩阵元。
3,自旋权重平均
上面公式中的|、|均为自旋耦合表象中的基矢,因此称为分道的微分截面。但如果给定的自旋初、末态、|按耦合表象看为(相干)
叠加态(仍为纯态),上面公式依然成立。这时可将|、按耦合基矢展开,设展开系数分别为和
{
,散射振幅为
f
fi
(,)θ?
)
{}
c
l
i()
x
f
> σθ(,
(if→ |x
>
x
f
>
x
i
c
m
f()
fcc
fi l
i
m
f
lm
(,) (,
() ( )
θ? θ?= ∑
这里为(分道的分道散射振幅。于是微分截面为 f
ml
(,)θ? l→
σθ? θ?(,) (,
() ( )
fi l
i
m
f
lm
cc f= ∑
当自旋末态为某个基矢时,σθ。显然,当|或|为叠加态时,
各分道之间一般将存在干涉,并不能表示为各分道截面
θ?) (,)
()
fi l
i
fl
l
cf= ∑
2
x
i
> x
f
>
σ θ?(,)
ml
按系数cc
l
i
m
f() ( )
2
或的权重叠加。若求在此|入射下的总微分截面,则应分别取x
i
< x
f
|为自旋耦合表象的全部基矢,求出各个出射分道截面后,非相干叠加,
σθ? θ?) (,) (,)
()
ifi
f
l
i
fl
lf
cf= ∑ = ∑∑
2
这是由于,伴随测量的波包坍缩总是导致相干性的破坏,因此出射分道截面的测量破坏了各出射分道之间的相互干涉,这导致,总截面总是等于各个出射分道截面之和,简称为“对末态求和”。
对于非极化情况,比如给定的初态|为一些耦合基矢(或无耦合基矢)的非相干混合
(不是相干叠加,不是纯态),将不存在初态中各成分之间的干涉项。这正是非极化
x
i
>
|x
i
> c
l
i()
93
粒子入射到非极化靶粒子的情况。这时,在此入射下,给定的出射道|的微分截面为
|x
i
> x
f
>
()
l
i
c
2
i
lf
= ∑∑
cos
sin
α
α
2
2
1
2
2
(|,
//
10
22
| x
f
>
ββ/i
ef
2
(,
/i
ef+
2
rdr′′
vv
k r()? ′? ′
vv
v
σθ? σθ?(,) (,)
fi fl
l
= ∑
而总微分截面,和上面一样,为各出射道的微分截面之和,f
σθ? σθ? σθ?(,) (,) (,)
()
if
f
l
i
fl
c= ∑
2
这就是说,非极化散射中,截面对自旋初态的依赖是:各分道截面分别以混态中相应系数模方为权重的平均,简称为“对初态平均”。至于对末态的总截面仍是等于各个出射分道截面之和。
前者例如,两个可分辨的
1
2
自旋粒子,自旋分别为|和χ>=
1
0
|,于是自旋初态|为
/
/
ξ
β
β
>=
2
2
e
e
i
i
x
i
>
|
cos
sin
cos |,sin |,
/
/
//
x
e
e
ee
i
i
i
ii
>=
=>+
1
0
2
2
1
2
1
22
1
2
2
2
α
α
αα
β
β
ββ
>
=>+ >+
cos |,sin |,)
αα
ββ
2
11
1
2
2
00ee
ii
>
第三步等号已用了从无耦合基向耦合基的转换关系。假定自旋末态一般为,不去研究它的展开,这时微分截面为
a
σθ?
α
θ?
α
θ?
α
θ?
β
(,) cos (,) sin (,) sin )
/
(,) (,) (,)fi
i
f f f
e f=+
2
1
2
2
1
2
2
2
11 10 00
2
说明,当相互作用与自旋有关时,σ
fi
一般并非是各分道截面按|展开系数的模方的权重平均。
x
i
>
后者例如,两个
1
2
自旋的可分辨粒子,散射势Vdssr=?
v v v
12
δ()时,非极化截面的计算。
这时
f
m
exsx
fi
ik
fi
() | | ( )θ
π
αδ=? <? >

2
2
12
h
vv
a
再进一步的计算要视|如何、自旋有关相互作用矩阵元情况。 x
f
>
94
=? <? >
α
π
m
xssx
fi
2
2
12
h
vv
||。
由于
vv
v
vv
v
ss S s s S
12
2
1
2
2
2
1
2
1
2
3
2
= =?()(),其中
v
v v
Ss s=+
12
是系统的总自旋,因此这一散射
过程中总自旋量子数和它的第三分量S M均为守恒量子数,就是说,初末态中(或S M )不
同的矩阵元均为零。这时
系统总的单态系统总的三重态
S
mm
S
mm
=
=?=
=
==
0
2
1
2
3
2
3
8
1
2
1
2
1
2 8
0
22
22
2
2
2
2
1
22
22
2
2
2
2
:
σθ
α
π
α
π
σθ
α
π
α
π
()
()
()
()
()
()
()
()
()
()
h
h
h
h
对自旋初态按非相干混合权重平均之后的微分散射截面,即非极化散射截面为
<>= + =σθ σ θ σ θ
α
π
() () ()
()
1
4
3
4
3
8
01
22
2
m
平均时已假定各个自旋态的权重相同,即几率相等。这是由于已经假定为非极化情况。
对于非极化情况的自旋权重系数问题,可将上面例子稍作推广。假定两个自旋为s的全同粒子散射,并假定为非极化情况。按前面所说,计算截面时应对自旋初态取平均。结果为
s
s
s
s
s
s
s
s
s
s
sa
sa
=<>=
+
+
+
+
+
+
+
+
半整数两费米子整数两波色子
,(,) (,) (,)
,(,) (,) (,)
,
,
σθ? σθ? σθ?
σθ? σθ? σθ?
21
1
21
1
21 21
这里,相应于空间对称化波函数,σθ?(,)
s
σ θ?(,)
a
相应于空间反称化波函数。这两个公式由来是:这个全同粒子系统总自旋的可能取值由反平行取向时的零到平行取向时的2s,逐一增加,所以系统自旋态的总数目为
()(21 21
0
2
2
Ss
S
Ss
+∑ =+
=
=
)
其中,
Ssss
ss s
Sss
ss s
取偶数值的态数目对半整数情况对整数情况取奇数值的态数目对半整数情况对整数情况
:( )
()( )
:( )( )
()
( )
( )
()
( )
21
12 1
12 1
21
+=
++ =
++ =
+=
而由前面叙述,当总自旋为偶数时,空间波函数为对称的;S为奇数时,空间波函数为反对称的。于是即得上面的结果。
S
4,计算举例。
95
i,为方便地描述两个
1
2
自旋可分辨粒子(如质子和中子)的各种极化与非极化散射,引入散射振幅算符,它被定义为
$
f
$
||fx f x
00 1 00
>= >,
$
||fx f x
MM131
>= >
这里为自旋单态,为自旋三态,和分别为它们的散射振幅。a) 求的表达式;b) 若散射前质子处于
态,中子为态,求散射后、自旋反向的几率;c)
若初态为
| x
00
> | x
M1
>
1
0
f
1
0
1
f
3
$
f
p n
n p
1
0
2
2
p
i
i
e
e
cos
sin
/
α
α
β
β
2
2
n
/
,求散射总截面。
解:a) 设,这里
$
f
p
=+?αασσ
12
vv
n
α
1
、α
2
为两个待定系数。引入质子中子自旋交换算符
P
Px x Px x
spn
ssM
=+?
>=? > >= >
1
2
1
00 00 1 1
()
||,||
vv
σσ
M
M
3
于是将分别作用到态和态上,
$
f | x
00
> | x
M1
>
$
| ( ( ))| ( ( ))|
()| |
$
| ( ( ))| ( ( ))|
()| |
fx P x x
xfx
fx P x x
xfx
s
MsM
MM
00 1 2 00 1 2 00
120 10
112 112 1
121 31
21 21
3
21 21
>= +? >= + >
=? >= >
>= +? >= +? >
=+ >= >
αα αα
αα
αα αα
αα
由联立方程即得 αα αα
1 2112
3?= +=ff,
$
(( )( )f
ffff
ff ffP
pn s
=
+
+ )
= ++?
1331
13 31
3
44
1
2
1
2
vv
σσ
b) 散射初态为
||,(|x
i
pn
>=
=?>= >+>
1
0
0
1
1
2
1
2
1
2
00 10|)
按题设反向的要求可知自旋末态为
||,(|x
f
pn
>=
=? >=? >+ >
0
1
1
0
1
2
1
2
1
2
00 10|)
96
于是
{}
{}
<>=<?< ++?
>+>
xfx f f f fP
fi s
|
$
|||()(
||
1
2
10 00
1
2
1
2
00 10
13 31
)
=<?<>+>
1
2
10 00
1
2
00 10
31
(| |)( )(| |)ff
=?
1
2
31
()ff
相应的微分截面为σ

=?
1
4
31
2
ff
0 >
。由于散射中总自旋的第三分量守恒,因此散射后的出射道也只能是和|1这两个。这两个出射分道的分道微分截面分别为 |00 >
{}
ffffP
is00 1 3 3 1
00
1
2
1
2
1
2
00 10
,
|( ) ( ) | |=< + +?
>+ >
=+=
1
22
1
22
1
2
13 31
()()ff ff f
1
{}
ffffP
is10 1 3 3 1
10
1
2
1
2
1
2
00 10
,
|( ) ( ) | |=< + +?
>+ >
=++?=
1
22
1
22
1
2
13 31
()()ff ff f
3
,
于是
σ
00 1
21
2
= f,σ
10 3
21
2
= f
因此反转的几率P为
P
ff
ff
=
+
=
+
σ
σσ

00 10
31
2
1
2
3
2
2( )
注意,这里分子上已表现出在两粒子自旋反转中分道之间存在干涉。
c) 这时自旋初态
|
cos
sin
cos |,sin |,
/
/
//
x
e
e
ee
i
p
i
i
n
ii
>=
=>+
1
0
2
2
2
1
2
1
22
1
2
1
2
2
2
α
α
αα
β
β
ββ
>
=>+ >+
cos |,sin (|,|,)
//
>
α α
ββ
2
11
1
2
2
10 00
22
ee
ii
于是各个出射分道(这时仍有三个结果不为零的:<11|、<10|、< 00| )的散射振幅分别为
< >=< + +?
>+
11 11
1
2
1
22
11
13 31
2
,|
$
|,|()()(cos|,
/
fx f f f f P e
is
i
α
β
97
+>+
1
2
2
10
1
2
2
00
22
sin |,sin |,
//
>
α α
ββ
ee
ii
=
fe
i
3
2
2
cos
/
α
β
<>=10
1
2
2
3
2
,|
$
|sin
/
fx f e
i
i
α
β
<>=00
1
2
2
1
2
,|
$
|sin
/
fx f e
i
i
α
β
从而微分总截面为
σθ?(,),|
$
|,|
$
|,|
$
|=< > +< > +< >11 10 0 0
22
fx fx fx
ii
2
i
=+ +fff
3
2
2
3
2
2
1
2
2
2
1
22
1
22
cos sin sin
α α α
=++σ
α α
σ
α
3
22
1
2
2
1
22
1
22
(cos sin ) sin
=++?
1
4
3
1
4
31 31
()()coσσ σσ αs
ii,上例中,若初态为|的两个
cos
sin
/
/
x
e
e
i
i
i
>=
α
α
β
β
2
2
1
0
2
2
入射靶
1
2
自旋全同粒子,求散射的非极化截面。这个问题还可以提得更一般些:若两个
1
2
自旋的全同粒子,它们各处于自旋平均值为<和的自旋初态上,求(非极化)散射截面。 >
v
s
1
<
v
s
2
v v
>
假定这个自旋初态为|,Vrx
i
> s s r rs s(,,) () ()
v v v v v
12 1 2 1 2
= +?α α,于是,总的非极化截面为对全部末态|的截面求和,x
f
>
σθ θ() () | | | |

=

=?<><>′′′


′+?′′
fce xVxxVxdr
fi
x
iq r iq r
if fi
f
f
2
2
vv vv
vv
= ′′′ < ′′ ′ >

′? ′′
cdrdre xVrssVrssx
iq r r
ii
2
12 12
vv v v v v v v
v v v
()
|(,,)(,,)|
由于()( )(
vv vv vv
ss
12
2
12
2
12
1
4
1
4
32?=?=σσ σσ
ββ
1212
(,) (,)
vv vv vv
′′′+ ′
),因此V的二次幂乘积算符不外乎为
′′?rr rr ss的形式,于是可得到σ θ()
非对极化矢量的一般依赖关系为
σθ() | |

=+<? >= + < >?< >abxssx abs s
ii
v v v v
12 1 2
现在可以根据两种极端情况来决定表达式中的系数和。首先,靶和入射粒子均为非极化a b
98
的情况,有<>=<>=
v v
ss
12
0,同时σθ σθ σθ() (() ())

== +a
s
1
4
3
a
,这里
σθ()
s
θ π θ() ( )ff=+?
2
,σθ;其次,为了靶和粒子均沿同一方向极化的情况,有
θ π θ() () ( )
a
ff=
2
<>?<>
vv
ss
12
=
1
4
,而这时系统总自旋必为1,又得σθ σθ() ()

=+ =ab
a
1
4

解出、,即得 a b
σθ()

1
4
σ((
<>?<>=
vv
ss
12
1
4
cos
sin
/
/
e
e
i
i
α
α
β
β
2
2
2
2
入射
i
θ σθ σθ σθ) ()) (() ())=++?<>?<3
12sa as
ss
vv
>
将此一般结果应用于本题上面特殊的,可得 | x
i
>
{}{}
=001
1
4
sin cos,sin sin,cos,,cosαβαβα α
于是在|情况下散射截面为 x >=?
1
0

σθ σθ σθ σθ σθ α() (() ()) (() ())cos

=++?
1
4
3
1
4
sa as
99
第十一章 含时问题与量子跃迁
本章讨论量子力学中的时间相关现象。它们包括:时间相关问题的一般讨论、含时微扰论、量子跃迁也即辐射的发射和吸收问题。如果说,以前各章研究的是量子力学中的稳态问题,则本章讨论的是量子力学中的非稳态问题。
众所周知,由于我们所需的时空结构中时间轴所固有的均匀性,孤立的量子系统必将遵从不显含时间的Schr?dinger方程。因此,这里的含时Schr?dinger方程所表述的量子系统必定不是孤立的量子系统,而是某个更大更基本的孤立系的一部分,是这个孤立系的一个子系统。当这个子系统和孤立系的其他部分存在着能量、动量、角动量、电荷甚至粒子的交换时,
便导致针对这个子系统的含时Schr?dinger方程问题。在掌握下面内容前应当记取这一点。
§11.1 含时Schr?dinger方程求解的一般讨论
1,量子力学中时间相关问题的一般分析
量子力学中,时间相关问题可以分为两类,
i,系统的哈密顿量并不含时
这时,要么是散射或行进问题,要么只是由于初始条件或边界条件使问题成为与时间相关的现象。行进问题例如中子初始以一定的自旋取向进入一均匀磁场并穿出,这是一个自旋沿磁场方向进动的时间相关问题; 初始条件问题比如一个波包(而非一个平面波)的自由演化,这是一个与时间相关的波包弥散问题(其实,由初态引起的含时问题也可以换个提法,
这就是,由于哈密顿量中的某中相互作用导致系统初态的不稳定。例如哈密顿量中的弱作用导致初态粒子的β衰变等); 最后是阱壁位置随时间振荡的势阱问题,这当然是由于边界条件使问题成为一个与时间相关的现象。
ii,系统的哈密顿量含时
这比如,频率调制的谐振子问题或是时间相关受迫谐振子问题等。
如果问题允许有精确的、解析的解,就称相应的哈密顿量H为可积的,这个系统为可积系统。所谓“解是精确的”是指相应的波函数ψ()
v
rt能够被表述为解析的形式或是一个积分
a
。情况和经典力学相似,在量子力学中,时间相关的可积系统比定态可积系统更少。绝大多数时间相关问题只能以各种近似方法求解。
由于篇幅所限,这里只限于叙述时间相关的部分问题和某些近似方法。
2,含时系统初始衰变率的一个普遍结论
这里研究的问题一般可提为
i
drt
dt
Ht rt
rt r
t
|()
()| ( )
|()| |( )
ψ
ψ
ψψ
v
v
vv
>
=>
>= >
=0
0
(1.1)
注意,一般而言,由于不同时刻的彼此可能不对易,即 Ht()
[]
Ht Ht t t(),(),( )
12 1
0≠≠
2
a
含时问题精确求解可参见M,Kleber,Exact Solutions for Time-dependent Phenomena in Quantum Mechanics,
100
因而,一般会有
[ ]
Ht H d
t
t
(),( )ττ
0
0


也即
Ht e
iHd
t
t
(),
()?


ττ
0
0
于是,解并不能想当然地简明紧凑地写成。 |()ψ
v
rt > er
iHd
t
t

>
()
|( )
ττ
ψ
0
0
v
然而,可以一般地证明:任一含时系统,其初始时刻的衰变(或跃迁)速率必为零。这里,
初始时刻衰变(或跃迁)速率定义为
dP t
dt
Pt r rt
t
()
|
() ( )| ( )
=
=
=< >
0
2
0
0ψψ
vv
(1.2)
这里是演化到时刻时,系统仍存活(保持)在初态的几率。 Pt() t
证:由于
dt
dt i
Ht t
|()
()| ()
ψ
ψ
>
=>
1

dt
dt
t
i
Ht
<
=?<
ψ
ψ
()|
()| ()
1
于是
dP t
dt
d
dt
t
d
dt
t
()
( )|{ | ( ) } { ( )|}| ( )=< > + < >ψψ ψψ00
=< >?< >
1
0
1
0
i
Ht t
i
tHtψψ ψψ( )| ()| () ()| ()| ( )
令取极限,即得 t → 0
dP t
dt
t
()
|
=
=
0
0
(1.3)
注意,这是含时系统的一个普遍结论,当然也是下面分类含时微扰论的共同特征。此外还应当强调指出,这一量子力学结论和放射源的负指数衰变规律是想抵触的,但这不等于说明,
对于一个属于统计量子系综(先先后后被制备出的大量不稳定核素)的放射源,这一负指数的统计衰变规律不成立。详细参见文献
a

3,初态衰变系统长期衰变规律的一个分析
b
和上面给出的初始时刻衰变特性相对照,下面证明,相互作用引起的初态的衰变,当时也将偏离负指数规律。 t →+∞
Phys,Reports,236,No.6(1994)。
a
Yong-de Zhang,Jian-wei Pan,H.Rauch,Annals of the New York Academy of Sciences,Vol,755,353 (1995);
张永德,潘建伟,<量子Zeno效应新论>,大学物理,第14卷,第7期,第34页,1995年。
b
本段内容可见L,Fonda,G.C,Ghirardi and A,Rimini,Decay Theory of Unstable Quantum Systems,Rep,Prog,
Phys.,Vo l,4 1,1978。
101
假定这一不稳定的系统是个孤立系,它初态的衰变完全由于内部的相互作用所致。于是哈密顿量H将不显含,并且有 t
|() |()ψψte
iHt
>= >
0
记,于是到时刻初态的存活几率。 At t() ( )| ()=< >ψψ0 t Pt At() | ()|=
2
我们假定,哈密顿量H的能谱有一个下限。这个假定从物理上看是必要的,因为由于跃迁(特别是自发跃迁)的存在,没有这个下限的量子系统将会因不断向下跃迁、不断释放能量而坍缩掉,从而失去研究的价值。并假设H的本征函数族为{ }|(,)?αE >。于是

>ψα?><α?ψ<α>=ψψ=<

)0(|e|),E(),E(|)0(dEd)0(|e|)0()t(A
iHtiHt
∫∫
+∞
>ψα?<α=
min
E
iEt2
e|)0(|),E(|ddE

+∞
ω=
min
E
iEt
dEe)E(,
这里,并且有 ωα?αψ() | (,)|()|Ed E=< >

0
2
<>=

=
+∞
ψψ ω()| () ( )
min
00 EdE
E
1
由于的这个绝对可积性质,可以直接引用傅立叶变换理论中的Riemann-Lebesque定理ω()E
a
,得
lim ( )
t
At
→∞
= 0
不但如此,根据傅立叶变换理论中的Payley和Wiener定理:如果A(t)的傅立叶变换像函数在某个下限频率以下恒为零(如现在的)E(ω在以下恒为零),则A(t)必定满足
min
E
dt
At
t
log| ( )|
1
2
+

<+∞

+∞
按此定理,若要积分在时收敛,则必须要求 t →+∞
log| ( )|At Ct
t
q
→+∞

1
2
这里C、为某两个常数,。由于,得 q q <1 At
t
()
→+∞
→ 0
At e C q
t Ct
q
(),(,)
()→+∞?
→ > < 0 1 (11.4)
a
参见,例如,河田龙夫著,富里哀变换与拉普拉斯变换,(现代应用数学丛书),第三页,上海科学技术出版社,1961年。
102
这一结果表明,量子系统基态的存在性要求,在大衰变时间下,不衰变几率偏离负指数规律,
并且要慢于负指数衰减。
4,量子Zeno效应
理论研究发现
a
,频繁地对一个不稳定系统进行量子测量将会抑制或阻止它的衰变(或跃迁),若极端而言,连续的量子测量将使不稳定系统稳定地保持在它的初态上,完全不发生衰变或跃迁。这种初态存活几率随测量频率增加而增加的现象就是量子Zeno效应。这种纯粹的量子效应其实是量子测量理论的一个直接推论,完全不存在经典对应的东西。应当强调指出,这里的测量是真正意义上的量子测量,也即第一章中所论述的可以分解为谱分解、随机坍缩和初态演化三个阶段的完整定义下的量子测量。
设一个含时量子系统的初态为|()ψ 0 >
Pt() |=<
0
,按上面Riemann-Lebesque定理,随着这个不稳定系统的演化,其初态的存活几率将越来越小。当然,这个按它的物理含义应当只适用于,自t开始演化之后,直到时刻才进行检验初态存活与否的量子测量,在(,时间间隔内不另进行任何这类量子测量。现在问,如果在(,之间再附加以若干次这类量子测量,上面意义下的这个实测值会不会发生变化? 根据量子测量理论所作的理论分析表明,的实测值应当增加。论证如下,
t( )| () |>ψψ0
2
Pt()
Pt()
)
= t
)0 t 0 t
Pt()
将[,区间等分为]0 t N份,在每一时刻t
nt
N
n
=,进行一次量子测量,以确认体系是否仍在上。按上面关于含义的叙述,第一次在|(ψ 0)> Pt()
t
N
时刻测量时,初态存活几率为
P
t
N
(),按测量理论,除衰变或跃迁掉的已经不计入了以外,剩下的这P
t
N
()部分将坍缩为
,并以|(为初态,以|()ψ 0 > )ψ 0 >
t
N
时刻为初始时刻再次重新开始演化,演化到2时刻,
再次作类似测量,于是,经两次测量后到
t
N
2
t
N
时刻,总计的初态存活几率为P
t
N
2
()。如此继续推论下去,最后可得:在[,内经受]t0 N次测量后,初态|()ψ 0 >的存活几率为
Pt P
t
N
N
N
() ( ( ))=
当N足够大时,可将P
t
N
()展开并保留到一阶项
P
t
N
PP
t
N
P
t
N
()[() ()] ()≈+′ =+′0010
于是,令,即过渡到在内为连续测量的理想极限情况,设这时存活几率为,

N →∞ [,]0 t P
c
a
这个纯量子效应最早在理论上由Sudarshan等人提出,参见J,Math,Phys.,18,756 (1977); Phys,Rev,D,
16,520 (1977)。有关论述很多,这里的简化改进论述主要取自文献Y.D,Zhang,J.W,Pan and H,Rauch,Some
Studies about Quantum Zeno Effects,该文被包括在《Fundamental Problems in Quantum Theory》,edited by
D.M,Greenberger and A,Zeilinger,Annals of the New York Academy of Sciences,Vol,755,353 (1995)。
103
PP
t
N
e
c
N
NP
=+′ =
→∞

lim ( ( ) )
()
10
0 t
注意2,中的结论,′ =P ()00,最后得到
P
c
=1 (1.5)
即当一个不稳体系经受连续量子测量时,将一直处于它的初态而将不发生(本应发生的)衰变或跃迁。当然,连续测量尽管原则上是存在的,但实验上常常不易实现,因此用实验检验这一效应存在与否只需作到:对于给定的区间[,,存活几率 ]0 t
Pt Pt N N
NN
21
21
() (),>> 当
5,含时问题在相互作用图象中的处理
时常,哈密顿量中含时部分只是一小部分,说它是一小部分是其中含有一个小参量,因此常常可以将含时哈密顿量分解成为一个不含时的而且其本征值本征矢量为已知的部分,加上一项含时的不便于严格处理的部分
Ht()
H
0
′H,即时常可以将表示为 Ht()
Ht H H t() ()= + ′
0
(1.6)
这时对Schr?dinger方程
i
t
t
HHt th
ψ
ψ
|()
(()|(
>
=+′ >
0
)
作一个幺正变换Ut,并记 e
iH t
0
0
()
/
=
h
ett
iH t
I
0
/
|() |()
h
ψψ>= >
于是有
{}
i
t
etHHet
iH t
I
iH t
I
h
hh
ψψ

>= +′ >
00
0
//
|()( )|()
最后得
i
t
t
Ht t
Ht e Hte
I
I
I
iH t iH t
h
hh
ψ
ψ
|()
()| ()
() ()
//
>
= ′ >
′ = ′
00
I
(11.7)
其解为
|() |() ()|()ψψ ττψτt
i
dH
II
t
>= >+ ′ >

0
1
0
h
I
(11.8)
如果采用迭代法求解这个积分方程,即得
|() ()
()
( ) ( ),....,| ( )ψτττττ
τ
t
i
dH
i
ddH H
II
t
II
t
>= + ′

+ ′′
∫∫
+
>1
11
0
0
2
121 200
1
h h
ψ (11.9)
由于算符正比于中所含的小常数,上面表达式中的大括号内各项即成为关于这个小常数的幂级数展开式,很便于作各级近似截断处理。这正是由于从原先的哈密顿量中经变换减去了主要部分的缘故。
′Ht
I
() ′Ht()
H
0
Ht()
104
§11.2 时间相关微扰论与量子跃迁
1,含时扰动及状态之间量子跃迁的概念
系统的哈密顿量原来为,自某一时刻起经受一扰动H
0
′H,总哈密顿量成为
。这时可按HH H=+′
0
′H与时间是否有关而区分为两种情况:若′H与时间无关,要么是个定态微扰论问题,是定态波函数及其本征值的修正问题,要么是属于定态框架下的散射或跃迁问题,这视问题的提法及初态情况而定。与此相应,作为展开基矢的本征函数族,既可选的也可以选择H
0
H的。若′H与时间有关,则是个非定态问题,体系的能量已不再守恒,其空间几率分布一般也将随时间变化。这时只能选择的本征函数族作为展开的基矢。
这时应当注意的是,用的本征函数族
H
0
H
0 { }
|n>=|,|
/
ne H E
iE t
n
n
>
h
n
0
>
m
>
2
对含时问题的未知态展开时,展开式的系数一般应是时间相关的。即有 |()ψ t >
| () ()|,| () ()|
||
ψψtctm c
Hm Em
Im
m
m
m
m
>= >∑ >= >∑
>= >
00
0
于是
|() |() () |
() | ()
//
/
ψψ
ψ
te t cte m
ct n t e
iH t
Im
iE t
m
n
iE t
m
n
>= > = >

=< >

0
hh
h
从而系统到时刻处于|态的几率即为 t n >
Pt c t n t
nn
() | ()| | | () |==<
2
ψ (11.10)
显然,这正是系统自初态向态的量子跃迁,而状态的这类跃迁正是由|n > ′H所引起的。
2,量子跃迁系数基本方程组及其一阶近似
这里就用的本征函数族作变系数展开法来研究量子跃迁几率的计算。由相互作用表象中态矢的方程
H
0
)
I
>|(ψ t
|() |() ()|()ψψ ττψτt
i
dH
II
t
>= >+ ′ >

0
1
0
h
I
两边作用以,并在积分号下的< m| ′ >H
I
()| ()
I
τ ψ τ中间插入完备关系,即得 |n
n
><∑ |n
ct c
i
dme He nn
mm
iH iH
I
t
n
() ( ) | ( ) | | ( )
//
=+ < ′ >< >


0
1
00
0
h
hh
ττ ψ
ττ
τ

ct ct
i
dH e c m
mm mn
i
nt
t
n
mn
( ) ( ) ( ) ( ) (,,2,......)=+ ′


=
0
1
01
0
h
ττ τ
ωτ
,(11.11)
这里ω,并且已将刚刚加上扰动的初始时刻记为,它既可以为有限值,
也可以是即无限的过去。
mn m n
EE=?
1
h
(

) t
0
105
如果在t时刻体系处于的一个本征态|上,在经受扰动
0
H
0
n > ′Ht()后到时刻,系统的跃迁系数方程组即为
t
ct
i
dH e c m
mn mn ml
i
t
t
l
ml
( ) ( ) ( ) (,,,......)=+ ′


=δτττ
ωτ
1
012
0
h
ln
,(11.12)
若考虑向态的跃迁( ),则有 |m> mn≠
ct
i
dH e c m
mn ml
i
t
t
l
ml
( ) ( ) ( ) (,,,......)= ′


=
1
012
0
h
ττ τ
ωτ
ln
,
这里已在系数的脚标上标记了初态|的记号,以表明是由态的量子跃迁系数。
这时相应的跃迁几率。
n >
t)|
2
||nm>→ >
Pt c
mn mn
() | (=
如果体系初始时刻处于混态
{
(即,以的几率处于|态上,等等),则向|
态(全部的)的跃迁几率为
}
n
pn
n
,| > p
n
n > m>
m ≠ n
Pt pP t
mnm
n
() ()= ∑
上面这组跃迁系数的方程组是个积分方程组,为进一步求解,假定含有一个属于小量的常系数,于是可对这个方程组作逐级迭代近似。最简单的一阶近似是将方程组右边积分号下的未知系数代以零阶近似
′Ht()
c
ln
δ
ln
,由此即得方程组左边跃迁系数的一阶近似值ct
mn
()
a
,
ct
i
dH e
mn mn mn
i
t
t
mn
()
() ( )
1
1
0
=+ ′

δττ
ωτ
h
于是在作用下,从至时刻系统自|态跃迁到|态的跃迁几率即为 ′Ht() t
0
t n > m>
Pt d mH ne
mn
i
t
t
mn
() | ( )|=<′ >

1
2
2
0
h
ττ
ωτ
(11.13)
这是讨论在(,区间以外都撤除(当然,前面已说过,t、t也可以分别假定为和
)的含时(或不含时微扰所造成的量子跃迁问题的出发点。注意,只要积分限的时间尺度比
′Ht() )tt
0 0

+∞
1
ω
mn
大很多,便可相应取为。由于±∞ ′H在(,区间之外已撤除,这里的傅立叶积分在上下限处应当是收敛的。附带指出,这里的表达式显然满足前面的普遍结论
)tt
0
P
mn
(t)
dP
dt
mn
t()
|
tt=
=
0
0。
a
由推导知,此结果亦应包括,若假定mn= ′H与无关,将积分积出,得 t
c
i
Ht
nn nn
()1
1=? ′
h
,
从而,因此,此处近似要求|()|
()
ct
nn
12
1> ′H的对角元对时间的积分值很小(虽然时间比t
1
ω
mn
大很多)。
106
§11.3 几种常见含时微扰的一阶近似计算
1,常微扰情况
假定在足够大的时间(,?
TT
22
)内加在系统上的微扰′H与时间无关。这时按上面一阶近似,体系从|态向|态的跃迁速率(单位时间内的跃迁几率)为 n > m>
p
PT
T T
He
mn
T
mn
T
mn
it
T
T
mn
==′?

→∞ →∞?
lim
()
lim
1
2
2
2
2
2
h
ω
dt
= ′ = ′
→∞
lim () ()
T
mn mn mn mn
T
HT H
1
2
2
2
2
2
2
hh
πδ ω
π
δω
或记为
pHE
mn mn m n
= ′?
2 2π
δ
h
( )E (11.14)
这里只涉及单态之间的跃迁,量纲为p
mn
1

,其中δ(EE
mn
)?表示能量守恒,′H
mn
2
代表扰动′H在态和|之间的扰动强度。|n > m> δ函数存在说明,在足够遥远的过去和将来均被撤除,并且在扰动期间微扰的变化十分缓慢时,所有改变能量的跃迁几率均趋于零。同时也说明,在足够缓慢地加上和撤除这种绝热微扰下,处于无简并定态下的系统将仍留在该态上
a

若往连续态跃迁(比如,在静电场扰动下原子的电离),设内有态数目
,则单位时间内向附近单位能量间隔跃迁的几率
EEdE
mm
→+
m
m
ρ()EdE
m
E
m
pHE
mn m
= ′
2 2π
ρ
h
( ) (11.15)
这就是著名的No.2黄金规则(Golden Rule No.2)。
2,周期微扰情况
设微扰呈周期变化,即
′ =+
Ht We e
it it
() ( )
ωω
这里W与无关。于是 t
cT
i
He dt
W
i
eee
mn mn
it
T
T
mn i t it it
T
T
mn mn
()
() ( )
1
2
2
2
2
1
= ′

=+

hh
ωωω
dt
ω
当T充分大时,
c
W
i
mn
mn
mn mn
()
[( ) ( )]
1
2
→++?
π
δω ω δω ω
h
由于ω,方括号中第一个δ -函数表示从|态放出光子的向下跃迁; 第二个> 0 |nm>→ > δ -
a
朗道,非相对论量子力学,上册,第179页。
107
函数表示吸收光子的向上跃迁。假定是吸收扰动电磁场的入射光子而向上跃迁,可得单位时间内由|态的跃迁速率 |nm>→ >
ct
mn
()
()
1
ct
mn
()1
( )tc
m
>= ∑
=>
n
pWEE
mn mn m n
=?
2 2π
δ
h
h( )ω (11.16)
注意此结果和常微扰很相象,只是δ -函数中多了hω项。这表明,周期变化的电磁场可看为一束该频率的光子,量子跃迁系数的一阶近似只考虑单个该频率的光子的吸收和发射,也即单光子过程近似。
§11.3 Sudden及不撤除微扰的情况
现在来考虑这类的微扰:′?∞ =H ()0,′ +∞ =H ()有限。换句话说,加上之后就一直持续下去不撤除。这当然也包括了在某个时刻突然加在系统上并一直不变地持续下去的所谓
Sudden微扰这一特殊情况。
这时,上节的基本公式不适用,因为在t = +∞处,′ +∞H ()不为零,积分在上限急剧振荡而不定。但这种不定性从物理上看是很清楚的,从数学上看是可以绕过的。为了看到这一点,对实施分部积分(设),mn≠
i
He d
mn
it
mn
()
1
= ′


h
ττ
ωτ
= ′?



11 1
ii
Hte
i
H
ed
mn
mn
it
mn
mn i
t
mn mn
h ωω
τ
τ
τ
ωω
()
()
=




Hte
EE EE
H
ed
mn
it
nm nm
mn i
t
mn
mn
() ( )
ω
ωτ
τ
τ
τ
1
于是
| ( )| ( ( ))|
/()
ψδtme ctme
m
iE t
mn mn
iE t
m
mm
> =+ >∑
hh1 /
+


>



>

||
/
/
H
EE
me
e
EE
H
edm
m
mn
nm
iE t
m
iE t
nm
mn i
t
n
m
mn
ΣΣ
h
h
τ
τ
ωτ
|
这里,右边第一项是′H (趋于无限将来也不为零的恒定部分)扰动所造成的对原先|态的一级修正,是个一级定态微扰修正,它只涉及t
n >
→ +∞时,HH H= + ′ +∞
0
()本征态的一级近似表示,描述了原先|态变形,并不涉及态之间的量子跃迁。决定量子跃迁的是第二项,
它前面的系数是含时的。即在此类扰动下,由的跃迁几率
n >
|n>→|m>
P
EE
H
t
edt
mn
nm
mn it
mn
()
()
+∞ =



+∞
1
2
2
ω
(11.17)
对于时刻突然加上的常微扰t = 0 ′ =Ht tW() ()ε (这里ε()t为t = 0时刻的单位阶跃函数),即
108
Sudden微扰情况,有
δ

=
H
t
Wt
mn
mn
()
P
W
EE
mn
mn
nm
()
()
+∞ =
2
2
(1.18)
以上是当H的本征态不好求,只使用的本征态展开的情况,于是跃迁也就是在的基矢之间进行。
H
0
H
0
另有一类Sudden扰动,也是从突然改变为H
0
HH H= + ′
0
,所不同的是这时H也可解,于是跃迁便有可能体现为两套基矢(的和H
0
H的)之间的跃迁。比如,谐振子系统,当时其弹性系数t = 0 k突然改变为′k,假定原先在的基态上,问(通过测量能量)粒子在新基矢中处于基态的几率
H
0
Pxkxk=< ′ >ψψ
00
2
(,)| (,)
这种Sudden扰动的一般公式适用于H的本征态好求,因此扰动也不必很小。可以证明,只要′H很小,这里的和前面的两种Sudden微扰是一致的
a
。因为,设初态为的,末态为
H
0
E
i
()0
H的,即有 E
f
HE HE
fff iii
||,||
()
ψψ?>= > >= >
0
0
于是
()| | ||
()
EE H H
fi fi i f f i
< >=< >?< >
0
0
ψ ψ ψ?
=< >? < >=< ′ >ψ? ψ? ψ?
fi f i f i
HHH|| | | | |
0
∴< >=
< ′ > ψ? ψ?
fi
fi
fi
EE
H||
()
1
0
|
若′H是微扰,则,|EE
f f

()0
|ψ?
ff
>≈ >,这样便得到
P
H
EE
fi
fi
f i
=

2
00() ()
即转化为前面用基表示的Sudden微扰。 H
0
§11.4 光辐射的半量子理论
109
1,概论
众所周知,光辐射和物质之间存在相互作用,这种相互作用不仅影响着光辐射的传播,
更导致光辐射被物质的发射和吸收。经典理论成功地描述了光辐射的传播,然而却无法正确地描述光的吸收和发射。量子理论最辉煌的成就之一正在于能够正确全面地描述光和物质的相互作用,特别是相互作用所导致的光的发射和吸收。
光辐射和物质相互作用的全量子理论应当是从统一的量子化观点处理相互作用着的双方:电磁场和物质原子。就是说,原子(及其中的电子)遵从Schr?dinger方程,电磁场也是被量子化了的。这导致量子电动力学的辐射理论。尽管这个理论本身还存在不少带根本性的问题,但可以说,它是人类迄今为止建立的最成功、最精确的物理理论。
由于课程所限,也考虑到认识的顺序,这里只给出半量子理论。这个理论的实质是对物质中的原子、分子、电子采用量子观点,但对辐射场采用经典电磁波观点。于是就成为如下的一幅物理图象:量子力学中的原子(及原子中的各层电子)在经典电磁场的强迫振动下,产生能级之间的量子跃迁,与此同时便产生出光子或湮灭着光子。
用半量子理论能够给出光辐射和物质相互作用的正确结果,给出产生的或湮灭的光子的能量、强度和偏振(极化)状态,并成为以后全量子理论的正确的出发点。但是,由于它的不彻底性,也如同非相对论量子力学一样,不能解释处于激发态原子的自发辐射问题以及强辐射场中的多光子过程问题等。关于自发辐射问题,爱因斯坦曾依据热力学平衡的一般观念,
半唯象地但却是普适地处理了自发辐射和受激辐射之间的关系。
2,原子的受激量子跃迁。
按这里的半量子理论,原子和电磁场所组成的体系的哈密顿量为
H
m
p
e
c
AVr=?+
1
2
2
()
v
()
v
这里,、、是原子中的电子的、Vr是原子核的库仑场,m
v
p e ()
v
v
vv
v
Art A k r t() cos( )=
0
ω是作为经典场处理的外加电磁场。选择
v
A为横向的,即取库仑规范 =
v
A 0,上述哈密顿量成为
HH H
H
p
m
Vr H
ie
mc
A
e
mc
A
=+′
=+ ′ =+
0
0
2
2
22
v
h
v
(),,
v (11.19)
如果电磁场不十分强,可如Zeeman效应中所作的那样,将′H中的
v
A
2
项略去。
由于电子位置矢径
v
r只局限于原子尺度内变化,因此当电磁场的波长远大于原子的尺度时,就意味着在电子运动的空间范围内,电磁场可看成是空间均匀的,只随时间振荡着。
这便是常说的电偶极近似。在这种
a
ka <<1的近似下,可取e
ik r±?

v
v
1,于是
′ =?Ht
ie
mc
tA() cos
h
v
ω
0
a
参见朗道,非相对论量子力学,上册,第180页。
110
这就是偶极近似下不十分强的电磁场对原子中电子的扰动算子。
假设跃迁前后的初末态分别为|和,能量分别为i > | f > E
ii
= hω和,并记
,利用等式
E
f
= hω
f
EEE
fi f i fi
=?=hω
<>=?<f
x
i
m
fxi
fi
|| ||
ω
h
>
或一般地
<>= < >fpi im fri
fi
|| ||
v v
ω
于是′H在初末态之间的矩阵元为
< ′ >=? +?< >
fHi
i
c
eeAfDi
fi
it it
|| ( ) ||
2
0
ω
ωω
v v
这里
v
v
D er=
E
f
>
是电子的电偶极矩。由上面周期微扰的叙述可知,对于吸收光子而发生跃迁的情况( ),只需取第二项,即有 E
i
< ′ >=< >
fH i
i
c
eA fDi
fi
it
|| ||
2
0
ω
ω
v v
此外,注意到
v
v
vv
v
v v
v
E
c
A
tc
A krtE krt=? = =
1
00
ω
ωωsin( ) sin( )
可得
v v
E
c
A
0
=?
ω
0
,于是上式也可以写为
< ′ >=?< >
fH i
i
eE fDi
it
|| ||
2
0
ω
v v
再注意到
v v v v v
v
BAkAkr=?× =? ×()sin(
0
ωt)
v vv
v
v
v
S
c
EB
A
c
kr t=×=
44
2
0
2
2
π
ω
π
ωsin ( )
若记为ωωρω() ω→ + d频率中单位频率间隔内电磁场的平均能量,有
ρω
ω
π
() ||=<> =
1
8
2
0
2
2
c
S
A
c
v
v
对时间平均
于是
′ =
< >
H
i
c
eA fDi
fi
fi it
2
0
2
2
ω
ω
v v
||
111
= 2
2
0
2
πρ ω()cos(,)
v v
ADD
fi
=
2
3
2
π
ρωD
fi
()
这里已假定了原子中的指向是无规的,取
v
D
fi
v
E
0
方向为z轴,即得方向余弦平方的平均值为
<>==

=cos (,) cos cos
2
0
22
1
4
1
3
v v
ED d
fi
θ
π
θ?
于是,按周期微扰论的公式,得到在辐射场
v v v
v
At A k r t() cos( )=
0
ω扰动和电偶极近似下,
单位频率间隔内的吸收辐射跃迁速率
pHE
fi fi fi
() ( )ω
π
δω= ′?
2
2
h
h
=?
4
3
2
2
π
ρωδ ω
h
hDE
fi fi
()( )
总吸收跃迁速率为
ppd D
fi fi fi fi
=

=() ( )ωω
π
ρω
4
3
2
2
2
h
(11.20)
3,电偶极辐射
上面的受激跃迁将伴随着光子的发射与吸收。所辐射的光子称为电偶极辐射。现对上面这一结果作些讨论。
i,和入射光的频谱有关,并正比于其中能量符合共振条件的成分。 p
fi
ii,在这种电偶极扰动下,分立的初末态之间跃迁的选择定则。
由于
< ′′ ′ >=< ′′ ′ +?
>

nlm rnlm nlm
r
ee
r
i
ee r nlm
ii ii
| | | sin ( ),sin ( ),cos |
v
22
θθ

θ
并注意到
eY
lm lm
ll
Y
lmlm
ll
Y
Y
lm
ll
Y
i
lm l m
lm
lm l
±

±
+
=
±+ ±+
++
±
±
+
=
+?
++
θθ? θ?
θ?
θθ?
sin (,)
()( )
()()
(,)
()( )
()()
(,)
cos (,)
()
()()
,
,
,
m
mm
12
2123
1
2121
1
2123
11
11
22
1
ml
lm
ll
Y(,)
()()
(,)
,
θ? θ?+
+
22
1
2121
m
这里采用了费瑞尔定义,于是可知,矩阵元三个分量不全为零的条件为 Px
l
m
()
112
ll l
mm m
= ′?=±
= ′?=±
1
01,
(11.21)
这便是电偶极跃迁的选择定则。
iii,关于极化和角动量守恒问题
<a>电子运动:
v
D
fi
x分量位相为零时,分 <b>电子y
v
D
fi
运动:x分量位相已到
π
2
,分
量位相已为
y
π
2
,左手螺旋。与此相关,发出 量位相才到零,这导致右手螺旋。与此相关,
的光子沿z方向观察为左手螺旋圆偏光; 而 被吸收的光子为沿z方向右手螺旋圆偏光。
在x -平面内观察它则为垂直y z轴的线偏光。
(如果原子向电磁场放出一个角动量为的光子),h
按角动量守恒,这时原子中电子的应减少一个,即应L
z
L
z
h ′ =?mm1 (? ),
而这又导致的
m =?1
v
D
fi
z分量为零,而x和分量中项不为零,并且它们之间有如下关系,
y <>
e
i
fi
< > = < >DiD
yfi xfi
(11.22)
由于电子自|态→态跃迁时,减少,这相应于电子的i > >| f L
z
h
v
D
fi
为绕z轴左手旋转,如图<a>。
(如果原子从电磁场吸收一个角动量为的光子),h
这时原子中电子的应增加一个,即应有L
z
h ′ = +mm1(?m = +1)。这时,除的
v
D
fi
z分量为零外,x和分量中只有含的项不为零,于是它们之间有关系 y <e
i?
>
< > = < >DiD
yfi xfi
113
由于电子自|态态跃迁时,L增加,这使电子的i > →>| f
z
h
v
D
fi
为绕z轴右手螺旋,如图<b>。
对于的发射光子的情况,只有?m = 0 < >D
zfi
不为零,因此沿z轴观察不到这种辐射,
而在x -平面内观察时发出的光子是沿y z轴线偏振的。这个光子不带走角动量,因为电子自态向态跃迁时、L、L的期望值均未改变(前两者仍为零,后者仍为),因此被光子带走的角动量的期望值也应为零。
|i > | f > L
x y z
mh
当然,上面是就单个原子分析的,若介质中大量原子的
v
D
fi
杂乱无章地取向,实验上将观察不到以上的这些分析。
4,受激氢原子的光电效应
上面的量子跃迁过程是针对初、末态均为分立态的情况,这就是通常的原子由辐射场所激发(或退激发)的情况。但也存在原子被辐射场所电离的光电效应,这时末态在渐近意义下为自由电子状态。
对于现在吸收光子的情况,辐射场扰动为
′ ==?

H
ie
mc
eA
e
mc
eA
it it
h
vv
v
22
00
ωω
p
为计算简化,这里略去原子核库仑场对逸出电子状态的影响,从而取末态为<=<fp
f
||
v
a

同时取初态为s态,即有
<>= <>=
vv
h
v
vv
h
rp e ri
a
e
f
ip r
r
af
|
()
,|
/
/1
2
1
32
0
3
0
π
π
于是
′ =< ′ >=


HfHi
e
mc
e
a
eApe
fi
it
ip r
r
a
f
||
()
/
/
2
1
2
1
32
0
3
0
ω
π
π
h
v
vv
vv
h
dr


= rdepAe
a
mc
e
a
r
rpi
f
ti
f
vv
v
h
h
vv
0
/
0
3
0
2/3
)(
1
)2(
1
2
ω
π
π
其中积分为
edr e rdr
ip r
r
a
ip r
r
a
ff



=
∫∫∫
vv
hh
v
/cos/
cos
00
2
0
2
1
1
0
θ
π
θ?d
a
这一近似与扰动的频率ω及初态|有关。当hi > ω>>原子电离能,和|处于i > s态时,误差可以忽略。
详细可参见R,Shankar,Principles of Quantum Mechanics,Plenum Press,New York,1980。
114
=
+
8
1
0
0
22
2
π /
() ( )
a
a
p
f
h
跃迁速率为
PHEE
fi fi p i
= ′
2
2
π
δω
h
h()
=
+

2
2
1
8
1
64
1
2
33
0
3
0
2
2
0
6
0
2
4
π
ππ
π
δ
h
h
ω
v
v
h
h()
()
(
e
mc
a
Ap a
ap
EE
f
f
pi
) (11.23)
注意,这是向的动量本征态的跃迁。实际上,对于这种属于连续分布的末态,假定在
v
p
f
方向和值附近单位能量间隔和单位立体角内的出射电子态密度为E
p
ρ(,E
p
)
m
,则有
ρdE d p dp d
fff
=
2
由于,代入上式即得Ep
ff
=
2
2/ ρ= mp
f
。于是向d?内跃迁速率为
PdPEd
di fi f f
E
f
E?
←←
=

ρ(,)
=
+
4
1
2
0
3
42
0
2
4
0
2
eap
mc
ap
Ap d
f
f
f
π h
h
v
v
()
这里值取符合能量守恒的数值 p
f
pmE
fi
=+2( )hω
对立体角积分,即得总的电离速率 4π
P
eap A
mc
ap
i
f
f
=
+
16
31
2
0
33
0
2
42
0
2
4
v
h
h
()
(11.24)
注意上面已叙述过电磁场的能量密度ρ
ω
π
=
2
0
2
2
8
v
A
c
,于是入射到原子上的能流密度
115
Jc
A
c
==ρω
ω
π
()
2
0
2
8
v
于是得到总光电截面
σ=
电磁场因电离而导致的能量损失率能秒电磁场入射的能流密度能厘米秒
()
(/ )
2
==
+
h
v
h
h
ω
ω
π
π
ω
P
A
c
eap
mc
pa
i
f
f
2
0
2
2
0
33
3
2
0
2
2
4
8
128
31
(11.25)
注意这个结果是在i,对电磁场作电偶极近似; ii,对末态作自由电子近似这两个假设下得到的。满足第二个近似,要么电磁场频率足够高,但这和第一个近似会产生矛盾,要么要求初态为s态。
116
习题
第一章
1,在宏观世界里,量子现象常常可以忽略。对下列诸情况,在数值上加以证明,
(1)长ml 1=,质量kgM 1=的单摆的零点振荡的振幅;
(2)质量gM 5=,以速度scm /10向一刚性障碍物(高,宽1 )运动的子弹的透射几率;
cm5 cm
(3)质量kgM 1.0=,以速度sm /5.0运动的钢球被尺寸为1
2
5.1 m×窗子所衍射。
2,用(电子质量),mce,,,h M (质子质量)表示下列每个量,给出粗略的数值估计,
(1)玻尔半径( ; (2)氢原子结合能( ; (3)玻尔磁子; )cm )ev
(4)电子的康普顿波长( ; (5)经典电子半径( ; (6)电子静止能量( ; )cm )cm )mev
(7)质子静止能量( ; (8)精细结构常数; (9)典型的氢原子精细结构模型。 )mev
3,从下列实验中的两个实验能够了解到辐射或力学体系的什么性质?
(1)光电效应; (2)黑体辐射谱; (3)夫兰克-赫兹实验; (4)戴维孙-革末实验;
(5)康普顿散射。详细描写所选的实验,指出测量的哪个效应是非经典的,说明原因并用量子概念解释之,给出适当方程。
第二章
1,写出动量表象中的不含时间的dingeroSchr &&方程。
(答,)()()()(
2
2
pEp
p
iVp
m
p vv
v
h
v
v
=
+。)
2,设粒子在势场)(rV中运动,
v
(1)证明其能量平均值为,
117
∫∫

+== ψψψψ V
m
xdxWdE
2
[
2
22
h
W称为能量密度;
(2)证明能量守恒公式,0=+
S
t
W
v
其中,)(
2
2
+?
= ψ
ψ
ψ
ψ
ttm
S
h;
3,一个质量为m的粒子受力)()( rVrF
vv
v
=作用,使其波函数满足动 量空间的
dingeroSchr &&方程,
),()),()
2
(
2
2
tp
t
itpa
m
p
p
vv
v

=
求力。 )(rF
v
v
(答,rarF
vv
v
=)(。)
4,考虑任意势)(xV的时间无关的一维dingeroSchr &&方程,证明如果一个解具有性质,
当±∞→x时0→)(xψ,则此解必然非简并,进而是实的,除了某一可能的相因子。
5,考虑一个一维束缚粒子,
(1)证明,0),(),( =

∞+
∞?
txtx
dt
d
ψψ ;
(2)证明某粒子在一给定时刻是定态,则它将永远保持定态;
(3)若在0=t时,波函数在axa <<?范围内是常数,而在其它处为零,利用系统的本征态表达以后时间的完整波函数;
(答:(3)

=
n
n
t
E
i
n
xatx
e
n
)(),( ψψ
h
,

=
a
a
n
dxx
a
a )(
2
1
ψ。)
6,),( txψ是质量为m的自由粒子的一维dingeroSchr &&方程的解,
118
e
ax
Ax
22
)0,(
=ψ。
0=(1)求出t时,动量空间的几率振幅; (2)求出),( txψ。
(答:(1)
e
pa
Aa
p
2
22
4
2
)0,(
h
h
=ψ ;
(2) }
2
exp{
2
),(
2
2
2
m
ti
a
x
m
ti
a
Aa
tx
h
h
+

+
=ψ。)
7,设),( pxF是,的整函数,证明,
k
x
k
p
k
k
x
F
i
Fp
=
h
],[,
k
k
p
F
Fx
= h],[。
整函数是指可展开成的函数。
∑∑
=
=
nmlk
n
l
m
k
mn
kl
pxCpxF
,
3
1,
][),(
8,给定算符A
,B,令
L],
,
[
],
,
[
,

1210
CACBACBC ===
L]
,
[
1?
=
nn
CAC,证明,


=
=
0

!
1
n
n
AA
C
n
B
ee
9,证明:
eeeeeee
CABCBABA
2
1

2
1

)

(
22
λλλλλλλ
==
+
其中,且与]
,
[
BAC = A
,B都对易。
第三章
1,一个质量为的粒子被限制在一维区域0m 0,=≤≤ tax时,初态波函数为,
a
x
a
x
a
x
ππ
ψ sin)cos1(
5
8
)0,( +=
(1)在后来某一时刻似的波函数是什么?
0
t
(2)体系在0=t和t时的平均能量是多少?
0
t=
119
(3)在时,于势箱左半部
0
tt = )
2
0(
a
x ≤≤发现粒子的几率是多少?
(答:(1)

=
n
nn
xtAtx )()(),( ψψ,)sin(
2
)(
a
xn
a
x
n
π
ψ =,
,
e
tiE
nn
n
AtA
h/
)0()(
=
2
222
2ma
n
E
n
πh
= ; (2)
2
22
5
4
ma
E
πh
= ;
(3) )
2
3
cos(
15
16
2
1
2
0
2
ma
th
p
π
π
+=。)
2,一个刚体具有惯性矩,自由地在
z
I yx?平面中转动.令φ是?x轴与转动轴间
的夹角。
(1)找出能量本征值和相应的本征波函数)(φψ
n;
(2)在0=t时转子由波包φψ
2
sin)0( A=描述,求在0>t时的)(tψ。
(答:(1),
2
22
z
m
I
m
E
h
=
e
im
m
φ
π
φψ
2
1
)( =
(2) )(
42
)(
)
2
2()
2
2(
ee
t
I
it
I
i
zz
AA
t
hh
+?+
+?=
φφ
ψ。)
3,一个电子被禁闭在一维盒子中,并处于基态上,盒宽为10,电子能量为38。计算,m
10?
ev
(1) 电子在其第一激发态的能量; (2) 当电子处于基态时盒壁所受的平均力。
(答:(1) ; (2)evE 152
2
= evcmF?×=
19
106.7。)
4,求在一维短程势V )()(
0
xVx δ?=中质量为的束缚能。 m
(答:
2
0
2h
mV
E?=。)
5,证明,若
2
h
= px,则简谐振子最小能量为
2
ωh

120
6,考虑作一维运动的粒子,哈密顿量为,)(
2
2
xV
m
p
H +=,其中0)( ≤xV,
对所有x值成立,0)( =±∞V,且V并非处处为零。证明至少存在一束缚态。
7,一个质量为的粒子在势m )(xV的作用下作一维运动.假如它处在
m
E
2
22
γh
=的能量本征态
e
x
2
22
γ
x
4
1
2
)/)( πγ(ψ
=。
(1)求粒子的平均位置; (2)求粒子的平均动量; (3)求)(xV ; (4)求粒子动量在
dppp +→之间的几率dppP )(,
(答:(1) 0=x ; (2) 0=p; (3)
m
x
x
2
)(
242
V
γh
=;
(4) dpdppP
e
p
22
2
2
1
22
)
1
()(
γ
πγ
h
h
=。)
8,一个质量为的粒子处于频率为m ω的一维谐振子势阱的基态上,受到一冲力)(tpδ,求它处于基态的几率。
(答:
e
m
p
P
hω2
0
2
=。)
9,证明:

为一不依赖于?
n
n
xnEE
2
0
0)( )(xV的常数。其中,为本征态
n
E n的能量,哈密顿量为)(
2
2
xV
m
p
H +=。
10,粒子在深度为V,宽度为的对称方势阱中运动,求,
0
a
(1)在阱口附近刚好出现一条束缚能级)(
0
VE ≈的条件; (2)束缚态能级总数。
(答:(1)
2222
0
/2 πnamV =h ; (2) ]2[1
0
mV
a
N

+=。)
121
第四章
1,一电子被束缚在半径为的匣子中,求处于基态的电子对匣壁的压力。 R
(答:
3
22
R
F
μ
π h
=。)
2,一个质量为的粒子被限制在半径为m ar =和br =的两个不可透穿的同心球面之间运动,不存在其它势.求粒子的基态能量和归一化波函数。
(答:
ab
ar
rab
r
=
)(
sin
12
4
1
)(
π
π
ψ
v
,bra ≤≤。)
3,求证在l的本征态下,
z
0

==
yx
ll。
4,设体系处于态
222111
YcYc +=ψ,求,
(1)l的可能测量值及平均值;
z
(2)
2
l
v
的可能测量值及相应的几率;
(3)l及的可能测量值。
x y
l
5,质量为的粒子在势场 m
2
3
/)( rr λ?=V,( 0>λ )中运动,用测不准关系估
算其基态能量。
6,一质量为m的粒子在对数势 V )/ln()(
0
rrcr = 中运动。证明,
(1) 所有的能量本征态都有相同的均方速度,并求之;
(2) 任何两个能量本征态间的能量间隔是与质量m无关的。
7,质量为μ的粒子在中心力场
s
r
rV
α
=)(,( 0>α )中运动。证明,存在束缚态的条件为20 < <s ; 再进一步证明在
0~E附近存在无限多束缚态能级。
8,质量为μ的粒子在球势阱 )()( arrV= γδ,( 0,>aγ )中运动,求存在束缚态的条件。
122
(答:μγ 2/
2
h≥a。)
9,设粒子在无限长的圆筒中运动,筒半径为,求粒子能量。 a
10.设
2
)(
r
A
r
a
r +?=V,( 0,>Aa )。求粒子能量本征值。
第五章
1,证明,


=
+
0,
,
2

mn
nmmn
mn
pxxp
A (A是实数)是厄米算符。
mn,
2,厄米算符A
与满足B
1

22
== BA,0

=+ ABBA,求,
(1) 在A表象中A
与的矩阵表示式,并求的本征函数表示式; B
B
(2) 在B表象中A
与的矩阵表示式,并求B
A
的本征函数表示式。
(3) A表象到B表象的幺正变换矩阵S,
3,证明,BAAB detdet)det(?=
AASS det)det(
1
=
)()( BATrABTr =
)()()( BCATrCABTrABCTr ==
Tr。 TrAASS =
)(
1
4,设A为厄米或幺正矩阵。证明,在任何表象中有,
)exp()exp(det TrAA =。
第六章
1,多粒子系,如不受外力,Hamilton量表成
123
()
∑∑
<
+=
ji
ji
i i
i
rrV
m
p
H
vv
v
2
2
证明 总动量守恒。

=
i
i
pP
v
v
2,多粒子系,如所受外力矩为0,则总角动量

=
i
i
lL
vv
守恒。
3,证明 对于经典力学体系,若,A B为守恒量,则{ }(Poisson括号)也是守恒量(但不一定是新的守恒量)。对于量子力学体系,若,
BA,
A
B
为守恒量,则[ ]BA
,
也是守恒量
(不一定是新的守恒量)。
4,() {}
=?=
x
aiapaD
xx
expexp h表示沿x方向的平移算符,设与对易,求的一般形式。
()xf ()aD
x
()xf
(答:() ( )axfxf?=,即为()xf x的周期函数,周期为。) a
5,证明 周期场中的Bloch波函数
( ) ( ) ( )xikxx
k
φψ exp=
( ) ( )xax
kk
φφ =+
是的本征态,相应本征值为()aD
x
[ ]ika?exp。
6,设Hamilton算符表为
()zayaxa
J
l
J
l
J
l
H
z
y
x
321
3
2
2
2
1
2
222
++?++=
(1) 写出r
v
和l
v
(角动量)的Heisenberg方程;
(2) 若,,列出主要的守恒量。
21
JJ = 0
21
== aa
第七章
1,在
z
σ表象中,求
x
σ的本征态。
124
(答:
1
1
2
1
,对应本征值;1+
1
1
2
1
,对应本征值1?。)
2,在自旋态()
=
0
1
2
1 z
sχ下,求
2
x
s?与
2
y
s?。
(答:
4
2
22
h
=?=?
yx
ss。)
3,设矩阵,A B,满足,C 1
222
=== CBA iACBBC =?。
(1) 求证 0=+=+ CAACBAAB;
(2) 在表象中,求出A B,C的矩阵(设无简并)。
(答:(2),B,C,参数,c满足b。)
=
10
01
A
=
0
0
1
b
b
=
0
1
c
co
b ibcc =?
22
4,矩阵,A B满足,0
2
=A 1=+
++
AAAA,AAB
+
=。
(1) 证明 BB =
2;
(2) 在B表象中求出的矩阵表示式。 A
5,满足U,1==
++
UUU 1det =U的维矩阵称为矩阵。求的一般形式。 n
n
SU
2
SU
(答:U,其中
=
αβ
βα
ωω
ωω
ii
ii
ee
ee
cossin
sincos
α,β,ω为实参量。)
6,某个状态ψ是和的本征态,
2
L
z
L
( ) ψψψψ hh mLllL
z
=+=
,1
22
在这个态下计算
x
L

2
x
L。
(答:0
=
x
L,()[]
222
1
2
1
hmllL
x
+=。)
7,一粒子的某一量子力学态,在直角坐标x,,中由归一化波函数 y z
() ()
++?=
2
1
222
2
5
exp,,zyxZzyx α
π
α
ψ
描述。证明系统处在一个具有确定角动量的态上。并给出该态相应的和值。
2
L
z
L
125
(答:,。)
22
2h=L 0=
z
L
8,(1) 考虑自旋为
2
1
的系统。求出算符
zy
sBsA +的本征值及归一化的本征函数。其中,
是角动量算符,且,
y
s?
z
s? A B是实常数。
(2) 假定此系统正处在以上算符的一个本征态上。求测量得结果
y
s? 2h的几率。
9,考虑一个电子在均匀沿方向的磁场中运动。在z 0=t时刻测量到电子自旋沿方向。
求在时自旋的态矢及沿
y+
0>t dingeroSchr && x方向的平均极化率(正比于的期望值)。
x
S
(答:()
=
ti
ti
ie
e
t
ω
ω
ψ
2
1
,tS
x
ω2sin
2
h
=。)
10,两个自旋为21的粒子组成一个复合系统。自旋在A 21+=′
z
S的本征态,自旋B在
21+=′
x
S的本征态。求发现系统总自旋为零的几率。
(答:。) %25=P
第八章
1,设带电粒子在相互垂直的均匀电场?及均匀磁场B
v
中运动,求其能谱及波函数。(取磁场方向为轴方向,电场方向为z x轴方向。)
2,设带电粒子在均匀磁场B
v
及三维各向同性谐振子场()
22
0
2
1
rr μω=V中运动,求能谱公式。
3,一个处于磁场AB
vv
×?=中的无自旋带电粒子的Hamilton量为
()
2
2
1
= rA
c
e
p
m
H
v
v
v
其中( )
zyx
pppp,,=
v
是粒子位置r
v
的共轭动量。设
x
eyBA?
0
=
v
,对应着一个均匀磁场
z
eB?
0
B =
v

126
(1) 证明和是运动恒量;
x
p
z
p
(2) 求该体系的能级。
(答:(2) ()Lh,2,1,0,
2
1
2
0
2
=
++= n
mc
eB
n
m
p
E
z
n
。)
4,在一次经典的台面实验中,一单色中子束()在干涉仪的A点发生布拉格反射分成两束,然后(经过另
0
4445.1 A=λ
2
1
C
B
B
v
A
一次反射)又汇交于D点。其中
一束穿过一磁场强度为B
v
的横向
D
磁场区域,经过的距离为l(如图)。
假定从A到D的两条路径除有无
磁场外完全一样。在中子极化方向
平行或反平行与磁场的情况下,找出D点强度依赖于B
v
,l,及中子波长的明显表达式。
5,考虑一个无限长的螺线管。其中通有电流I,结果在螺线管内产生了一个均匀恒定的磁场。假设在螺线管外的区域中,电荷为、质量为的粒子的运动可以用方程描述。假定对于,方程的解由下式给出,
e m dingeroSchr &&
0≡I
( ) ( )xetx
tiE
vv
00
0
,ψψ =
(1) 对情况给出并求解螺线管外区域中的方程。 0≠I dingeroSchr &&
(2) 考虑一个为上述粒子
准备的双缝衍射实验(如图),
假定两缝间距远大于螺线 d
管直径。计算由于的 0≠I
螺线管的存在而使衍射图象
在屏上产生的移动。假定lS? S?>>。

(提示:() ()(xtxtx
A
)
vvv
v
ψψψ,,
0
=,其中
i
第九章
螺线管
d

() ()=
xxA
c
e
A
vv
v
v
ψ 0,( )1=h。)
127
1,设非谐振子的Hamilton量为
322
0
2
22
2
1
2
xx
dx
d
H βμω
μ
++?=
h
,(β为常数),取
22
0
2
22
0
2
1
2
x
dx
d
H μω
μ
+?=
h
3
xH β=′
试用微扰论计算其能量及能量本征函数。
2,一维谐振子Hamilton量表为
22
2
22
0
2
1
2
x
dx
d
H μω
μ
+?=
h
设再加上一个微扰
()1,
2
22
<<=′ λμω
λ
xH
试用微扰论求能级的修正(到三级近似),并和精确解比较。
3,自旋为
2
1
的三维各向同性谐振子,处于基态。设粒子受到微扰rH
vv
=′ σλ作用,求能级修正(二级近似)。
(答:,。)
()
0
1
0
=E
() 222
0
2/3 μωλ?=E
4,自旋为的两个全同粒子在谐振子势中运动,0
()
2
2
2
1
2
2
2
2
2
1
22
0
2
1
2
xx
xx
H ++
+
= μω
μ
h
设粒子之间有相互作用
( )[ ]
2
21
2
0
exp xxVH=′ β
试用微扰论求体系的基态能级修正(一级近似)。
(答:
() 22
0
1
0
/21 αβ+=VE。)
5,一维无限深势阱( )ax <<0中的粒子,受到微扰
128
()
<<
<<
=′
axa
a
x
ax
a
x
xH
212
202
λ
λ
的作用,求基态能量的一级修正。
(答:
()
λ
π
+=
2
1
1
2
2
1
E。)
6,一带电粒子被约束在谐振子势
2
2
1
kx=V内,系统处于一恒定常数外电场E中,试计算基态能级移动(二级近似)。
7,假设原子有非零半径,且其电荷沿该尺寸均匀地分布。求由于点电荷与此扩展电荷间差异导致的氢原子的1态和态的能移。
cmr
p
13
10

s p2
8,质量为的粒子约束在一半径为的圆周上,除此之外它是自由的。加一个微扰势m a
θθ cossinAH =′(其中θ是圆周上的角位置)。对于最低的两个态求出修正后的零级波函数,并计算它们的微扰能量修正(二级近似)。
9,讨论和计算氢原子基态的斯塔克效应。
第十章
1,质量为μ的粒子被中心势() (0,
2
>= α )
α
r
rV散射
(1) 求各分波的相移
l
δ;
(2) 设作用势较弱:8,求相移,散射振幅和截面的表达式; 1/
2
<<hμα
(3) 用Born近似计算散射振幅和截面,并与(2) 结果比较。
2,粒子被势场() (0,
4
>= αα rr )V散射,求低能极限下(只考虑波)散射的散射长度,
相移,散射振幅和截面。
s
(提示:引进无量纲变量μαξ 2/rh=。)
3,质量为的粒子被一个很重的靶粒子散射,两粒子的自旋均为m
2
1
。设相互作用势为
()rsAV
vv
s
v
δ
21
=,r
21
rr
vvv
=,是很小的常量,因此可以用Born一级近似来处理。设入A
129
射粒子的自旋“向上”,靶粒子的自旋取为无规分布。求散射总截面以及散射后粒子自旋仍然保持“向上”的几率。
(答:
22
16
3
Am
t
π
σ =,。) 3/1=
r
P
4,设中性原子的电荷分布为球对称,密度( )rρ具有如下性质:∞→r,()rρ迅速趋于0,
并有,但(正负电荷分布不均匀)。今有质量为m,电荷为的粒子沿轴方向入射,受到此电荷分布所产生静电场的散射。试用Born近似计算向前散射(
()

= 0
3
xdrρ
e z
0=
()

=
32
Axdrrρ ≠ 0
θ)的微分截面。
(答:()
4
222
9
0
h
Aem
=σ。)
5,对吸引的球方势阱(V,
0
V?= ar <;0=V,ar >),求出正能量下l的相移0=
0
δ
的能量依赖关系。由此证明,高能时()
k
maV
2
h
→kδ
0
,并从Born近似得出这个结果。
6,在Born近似下计算一质量为m的粒子被δ -函数势( ) ( )rgrV
vv
3
δ=散射的微分截面及总截面。
7,一个质量为的粒子,在排斥势m
22
ar
Ae
=V中运动,用Born近似求出微分散射截面,
确定到一个相乘的常数。
第十一章
1,具有电荷的离子,在其平衡位置附近作一维简谐运动,在光的照射下发生跃迁。入射光能量密度为
q
()ωρ,波长较长。求
(1) 跃迁选择定则;
(2) 设离子原来处于基态,求每秒跃迁到第一激发态的几率。
2,一维运动的体系,m态跃迁到n态所相应的振子强度定义为
22
mxnf
nm
nm
h
μω
=
μ为振子质量。求证
130
=
∑∑
和指对一切能量本征态求
nn
nm
f,1
(Thomas-Reich-Kuhn求和规则)
3,一个处于第一激发态()的氢原子位于一空腔中,当空腔的温度等于多少时,自发跃迁几率和受激跃迁几率相等?
p2
(答:T。) K
5
1076.1 ×=
4,一个电荷为的粒子被禁闭在各边为的立方盒子中,给定一个电场 e b2
()
>>
<
=
0,0,
0,0
0
teE
t
E
t
α
α
v
v
0
E
v
垂直于盒子的某一面。t时,带电粒子处于基态,求0= ∞=t时粒子处于第一激发态的几率。
131